PASTEST Flashcards

1
Q

A 45-year-old man is admitted with right-sided abdominal pain and his investigations include an abdominal ultrasound scan. The ultrasound reveals right-sided hydronephrosis with a dilated right ureter.
Which of the following inflammatory processes might impinge upon the right ureter and cause obstruction?

1) Acute appendicitis
2) A perforated caecal carcinoma
3) Crohn’s disease affecting the terminal ileum
4) All of the above
5) None of the above

A

All of the above
As stated, given the ultrasound findings, it is important to consider acute appendicitis, Crohn’s disease and a perforated caeceal carcinoma as a possible cause of his symptoms, due to their close proximity anatomically to the right distal ureter.
Acute appendicitis
The appendix lies anterior to the right ureter. As such, acute appendicitis and associated inflammatory processes can cause hydronephrosis and a hydroureter.
A perforated caecal carcinoma
Inflammatory processes, including local and generalised peritonitis, could cause obstruction, as the caecum lies anterior to the right ureter. This question refers specifically to inflammatory processes causing obstruction. However, tumour bulk could also cause obstruction of the right ureter.
Crohn’s disease affecting the terminal ileum
Again, the terminal ileum is found just anterior to the right ureter, and inflammation of Crohn’s disease affecting the terminal ileum could cause obstruction.
None of the above
Although a calculus in the distal right ureter would be included in the differential diagnosis for this patient, local anatomy and associated pathologies must be considered, given the ultrasound findings. Anteriorly, the right ureter is related to the terminal ileum, caecum, appendix, and ascending colon and their mesenteries.

How well did you know this?
1
Not at all
2
3
4
5
Perfectly
2
Q

A 17-year-old male presents with fever, malaise and severe anorectal pain with swelling. This is on a background of six month history of diarrhoea and weight loss.
What is the most likely diagnosis?

1) Intersphincteric abscess
2) Ischiorectal abscess
3) Necrotising fasciitis
4) Perianal abscess
5) Supralevator abscess

A

Explanation
Perianal abscess
The most common type of anorectal sepsis is a perianal abscess (60%), and it is more commonly seen in men than women. Crohn’s disease is associated with an increased incidence of anorectal sepsis.
Intersphincteric abscess
Intersphincteric abscesses only account for a small number (around 5%) of anorectal abscesses.
Ischiorectal abscess
Ischiorectal abscesses account for around 20% of anorectal abscesses. Therefore, a perianal abscess is more likely, given the clinical scenario.
Necrotising fasciitis
If untreated, perianal sepsis can lead to necrotising fasciitis, so prompt incision and drainage of abscesses is required. However, there are no features of necrotising fasciitis described in the question stem. Necrotising fasciitis is a surgical emergency caused by bacterial infection of the fascia and subsequent necrosis of the subcutaneous tissues. It requires emergency debridement of affected areas with wide margins and intravenous antibiotics. Even with prompt treatment, mortality is still approximately 25–35%.
Supralevator abscess
Supralevator abscesses can develop from abdominopelvic pathology and can produce lower abdominal or rectal pain.

How well did you know this?
1
Not at all
2
3
4
5
Perfectly
3
Q

A 3-week-old boy presents with vomiting, jaundice and dehydration. Investigations reveal hypokalaemia and metabolic alkalosis.
What is the most appropriate initial management?

1) Correction of metabolic derangements
2) Feeding jejunostomy
3) Ramstedt’s pyloromyotomy
4) Total parenteral nutrition
5) Upper GI endoscopy

A

Explanation
Correction of metabolic derangements
The classical electrolyte abnormality of infantile pyloric stenosis (IHPS) is hypokalaemic hypochloraemic alkalosis. Surgery should be undertaken only after careful correction of the abnormalities, with the consult of an experienced paediatrician and anaesthetist.
Feeding jejunostomy
This would not be appropriate initial management of IHPS. Furthermore, pyloromyotomy is required as a curative procedure.
Ramstedt’s pyloromyotomy
This is the definitive surgical procedure required. However, correction of electrolytes before surgery is paramount. Of note, the umbilicus should be excluded from the operative field because of the risk of Staphylococcus aureus infection.
Total parenteral nutrition
Correction of electrolyte abnormalities is required as initial management. Given the significant electrolyte derangements, total parenteral nutrition would be challenging and ill-advised.
Upper GI endoscopy
The tumour is most commonly diagnosed clinically as a palpable tumour on test feed, alongside a history of projectile vomiting and hungry feeding, with no bile in the vomitus. This prevents the need for a diagnostic endoscopy, while correction of electrolyte abnormalities should be the first priority.
The incidence of IHPS is 3–4/1000 (1 in 300–900) live births. It is more common in males and has a slightly higher incidence in first borns. If the mother had IHPS, the risk is 20%, and if the father had IHPS, the risk is 5%. The pylorus is increased in length and diameter with hypertrophy of the circular muscle layer and autonomic nerves. There may be jaundice in 5–10% of cases due to a reduction of glucuronyl transferase.

How well did you know this?
1
Not at all
2
3
4
5
Perfectly
4
Q

A 60-year-old gentleman with a past history of atrial fibrillation attends the Emergency Department, complaining of a 3-day history of progressively worsening generalised abdominal pain and bloody diarrhoea. He has had intermittent colicky postprandial abdominal pain for the last 6 months. There are no exacerbating factors, and his weight has gradually declined, presumably due to reduced oral intake from apprehension to eat. Clinical examination reveals sinus tachycardia and hypotension. He has severe generalised abdominal pain and distension, but no specific tenderness. What is the most likely diagnosis?

1) Acute diverticulitis
2) Gastrointestinal ischaemia
3) Perforated viscus
4) Ruptured abdominal aortic aneurysm
5) Ulcerative colitis

A

Explanation
Gastrointestinal ischaemia
The history suggests a gastrointestinal cause, for which the most likely answer is gastrointestinal ischaemia, mostly likely ischaemic colitis. It frequently presents as severe abdominal pain that is out of proportion to the clinical signs, bloody diarrhoea and a significantly raised serum lactate level that is poorly responsive to fluid resuscitation. The concurrent atrial fibrillation is also a potential thromboembolic source that may trigger such acute ischaemia.
Acute diverticulitis
If the causative pathology were acute diverticulitis, signs of sepsis would be expected. Additionally, diverticulitis presents with localised abdominal pain, usually the left iliac fossa, as diverticular disease most commonly affects the sigmoid colon.
Perforated viscus
A perforated viscus is merely likely to present with localisable signs and symptoms. For example, a perforated gastric ulcer would produce localised tenderness and guarding within the upper abdomen. Moreover, the history of colicky postprandial pain and atrial fibrillation is suggestive of abdominal ischaemia.
Ruptured abdominal aortic aneurysm
A ruptured abdominal aortic aneurysm is unlikely, as this patient has no risk factors suggesting the diagnosis.
Ulcerative colitis
Although ulcerative colitis is a possible cause, in the acute setting, it is more likely in a younger individual.

How well did you know this?
1
Not at all
2
3
4
5
Perfectly
5
Q

A 70-year-old gentleman undergoes a difficult laparoscopic cholecystectomy where the cystic duct is found to be wide. Before this procedure, he had undergone an endoscopic retrograde cholangiopancreatography (ERCP) and a sphincterotomy for common bile duct stones. In the post-operative period, he develops a subdiaphragmatic collection which is drained. The fluid is bile, and so a stent is placed within the common bile duct by ERCP. Within 24 hours, he becomes tachycardic and hypotensive, passes black stools and develops severe abdominal pain.
What is the most likely diagnosis?

1) Acute pancreatitis
2) Enterocutaneous fistula
3) Gastrointestinal haemorrhage
4) Small bowel obstruction
5) Small bowel perforation

A

Gastrointestinal haemorrhage
Division of the sphincter of Oddi with sphincterotomy may cause pancreatitis, duodenal perforation or bleeding. The black stools suggest the passage of melaena, and therefore upper gastrointestinal bleeding post-ERCP.
Acute pancreatitis
Pancreatitis is one of the most common complications post-ERCP, with an incidence of around 20%, and can also present with severe abdominal pain (localised to the epigastric region, with radiation to the back), hypotension and tachycardia. However, black stools are not consistent with acute pancreatitis.
Enterocutaneous fistula
An enterocutaneous fistula would not cause the passage of black stools.
Small bowel obstruction
Gallstone ileus as a form of small bowel obstruction post-ERCP is rare. Again, passage of black stools is more indicative of upper gastrointestinal bleeding which would not be present in small bowel obstruction.
Small bowel perforation
Perforation of the duodenum (usually periampullary or ductal perforation due to sphincterotomy or guidewire manipulation) is a recognised complication of ERCP. However, it is rare, with a reported rate of 0.6%. Therefore, gastrointestinal haemorrhage would be more likely.

How well did you know this?
1
Not at all
2
3
4
5
Perfectly
6
Q

A 46-year-old male presents with an acutely painful left groin. He is tender over the affected area and you notice that his swelling originates inferior and lateral to the pubic tubercle.
What is the most likely diagnosis?

1) Direct inguinal hernia
2) Femoral hernia
3) Indirect inguinal hernia
4) Obturator hernia
5) Spigelian hernia

A

Explanation
Femoral hernia
Femoral hernias always arise inferior and lateral to the pubic tubercle.
Direct inguinal hernia
Inguinal hernias originate superior and medial to the pubic tubercle, not inferior and lateral. Although clinical assessment can classify inguinal hernias as direct (through weakness in the abdominal wall) or indirect (through the inguinal canal), inguinal hernias can only be truly classified into ‘direct’ or ‘indirect’ at operation, when their relation to the inferior epigastric artery can be observed (direct – medial, indirect – lateral).
Indirect inguinal hernia
As above, inguinal hernias originate superior and medial to the pubic tubercle. Obturator hernia
Obturator hernias are rare and usually found in women.
Spigelian hernia
Spigelian hernias originate at the linea semilunaris of the abdominal wall.

How well did you know this?
1
Not at all
2
3
4
5
Perfectly
7
Q

A 54-year-old woman presents to the Emergency Department with severe upper abdominal pain which is constant and localised to the right upper quadrant. She is tachycardic and pyrexial, with a positive Murphy’s sign. A diagnosis of cholecystitis is made. She is treated with antibiotics and is offered a ‘hot’ cholecystectomy.
Concerning this diagnosis and its treatment, which of the following statements is correct?

1) Boas’ sign is pain in the right upper quadrant which radiates to the right iliac fossa as inflammatory fluid tracks down the right pericolic gutter
2) Hartmann’s pouch may be found at the junction of the cystic duct and common hepatic duct
3) The cystic duct is a boundary of the cystohepatic triangle
4) The bile duct lies in the free edge of the greater omentum
5) Ultrasound is more sensitive than computerised tomography (CT) in the diagnosis of cholecystitis

A

Explanation
The cystic duct is a boundary of the cystohepatic triangle
The boundaries of the cystohepatic triangle (Calot’s) are the cystic duct, the common hepatic duct and the inferior surface of the liver. The triangle’s contents include the right hepatic artery, cystic artery, cystic lymph node, connective tissue and lymphatics.
Boas’ sign is pain in the right upper quadrant which radiates to the right iliac fossa as inflammatory fluid tracks down the right pericolic gutter
Boas’ sign is hypersensitivity below the right scapula and can also be caused by phrenic irritation.
Hartmann’s pouch may be found at the junction of the cystic duct and common hepatic duct
Hartmann’s pouch is found at the junction of the gall bladder neck and the cystic duct. It is a pathological variant where a stone may become impacted.
The bile duct lies in the free edge of the greater omentum
The common hepatic duct is anterior to the portal vein, and the bile duct runs in the free edge of the lesser omentum.
Ultrasound is more sensitive than computerised tomography (CT) in the diagnosis of cholecystitis
Although CT scanning has a higher sensitivity and specificity, when compared with ultrasound scanning, it is more expensive and cannot visualise non-calcified gallstones. Additionally, the patient is not exposed to ionising radiation.

How well did you know this?
1
Not at all
2
3
4
5
Perfectly
8
Q

A patient undergoes intramedullary nailing for a mid-shaft fracture of the tibia three hours ago. The ward nurses are concerned as he is in a lot of pain. On examination, he has no neurovascular deficit. He complains of excruciating pain on passive plantar flexion of the big toe, but not on passive dorsiflexion.
What is the likely diagnosis?

1) Compartment syndrome of the deep posterior compartment of the leg
2) Compartment syndrome of the superficial posterior compartment of the leg
3) Compartment syndrome of the anterior compartment of the leg
4) Compartment syndrome of the lateral compartment of the leg
5) Compartment syndrome of the medial compartment of the leg

A

Explanation
Compartment syndrome of the anterior compartment of the leg
In compartment syndrome, pain is worsened by passive stretching (ie extension) of the affected compartment. In this case, the muscle being stretched is the extensor hallucis longus. This muscle is in the anterior compartment which also contains the tibialis anterior, the extensor digitorum longus and the peroneus tertius. This is a surgical emergency, as increasing pressure within the compartment exceeds perfusion pressure, causing hypoxia and ischaemia. It requires urgent fasciotomy, in which all osseofascial compartments are opened.
Compartment syndrome of the deep posterior compartment of the leg
The deep posterior compartment of the leg contains the flexor hallucis longus, along with the flexor digitorum longus and the tibialis posterior.
Compartment syndrome of the superficial posterior compartment of the leg
The superficial posterior compartment contains the gastrocnemius, the plantaris and the soleus.
Compartment syndrome of the lateral compartment of the leg The lateral compartment contains the peroneus longus and brevis. Compartment syndrome of the medial compartment of the leg There is no medial compartment of the leg.

How well did you know this?
1
Not at all
2
3
4
5
Perfectly
9
Q

A 45-year-old man has been sent to the accident and emergency department by his general practitioner who suspects that he is suffering from cauda equina syndrome. There is no history of spinal surgery in the patient.
Which of the following clinical findings supports this diagnosis?

1) Magnetic resonance imaging (MRI) of the spine which reveals a disc prolapse at T11
2) There is a positive Babinski sign
3) The patient’s chief complaint is of a shooting pain radiating down the back of his legs
4) The patient is in urinary retention, with reduced anal tone and bilateral lower limb weakness
5) The patient has purely sensory loss in all of the lumbar dermatomes

A

The patient is in urinary retention, with reduced anal tone and bilateral lower limb weakness As above, the signs of cauda equina syndrome can include urinary retention, reduced anal tone and bilateral lower limb weakness. Following clinical assessment, investigation with an MRI spine would be appropriate and surgical decompression may be required.
Magnetic resonance imaging (MRI) of the spine which reveals a disc prolapse at T11
Cauda equina syndrome affects the cauda equina which is situated below the level of the termination of the cord, at around the L1/2 disc space. As such, the disc prolapse at T11 would cause compression of the spinal cord.
There is a positive Babinski sign
A positive Babinski sign represents an upper motor neurone defect.
The patient’s chief complaint is of a shooting pain radiating down the back of his legs
Below the conus medullaris, the spinal canal contains the cauda equina which branches off the lower end of the spinal cord and contains the nerve roots from L1 to 5 and S1 to 5. These roots from L4 to S4 join in the sacral plexus. Compression to this area results in cauda equina syndrome. Signs include weakness of the muscles of the lower extremities, detrusor weaknesses causing urinary retention and post-void residual incontinence. There may be decreased anal tone and faecal incontinence, saddle anaesthesia, bilateral leg pain and weakness and bilateral absence of ankle reflexes. Pain may be absent.
The patient has purely sensory loss in all of the lumbar dermatomes
Sensory loss can be present in cauda equina syndrome, but primarily in the sacral dermatomes (S4–5) affecting the perianal region.

How well did you know this?
1
Not at all
2
3
4
5
Perfectly
10
Q

A 56-year-old man presents to the Accident and Emergency Department with severe vomiting and chest discomfort. His chest X-ray shows air in the mediastinum.
Which one of the following disease processes may cause this appearance?

1) Aortic rupture
2) Aortic dissection
3) Cardiac tamponade
4) Oesophageal perforation
5) Pericarditis

A

Explanation
Oesophageal perforation
Pneumomediastinum is the presence of air in the mediastinal tissues and can be readily seen on a chest radiograph. It is a hallmark of oesophageal perforation and large airway (trachea or bronchus) injury, and therefore must be taken seriously.
Aortic rupture
Isolated aortic rupture may cause mediastinal widening on the chest radiograph, but will not present as pneumomediastinum. Additionally, a patient with aortic rupture will likely present in extremis due to sudden circulatory collapse.
Aortic dissection
An aortic dissection occurs when there is an intimal tear of the aortic endothelium creating a false lumen. This can cause mediastinal widening on the chest radiograph, however there will be no air within the mediastinum.
Cardiac tamponade
Cardiac tamponade causes compression of the heart due to the accumulation of fluid, usually blood, impairing diastolic filling and so cardiac output. This may cause an enlarged cardiac shadow on the chest radiograph but no air would be visible within the mediastinum.
Pericarditis
Pericarditis is inflammation of the pericardium and has a number of aetiologies but it will not cause pneumomediastinum.

How well did you know this?
1
Not at all
2
3
4
5
Perfectly
11
Q

Acute limb compartment syndrome is characterised by increased pressure within an unyielding osteo-fascial compartment, resulting in local tissue hypoxia. Urgent surgery to decompress with responsible compartment(s) is necessary to reduce longer term morbidity and mortality.
Of the following symptoms, which one is a late sign of compartment syndrome and indicates the poorest prognosis?

1) Anaesthesia
2) Paraesthesia within distribution of sensory nerves
3) Pulses present
4) Severe pain on passive muscle stretch
5) Swollen limb

A

Anaesthesia
Complete anaesthesia is a late sign due to myoneural necrosis and indicates a poor prognosis.
Compartment syndrome is the term used to describe the condition in which the tissue pressure in an enclosed fascial compartment rises above the capillary pressure, so reducing blood flow to the distal tissues. Although direct measurement of compartmental pressures can be made, the condition should be treated on clinical grounds with removal of any occlusive dressings and elevation followed by fasciotomy if needed.
Paraesthesia within distribution of sensory nerves
Paraesthesia is a relatively late sign, however, progression to complete anaesthesia indicates a worse prognosis.
Pulses present
Distal pulses and capillary refill may be present even in the presence of significant increases in compartmental pressure.
Severe pain on passive muscle stretch
The earliest sign is pain out of proportion with the injury, particularly severe pain on passive muscle stretch. Other early signs are pink shiny skin and a feeling of pressure.
Swollen limb
This is an early sign of compartment syndrome and if diagnosis and treatment at this stage are swift then the prognosis would be good.

How well did you know this?
1
Not at all
2
3
4
5
Perfectly
12
Q

A 54-year-old man presents as an emergency to casualty following a crush injury to his left femur sustained on a building site. On examination a diagnosis of compartment syndrome is suspected.
Which one of the following early signs might be expected on examination of this man’s left lower limb to support this presumptive diagnosis?

1) Absent dorsalispedis pulse
2) Motor loss before sensory loss
3) Severe pain on passive stretch of the affected group of muscles
4) Greatly prolonged capillary refill
5) Blue or grey extremities

A

Explanation
Severe pain on passive stretch of the affected group of muscles
Severe pain in response to passive stretch of the ischaemic muscles is by far the most dramatic and reliable clinical sign.
Compartment syndrome is defined as an increase in the interstitial fluid pressure within an osteofascial compartment of sufficient magnitude to cause microcirculatory compromise and later myoneural necrosis. The limb becomes tense and swollen, and if not treated, the muscle weakness progresses to paralysis. Alternatively, areas of muscle may infarct, giving rise to rhabdomyolysis, hyperkalaemia, hyperphosphataemia, high uric acid levels and metabolic acidosis. It is a devastating early complication seen after long-bone fractures and crush injuries. It can also be caused by deep thermal burns, electrical injuries, restricting tourniquets and fluid extravasation (eg caused by iv regional anaesthesia).
Classically, compartment pressures are measured using a slit-catheter device. The normal resting pressure within the compartment tissues is approximately 3–4 mmHg. Compartment pressures in excess of 30–35 mmHg in a normally perfused patient have previously been taken to indicate the need for open-compartment fasciotomy. Recent evidence, however, suggests that fasciotomy should be undertaken if the difference between the diastolic pressure and the measured compartment pressure is < 30 mmHg.
Absent dorsalispedis pulse
Early in its development, the peripheral pulses are normal, as are fingertip/toe colour, temperature and capillary refill, as it is the microvasculature that is initially affected. Loss of peripheral pulses is usually a late sign and the diagnosis should be made before this progression of signs.
Motor loss before sensory loss
Thin cutaneous nerve fibres are more susceptible to ischaemia than the motor fibres, and distal paraesthesias occurs before motor loss.
Greatly prolonged capillary refill

Capillary refill may be normal in the early stages, so waiting until capillary refill time is prolonged is not appropriate as this will lead to a poor outcome for the patient. As such, prolonged capillary refill should not be used to support the presumptive diagnosis as ideally treatment would have taken place before this sign developing.
Blue or grey extremities
This would indicate significant disruption to the vasculature of the limb and ideally the diagnosis and subsequent treatment should have occurred before colour changes in the affected limb.

How well did you know this?
1
Not at all
2
3
4
5
Perfectly
13
Q

You are completing your paediatric surgical rotation and discussing with your consultant the indications for urgent surgery in an infant. Certain presentations must be dealt with swiftly to reduce future morbidity and mortality.
For which one of the following would a 4-month-old infant need urgent treatment?

1) A 6 cm strawberry naevus over the sacrum
2) Bat ears
3) Bilateral hydroceles
4) Metatarsus varus
5) Redcurrant coloured stool

A

Explanation
Redcurrant coloured stool
Redcurrant coloured stool is a late presentation suggesting a severe intussusception, and urgent treatment is required with a surgical opinion as soon as possible. Intussusception occurs when one section of the intestines invaginates another, the most common form is ileocolic. Surgical treatment is aimed at reducing the intussusception manually, usually an air enema or, if severe, resection of the affected bowel.
A 6 cm strawberry naevus over the sacrum
Strawberry naevus only requires treatment if the lesion is impairing sight, as failure to develop stereoscopic vision is a possibility. As this naevus is located on the sacrum, no treatment is required.
Bat ears
This is primarily a cosmetic concern for parents and is not an indication for urgent surgery.
Bilateral hydroceles
Bilateral hydroceles do not require urgent treatment as usually they resolve spontaneously.
Metatarsus varus
Metatarsus varus may correct with physiotherapy in due course and is not urgent.

How well did you know this?
1
Not at all
2
3
4
5
Perfectly
14
Q

You admit an elderly man through the Emergency Department with a 10-day history of abdominal distension, crampy abdominal pain and absolute constipation for the past 24 h. He has also been vomiting large amounts of faeculant material.
Which one of the following is the most common cause of colonic obstruction?

1) Endometriosis
2) Gallstone ileus
3) Irritable bowel syndrome
4) Sigmoid volvulus
5) Solitary rectal ulcer

A

Explanation
Sigmoid volvulus
The most common causes of colonic obstruction include carcinoma of the colon (approximately 65% incidence), diverticulitis (10% incidence), volvulus (approximately 5% incidence), others including pseudo-obstruction, radiational and inflammatory stricture, external hernias and ischaemic hernias approximately 20% incidence. A sigmoid volvulus may be resolved simply by passing a flatus tube rectally, if unsuccessful the patient will require surgery and manual reduction or resection of the volvulus.
Endometriosis
Endometriosis is a condition in which functioning endometrial tissue is found outside of the uterus, most commonly in the abdominal or pelvic cavity. If endometrial tissue is found on the bowel wall it can cause a stenosis and subsequent obstruction, however it is uncommon.
Gallstone ileus
Gallstone ileus is a rare form of bowel obstruction and occurs when a gallstone becomes lodged in the terminal ileum. It is a rare, but recognised complication of endoscopic retrograde cholangio-pancreatography (ERCP).
Irritable bowel syndrome
Irritable bowel syndrome does not cause bowel obstruction but may cause functional constipation.
Solitary rectal ulcer
Solitary rectal ulcers can cause obstruction if severe due to pain, however it is uncommon.

How well did you know this?
1
Not at all
2
3
4
5
Perfectly
15
Q

A 45-year-old man presents to the Acute Surgical Admissions Unit with a 1-day history of worsening epigastric pain, radiating through to the back, and vomiting. He is not able to find any comfortable position and analgesia is not helpful. Clinical examination reveals generalised abdominal guarding and rigidity.
What is the most definitive investigation to confirm diagnosis?

1) Amylase of 450 international units is suggestive of acute pancreatitis
2) Computed tomography (CT) scan of the abdomen
3) Ultrasound scan
4) Normal serum amylase excludes acute pancreatitis
5) The modified Glasgow score

A

Explanation
Computed tomography (CT) scan of the abdomen
In this context, a CT scan of the abdomen is the best investigative modality in confirming acute pancreatitis. It is also useful when the diagnosis is not clear and other conditions such as perforation and peritonitis are being considered.
Amylase of 450 international units is suggestive of acute pancreatitis
Amylase is often used to confirm the diagnosis of acute pancreatitis in conjunction with clinical symptoms and signs. However, the use of arbitrary values (eg >1000 or 3–4 times the upper limit of normal) are dependent upon the half-life of amylase. When available, plasma lipase has more accuracy than amylase. Lipase is produced by the pancreas alone and persists for longer than amylase. It is therefore more sensitive and specific in the diagnosis of acute pancreatitis.
Ultrasound scan
The pancreas is poorly visualised by ultrasonography but is useful to assess the aetiology (eg gallstones) or to detect other pathologies (eg abdominal aortic aneurysm). Normal serum amylase excludes acute pancreatitis
Amylase level gradually returns to normal over 3–4 days and delay in performing the test can lead to a false-negative result. A raised amylase can also occur in upper gastrointestinal (GI) perforation, mesenteric infarction, small bowel obstruction, tubo- ovarian disease, renal failure, or macroamylasaemia.
The modified Glasgow score
This is clinical severity and prognosis score, is not an investigation for acute pancreatitis.

How well did you know this?
1
Not at all
2
3
4
5
Perfectly
16
Q

A 55-year-old man with severe epigastric pain radiating through to the back presents with the following results:

Amylase (urine test): 500 IU, Total bilirubin: 50 mmol/l, Total bilirubin: 50 mmol/l, Alkaline phosphatase: 250 mmol/l, Aspartate transaminase: 50 mmol/l, Alanine transaminase: 95 mmol/l

Severity can be predicted with which one of the following?

1) APACHE I score
2) C-reactive protein
3) Glasgow Score
4) White-cell count
5) Serum amylase

A

Explanation
Glasgow Score
The Glasgow score can be used to predict severity and takes into consideration a number of parameters including; age, white-cell count, p (O ), serum glucose, calcium, albumin, LDH and urea. The British Society of Gastroenterology recommends that the Glasgow score be used to predict severity within 48 h following admission. It is only applicable when the presumed cause is either alcohol or gallstones. Computed tomography (CT) scan can also be used to assess prognosis by grading the extent of pancreatic necrosis (Balthazar score). Other CT features such as necrosis of the head of pancreas, mesenteric oedema and intraperitoneal fluid are also associated with a poorer outcome.
APACHE I score
The APACHE II score, not APACHE I, is a general severity of disease tool and mortality predictor used for the acutely unwell patient admitted to intensive care. It is not specific to pancreatitis. An APACHE II score > 8 could be used to be predictive of severe acute pancreatitis. It takes into consideration the physiological state of the patient, including oxygenation, haemodynamic stability, biochemical abnormalities, and neurological status.

C-reactive protein
C-reactive protein (CRP), an acute phase reactant, can be used in isolation as a predictor of severity. A CRP > 150 mg/l more than 48 h after the onset of symptoms is predictive of severe acute pancreatitis, however, there are more reliable indicators and scoring systems as CRP can be raised with any inflammatory process.
White-cell count
White-cell count is one of the parameters included in the Glasgow score, which can be used to predict severity, however in isolation it is not useful.
Serum amylase
Levels of serum amylase and lipase are not predictive of severity.

How well did you know this?
1
Not at all
2
3
4
5
Perfectly
17
Q

A 6-year-old boy is brought to the Accident and Emergency department with a 12-h history of vomiting, severe abdominal pain and being generally unwell. His parents say that he also had two episodes of convulsions during this period. On examination, he appears pale and dehydrated. The abdomen is rigid and tender, and his pain is worse over the right iliac fossa. His temperature is 40.2°C, blood pressure 82/64 mmHg, and pulse rate 172 beats/min. There is no discoloration over the anterior abdominal wall. Bowel sounds are absent.
From the options below choose the one that you think is the most likely pathological process in this child.

1) Necrotising enterocolitis
2) Uncomplicated acute appendicitis
3) Volvulus neonatorum
4) Meckel’s diverticulitis
5) Bacterial Peritonitis

A

Explanation
Bacterial Peritonitis
The signs and symptoms in this child are suggestive of spreading/established infection in the peritoneal cavity. Bacterial peritonitis in children may occur as a result of a ruptured viscus such as ruptured appendicitis or ruptured Meckel’s diverticulitis, or as a complication of abdominal surgery. The child may present with classical signs of peritonitis such as abdominal pain, pyrexia, nausea, vomiting, tachycardia, low blood pressure and decreased urine output. High pyrexia may result in febrile convulsions. Abdominal examination may reveal a board-like rigidity, guarding and rebound tenderness. Bowel sounds are absent if the peritonitis becomes established. Plain abdominal X-rays should be performed in both supine and upright positions to identify the presence of free gas beneath the diaphragm, which suggests perforation of a viscus. The common organisms responsible for bacterial peritonitis in children
include Escherichia coli, Klebsiella pneumoniae and Pseudomonas species.
Necrotising enterocolitis
This condition is primarily seen in premature neonates in which the bowel necroses and is a common cause of morbidity. As the child described in the case history is 6 years old, this answer is unlikely.
Uncomplicated acute appendicitis
Acute appendicitis would result in many of the signs and symptoms described in the case history, however, given the child is in extremis, it would suggest bacterial peritonitis. A ruptured appendix as a sequelae of appendicitis would cause bacterial peritonitis but this answer does not specify that the appendicitis has progressed to rupture.
Volvulus neonatorum This is a condition seen in newborns and would present in the neonatal period, therefore this child is too old to present with volvulus neonatorum. Additionally, the high grade fever, febrile convulsions and cardiac instability are suggestive of an extensive infective process which would not be seen with volvulus neonatorum.
Meckel’s diverticulitis
Merkel’s diverticulum is a congenital diverticulum most commonly found in the distal ileum, if bowel contents become trapped then diverticulitis can occur producing infective symptoms. As above however, only if there was perforation of a viscus would the severity of symptoms described in the case history be seen.

How well did you know this?
1
Not at all
2
3
4
5
Perfectly
18
Q

A 54-year-old man presents to the Accident and Emergency department with a 6 h history of gradual onset severe epigastric and central abdominal pain radiating through to his back. The pain reduces when he sits forward. He has also had three episodes of vomiting, mostly bilious.
He admits to drinking up to 40–50 units of alcohol per week. He has experienced similar episodes in the past but less severe. On examination, his pulse rate is 94 beats/min and respiratory rate is 18 breaths/min. Abdominal examination reveals that he is very tender over the epigastric region with moderate degree of guarding. Plain radiographs of the chest (erect) and abdomen (supine) are unremarkable.
From the options below choose the one that you think is the most likely diagnosis in this patient.

1) Intestinal obstruction
2) Mesenteric ischaemia
3) Acute pancreatitis
4) Perforated peptic ulcer
5) Ruptured abdominal aortic aneurysm

A

Explanation
Acute pancreatitis
The signs and symptoms in this patient are very suggestive of acute pancreatitis, with the most common causes being alcohol and gallstones. Pancreatitis is thought to result from early activation of pancreatic enzymes, producing auto-digestion of the pancreas and surrounding tissues. The severity of acute pancreatitis is validated using various prognostic scoring systems. Currently in the UK, the Glasgow–Imrie (modified Glasgow score) scoring system is widely used for assessing the severity, while the APACHE II is useful predicating the prognosis in acute pancreatitis. Serum C-reactive protein concentration, although not part of the Glasgow criteria, has an independent prognostic value if the peak level is >210 mg/litre in the first 4 days of the attack. Serum amylase is a useful indicator to diagnose acute pancreatitis; a diagnosis of acute pancreatitis is likely if the level is three times the upper limit of normal although this may vary between laboratories depending on the hospital policy/guidelines. An ultrasound of the abdomen is indicated in all patients with acute pancreatitis to determine the presence/absence of biliary calculi. A computed tomography (CT) scan of the abdomen should be performed on all patients with severe acute pancreatitis between the third and tenth days following the onset of symptoms to rule out pancreatic necrosis, in addition to use of the Balthazar score, which examines the radiographic features of acute pancreatitis.
Intestinal obstruction
Intestinal obstruction would more commonly present with generalised abdominal pain and distension, vomiting and absolute constipation. If intestinal obstruction was the causative pathology in this case history abnormalities would likely be seen on the abdominal X-ray, such as dilated loops of bowel.
Mesenteric ischaemia Intermittent, colicky, generalised abdominal pain, worse in the post prandial period, is the most common presentation of mesenteric ischaemia. A history of atrial fibrillation is a common co-morbidity. Furthermore, mesenteric ischaemia usually has a chronic, progressive history with the avoidance of food (due to pain) and weight loss.
Perforated peptic ulcer
This presentation could also be consistent with a perforated abdominal viscus. However, it is slightly less likely than pancreatitis in this scenario given that chest radiograph excluded free air. This patient has an unremarkable chest radiograph. It should be noted that air is not always seen under the diaphragm in hollow viscus rupture, but given that this patient has also had previous less severe episodes and drinks alcohol heavily, pancreatitis is more likely. It is also more likely given the gradual onset – patients with a perforated viscus often describe sudden onset abdominal pain. Guarding itself may indicate peritoneal irritation.
Ruptured abdominal aortic aneurysm
A ruptured abdominal aortic aneurysm is likely to result in a haemodynamically unstable patient due to circulatory collapse. Given that neither the patient’s heart rate or respiratory rate are raised, this diagnosis is less likely. Furthermore, the history is often of sudden onset pain, not of a gradual course over 6 h.

How well did you know this?
1
Not at all
2
3
4
5
Perfectly
19
Q

A 47-year-old barmaid presents to the Accident and Emergency department with a 12 h history of right upper quadrant pain and vomiting. She says that the pain is radiating to her right scapula and is exacerbated by breathing. She appears pale but not jaundiced. On examination, her pulse rate is 98 beats/min, blood pressure is 126/84 mmHg and temperature is 37.6°C. Abdominal examination reveals tenderness over the right hypochondrium but no mass is palpable. Plain radiographs of the abdomen (supine) and chest (erect) are unremarkable.
From the options below choose the one that you think is the most likely diagnosis in this patient.

1) Perforated peptic ulcer
2) Acute pancreatitis
3) Acute biliary cholangitis
4) Acute cholecystitis
5) Infective hepatitis

A

Explanation
Acute cholecystitis
The history, signs and symptoms in this patient are suggestive of acute cholecystitis. Acute cholecystitis is more common in women over the age of 40 and with high body mass index (BMI). Gallstones are the commonest cause for acute cholecystitis. Obstruction of the common bile duct due to stones leads to accumulation of bile and inflammation, resulting in an acutely inflamed gall bladder. Other risk factors for acute cholecystitis include alcohol abuse and tumours of the gall bladder. The signs and symptoms of acute cholecystitis include: severe right hypochondrial pain exacerbated by respiration, nausea and vomiting, and increase in temperature. The rise in temperature is frequently mild to moderate; a very high temperature with or without chills and rigours may point to a diagnosis of acute cholangitis. A tender, inflamed gall bladder may be palpable in some patients. Likewise, jaundice may or may not be present.
Perforated peptic ulcer
A perforated abdominal viscus would be likely to be identified on an erect chest radiograph with the presence of free air under the diaphragm, and cause more centralised pain. This patient has an unremarkable chest radiograph.
Acute pancreatitis
Pancreatitis more commonly presents with epigastric pain radiating through to the back.
Acute biliary cholangitis
Ascending infection of the biliary tree and ducts requires urgent treatment, but it generally presents with high grade fever, rigours and jaundice. Charcot’s triad of jaundice, fever (often with rigours) and right upper quadrant pain is often used to diagnose cholangitis.
Infective hepatitis Hepatitis broadly refers to the inflammation of the liver, and infective causes most commonly include the hepatitis viruses (A, B, C, D, E). Infective hepatitis is not often an acute presentation as described in the case history.

How well did you know this?
1
Not at all
2
3
4
5
Perfectly
20
Q

A 51-year-old woman presents to the Surgical Emergency Assessment Unit with a 12 h history of central colicky abdominal pain and vomiting. She has undergone a subtotal colectomy and formation of an end ileostomy for ulcerative colitis 7 years ago. Her ileostomy has not functioned for 2 days. On examination, she is tender over the upper abdomen and the abdomen is mildly distended. Plain abdominal radiograph reveals a number of small loops in the centre of the abdomen.
From the options below choose the one that you think is the most likely diagnosis in this patient.

1) Acute colonic pseudo-obstruction
2) Incarcerated incisional hernia
3) Bacterial peritonitis
4) Adhesional small bowel obstruction
5) Sigmoid volvulus

A

Explanation
Adhesional small bowel obstruction
The cardinal features of small bowel obstruction are pain, vomiting and abdominal distension; untreated, this leads to constipation with reduction in flatus which then becomes absolute. The pain is usually colicky due to excessive peristalsis, but may become continuous if strangulation or perforation occurs. Vomiting is early in high small bowel obstruction, late in low small bowel obstruction and delayed or absent in large bowel obstruction. The management involves appropriate resuscitation of the patient and surgical exploration of the abdomen to relieve the obstruction. Small bowel obstructions make up the majority of intestinal obstructions. Of these, adhesions following laparotomy and/or surgery to the bowel is the leading cause of small bowel obstruction. It can occur as a sequelae of ‘minor’ abdominal surgeries such as appendicectomies or ‘major’ surgeries such as resection of large sections of the bowel. In women, gynaecological procedures are an important cause. In addition, pelvic inflammatory disease can also lead to adhesions even in the absence of a surgical intervention in the abdomen.
Acute colonic pseudo-obstruction
This is obstruction and massive dilation of the colon in the absence of a mechanical blockage. As this patient has undergone a subtotal colectomy, this is not a possibility.
Incarcerated incisional hernia
Obstruction in a patient with known hernia can occur when a section of bowel becomes incarcerated within a hernial orifice. Abdominal examination would reveal localised tenderness over the incisional hernia, and there is not suggestion of a hernia in the case history.
Bacterial peritonitis
Bacterial peritonitis can occur as a result of a perforated abdominal viscus with infection of the peritoneal cavity. The patient may present with classical signs of peritonitis such as abdominal pain, pyrexia, nausea, vomiting, tachycardia, low blood pressure and decreased urine output. Abdominal examination may reveal a board-like rigidity, guarding and rebound tenderness.
Sigmoid volvulus This patient has previously undergone a subtotal colectomy with formation of an end ileostomy in which the sigmoid colon would have been resected.

How well did you know this?
1
Not at all
2
3
4
5
Perfectly
21
Q

A 75-year-old woman is admitted with small bowel obstruction and
pain radiating down the medial aspect of the right thigh to the knee. There is no palpable abnormality in the groin but the inner aspect of the groin is tender.
Which one of the following is the most likely diagnosis?

1) Femoral hernia
2) Gluteal hernia
3) Lumbar hernia
4) Obturator hernia
5) Sciatic hernia

A

Explanation
Obturator hernia
An obturator hernia is six times more common in women and twice as common on the right side. It particularly affects elderly women who have had recent rapid weight loss. The hernial sac protrudes through the obturator canal potentially compressing the geniculate branch of the obturator nerve causing referred pain.
Femoral hernia
Strangulated femoral hernias are generally palpable within the groin, lateral and inferior to the pubic tubercle.
Gluteal hernia
Gluteal hernias are very rare and are suggested by the presence of a painful swelling in the buttock or pain.
Lumbar hernia
Lumbar hernias are associated with paralysed muscles especially by poliomyelitis or spina bifida.
Sciatic hernia
Again, sciatic hernias are very rare but would be suggested by pain in the distribution of the sciatic nerve.

How well did you know this?
1
Not at all
2
3
4
5
Perfectly
22
Q

As a CT2 surgical trainee you are asked to perform a tracheostomy on a 52-year-old man currently on the Intensive Care Unit (ITU). The supervising consultant asks you where on the trachea would you normally perform the tracheostomy.
Selecting from the following, how would you answer?

1) Cricoid cartilage
2) Crycothyroid membrane
3) First tracheal cartilage
4) Second tracheal cartilage
5) Thyroid isthmus

A

In a typical surgical tracheostomy, the preferred site is usually between the second and third tracheal cartilages. Based on the given options, the most appropriate answer would be:

4) Second tracheal cartilage

Explanation:

•	Cricoid cartilage (1): A tracheostomy is not performed at this level because it is part of the larynx and performing the procedure here can damage the larynx.
•	Cricothyroid membrane (2): This is the site for an emergency cricothyrotomy, not a tracheostomy. A cricothyrotomy is usually performed in acute situations where airway access is needed immediately.
•	First tracheal cartilage (3): This is too high and close to the cricoid cartilage, posing a risk of laryngeal damage.
•	Second tracheal cartilage (4): This is the most common site for a surgical tracheostomy, as it allows for secure placement of the tracheostomy tube while minimizing risks to the laryngeal structures.
•	Thyroid isthmus (5): The thyroid isthmus often lies over the second and third tracheal rings, and while the tracheostomy may involve moving or dividing the isthmus, the actual tracheal opening is made in the second or third tracheal cartilage.

Conclusion:

The most appropriate site for performing a tracheostomy is at the level of the second tracheal cartilage.

How well did you know this?
1
Not at all
2
3
4
5
Perfectly
23
Q

A 56-year-old woman with known metastatic breast cancer presents to the Emergency Department with a calcium concentration of 3.22 mmol/l (normal corrected Ca2+ 2.20– 2.60 mmol/l).
Which one of the following is the most appropriate initial management?

1) Intravenous hydrocortisone
2) Intravenous infusion of 0.9% sodium chloride
3) Intravenous infusion of sodium phosphate
4) Oral bisphosphonate
5) Oral thiazide diuretic

A

Explanation
Intravenous infusion of 0.9% sodium chloride
The priority in symptomatic hypercalcaemia is to rehydrate the patient and establish diuresis; 0.9% sodium chloride is the fluid of choice, and patients often require around 4–6 litres over a 24 h period.
Intravenous hydrocortisone
Corticosteroids may occasionally be helpful but are not first-line treatment. Intravenous infusion of sodium phosphate
Sodium phosphate infusion is dangerous: it lowers the calcium concentration rapidly, but risks causing metastatic calcification.
Oral bisphosphonate
Bisphosphonates are effective, but must be given intravenously to have a rapid effect. Oral thiazide diuretic
Diuretics are only helpful for management of fluid overload in patients receiving rehydration therapy, and so copious volumes of intravenous fluids. However, they are not useful for reducing serum calcium levels. Loop diuretics would be the diuretic of choice, not thiazide diuretics.

How well did you know this?
1
Not at all
2
3
4
5
Perfectly
24
Q

A 71-year-old man is taken to theatre as an emergency following perforation of his colon resulting in generalised peritonitis. A subtotal colectomy and end ileostomy is performed.
What is the most common cause of peritonitis?

1) Acute appendicitis
2) Acute cholecystitis
3) Perforated peptic ulcer
4) Post-operative complications
5) Secondary to an initial infection Explanation

A

Explanation
Post-operative complications
The most common cause of peritonitis is post-operative complications, accounting for approximately 30% of cases. Peritonitis is inflammation of the peritoneum that may be localised (peritonism) or generalised. It is classified according to the causative agent, examples including bacterial, chemical or biliary.
Acute appendicitis
Acute appendicitis accounts for around 20% of cases.
Acute cholecystitis
Acute cholecystitis rarely leads to biliary peritonitis because the inflamed gall-bladder rarely becomes gangrenous or perforates.
Perforated peptic ulcer
Perforated peptic ulcers account for around 20% of cases of peritonitis. Secondary to an initial infection
If an infection is severe causing a bacteraemia then peritonitis can occur as a consequence of the initial infection, however this is uncommon.

How well did you know this?
1
Not at all
2
3
4
5
Perfectly
25
Q

An 81-year-old woman is admitted with bowel obstruction, pain and dehydration. An abdominal radiograph is performed by the admitting doctor with features suggesting volvulus of the large bowel.
What is the commonest form of volvulus in the elderly?

1) Caecal
2) Sigmoid
3) Small bowel
4) Stomach
5) Transverse colon

A

Explanation
Sigmoid
Volvulus occurs when a segment of bowel twists on its mesentery. Sigmoid volvulus is the commonest form of volvulus in the elderly and occurs in an anticlockwise direction. It is seen in approximately 5% of cases of large bowel obstruction in developed countries. The characteristic radiological appearance is known as a ‘coffee bean’ which arises from the left iliac fossa. The volvulus may be resolved by passing a flatus tube and, if unsuccessful, more aggressive surgical management.
Caecal
Caecal volvulus is less common than volvulus of the sigmoid colon.
Small bowel
A small bowel volvulus is rare; more commonly obstruction of the small bowel is due to adhesions from previous surgery.
Stomach
Gastric volvulus is uncommon and can occur at any age but a higher incidence is seen in the elderly.
Transverse colon
Again, this is an uncommon form of volvulus.

How well did you know this?
1
Not at all
2
3
4
5
Perfectly
26
Q

A 55-year-old obese man with known diabetes is admitted to an acute medical ward with cellulitis of the lower limb. Over the next few hours he becomes unwell and complains of increased pain. His cellulitis spreads, he becomes
haemodynamically unstable and he requires inotropic support to maintain his blood pressure (BP). You do an arterial blood gas.
Which one of the following would you most expect to find?

1) Metabolic acidosis
2) Metabolic alkalosis
3) Mixed metabolic alkalosis with respiratory compromise
4) Respiratory acidosis
5) Respiratory alkalosis

A

Explanation
Metabolic acidosis
Metabolic acidosis can be due to impaired hydrogen excretion, increased hydrogen ion production or ingestion, or loss of bicarbonate. In this patient, the rapidly spreading cellulitis coupled with significant cardiovascular compromise could indicate necrotising fasciitis and should be suspected. The acidosis is likely due to a diabetic ketoacidosis; diabetes is also a risk factor for necrotising fasciitis. Other causes of metabolic acidosis include renal disease, lactic acidosis, salicylate poisoning and chronic diarrhoea and intestinal fistulas.
Metabolic alkalosis
Given the severe infection and diabetes a metabolic acidosis, not alkalosis, is likely. Mixed metabolic alkalosis with respiratory compromise
Metabolic alkalosis with respiratory compensation would be unlikely. Respiratory compensation would be through hypoventilation, and given this patient is haemodynamically unstable and septic it is likely he would be tachypneic. Additionally, sepsis causes a metabolic acidosis due to anaerobic respiration.
Respiratory acidosis
A respiratory acidosis would occur in the presence of hypoventilation. This patient is likely to have a high respiratory rate due to sepsis.
Respiratory alkalosis
This occurs during hyperventilation as more carbon dioxide is ‘blown off’ causing a reduction in hydrogen ions. Carbonic anhydrase in erythrocytes facilitates a reaction between carbon dioxide and water that results in hydrogen ions. As such, a reduction in carbon dioxide, through hyperventilation, will reduce the number of hydrogen ions and so acidity. The primary issues for this patient however is a metabolic acidosis due to sepsis.

How well did you know this?
1
Not at all
2
3
4
5
Perfectly
27
Q

A patient awaiting surgery for a parathyroid adenoma is admitted with confusion and drowsiness. The diagnosis of hypercalcaemic crisis is made.
What should be the initial treatment?

1) Intravenous magnesium
2) Intravenous fluids
3) Steroids
4) Calcitonin
5) Intravenous phosphate

A

Explanation
Intravenous fluids
Hypercalcaemic crisis occurs when a patient with mild hypercalcaemia has a rapid rise in plasma calcium. This is associated with rapid deterioration in the patient’s condition, in the form of confusion, drowsiness, vomiting and dehydration. The first-line treatment of hypercalcaemia is intravenous fluid replacement with 0.9% saline. Crises may be precipitated by intercurrent illness, operation or infection.
Intravenous magnesium
This has no role in the treatment of a hypercalcaemic crisis.
Steroids
Hypercalcaemic crises due to hyperparathyroidism do not respond to treatment with steroids. Even in hypercalcaemia of different aetiologies, corticosteroids are considered second-line treatment.
Calcitonin
The condition may be controlled pre-operatively by the careful use of calcitonin or mithramycin but these would not be initial treatment.
Intravenous phosphate
Sodium phosphate infusion is dangerous; it lowers the calcium concentration rapidly, but risks causing metastatic calcification.

How well did you know this?
1
Not at all
2
3
4
5
Perfectly
28
Q

A patient is admitted with diverticulitis. On assessment they are found to have a high fever, are tachycardic and have localised peritonism in the left iliac fossa.
Which one of the following statements fulfils the criteria for the correct definition of sepsis?

1) The presence of bacteria in the bloodstream
2) The presence of micro-organisms within a normally sterile viscus
3) Hypotension refractory to resuscitation in the presence of demonstrable infection
4) Sepsis is a life-threatening organ dysfunction due to a dysregulated host response to infection.
5) Low blood pressure, signs of fluid overload and the presence of a proven source of infection.

A

Explanation
Sepsis is a life-threatening organ dysfunction due to a dysregulated host response to infection.
Sepsis is a life-threatening condition associated with significant morbidity and mortality, yet with early recognition and management these can be improved greatly. As such, in recent years there has been a concerted effort to improve the recognition and management of sepsis which includes increasing public awareness and producing a clear definition of sepsis. Recent National Institute of Clinical Excellence (NICE) Guidance was released in 2016 in response to a UK Parliamentary inquiry examining sepsis. The ‘Time to Act’ report found failures in the recognition, diagnosis, and early management of those who died from sepsis. Part of this guidance was clarifying the definition of sepsis as previously there was a confusing mix of terminology being used to describe the disease process, including; septicaemia, sepsis, septic shock and systemic inflammatory response syndrome (SIRS). Efforts are now being made to improve the recognition, diagnosis and management of sepsis throughout the UK.
The presence of bacteria in the bloodstream
This simply describes a bacteraemia that may or may not be related to sepsis depending if there is organ dysfunction or a dysregulated host response to infection.
The presence of micro-organisms within a normally sterile viscus
Sepsis is a systemic dysregulated response to infection and is not limited to a viscus. Hypotension refractory to resuscitation in the presence of demonstrable infection
In the past this would have been referred to as septic shock. A universal definition of sepsis is now in use and terminology such as septic shock should be avoided and replaced simply with sepsis.
Low blood pressure, signs of fluid overload and the presence of a proven source of infection. This is not the universally agreed definition of sepsis.

How well did you know this?
1
Not at all
2
3
4
5
Perfectly
29
Q

A 69-year-old immunosuppressed man presents to the medical ward with cellulitis of the leg. Over the next 24 h he becomes systemically unwell. He is anuric and develops acute renal failure. His systolic blood pressure does not respond to fluids and he requires noradrenaline support. He has radiological signs of acute respiratory distress syndrome (ARDS) on his chest X-ray.
Which one of the following is true of multi-organ dysfunction syndrome?

1) Jaundice is usually due to gallstones
2) Mortality rates are low
3) Sepsis is a well recognised cause
4) There is a decreasing alveolar–arterial oxygen gradient
5) There is increased lung compliance

A

Explanation
Sepsis is a well recognised cause
There are several causes of multi-organ dysfunction syndrome, however infections (resulting in sepsis) and injuries (either trauma or surgery) are the most common causes. If sepsis is left untreated, or unrecognised, then this can progress to septic shock and multi-organ dysfunction syndrome.
Jaundice is usually due to gallstones
Intrahepatic cholestasis and hepatocyte necrosis is the usual cause of jaundice in these patients.
Mortality rates are low
Multi-organ dysfunction syndrome unsurprisingly is associated with a high mortality rate, approximately 30–100%.
There is a decreasing alveolar–arterial oxygen gradient
A decreasing alveolar–arterial (A–a) gradient implies there is a reduced diffusion barrier to Ocrossing from the alveolus to 2 blood and would enhance gas transfer. In ARDS the A–a gradient increases due to alveolar–capillary membrane thickening caused by endothelial leakage and accumulation of interstitial fluid.
There is increased lung compliance
Increased lung compliance means the lung is more distensible so that a smaller fall of intrapleural pressure would be required for a given tidal volume during inspiration and respiratory work would decrease (in emphysema this is a problem as expiratory recoil is reduced). In ARDS lung compliance is reduced. In multiple organ dysfunction syndrome there is an uncontrolled inflammatory response and mal-distribution of blood flow, as a result of microembolic phenomenon, excessive release of various vasoactive compounds (vasoconstrictors, vasodilators and those that increase capillary permeability) which leads to interstitial oedema and a mismatch between supply and demand for oxygen within the microcirculation. Other hypotheses include inappropriate and uncontrolled release of cytotoxic mediators from bacteria.

How well did you know this?
1
Not at all
2
3
4
5
Perfectly
30
Q

A 40-year-old woman with acute onset abdominal pain and fever is found to be tachycardic and hypotensive with localised guarding and tenderness in the epigastric region. Despite initiating therapy for her pancreatitis, she develops refractory hypoxaemia secondary to acute respiratory distress syndrome (ARDS).
Which one of the following is the most appropriate initial management considering her current clinical state?

1) Close monitoring within a level 1 care facility
2) Computed tomography (CT) scan
3) Dopamine therapy to support renal function
4) Early endoscopic retrograde cholangiopancreatography (ERCP)
5) Initiate nursing in a prone position

A

Explanation
Initiate nursing in a prone position
Nursing in a prone or seated position has been shown to improve refractory
hypoxaemia caused by ARDS.
Close monitoring within a level 1 care facility
Patients should be managed in a level 2 or level 3 facility, which refers to an acute district hospital (level 2) or level 3 which is a tertiary referral centre (usually a teaching hospital). A level 1 care facility refers to a community hospital.
Computed tomography (CT) scan
This patient is critically unwell and a CT scan is unlikely to change current management; she primary requires treatment for ARDS and pancreatitis. Furthermore, transfer to the radiology department carries inherent risk for limited benefit.
Dopamine therapy to support renal function
Dopamine is not recommended as it has been shown to cause arrhythmias, so should be avoided.
Early endoscopic retrograde cholangiopancreatography (ERCP)
This patient is critically unwell and in their current condition ERCP is extremely risky. Early ERCP is only recommended for patients with co-existing cholangitis or biliary obstruction.

How well did you know this?
1
Not at all
2
3
4
5
Perfectly
31
Q

A 19-year-old girl is admitted to the Emergency Department with a severe anaphylaxisreaction following a bee sting. She becomes severely hypotensive and her airway may be threatened.
Which one of the following management options would you be most appropriate?

1) Intramuscular adrenaline
2) Nebulised bronchocdilators
3) Beta blockers
4) Intravenous nitrates
5) Intravenous morphine

A

Explanation
Intramuscular adrenaline
The principles of management of severe anaphylactic shock include: airway maintenance and oxygen; intravenous fluids to provide circulatory support; and subcutaneous or intramuscular adrenaline (adrenaline). The dose for an adult is 0.5 mg adrenaline, repeated after 5 min as required. Subsequent treatment is with an anti- histamine (chlorphenamine 4 mg) and corticosteroid (hydrocortisone 100 mg), however, adrenaline should be given first as this is the life-saving treatment.
Nebulised bronchocdilators
Nebulised bronchodilators can help counteract the bronchospasm, however, intramuscular adrenaline is the primary treatment.
Beta blockers
Beta blockers are contraindicated as they lower blood pressure and may cause bronchoconstriction, so have no role in the treatment of anaphylaxis.
Intravenous nitrates
Intravenous nitrates cause vasodilation and have no role in the treatment of anaphylaxis.
Intravenous morphine
Intravenous is primarily reserved for severe pain management, therefore would not be indicated in the treatment of anaphylaxis.

How well did you know this?
1
Not at all
2
3
4
5
Perfectly
32
Q

A 25-year-old woman exhibiting features of septic shock is brought to your
attention. She has been admitted with acute appendicitis, unfortunately there has been a delay with CEPOD and she has deteriorated.
Which one of the following is correct?

1) Tissue uptake of oxygen by cells is normal
2) Peripheral vasoconstriction is seen
3) Blood cultures are commonly positive
4) Endotoxins act as a negative inotrope
5) Gram-negative organisms are the most common cause

A

Explanation
Endotoxins act as a negative inotrope
Endotoxins act as a negative inotrope and can initiate both ARDS and disseminated intravascular coagulation (DIC).
Tissue uptake of oxygen by cells is normal
Tissue uptake of oxygen by cells is reduced, contributing to the metabolic acidosis of sepsis due to anaerobic respiration and so increased lactic acid production.
Peripheral vasoconstriction is seen
Peripheral vasodilation is seen throughout the systemic circulation causing reduced tissue perfusion and organ dysfunction if not treated.
Blood cultures are commonly positive
Blood cultures are rarely positive, however, if the causative bacteria is grown from blood cultures then this significantly aids treatment as targeted antimicrobial therapy is possible.
Gram-negative organisms are the most common cause
Septicaemic shock is most commonly due to Gram-positive organisms. While Gram- negative bacteria were previously considered the most common cause of septic shock, recent epidemiological studies revealed approximately 200 000 cases of Gram-positive sepsis annually, compared with 150 000 cases of Gram-negative sepsis in recent years.
Martin GS, Mannino DM, Eaton S, Moss M. The epidemiology of sepsis in the United States from 1979 through 2000. N Engl J Med. 2003;348:1546–54.

How well did you know this?
1
Not at all
2
3
4
5
Perfectly
33
Q

A 68-year-old woman is admitted to the Intensive Care Unit (ITU) having sustained 40% burns in a house fire. Over the next 3 days she develops increasing respiratory needs , wheeze and reduced pa (O2 ) level . You suspect she has developed ARDS.

Which one of the following statements regarding ARDS is correct?

1) It has an increased pa (O2 ) (in mmHg) to FI O 2 ratio

2) It has < 20% mortality in established cases

3) It is caused due to collapse of the alveoli

4) It is characterised by decreased lung compliance

5) It usually develops in the first 24 h after injury

A

Explanation

It is characterised by decreased lung compliance. Pulmonary fibrosis in the later stages of the disease leads to a decrease in the functional residual capacity, further decrease in lung compliance, and an increase in the shunt effect. It has an increased pa (O2 ) (in mmHg) to FI O 2 ratio. ARDS is characterised by refractory hypoxemia (pa (O2 ) < 8 kPa at FI O 2 > 0.4), alveolar
inflammation and oedema, reduced total compliance (< 30 ml/cmH2 O) and a pa (O2 ) (in mmHg) to FI O 2 ratio of < 200 (normal is approximately 500). It has < 20% mortality in established cases. The mortality of this condition is as high as 50–70%.
It is caused due to collapse of the alveoli. Indirect or direct lung injury initiates abnormal behaviour and movement of neutrophils, platelets and macrophages. Neutrophils and platelets attach to capillary endothelium causing capillary leak. This leads to oedema of lung tissue and movement of neutrophils and erythrocytes into the lung parenchyma. Lung lymph flow is increased and there is thickening of the alveolar–capillary membrane. This results in impaired oxygen diffusion and reduced lung compliance as the alveolus is surrounded by fluid. In addition, some of the fluid in the pulmonary parenchyma may leak into the alveoli, giving the characteristic appearance of a hyaline membrane.

It usually develops in the first 24 h after injury. ARDS can present as soon as 2 h following injury. ARDS is an acute, diffuse inflammatory process resulting from direct or indirect pulmonary injury. It is most commonly seen in sepsis but can also occur after trauma, burns, inhalation injuries, shock and pancreatitis. In post-operative surgical patients, abdominal sepsis or central- line sepsis should be considered. Pulmonary signs are often minimal or non-specific, the patient simply being breathless, progressively tachypneic, hypoxic and then cyanotic.

Chest X-ray may be normal in the early stages but later shows bilateral diffuse pulmonary infiltration. Treatment, in addition to eliminating the precipitating cause, involves ventilating the patient in an Intensive Care Unit. Patients are usually nursed in
the prone position. The tidal volume should be kept low (approximately 6 ml/kg) and so should the pulmonary capillary wedge pressure (high pressures exacerbate pulmonary oedema).

How well did you know this?
1
Not at all
2
3
4
5
Perfectly
34
Q

You are treating a patient who underwent emergency laparotomy for faecal peritonitis 2 days ago. The patient is tachycardic and hypotensive and has developed acute respiratory distress syndrome (ARDS). What is the most likely cause of this?

1) Abdominal sepsis

2) Fat embolism

3) Pancreatitis

4) Pulmonary fibrosis

5) Trauma

A

Explanation

Abdominal sepsis

ARDS is an acute diffuse inflammatory process resulting from direct or indirect

pulmonary injury. It is most commonly seen in sepsis but can also occur after trauma,

burns, inhalation injuries, shock and pancreatitis. In post-operative surgical patients,

abdominal sepsis or central-line sepsis should be considered. Pulmonary signs are often

minimal or non-specific in the early stages, the patient simply being breathless, but, if

untreated, they could soon become hypoxic and cyanotic. Chest X-ray may be normal in

the early stages but later shows bilateral diffuse pulmonary infiltration. The mortality of

this condition is as high as 50–70%.

Fat embolism

Fat embolism is a recognised complication of long bone fractures which are associated

with major trauma. As described above, major trauma can cause ARDS, however, given

this patient has been treated for peritonitis, abdominal sepsis is the most likely cause.

Pancreatitis

Pancreatitis is a recognised cause of ARDS, however abdominal sepsis is more likely

given the case history.

Pulmonary fibrosis

Pulmonary fibrosis occurs as a result of ARDS. Fibrosis in the later stages of the disease

leads to a decrease in the functional residual capacity, further decrease in lung

compliance and an increase in the shunt effect.

Trauma

As mentioned, major trauma is a recognised cause of ARDS, however, abdominal sepsis

is the likely cause in this patient. ARDS is characterised by refractory hypoxaemia

(pa (O2 ) < 8 kPa at Fi O2 > 0.4), alveolar inflammation and oedema, reduced total

compliance (< 30 ml/cm water) and a pa (O2 ) (in mm Hg):Fi O 2 ratio of < 200 (normal is

approximately 500). Indirect or direct lung injury initiates abnormal behaviour and

movement of neutrophils, platelets and macrophages. Neutrophils and platelets attach

to capillary endothelium causing capillary leakage. This leads to oedema of lung tissue

and movement of neutrophils and erythrocytes into the lung parenchyma. Lung lymph

flow is increased with thickening of the alveolar–capillary membrane. This results in

impaired oxygen diffusion and reduced lung compliance as the alveolus is surrounded

by fluid. In addition, some of the fluid in the pulmonary parenchyma may leak into the alveoli, giving the characteristic appearance of a hyaline membrane. Treatment, in

addition to eliminating the precipitating aetiology, involves ventilating the patient in

intensive care. Patients are usually nursed in the prone position. The tidal volume

should be kept low (approximately 6 ml/kg), as should the pulmonary capillary wedge

pressure (high exacerbates pulmonary oedema).

How well did you know this?
1
Not at all
2
3
4
5
Perfectly
35
Q

You review a patient who underwent emergency laparotomy for abdominal sepsis 36 h

ago. The patient clearly has acute respiratory distress syndrome (ARDS).

Which one of the following would you expect in ARDS?

1) Focal infiltration on chest X-ray

2) Leakage of protein rich oedema fluid into the lung tissue

3) Only late occurrence of tachypnoea and tachycardia

4) Steroids promote recovery in the late phase

5) Thinning of the alveolar membrane

A

Explanation

Leakage of protein rich oedema fluid into the lung tissue

The underlying pathological process is leakage of cells and fluid through the capillary

walls of the alveoli. The alveolar membrane thickens and ultimately fibroses.

Focal infiltration on chest X-ray

Typically the chest X-ray shows diffuse infiltration.

Only late occurrence of tachypnoea and tachycardia

In the early stages, tachycardia and tachypnoea may be the only features. Later there is

cyanosis and worsening dyspnoea.

Steroids promote recovery in the late phase

Steroids have been used in the treatment of ARDS but there is no good evidence to

support their use. The cornerstones of treatment are support of the circulation,

respiratory support by ventilation and the use of appropriate antibiotics.

Thinning of the alveolar membrane

As mentioned thickening of the alveolar membrane occurs, and in the later stages of the

disease, fibrosis occurs.

Acute respiratory distress syndrome (ARDS) is a well recognised complication of sepsis,

trauma and other pulmonary insults, both direct and indirect. The commonest clinical

signs are tachypnoea, tachycardia, cyanosis and high-pitched coarse crepitations on

auscultation. The alveolar membrane becomes thickened. Differential diagnoses include

left ventricular failure, bronchitis, pneumonia and asthma.

How well did you know this?
1
Not at all
2
3
4
5
Perfectly
36
Q

Systemic Inflammatory Response Syndrome (SIRS) is an uncontrolled immune reaction, and can be classified by aetiology as either infectious or non-infectious SIRS has a
defined criteria for diagnosis. Recognition is essential as early intervention is more likely to prevent severe morbidity and mortality associated with pathologies such as sepsis.

Which one of the following would meet the requirements for SIRS?

1) Temperature of 37.5°C, a leucocyte count of 11 000/mm 3 , heart rate 150 beats/min

2) Temperature of 38°C, a leucocyte count of 5x10 9 /l, respiratory rate 15 breaths/min

3) Temperature of 35.5°C, a leucocyte count of 8x10 9 /l, pa (CO2 ) 50 mmHg

4) Temperature of 38.0°C, a leucocyte count of 2x10 9 /l, respiratory rate 30 breaths/min

5) Temperature of 37.5°C, a leucocyte count of 11x10 9 /l, heart rate 100 beats/min

A

Temperature of 38.0°C, a leucocyte count of 2x10 9 /l, respiratory rate 30 breaths/min

Pyrexia, leucopenia and tachypnoea are all consistent with SIRS. A patient is said to have SIRS if they satisfy two or more of the following criteria:

*temperature > 38°C or < 36°C

*heart rate > 90 beats/min

*respiratory rate > 20/min (or pa (CO2 ) < 32 mmHg)

*leucocyte count > 12 000/mm 3 or < 4000/mm 3 [or > 10% immature (band) cells].

*Temperature of 37.5°C, a leucocyte count of 11 000/mm 3 , heart rate 150 beats/min

This patient is normothermic, has a leucocyte count in the higher range of normal, and a significant tachycardia, as such they do not fit the criteria for SIRS.

Temperature of 38°C, a leucocyte count of 5x10 9 /l, respiratory rate 15 breaths/min

These parameters do not fill the criteria for SIRS.

Temperature of 35.5°C, a leucocyte count of 8x10 9 /l, pa (CO2 ) 50 mmHg

The low temperature fulfils the criteria, however, the leucocyte count is within the normal range (leucocyte count: 4–11 × 10 9 /l), and the pa (CO2 ) is satisfactory.

Temperature of 37.5°C, a leucocyte count of 11x10 9 /l, heart rate 100 beats/min Only the tachycardia would satisfy the SIRS criteria.

How well did you know this?
1
Not at all
2
3
4
5
Perfectly
37
Q

15-year-old post-splenectomy patient is admitted to hospital with sepsis. Since becoming a teenager she has been inconsistent in taking her prophylactic antibiotics.

Which one of the following is a sign indicative of septic shock ?

1) Confirmed or suspected infection

2) Core temperature < 36°C or > 38.3°C

3) Heart rate higher than 90 beats per min

4) Respiratory rate higher than 20 breaths per min

5) Urine output < 0.5ml/kg/h for more than 2 h

A

Explanation

Urine output < 0.5ml/kg/h for more than 2 h

Decreased urine output is an indicator of reduced renal perfusion, which can be used as a surrogate marker for reduced generalised tissue perfusion. This is therefore maybe a sign of septic shock as it represents renal dysfunction which occurs in severe circulatory abnormalities or circulatory collapse. Post-splenectomy sepsis can present 20 years after splenectomy and carries a high mortality. It is prevented by penicillin prophylaxis.

The micro-organisms most likely to lead to infection include pneumococcus and meningococcus species and H. influenzae. Other infections include those due to Escherichia coli and Plasmodium (malaria). The Surviving Sepsis Campaign has published international guidelines on the recognition and management of severe sepsis in an attempt to decrease the high mortality rates. Although NICE guidance lists ‘red- flags’ for sepsis (see below), the National Early Warning System (NEWS) introduced by the Royal College of Physicians is an alternative.

Red-flags for sepsis according to NICE Guidelines:

*systolic blood pressure <90 mmHg (or >40 mmHg fall from baseline)

*heart rate >130 beats per min

*oxygen saturations <91%

*respiratory rate >25 breaths per min

*responds only to voice or pain/unresponsive

*lactate >2.0 mmol.

Confirmed or suspected infection

A patient can have an ongoing infection but may not have sepsis which is defined as ‘life-threatening organ dysfunction due to a dysregulated host response to infection’.

Core temperature < 36°C or > 38.3°C

Although temperature is commonly used as an indicator of infection, it is not a helpful indicator of either severe sepsis or septic shock. Septic shock is defined as ‘a subset of sepsis in which underlying circulatory and cellular/metabolic abnormalities are profound enough to substantially increase mortality’ – The Third International Consensus Definitions for Sepsis and Septic Shock (Sepsis-3). It indicates a patient who is septic with refractory hypotension when given an intravenous fluid challenge/bolus.

While sepsis is referred to as life-threatening organ dysfunction due to a dysregulated host response to infection, so a patient may have an infection causing pyrexia but may not have sepsis.

Heart rate higher than 90 beats per min

Heart rate can be used as a ‘red-flag’ sign for sepsis according to NICE guidance, however, it suggests a tachycardia above 130 beats per min.

Respiratory rate higher than 20 breaths per min

Again, NICE guidance suggests a ‘red-flag’ sign for sepsis is a respiratory rate above 25 breaths per min.

How well did you know this?
1
Not at all
2
3
4
5
Perfectly
38
Q

A 76-year-old patient is admitted to the acute admission unit with septic shock. Pulse is 106 beats per minute and blood pressure 90/40 mmHg. Urinary catheterisation produces 75 ml of concentrated urine (normal 800–2000 ml in 24 h).

Which one of the following principles applies to the choice of an appropriate intravenous fluid for resuscitation?

1) Certain intravenous solutions, which would be hypo-osmolar, have dextrose added to ensure they are iso-osmolar

2) Hartmann’s solution contains sodium, potassium, chloride, calcium and bicarbonate

3) An advantage of crystalloid solutions is that relatively small volumes have to be infused to restore an intravascular volume deficit

4) The normal colloid oncotic pressure is 70 mmHg

5) Albumin is indicated

A

The most appropriate principle regarding the choice of intravenous fluid for resuscitation in a patient with septic shock is:

1) Certain intravenous solutions, which would be hypo-osmolar, have dextrose added to ensure they are iso-osmolar

Explanation:

In the context of intravenous fluid resuscitation for septic shock, the principles involve understanding the properties and functions of different types of fluids. Here’s a breakdown of the options:

1.	Certain intravenous solutions, which would be hypo-osmolar, have dextrose added to ensure they are iso-osmolar: This is correct. Solutions like 5% dextrose are isotonic and provide free water once dextrose is metabolized, helping to ensure fluids remain iso-osmolar to avoid causing cellular dehydration or overhydration.
2.	Hartmann’s solution contains sodium, potassium, chloride, calcium, and bicarbonate: This is incorrect. Hartmann’s solution (or Ringer’s lactate) contains sodium, potassium, chloride, calcium, and lactate (which is metabolized to bicarbonate), but not bicarbonate directly.
3.	An advantage of crystalloid solutions is that relatively small volumes have to be infused to restore an intravascular volume deficit: This is incorrect. Crystalloids generally require larger volumes compared to colloids to achieve the same intravascular volume expansion due to their distribution across the extracellular space.
4.	The normal colloid oncotic pressure is 70 mmHg: This is incorrect. Normal colloid oncotic pressure (oncotic pressure) is around 25-30 mmHg, not 70 mmHg.
5.	Albumin is indicated: This is incorrect as a blanket statement. Albumin is a colloid and can be used in specific situations, but it is not the first-line treatment for fluid resuscitation in septic shock due to its cost and lack of evidence showing superiority over crystalloids.

Correct Answer: 1) Certain intravenous solutions, which would be hypo-osmolar, have dextrose added to ensure they are iso-osmolar

How well did you know this?
1
Not at all
2
3
4
5
Perfectly
39
Q

You are treating a 48-year-old man with acute severe pancreatitis. He has been in hospital for 48 h and you are assessing his modified Glasgow score. His modified Glasgow score is 5.

What is his mortality risk?

1) 2%

2) 15%

3) 30%

4) 40%

5) 100%

A

Explanation

40%

The mortality risk for a patient scoring 5 on the modified Glasgow score is 40%. The following criteria are used to calculate the score.

2%

This would represent the mortality risk for a patient with a modified Glasgow score of

0–2.

15%

This would represent the mortality risk for a patient with a modified Glasgow score of

3–4.

30%

The mortality risk for a patient scoring 5 on the modified Glasgow score is greater than

30%.

100%

Mortality risk reaches 100% when the patient scores between 7 and 8 on the modified

Glasgow score. Pancreatitis is a common presentation in the UK, of which gallstones and alcohol are the two primary aetiologies. Gallstones account for up to 50% of pancreatitis
in the UK, while alcohol accounts for 20–25%. The modified Glasgow score takes into account a number of parameter taken within the first 48 h following admission which can then be used to assess mortality risk These criteria are as follows; pa (O2 ) < 60 mmHg; age > 55 years; neutrophils > 15 × 10 9 /l; calcium < 2 mmol/l; raised urea > 16 mmol/l; enzyme (LDH) > 600 units/l; albumin, 32 g/l; sugar (glucose) > 10 mmol/l.

How well did you know this?
1
Not at all
2
3
4
5
Perfectly
40
Q

A 42-year-old man with type 1 diabetes mellitus, which was diagnosed 23 years ago, is admitted to hospital with diabetic ketoacidosis (DKA). He is complaining of epigastric abdominal pain and vomiting.

Which one of the following findings on admission would most specifically suggest the precipitating cause of this illness?

1) Serum amylase activity of 1244 U/l (upper limit of normal, 150 U/l)

2) Serum creatinine concentration 140 µmol/l (normal value creatinine: 50–120 µmol/l)

3) Serum lactate concentration 4.3 mmol/l (normal value lactate: 0.5–2.2 mmol/l)

4) Serum triglyceride concentration 12.2 mmol/l (normal value triglycerides: 0–1.5 mmol/l)

5) White cell count of 15 × 10 9 /l (normal value WCC: 4–11 × 10 9 /l)

A

Explanation

Serum amylase activity of 1244 U/l (upper limit of normal, 150 U/l)

Patients in DKA often have elevated serum amylase activities as a result of decreased renal excretion of the enzyme, however, a level this high is very suggestive of pancreatitis and coupled with the epigastric pain, pancreatitis is the likely precipitant cause of his DKA.

Serum creatinine concentration 140 µmol/l (normal value creatinine: 50–120 µmol/l)

Serum creatinine concentration is often slightly raised in patients with DKA (even in the absence of diabetic nephropathy) as a result of dehydration and subsequently a
decreased glomerular filtration rate.

Serum lactate concentration 4.3 mmol/l (normal value lactate: 0.5–2.2 mmol/l)

Decreased tissue perfusion as a result of dehydration often leads to an element of lactic acidosis secondary to anaerobic respiration, in addition to the ketoacidosis.

Serum triglyceride concentration 12.2 mmol/l (normal value triglycerides: 0–1.5 mmol/l)

Hypertriglyceridaemia is common in uncontrolled diabetes.

White cell count of 15 × 10 9 /l (normal value WCC: 4–11 × 10 9 /l)

An increased white cell count can occur in any acute illness and does not specifically indicate infection.

How well did you know this?
1
Not at all
2
3
4
5
Perfectly
41
Q

Two hours after sustaining major trauma in a road traffic accident, a 22-year-old man, not known to have diabetes, is found to have a high blood glucose concentration. They have sustained significant injuries

Increased secretion of which one of the following substances is most likely to be responsible?

1) Adrenaline (epinephrine)

2) Cortisol

3) C-reactive protein

4) Growth hormone

5) Insulin

A

Explanation

Adrenaline (epinephrine)

During the metabolic response to trauma, there is increased secretion of catecholamines (such as adrenaline), cortisol, glucagon and growth hormone. Adrenaline acts most rapidly through stimulation of glycogenolysis causing blood glucose concentration to rise

Cortisol

Increased secretion of cortisol following trauma will increase blood glucose concentration, however, this occurs more slowly through the stimulation of gluconeogenesis.

C-reactive protein

C-reactive protein is an acute phase reactant and a marker of inflammation, but does not affect glucose homeostasis.

Growth hormone

Growth hormone is also released in response to major trauma. It can increase blood glucose concentration indirectly as it appears to potentiate the action of cortisol and opposes the action of insulin.

Insulin

Insulin is a hypoglycaemic hormone, therefore would lower blood glucose concentration.

How well did you know this?
1
Not at all
2
3
4
5
Perfectly
42
Q

An 18-year-old man presents to Accident and Emergency after vomiting blood following a night out for his birthday. On further questioning he admits to consuming a large amount of alcohol and then vomiting heavily. After vomiting around 12 times he noticed streaks of blood in his vomit.

What is the most likely diagnosis?

1) Aortic-enteric fistula

2) Boerhaave syndrome

3) Mallory-Weiss tear

4) Oesophageal varices

5) Peptic ulcer disease

A

Explanation

Mallory-Weiss tear

A Mallory–Weiss tea is a tear in the mucosal lining of the oesophagus that classically occurs after heavily vomiting (often following an alcohol binge as described in the case
history). Most cases of bleeding from this are self-limiting and usually minor.

Aortic-enteric fistula

Aortic-enteric fistulae occur mostly following abdominal aortic aneurysm (AAA) repair in which communication has developed between the aorta and the small bowel. They
result in very heavy blood loss into the small bowel and are often fatal.

Boerhaave syndrome

Boerhaave syndrome is perforation of the oesophagus secondary to vomiting. It is associated with a high morbidity and mortality, and would present with severe chest pain, fever and shock. The patient described in the case is not significantly unwell.

Oesophageal varices

Oesophageal varices are the result of portal hypertension as blood is shunted through arteriovenous anastomosis due to increased vascular resistance within the liver, most commonly as a result of cirrhosis secondary to alcohol. It would be rare for an 18-year- old to have significant liver cirrhosis to cause varices. Furthermore, variceal bleeding
would be profuse, and unlikely to present with just streaks of blood in the vomit.

Peptic ulcer disease

A peptic ulcer can cause upper gastrointestinal bleeding, however, it would be unlikely in a young adult, and does not fit with the case history.

How well did you know this?
1
Not at all
2
3
4
5
Perfectly
43
Q

An 82-year-old man is recovering in the Intensive Care Unit
seven days after an emergency repair of a difficult juxta-renal leaking abdominal aortic aneurysm. The nurse bleeps you because he has become tachycardic and his blood pressure has dropped to 80/50 mmHg. On assessment he is very pale and the nurse reports he has just had a large, fresh per rectal (PR) bleed.

What is the most likely cause for this?

1) Angiodysplasia

2) Aortic-enteric fistula

3) Diverticular disease

4) Gastric Cancer

5) Peptic ulcer disease

A

Explanation

Aortic-enteric fistula

Aortic-enteric fistulae occur mostly following abdominal aortic aneurysm (AAA) repair in which a communication has developed between the aorta and the small bowel. They result in very heavy blood loss into the small bowel and are often fatal.

Angiodysplasia

Angiodysplasia is the most common vascular lesion of the gut. Lesions are often multiple and presentation insidious, and may simply present as anaemia.

Diverticular disease

Diverticular disease may present with small amount of fresh PR bleeding, but given the proximity to surgery, an aortic-enteric fistula is more likely.

Gastric Cancer

Gastric cancer is likely to have a more insidious presentation. Again, as the bleeding would be upper gastrointestinal in nature, it would present as melaena not fresh blood.

Peptic ulcer disease

Peptic ulcer can cause bleeding, however this is likely to present as upper gastrointestinal bleeding. Occasionally, they can present as lower gastrointestinal bleeding but this will present as melena as opposed to fresh PR blood.

How well did you know this?
1
Not at all
2
3
4
5
Perfectly
44
Q

A 35-year-old male with known ulcerative colitis (UC) reports generalised itching, fatigue and abdominal pain. On examination, he is found to be jaundiced and his blood tests show a markedly raised bilirubin and alkaline phosphatase. A magnetic resonance cholangiopancreatography (MRCP) test shows multiple strictures in the biliary tree.

What is the most likely diagnosis?

1) Cholangitis

2) Cholecystitis

3) Primary biliary cirrhosis

4) Primary sclerosing cholangitis

5) Wilson’s disease

A

Explanation

Primary sclerosing cholangitis

Primary sclerosing cholangitis (PSC) is a condition in which inflammation, fibrosis and Strictures of the intrahepatic and extra-hepatic bile ducts occur. MRCP shows multiple strictures in the biliary tree and a characteristic ‘beaded’ appearance. Around 80% of patients with PSC will have a diagnosis of UC.

Cholangitis

Cholangitis is an ascending infection of the biliary tree, given this patient has no signs of infection this is unlikely. Furthermore, strictures would not be seen in cholangitis.

Cholecystitis

This refers to inflammation of the gallbladder, most common caused by gallstones. If the gallstones become lodged in the common bile duct then obstructive jaundice may be seen, however, the finding of strictures on MRCP is more suggestive of primary sclerosing cholangitis.

Primary biliary cirrhosis

Primary biliary cholangitis is an autoimmune disorder causing destruction of the small interlobular bile ducts, subsequent intrahepatic cholestasis causes fibrosis and
ultimately cirrhosis of the liver. The patient has a history of UC, so primary sclerosing cholangitis is a more likely, furthermore, strictures in the biliary tree would not be seen
on MRCP.

Wilson’s disease

Wilson’s disease is a rare inherited disorder of copper metabolism leading to a pathological accumulation of copper within various organ, particularly affecting the liver and brain. Presentation is usually in teenager years with either neuropsychiatric conditions due to accumulation of copper in the brain, or with coagulopathy and hepatic encephalopathy from accumulation in the liver. As such, this does not fit with the case history given.

How well did you know this?
1
Not at all
2
3
4
5
Perfectly
45
Q

An 18-year-old A-level student is referred from his GP with
right iliac fossa (RIF) pain and nausea. On examination, she has guarding and tenderness in the RIF. On deep palpation in the left iliac fossa (LIF) she still complains of pain in the RIF.
What is this sign called?

1) Aaron’s sign

2) Dunphy’s sign

3) Murphy’s sign

4) Rigler’s sign

5) Rovsing’s sign

A

Explanation

Rovsing’s sign

Rovsing’s sign describes pain in the RIF more than the LIF when the LIF is pressed, and is a sign of appendicitis.

Aaron’s sign

Aaron’s sign is pain referred to the epigastrium upon continuous firm pressure over

McBurney’s point and is indicative of appendicitis.

Dunphy’s sign

Dunphy’s sign is a medical sign in which increased abdominal pain occurs with coughing and is often present in appendicitis.

Murphy’s sign

Murphy’s sign occurs when pressing over the right upper quadrant (RUQ) with two fingers and asking the patient to breathe in, causes pain and arrest of inspiration as the
inflamed gall-bladder impinges on the examiner’s fingers. It is only positive if repeating the test on the left does not result in a similar pain.

Rigler’s sign

Rigler’s sign is a radiological sign in which air is seen on an abdominal X-ray on both sides of the intestine.

How well did you know this?
1
Not at all
2
3
4
5
Perfectly
46
Q

A previously well 83-year-old woman is brought into Accident and Emergency with abdominal pain and vomiting. On examination, she has a distended abdomen, and denies any previous surgery. On assessment of her groin, a tenderlump
is palpable on the left. It is below and lateral to the pubic tubercle.

What is the most likely cause of her symptoms?

1) Epigastric hernia

2) Femoral hernia

3) Direct inguinal hernia

4) Indirect inguinal hernia

5) Obturator hernia

A

Explanation

Femoral hernia

Femoral hernias classically emerge below and lateral to the pubic tubercle, and are

more common in women due to the wider pelvis. As the neck for femoral hernias is

relatively narrow and stiff, they are more likely to obstruct and strangulate.

Epigastric hernia

Given the patient has a lump in their groin, an epigastric hernia would be unlikely.

Direct inguinal hernia

Inguinal hernias emerge from above and medial to the pubic tubercle. Direct inguinal

hernias pass through Hasselbach’s triangle directly through a defect in the anterior

abdominal wall.

Indirect inguinal hernia

Again, inguinal hernias emerge from above and medial to the pubic tubercle, however,

indirect inguinal hernias pass through the inguinal canal.

Obturator hernia

An obturator hernia is rare but is more common in women and twice as common on the

right side. It particularly affects elderly women who have had recent rapid weight loss.

The hernial sac protrudes through the obturator canal potentially compressing the

geniculate branch of the obturator nerve causing referred pain. Given their anatomical

location they are difficult to diagnose clinically and does not present as a lump in the

groin.

How well did you know this?
1
Not at all
2
3
4
5
Perfectly
47
Q

A 17-year-old girl is brought in with a history of abdominal and left-sided chest pain. She gives a history of excessive vomiting earlier after a large meal, followed by sudden
onset of the pain. On examination she looks very unwell, is febrile and has a chest X-ray that shows a left pleural effusion. She is peripherally shutdown, hypotensive and
tachycardic. Her only past medical history of note is eating disorders for which she is known to the adolescent psychiatry team.

What is the most likely diagnosis?

1) Achalasia

2) Hiatus hernia

3) Mallory–Weiss tear

4) Oesophageal cancer

5) Oesophageal rupture

A

Explanation

Oesophageal rupture

Oesophageal rupture (Boerhaave syndrome) occurs mostly either secondary to trauma eg during endoscopy or due to forceful vomiting. Excessive vomiting may be due to
alcohol or in patients with a history of excessive eating and forced vomiting eg due to bulimia as suggested in this case. Perforation of the oesophagus causes a mediastinitis and an early diagnosis is crucial. Patients will deteriorate rapidly with systemic inflammatory response syndrome, unless the perforation is minimal. In patients with a significant perforation, resuscitation with surgery to debride the mediastinum and place a T-tube within the oesophagus is needed in most cases.

Achalasia

This refers to oesophageal dysmotility secondary to failure of smooth muscle of the lower oesophageal sphincter to relax during swallowing. It causes progressive dysphagia and regurgitation. The acute history of excessive vomiting is more suggestive of oesophageal rupture, achalasia would likely present with a longer history.

Hiatus hernia

The cardia protrudes into the thoracic cavity through the phrenoesophageal membrane into the thoracic cavity in a hiatus hernia. The most common is a sliding hiatus hernia,
which causes symptoms of reflux. It is unlikely to present with a patient in extremis.

Mallory–Weiss tear

A Mallory–Weiss tear is a tear in the mucosal lining of the oesophagus that classically occurs after heavily vomiting (often following an alcohol binge as described in the case
history). Given the patient is in extremis, this is not likely to be the cause as most cases of bleeding from this are self-limiting and usually minor.

Oesophageal cancer

Oesophageal cancer in a young patient would be rare.

How well did you know this?
1
Not at all
2
3
4
5
Perfectly
48
Q

A 24-year-old man is taken to the Emergency Department by his friends as they are concerned regarding blood in his vomit. The patient is very inebriated and a collateral history from them reveals he has been on a stag night and drinking heavily. After arriving home he had been vomiting excessively and recent vomitus has shown streaks of blood. He is haemodynamically stable, with unremarkable blood results, no past medical history and no change in bowel habits with normal stool.

Which one of the following conditions is most likely?

1) Achalasia

2) Diffuse oesophageal spasm

3) Mallory–Weiss tear

4) Para-oesophageal hernia

5) Oesophageal Ca

A

Explanation

Mallory–Weiss tear

A Mallory–Weiss tear occurs after excessive vomiting, often secondary to alcohol and involves a mucosal tear in the mucosa and submucosa. In most cases the bleeding is minor and self-limiting, and no intervention is required.

Achalasia

Achalasia refers to oesophageal dysmotility secondary to failure of smooth muscle of the lower oesophageal sphincter to relax during swallowing. It causes progressive dysphagia and regurgitation. The acute history of excessive vomiting and alcohol binge is more suggestive of a Mallory–Weiss tear, achalasia would likely present with a longer history.

Diffuse oesophageal spasm

This is a disorder of oesophageal dysmotility causing uncoordinated contractions of the oesophagus. Diagnosis can be difficult but it usually presents with intermittent non-
cardiac retrosternal pain and dysphagia.

Para-oesophageal hernia

The acute history given would not support the diagnosis of a para-oesophageal hernia as this tends to present with reflux symptoms and dysphagia.

Oesophageal Ca

This would present with weight loss and dysphagia for which the patient has no history. Additionally, oesophageal cancer in a 24-year-old would be rare.

How well did you know this?
1
Not at all
2
3
4
5
Perfectly
49
Q

An overweight 41-year-old woman presents to the Emergency department with central abdominal pain which is colicky in nature, and vomiting. On further questioning it
emerges she has not opened her bowels for 4 days, and on examination her abdomen is distendedand tender centrally. A plain abdominal radiograph shows dilated loops of small bowel, however, no air is seen in the large bowel or rectum. The radiograph also suggests evidence of pneumobilia.

What is most likely the cause of her symptoms?

1) Caecal volvulus

2) Crohn’s disease

3) Diverticular stricture

4) Gallstone ileus

5) Sigmoid volvulus

A

Explanation

Gallstone ileus

It is unusual for a gallstone to cause problems outside of the biliary tree, but sometimes a gallstone can pass into the small bowel and cause an obstruction known as a gallstone
ileus. This occurs after formation of a cholecystoduodenal fistula between the gall-bladder and duodenum. This causes the presence of air in the biliary tree, (ie pneumobilia), which can be seen on imaging.

Caecal volvulus

A caecal volvulus would cause dilated loops of small bowel, however, it would not cause pneumobilia, which is presence of air in the biliary tree, and is more indicative of gallstone ileus.

Crohn’s disease

Crohn’s disease is an inflammatory bowel disease and can affect the entire length of the gastrointestinal tract with transmural inflammation noted and although Crohn’s disease can cause obstruction, pneumobilia is unlikely. Additionally, the history is more indicative of cholecystitis secondary to gallstones.

Diverticular stricture

The lack of air in the rectum suggests an obstructive picture and the absence of dilated large bowel loops goes against the diagnosis of large bowel obstruction caused by a diverticular stricture.

Sigmoid volvulus

A sigmoid volvulus causes a classic coffee-bean sign on abdominal X-ray.

How well did you know this?
1
Not at all
2
3
4
5
Perfectly
50
Q

A 20-year-old man is admitted as an emergency with suspected appendicitis. This is the most common emergency surgical operation and approximately 80 000 people a year
are admitted with this diagnosis in the UK. How may the symptoms present?

1) Colicky central abdominal pain shifting to right iliac fossa

2) Dysuria, frequency of micturition and fever

3) Nausea and vomiting

4) Sharp localised pain, worse on movement and found at McBurney’s point

5) All of the above

A

Explanation

All of the above

The possibilities of presentation of appendicitis are numerous; good knowledge of the

signs and symptoms of appendicitis is paramount as it is often a clinical diagnosis,

supported with evidence of inflammation in the blood results, and occasionally with

imaging, primarily ultrasound or in older patients computed tomography (CT) scan.

Colicky central abdominal pain shifting to right iliac fossa

The usual presenting history is of colicky and vague central abdominal pain shifting to

the right iliac fossa after 24 h and becoming constant with associated nausea and

pyrexia.

Dysuria, frequency of micturition and fever

History may mimic urinary tract infection with dysuria, frequency and fever. This

generally occurs when the appendix lies on or adjacent to the bladder.

Nausea and vomiting

Nausea and vomiting are common associated symptoms of appendicitis, and are usually

seen as the condition progresses.

Sharp localised pain, worse on movement and found at McBurney’s point

The classical area of maximal tenderness is one-third of the distance from the anterior

superior iliac spine (ASIS) to the umbilicus. This is McBurney’s point. Specific signs on

abdominal examination are as follows:

Psoas stretch sign – passive extension or hyperextension of the hip increases abdominal

pain due to the psoas muscle being in contact with the inflamed appendix.

Rovsing’s sign – Palpation in the left iliac fossa may produce pain at the site of

tenderness in the right iliac fossa due to movement of the inflamed parietal peritoneum.

How well did you know this?
1
Not at all
2
3
4
5
Perfectly
51
Q

A 64-year-old woman presents with severe abdominal pain
and a history of vomiting initially that has now settled, although she continues to retch. On assessment she has widespread abdominal tenderness and looks unwell. A blood gas confirms a metabolic acidosis and the nursing staff report they are unable to pass a nasogastric (NG) tube.

What is the likely diagnosis?

1) Gastric volvulus

2) Oesophageal perforation

3) Perforated duodenal ulcer

4) Sigmoid volvulus

5) Small bowel obstruction

A

Explanation

Gastric volvulus

Gastric volvulus classically involves Borchardt’s triad; epigastric pain, retching without

vomiting and the inability to pass an NG tube. The twisting can occur in two ways: along

the long axis of the stomach (organoaxial) or around the axis perpendicular to the

stomach (mesenteroaxial).

Oesophageal perforation

Oesophageal rupture or perforation occurs mostly either secondary to trauma eg during

endoscopy or due to prolonged forceful vomiting. It is likely to also cause retrosternal

pain and would not prevent passage of an NG tube.

Perforated duodenal ulcer

A perforated ulcer will cause epigastric abdominal pain with peritonitis and possibly

guarding depending on the extent of the perforation. Additionally, it would not obstruct

the passage of a nasogastric tube.

Sigmoid volvulus

Sigmoid volvulus is more likely to present with abdominal pain and complete

constipation. If vomiting were to occur, it would be faeculent in nature.

Small bowel obstruction

This would not prevent the passage of an NG tube.

How well did you know this?
1
Not at all
2
3
4
5
Perfectly
52
Q

A 3-year-old boy presents to the Emergency Department with a severe sore throat, temperature of 38.7 °C and noisy breathing. The child is sat forwards and drooling.

What is the most important first step to perform in the management of this child?

1) Examination of the oropharynx

2) Flexible laryngoscopy

3) IV access and blood cultures

4) IV antibiotics

5) Urgently call the paediatric anaesthetist

A

Explanation

Urgently call the paediatric anaesthetist

The child has acute epiglottitis. Acute epiglottitis tends to present with a rapidly

progressive sore throat and in the late stages, may be associated with inspiratory

stridor. The child will be toxic with a raised temperature and tends to sit forwards and

drool. The loose connective tissue swelling in the epiglottis can occlude the airway and

care must be taken to avoid this. Any attempts to examine the child, including simple

cannulation can precipitate laryngospasm and airway occlusion. Therefore the most

important step in managing this patient is to get the help of someone with experience in

managing paediatric airways.

Examination of the oropharynx

Any attempts to examine the child, including examination of the oropharynx can

precipitate laryngospasm and airway occlusion, and therefore is ill advised and should

be avoided.

Flexible laryngoscopy

As mentioned, any attempt to examine this child may trigger life-threatening airway

occlusion and so should be avoided.

IV access and blood cultures

While this child is likely to be septic given the history, no attempt to cannulate the

patient should be undertaken as this may cause airway obstruction.

IV antibiotics

This would require cannulation, which should not be undertaken.

How well did you know this?
1
Not at all
2
3
4
5
Perfectly
53
Q

A 22-year-old woman student presents with epistaxis. This
fails to improve with pressure. Which one of the following methods would be attempted next?

1) Embolisation

2) External carotid artery ligation

3) Sphenopalatine artery ligation

4) Trans-sphenoidal ligation

5) Use of silver nitrate

A

Explanation

Use of silver nitrate

Application of sustained pressure for epistaxis is the primary first aid measure. If

unsuccessful, silver nitrate or chemical cautery can be utilised and can be very effective

in stemming epistaxis. However, in higher risk patients (particularly the elderly on

anticoagulation), nasal packing, both anterior and posterior, may be required.

Progression to surgery and artery ligation would then be the next management method.

Embolisation

Maxillary artery embolisation is becoming a more frequently used alternative to artery

ligation, however, this is reserved for specialist interventional radiology centres, and

would only be indicated for refractory bleeding despite cautery and nasal packing.

External carotid artery ligation

Ligation of the external carotid artery is reserved for life-threatening epistaxis, other

measures should be trialled to stem bleeding before artery ligation.

Sphenopalatine artery ligation

Ligation of local arteries, such as the sphenopalatine, may be required and are not

uncommon, however, more conservative measures should be

trialled before progression to surgical ligation.

Trans-sphenoidal ligation

This is not indicated for epistaxis.

How well did you know this?
1
Not at all
2
3
4
5
Perfectly
54
Q

An 18-year-old gap-year student presents to the Accident and Emergency Department (A&E) with right-sided abdominal pain and rigours. On examination, he is clammy to touch, febrile and tender in the right upper quadrant. His only medical history of note is dysentery while on a backpacking trip to Mexico 2 months ago. What is the most likely diagnosis?

1) Amoebic liver abscess

2) Cholangiocarcinoma

3) Gallstones

4) Hydatid disease

5) Pancreatitis

A

Explanation

Amoebic liver abscess

Amoebic liver abscesses often occur secondary to Entamoeba histolytica infection in the

gut entering the portal circulation and spreading to the liver. The condition occurs

mostly in endemic areas and affects travellers visiting them. Clinical features include

abdominal pain (most commonly in the right upper quadrant), fever, rigours and

nausea/vomiting. Some patients will report a history of dysentery recently. Imaging

with ultrasound scan or computed tomography (CT) is useful in the diagnosis, and can

be combined with aspiration which produces a thick fluid resembling anchovy sauce.

Treatment mostly consists of metronidazole sometimes with percutaneous drainage.

Cholangiocarcinoma

Cholangiocarcinoma would be rare in an 18-year-old adult.

Gallstones

Gallstones presents with colicky, right upper quadrant abdominal pain. It may be

associated with pyrexia if there is cholecystitis. It would be uncommon in young men,

more often presenting in overweight, middle-aged women. As such, the case history

does not fit with this diagnosis.

Hydatid disease

Hydatid disease is caused by parasitic Echinococcus tapeworms, it causes the formation

of slow growing cysts, usually in the liver or lungs, containing hydatid fluid. Patients can

often be asymptomatic. Intermediate hosts are grazing animals, with dog and cats often

acting as definitive hosts. Humans are only accidental hosts. Given this patient has a

history of dysentery while travelling, this supports the diagnosis of amoebic abscess

formation, as opposed to hydatid disease.

Pancreatitis

This commonly presents with epigastric abdominal pain radiating through to the back.

The most common aetiology in the UK are gallstones or alcohol, both of which would be

unlikely in a young adult.

How well did you know this?
1
Not at all
2
3
4
5
Perfectly
55
Q

A 75-year-old man who has had a prolonged intensive care unit admission following an emergency abdominal aortic aneurysm repair is referred to the surgical team because of
increased abdominal pain and discomfort noted when he is moved. His bloods and urine show normal amylase and lipase levels respectively. A computed tomography (CT) scan showed no leak from the anastomosis, but an ultrasound scan showed a dilated gall-bladder with oedema in the wall. However, no gallstones were noted. What is the most likely diagnosis?

1) Aorta-enteric fistulae

2) Biliary colic

3) Cholecystitis

4) Ileus

5) Pancreatitis

A

Explanation

Cholecystitis

Acalculous cholecystitis is rare, but can occur in critically ill patients often

on the intensive care unit. The mechanism for this is thought to involve gall-

bladder stasis due to analgesia or parenteral nutrition. Imaging includes

ultrasound, which can show a dilated gall-bladder with oedema in the wall.

Aorta-enteric fistulae

Aortic-enteric fistulae occur mostly following abdominal aortic aneurysm (AAA) repair

in which a communication has developed between the aorta and the small bowel. They

result in very heavy blood loss into the small bowel and are often fatal.

Biliary colic

Biliary colic secondary to gallstones is common and presents as colicky abdominal pain.

Given that no gallstones were identified on ultrasound and the prolonged stay in

intensive care, this is less likely than acalculous cholecystitis.

Ileus

Ileus is common in patients who have undergone major abdominal surgery and is often

due to handling of the bowel during the operation. It usually resolves after a few days

and ‘bowel rest’. The findings of a distended gall-bladder suggest a more likely

pathology.

Pancreatitis

Pancreatitis usually presents with epigastric pain, and the most common causes are

gallstones or alcohol. Given no gallstones were identified on ultrasound but the gall-

bladderis distended and oedematous, acalculous cholecystitis must be considered.

Moreover, this patient has normal lipase and amylase levels.

How well did you know this?
1
Not at all
2
3
4
5
Perfectly
56
Q

A 57-year-old woman is admitted under the general surgical team with severe epigastric pain radiating to her back. She has a history of profuse vomiting and a serum amylase comes back as 1200 U/ml. She is started on treatment for acute pancreatitis and further tests are done.

Result Normal

Serum amylase

1200 U/l

<200 U/l

White-cell count

12 × 10 9 /l

4–11 × 10 9 /l

Calcium

1.9 mml/l

2.20–2.60 mmol/l

Urea

17 mmol/l

2.5–6.5 mmol/l

LDH

672 IU/l

100–190 IU/l

Albumin

31 g/l

35–55 g/l

Blood glucose

11 mmol/l

3.5–5.5 mmol/l

Arterial blood gas pa (O2 )

13.1 kPa

10.5–13.5 kPa

How severe is her pancreatitis using the modified Glasgow criteria?

1) Mild

2) Moderate

3) Severe

4) Haemorrhagic

5) Necrotic

A

Explanation

Severe

The modified Glasgow criteria can be used to assess the severity of acute pancreatitis by

using parameters taken within the first 48 h following admission. The most common

aetiologies in the UK are either gallstones or alcohol. This patient has significantly

deranged physiological and biochemical markers, increasing their Glasgow score,

indicating a severe pancreatitis.

Mild = 1 positive factor, moderate = 2 factors, severe = 3 positive factor.

Mild

The patient has significant derangements of the physiological and biochemical markers

used to assess severity of pancreatitis, increasing the Glasgow score above the mild

category.

Moderate

As above, the patient has significant derangements in the parameters used to calculate

the Glasgow score, causing the severity to be categorised as severe.

Haemorrhagic

The terms haemorrhagic and necrotic describe the computed tomography

(CT) scan appearance of the pancreas, and influence the Balthazar score.

Necrotic

Again, this term is reserved for describing the appearance of the pancreas on CT.

How well did you know this?
1
Not at all
2
3
4
5
Perfectly
57
Q

A 45-year-old woman with known gallstones is admitted with epigastric pain and vomiting. Her serum amylase is 1194 U/1ml. An abdominal ultrasound shows gallstones in the common bile duct (CBD) with a dilated CBD and it is thought she has acute gallstone pancreatitis. Chest X-ray (CXR) and abdominal X-ray (AXR) are unremarkable and her other test results are as follows:

                                          Result                                      Normal

White-cell count. 9 × 10 9 /l. 4–11 × 10 9 /l

Calcium. 2.3 mml/l. 2.20–2.60 mmol/l

Urea. 9 mmol/l. 2.5–6.5 mmol/l

LDH. 352 IU/l. 100–190 IU/l

Albumin. 35 g/l. 35–55 g/l

Blood glucose. 7 mmol/l. 3.5–5.5 mmol/l

Arterial blood gas pa (O2 ) 12 kPa. 10.5–13.5 kPa

With this information, which scoring system can be best used to assess the

severity of her pancreatitis?

1) APACHE

2) Balthazar

3) Glasgow

4) Hinchey

5) Ranson

A

Explanation

Glasgow

The modified Glasgow criteria can be used to assess the severity of acute pancreatitis by

using parameters taken within the first 48 h following admission.

APACHE

The APACHE score requires additional information to those provided eg blood pressure,

urine output and is used to assess mortality risk for patients admitted to intensive care

regardless of causative pathology. Balthazar

The Balthazar grade is based on the computed tomography (CT) scan appearance of the

pancreas.

Hinchey

The Hinchey classification grades diverticulitis not pancreatitis.

Ranson

Ranson’s criteria are valid for alcohol-induced pancreatitis and can only be applied after

48 h.

The severity of acute pancreatitis is typically classified based on various criteria, including clinical, biochemical, and radiological parameters. Here are some commonly used criteria:

1.	Ranson’s Criteria: Developed by Dr. John Ranson in the 1970s, this system includes 11 criteria—5 assessed at admission and 6 during the first 48 hours. The presence of 3 or more of these criteria indicates severe pancreatitis and a poorer prognosis.
2.	APACHE II Score (Acute Physiology and Chronic Health Evaluation II): This scoring system evaluates physiological parameters within the first 24 hours of admission. A higher score indicates a more severe condition and a worse prognosis.
3.	Balthazar Score: This system assesses the severity of pancreatitis based on CT imaging findings. It uses a grading system from A to E, with E indicating the most severe form of pancreatitis.
4.	Imrie Score: Similar to Ranson’s criteria, the Imrie scoring system evaluates clinical and laboratory parameters to predict the severity of pancreatitis. A score of 3 or more indicates severe pancreatitis.
5.	CT Severity Index (CTSI): This index evaluates the extent of pancreatic and peripancreatic inflammation on CT imaging. It helps in predicting the severity of pancreatitis and guiding management decisions.
6.	Bedside Index for Severity in Acute Pancreatitis (BISAP): This simplified scoring system assesses five parameters at admission—blood urea nitrogen, impaired mental status, systemic inflammatory response syndrome, age, and pleural effusion—to predict the severity of acute pancreatitis.
7.	CRP (C-reactive protein): Elevated levels of CRP have been associated with severe acute pancreatitis and can be used as a marker for assessing severity.
8.	Modified Glasgow Criteria: Similar to Glasgow Coma Scale, this criteria assesses various clinical parameters and organ dysfunction to predict the severity of pancreatitis.

These criteria help clinicians in assessing the severity of acute pancreatitis, guiding treatment decisions, and predicting patient outcomes.

Sources:

•	https://www.ncbi.nlm.nih.gov/pmc/articles/PMC5734411/
•	https://www.ncbi.nlm.nih.gov/books/NBK482291/
How well did you know this?
1
Not at all
2
3
4
5
Perfectly
58
Q

A 38-year-old woman with a 1-year history of intermittent epigastric pain presents with vomiting and abdominal distension. A plain abdominal radiograph reveals dilated loops of small bowel and pneumobilia. What is the most appropriate management?

1) Emergency laparotomy

2) Endoscopic retrograde cholangiopancreatography (ERCP)

3) Laparoscopic cholecystectomy

4) Percutaneous drainage of the gall bladder

5) Surgical closure of a cholecystoduodenal fistula

A

Explanation

Emergency laparotomy

This patient has a small bowel obstruction secondary to gallstone ileus. This is a surgical

emergency requiring urgent laparotomy. The gallstone usually impacts proximal to the

ileocaecal valve, and so it should be massaged through into the large intestine or

extracted via an enterotomy, so relieving the obstruction.

Endoscopic retrograde cholangiopancreatography (ERCP)

This intervention is used for the diagnosis and treatment of gallstones located in the

common bile duct. Again, this will not solve the small bowel obstruction which requires

emergency intervention.

Laparoscopic cholecystectomy

Laparoscopic cholecystectomy is indicated for those with recurrent cholecystitis with

evidence of gallstones. This does not, however, address the small bowel obstruction

which is a surgical emergency requiring laparotomy.

Percutaneous drainage of the gall bladder

Small bowel obstruction is a surgical emergency and requires intervention.

Percutaneous drainage of the gall bladder will not resolve the bowel obstruction and

therefore would be inappropriate.

Surgical closure of a cholecystoduodenal fistula

The gallstone will have passed into the small bowel through a biliary-enteric fistula, but

this does not usually require surgical repair. Such fistulae often lead to pneumobilia,

which can be seen on plain abdominal radiographs.

How well did you know this?
1
Not at all
2
3
4
5
Perfectly
59
Q

A previously fit and well 54-year-old man attends the General Practice Surgery, as his family have told him he is ‘looking yellow’. On examination, he is aundiced, but his abdomen is soft and non-tender. In the right upper quadrant (RUQ), his
gall bladder is palpable. A triple-phase computerised tomography (CT) reveals a tumour at the head of
the pancreas.

Which of the following phenomena is demonstrated?

1) Cope sign

2) Courvoisier’s law

3) Murphy’s sign

4) Psoas sign

5) Rovsing’s sign

A

Explanation

Courvoisier’s law

Courvoisier’s law states that in the event of painless jaundice and a palpable gall

bladder, the cause is unlikely to be gallstones. This is because the presence of gallstones

tends to make the gall bladder fibrosed, and therefore it is unlikely to become distended

in the event of an obstruction of the biliary tree.

Cope sign

Cope sign involves flexion and internal rotation of the right hip, causing pain if the

appendix is lying in close relation to the obturator internus.

Murphy’s sign

Murphy’s sign occurs when, upon pressing over the RUQ with two fingers, asking the

patient to breathe in causes pain and arrest of inspiration, as the inflamed gall bladder

impinges on the examiner’s fingers.

Psoas sign

Psoas sign involves extension of the hip, causing pain if there is a retrocaecal appendix.

Rovsing’s sign

Rovsing’s sign describes pain in the right iliac fossa more than in the left iliac fossa

when the left iliac fossa is pressed.

How well did you know this?
1
Not at all
2
3
4
5
Perfectly
60
Q

A 37-year-old woman presents to the acute surgical take. She is
postpartum and on the oral contraceptive pill, she complains of right upper quadrant pain, nausea and vomiting. On examination she has
hepatosplenomegaly and ascites.

What is the most likely cause for this?

1) Alcoholic cirrhosis

2) Budd–Chiari syndrome

3) Pylephlebitis after acute appendicitis

4) Splenectomy

5) Tricuspid valve incompetence

A

Explanation

Budd–Chiari syndrome

Portal hypertension with a pressure of over 20 mmHg is commonly caused by pre-hepatic problems such as portal vein thrombosis, hepatic disease such as cirrhosis and post-hepatic problems such as tricuspid valve incompetence and Budd–Chiari syndrome of hepatic vein thrombosis. Budd–Chiari syndrome commonly results in
hepatosplenomegaly and ascites. It is associated with pregnancy and being postpartum.

Use of the oral contraceptive pill can also increase the risk for venous thromboembolic disease.

Alcoholic cirrhosis

There is no history of alcohol excess in the history given, so alcoholic cirrhosis would be unlikely.

Pylephlebitis after acute appendicitis

Pylephlebitis is an uncommon thrombophlebitis of the portal vein, caused by an infection such as appendicitis. The case history given, however, is not consistent with appendicitis, which commonly presents with vague, centralised abdominal pain, localising to the right iliac fossa over 24 hours.

Splenectomy

Palpable splenomegaly is noted on examination; therefore, splenectomy cannot be the cause.

Tricuspid valve incompetence

Although tricuspid valve incompetence can cause portal hypertension, and subsequently hepatosplenomegaly, as described in the case history, as the patient is postpartum, Budd–Chiari syndrome is the more likely diagnosis.

How well did you know this?
1
Not at all
2
3
4
5
Perfectly
61
Q

An elderly, confused man presents with a
grossly distended tympanic abdomen
and absolute constipation. A plain abdominal radiograph reveals a grossly
dilated large intestine , but no evidence of pneumoperitoneum. What is the most appropriate initial management?

1) Computerised tomographic scan of the abdomen and pelvis

2) Laparotomy and sigmoid colectomy

3) Laxatives

4) Sigmoidoscopy

5) Trephine ileostomy

A

Explanation

Computerised tomographic scan of the abdomen and pelvis

For this scenario, the differential diagnosis is pseudo-obstruction, sigmoid volvulus and carcinoma of the distal colon/rectum. Computerised tomography of the abdomen and pelvis is the best option, as it permits the diagnosis to be obtained and the correct treatment instituted. Pseudo-obstruction may be associated with an underlying precipitating condition such as renal failure, electrolyte abnormalities, myxoedema, stroke, myocardial infarction, chest infection and retroperitoneal malignancy (pseudo- obstruction in the presence of retroperitoneal malignancy is known as Ogilvie syndrome). Medical management comprises correction of electrolyte abnormalities, treatment of the underlying cause and use of the anticholinergic neostigmine (with heart monitor surveillance).

Laparotomy and sigmoid colectomy

At present, the diagnosis is not clear and further imaging is required, so the most
appropriate treatment can be initiated. Sigmoid colectomy is indicated in an elective setting for recurrent sigmoid volvulus. Sigmoid volvulus often has the characteristic ‘coffee bean’ sign on plain abdominal radiograph. Obstructed colon caused by a distal neoplasm is open to several surgical options following adequate resuscitation. These include stenting, defunctioning stoma, primary resection and anastomosis following on-
table colonic lavage or Hartmann’s procedure. Note if the patient was too unstable for the computerised tomography scanner, then exploratory laparotomy may indeed be indicated first.

Laxatives

Initiating treatment before identifying the causative pathology could cause further harm to the patient. For example, if the obstruction is caused by a carcinoma of the sigmoid colon, use of laxatives could precipitate a perforation.

Sigmoidoscopy

Sigmoidoscopy may be beneficial in decompressing the colon with a flatus tube when a sigmoid volvulus is the causative pathology. However, the diagnosis is not clear in this case, so further imaging is required before intervention. Fixation procedures for
recurrent sigmoid volvulus, such as sigmoidopexy, have been performed but are associated with high recurrence rates.

Trephine ileostomy

This is an operative technique to create a permanent end-colostomy without undergoing a laparotomy. Again, this would not be indicated until the diagnosis is clarified with further imaging.

How well did you know this?
1
Not at all
2
3
4
5
Perfectly
62
Q

An 82-year-old man presents with intermittent abdominal pain and vomiting. On further questioning, it is noted he has
not opened his bowels for 5 days and not passed wind for 2 days. On examination, he looks uncomfortable and his abdomen is grossly distended. His only past medical history of note is an open repair of an
abdominal aortic aneurysm (AAA) 4 months ago, from which he recovered well.

What is the most likely cause of his symptoms?

1) Adhesional small bowel obstruction

2) Aorto-enteric fistula

3) Incisional hernia

4) Diverticular disease

5) Sigmoid tumour

A

Explanation

Adhesional small bowel obstruction

This patient has a number of characteristic features of small bowel obstruction:

vomiting, constipation and abdominal distension. Adhesions are the most common

cause of small bowel obstruction in the developed world and can occur following any

form of abdominal surgery.

Aorto-enteric fistula

Aortic-enteric fistulae occur mostly following AAA repair, in which a communication has

developed between the aorta and the small bowel. They result in very heavy blood loss

into the small bowel, causing rectal bleeding, and are often fatal.

Incisional hernia

There is nothing in the history of the case to suggest the patient has an incisional hernia

which could be causing obstruction.

Diverticular disease

There are no features in the case history that suggest diverticular disease as the cause

for obstruction.

Sigmoid tumour

Carcinoma of the colon, particularly the sigmoid colon, is a common cause for large

bowel obstruction. However, there are no features in the history that point to a sigmoid

tumour as the cause for obstruction, and given this gentleman has had previous major

abdominal surgery, adhesional bowel obstruction is more likely.

How well did you know this?
1
Not at all
2
3
4
5
Perfectly
63
Q

An 88-year-old woman, who has a past medical history of dementia and is bed-bound in a nursing home, presents with a history of constipation for the last 7 days. Her carers have noted her abdomen has become grossly distended and is tender to touch. An abdominal X-ray demonstrates the coffee bean sign.

What would be an appropriate treatment option to manage this patient initially ?

1) Antibiotics

2) Computerised tomography (CT) scan

3) Exploratory laparotomy

4) Insertion of flatus tube

5) Sigmoid colectomy

A

Explanation

Insertion of flatus tube

The coffee bean sign and clinical history are highly suggestive of sigmoid volvulus,

which commonly occurs in elderly patients who have a chronic history of constipation.

Insertion of a flatus tube represents a relatively low-risk procedure that could

potentially resolve the patient’s symptoms and therefore would be a good first-line

option. If this fails, the next step would be flexible sigmoidoscopy.

Antibiotics

Antibiotics are not indicated, as there are no features in the case history suggestive of

infection.

Computerised tomography (CT) scan

A CT scan would confirm the diagnosis but offers no therapeutic benefit to the patient.

Exploratory laparotomy

Given the patient’s pre-morbid status of being a bed-bound nursing home patient, the

risks of any major surgery, eg laparotomy, would be great. Depending on the severity of

the patient’s dementia, they may also lack capacity to consent to surgery, and it is likely

to be against the patient’s best interests to undergo major surgery.

Sigmoid colectomy

Again, major surgery for this patient would be ill-advised.

How well did you know this?
1
Not at all
2
3
4
5
Perfectly
64
Q

An 85-year-old man is recovering 2 days after a hemiarthroplasty for a fractured neck of femur. The nurses call you as he has started vomiting profusely and his
abdomen is distended. He has not opened his bowels for 3 days and on examination, he has a distended, tender abdomen. He has never undergone any abdominal surgery and has no palpable herniae. His rectum is empty. His blood tests show a potassium level of 2.6 and a creatinine level of 144.

What is the most likely diagnosis?

1) Adhesional small bowel obstruction

2) Constipation secondary to opioids

3) Incisional hernia

4) Obstruction hernia

5) Pseudo-obstruction

A

Explanation

Pseudo-obstruction

The history of vomiting, constipation and a distended abdomen is suggestive of an

obstruction. In this case, this gentleman has developed pseudo-obstruction, which often

resembles bowel obstruction and can occur in any post-operative patients and can also

be triggered by deranged electrolytes, eg hypokalaemia. It is best managed

conservatively through correcting the underlying causes.

Adhesional small bowel obstruction

The most common cause for small bowel obstruction, namely adhesions, is unlikely in

this case, given the patient has never had abdominal surgery.

Constipation secondary to opioids

Although the patient is likely to have been prescribed opioid-based medication before

and following the hemi-arthroplasty, his empty rectum points away from constipation.

Incisional hernia

No hernias are palpable on examination and he has not previously undergone

abdominal surgery; as such, this is incorrect.

Obstruction hernia

This is unlikely, as there are no palpable hernias on examination and no history of

previous abdominal surgery.

How well did you know this?
1
Not at all
2
3
4
5
Perfectly
65
Q

A frail 87-year-old woman is brought in by her concerned family as she has started vomiting and not opened her bowels or passed wind for 3 days. On examination, she looks unwell with a distended abdomen, but no previous incisions, and on further questioning, she reports the pain radiates along the right upper medial thigh. Vaginal examination reveals a swelling in the right side wall of the vagina.

What is the most likely diagnosis?

1) Adhesional small bowel obstruction

2) Epigastric hernia

3) Para-umbilical hernia

4) Obturator hernia

5) Sigmoid volvulus

A

Explanation

Obturator hernia

Pain which radiates to the thigh is suggestive of pressure on the obturator nerve which supplies sensation to the medial thigh. This can be present in around 50% of patients with an obturator hernia. Obturator hernias occur most commonly in frail old women and often there are no signs. If pressure is present over the obturator nerve, patients often hold the leg flexed to reduce pain. Rectal, and more often vaginal, examination can reveal a swelling in the region of the obturator foramen.

Adhesional small bowel obstruction

Although the most common cause of small bowel obstruction is adhesions in the developed world, the lack of previous surgery makes this very unlikely.

Epigastric hernia

An epigastric hernia would not cause pain in the medial thigh and vaginal examination would be unremarkable. Furthermore, an epigastric hernia was not identified on examination of the abdomen.

Para-umbilical hernia

There were no hernias identified on abdominal examination, and a lack of previous abdominal surgery makes this diagnosis unlikely.

Sigmoid volvulus

Sigmoid volvulus is a common cause for bowel obstruction, particularly in the elderly.

However, it would not produce pain in the medial thigh and vaginal examination would be unremarkable.

How well did you know this?
1
Not at all
2
3
4
5
Perfectly
66
Q

A 32-year-old man develops an acutely painful arm after having been immobilised in a plaster cast, following a closed fracture of his distal radius 8 hours previously.

What is the most likely underlying diagnosis?

1) Compartment syndrome

2) Deep vein thrombosis

3) Early infection

4) Inadequate analgesia

5) Necrotising fascitis

A

Explanation

Compartment syndrome

Compartment syndrome is characterised by increased pressure within an unyielding osseofascial compartment, resulting in local tissue hypoxia. The earliest sign is pain out of proportion with the injury, particularly severe pain on passive muscle stretch. Pallor, paraesthesiae and pulselessness occur in late stages. Diagnosis is on clinical grounds, although compartment pressures can be measured. Raised creatine kinase (CK) levels are indicative of muscle necrosis. Fasciotomies to release the deep and superficial compartments should be performed early. Although more common in the lower limb, it is possible in the upper limb as well.

Deep vein thrombosis

While venous thromboembolic disease should be considered in those with prolonged immobilisation of a limb, development of a deep vein thrombosis within 8 hours is
improbable.

Early infection

Infection of the limb may be painful. However, there are no other signs of infection in the case history. Moreover, compartment syndrome should always be considered in an
acutely painful limb which has recently been immobilised in a plaster cast.

Inadequate analgesia

Adequate analgesia is important. However, effective immobilisation should provide sufficient pain relief. The history described is highly suggestive of compartment syndrome.

Necrotising fascitis

Necrotising fasciitis is a surgical emergency caused by bacterial infection of the fascia and subsequent necrosis of the subcutaneous tissues. It requires emergency debridement of affected areas with wide margins and intravenous antibiotics. In the case history, there is no suggestion of sepsis with haemodynamic compromise, which
would be seen with necrotising fasciitis.

How well did you know this?
1
Not at all
2
3
4
5
Perfectly
67
Q

A 2-year-old child is admitted to the Paediatric Emergency Department with malaise, rash, vomiting and fever. Mum reports that she has been off her food for the preceding 24 hours and has been running a low-grade temperature. She also points to a partial-thickness burn over her arm that has been de-roofed and treated with dressings by her general practitioner.

What is the likely diagnosis?

Select one answer only.

1) Anaphylaxis

2) Haemophilus influenza

3) Meningococcal septicaemia

4) Salmonella

5) Toxic shock syndrome

A

Explanation

Toxic shock syndrome

An unwell child with an unhealed burn must be treated for toxic shock syndrome until proven otherwise. Treatment will require management in the Intensive Care Department, with input from a paediatric consultant and the plastic surgery team. Circulatory support with cryoprecipitate may be necessary.

Anaphylaxis

This is a paediatric airway emergency which requires emergency administration of intramuscular adrenaline to prevent airway occlusion from a hypersensitivity reaction.

The history described is not consistent with anaphylaxis, which presents rapidly with lip and tongue swelling, airway compromise and difficulty breathing.

Haemophilus influenzae

There is no suggestion of respiratory signs or symptoms in the case history, so this is unlikely, despite being a common paediatric presentation.

Meningococcal septicaemia

Meningococcal meningitis leading to septicaemia is life-threatening and should always be considered in an acutely unwell child. However, given the unhealed burn, toxic shock
syndrome is more probable.

Salmonella

Salmonella infection would present with gastrointestinal symptoms and signs, of which there are none in the case history, other than anorexia which is a common and non-
specific symptom for an unwell child.

How well did you know this?
1
Not at all
2
3
4
5
Perfectly
68
Q

A 17-year-old man presents with right-sided abdominal pain, which is worse on movement, and anorexia. He is listed for an open appendicectomy which reveals a normal appendix. Further exploration intra-operatively reveals a normal caecum, and a terminal ileum with no obvious inflammation. However, an outpouching is noted on the terminal ileum around 60 cm from the ileocaecal valve.

What is the most likely diagnosis?

1) Abdominal tuberculosis (TB)

2) Appendicitis

3) Caecal carcinoma

4) Crohn’s disease

5) Meckel’s diverticulum

A

Explanation

Meckel’s diverticulum

The description of an outpouching in this location on the antimesenteric border is consistent with a Meckel’s diverticulum. These are embryological remnants of the vitello-intestinal duct and are a free diverticulum of the terminal ileum. They occur in 2% of the population, are commonly 2 feet (60 cm) from the ileocaecal valve, often 2
inches (5 cm) in length and twice as common in men than women. Acute inflammation may mimic appendicitis and it is important when performing an appendicectomy to search for a Meckel’s diverticulum if the appendix is normal.

Abdominal tuberculosis (TB)

The case history is not consistent with abdominal TB and no risk factor for TB is described.

Appendicitis

The normal appendix goes against a diagnosis of acute appendicitis.

Caecal carcinoma

The normal caecum makes a caecal tumour causes the symptoms very unlikely, furthermore, a caecal carcinoma in a 17-year-old adolescent would be rare.

Crohn’s disease

The lack of inflammation in the terminal ileum makes Crohn’s unlikely.

How well did you know this?
1
Not at all
2
3
4
5
Perfectly
69
Q

An 89-year-old woman is seen on the post-take ward round following CT scanning. She presented with tenderness in the left iliac fossa for 7 days. She has localised tenderness but feels better than on admission yesterday. Past medical history reveals a cerebrovascular accident (CVA) 5 years ago which has left her bed bound with chronic obstructive pulmonary disease (COPD), requiring multiple admissions. Her CT scan shows diverticulitis with a localised perforation. Blood results show a white-cell count of 18 × 10 9 /l (normal 4–11 × 10 9 /l) and C-reactive protein 160 mg/l (normal 0–10
mg/l).

What is the best initial management?

1) Discharge

2) Intravenous antibiotics and close monitoring

3) Laparotomy and primary anastomosis

4) Laparoscopic resection

5) Laxatives

A

Explanation

Intravenous antibiotics and close monitoring This elderly patient has a localised diverticular perforation with absence of peritonitis and is showing signs of improvement. She has significant past medical history that would make her a high risk candidate for surgery, therefore conservative management with IV antibiotics and close monitoring for signs of deterioration is the best management path.

Discharge

Discharge is obviously not appropriate at this point.

Laparotomy and primary anastomosis

A primary anastomosis would not be advisable in an acute case with potential contamination leading to a higher chance of anastomotic leak. An anastomotic leak could be disastrous in this patient and so a colostomy would be best surgical practice if she were to be taken to theatre.

Laparoscopic resection

If surgical intervention were required, laparoscopy may be difficult due to her COPD, particularly as carbon dioxide insufflation of the abdominal cavity will splint the diaphragm, making ventilation more difficult.

Laxatives

Laxatives are not the primary concern in this patient’s acute management.

How well did you know this?
1
Not at all
2
3
4
5
Perfectly
70
Q

A 78-year-old nursing home resident is admitted with severe diarrhoea and left iliac fossa pain. At flexible sigmoidoscopy diffuse pseudomembranes are seen. Biopsy is neutrophil rich.

What is the most likely diagnosis?

1) Crohn’s disease

2) Melanosis Coli

3) Malignancy

4) Pseudomembranous colitis

5) Ulcerative Colitis

A

Explanation

Pseudomembranous colitis

The appearance of pseudomembranes are typical for pseudomembranous colitis caused by Clostridium difficile. This pathogen is more common in nursing home or hospitalised patients, and is often associated with antibiotic use. High risk antibiotics include; fluoroquinolones, cephalosporins and clindamycin. Prevention of the spread of C. difficileinvolves isolation of infected patients, judicious hand hygiene and measured antibiotic use.

Crohn’s disease

Crohn’s disease is an inflammatory bowel condition affecting the entire length of the gastrointestinal tract. Biopsy would demonstrate transmural inflammation and presentation of Crohn’s disease is usually in early 20s.

Melanosis Coli

This is a benign condition identified on colonoscopy in which there is a disorder of pigmentation of the endothelium of the colon. As the condition is benign, it would not produce the symptoms described in the case history.

Malignancy

Malignancy can present with a change in bowel habit, although it is usually less dramatic than severe diarrhoea, while malignancies are usually painless until complications such as perforation or obstruction occur.

Ulcerative Colitis

Ulcerative colitis is an inflammatory bowel disease in which inflammation is limited to the colon. It presents in the early 20s, so a diagnosis of ulcerative colitis in the elderly is uncommon.

How well did you know this?
1
Not at all
2
3
4
5
Perfectly
71
Q

A 22-year-old right-hand-dominant woman is referred to the hand surgery unit with pain and swelling of her right index finger. She thinks she has sustained a small puncture wound
to the finger while helping her dad with gardening 2 days ago. On examination the finger is diffusely swollen, erythematous and held in a slightly flexed position. It is exquisitely painful to passively extend the finger , and most tender on the volar aspect.

What is the most likely diagnosis?

1) Cellulitis

2) Flexor tendon sheath infection

3) Foreign body

4) Mallet finger

5) Paronychia

A

Explanation

Flexor tendon sheath infection

A flexor tendon sheath infection is an infection of the sheath that forms the lubricating surface around the flexor tendons to the hand. This constitutes an emergency as severe infection can destroy these surfaces resulting in a painful and stiff finger. In most cases there will be a history of penetrating trauma before the infection.

There are four main signs of a flexor tendon sheath infection:

*diffuse swelling of the digit

*the digit will be partially flexed

*tenderness along the flexor tendon sheath

*pain on passive extension of the digit.

Cellulitis

Cellulitis would certainly be included in the differential diagnosis given the case history, however, cellulitis would not cause severe pain on passive extension of the finger.

Foreign body

This will likely produce localised inflammation and swelling around the site of the foreign body. Additionally, pain on passively extending the finger suggests tendon sheath infection.

Mallet finger

Mallet finger describes the deformity of disruption of the extensor digitorum tendon at the distal interphalangeal joint with blunt force trauma being the causative injury. The deformity is not described and the patient has not suffered blunt force trauma in the case history.

Paronychia

Paronychia is a local infection around a fingernail that often begins as a cellulitis before developing into an abscess which requires drainage. This is not described in the case history above.

How well did you know this?
1
Not at all
2
3
4
5
Perfectly
72
Q

Question:

A 60-year-old woman who had undergone a successful left renal transplant 1 week earlier presents with fever, oliguria, and rising serum creatinine. She has been referred back to the transplant team for ongoing care. Which is the best management option in such a scenario?

Options:

1.	It is a picture of acute rejection characterized by preformed antibodies to donor human leukocyte antigen (HLA) and should be treated with intravenous hydrocortisone
2.	Tacrolimus should be the first-line treatment
3.	The woman has chronic graft rejection
4.	Treatment is with prednisolone
5.	Urinary obstruction, infection, reduced blood supply and drug toxicity should be considered before treating for acute graft rejection
A

Correct Answer:

5.	Urinary obstruction, infection, reduced blood supply and drug toxicity should be considered before treating for acute graft rejection

Explanation:

In the immediate post-transplant period, a rise in serum creatinine accompanied by fever and oliguria can be due to several potential causes. While acute rejection is a possibility, it is crucial to first rule out other common causes such as:

•	Urinary obstruction: Check for any blockages in the urinary tract.
•	Infection: Infections can cause fever and impact kidney function, and must be treated appropriately.
•	Reduced blood supply: Ensure that the transplanted kidney is receiving adequate blood flow.
•	Drug toxicity: Medications used post-transplant can sometimes be nephrotoxic.

It is important to conduct a thorough evaluation to rule out these causes before initiating treatment for acute graft rejection. This approach helps to avoid unnecessary immunosuppressive therapy and addresses any reversible causes of graft dysfunction. Therefore, the correct management strategy in this scenario is to consider and investigate other potential causes of the symptoms before treating for acute graft rejection.

How well did you know this?
1
Not at all
2
3
4
5
Perfectly
73
Q

Question:

A 71-year-old diabetic man presents with cellulitis of his lower limb and pyrexia. He has pain on flexion of his ankle and it is tender to palpation. He develops a metabolic acidosis, and his blood pressure and urine output decline steadily throughout the day. Further examination reveals a reddish-bronze discoloration of the skin and decreased sensation, with the margin of infection advancing rapidly along the fascial planes. What is the diagnosis?

Options:

1.	Cellulitis
2.	Compartment syndrome
3.	Myositis
4.	Necrotising fasciitis
5.	Rhabdomyolysis
A

Correct Answer:

4.	Necrotising fasciitis

Explanation:

Necrotising fasciitis is a severe, rapidly progressing soft tissue infection characterized by widespread necrosis of the fascia and subcutaneous tissue. The key features in this case that point towards necrotising fasciitis include:

•	Rapid progression of infection: The infection is advancing rapidly along the fascial planes.
•	Severe systemic signs: The patient has metabolic acidosis, declining blood pressure, and urine output, indicating severe sepsis or septic shock.
•	Characteristic skin changes: The reddish-bronze discoloration and decreased sensation are typical signs of necrotising fasciitis.
•	Tenderness and pain: Severe pain and tenderness beyond what is expected for cellulitis.

These findings distinguish necrotising fasciitis from other conditions such as cellulitis, compartment syndrome, myositis, and rhabdomyolysis, which do not typically present with such rapid progression and systemic deterioration. Immediate surgical intervention and broad-spectrum antibiotics are critical in the management of necrotising fasciitis to prevent further tissue destruction and systemic complications.

Explanation

Necrotising fasciitis

The progressive symptoms and signs with progressive haemodynamic compromise is

highly suggestive of necrotising fasciitis. It is a surgical emergency requiring

debridement with wide margins and treatment with high dose intravenous antibiotics.

Necrotising fasciitis often involves an initial focus such as a puncture wound, insect bite,

or scratch, and usually begins like cellulitis with hot, red and tender skin. However,

there may be an initial dissemination in the deeper tissue planes in the absence of

surface changes. At the leading edge of infection, there is reddish or

bronze discoloration of the skin and decreased sensation. The margin of infection

advances rapidly along fascial planes with thrombosis of perforating vessels, oedema,

necrosis and sloughing of the skin occur. Haemorrhagic bullae are relatively common.

Clinical features can be divided into early findings, which include pain, cellulitis, pyrexia,

tachycardia, swelling, induration and skin anaesthesia. Late findings may include severe

pain, purple or black skin discoloration, blistering, haemorrhagic bullae, crepitus,

discharge of ‘dishwater’ fluid, severe sepsis or systemic inflammatory response

syndrome and multiorgan failure.

Cellulitis

The condition has progressed from a simple cellulitis to necrotising fasciitis given the

worsening condition of the patient and clinical signs of a rapidly spreading fascial

infection.

Compartment syndrome

Compartment syndrome is characterised by increased pressure within an unyielding

osseofascial compartment, resulting in local tissue hypoxia. It is often an early

complication of long bone fractures. The case history given is not consistent with

compartment syndrome.

Myositis

Myositis is a broad term used to describe inflammation of the muscles, of which there

can be many causes including infection, medications, trauma or be immune mediated. It would not be associated with rapidly advancing skin changes or significant

haemodynamic compromise.

Rhabdomyolysis

This is condition causing skeletal muscle to break down rapidly following either direct

or indirect injury, and is a common pathology following a crush injury. A rapid

accumulation of myoglobin can cause significant renal failure. Again, rhabdomyolysis

would not produce skin changes and the history is consistent with necrotising fasciitis.

How well did you know this?
1
Not at all
2
3
4
5
Perfectly
74
Q

Question:

A 22-year-old man presents with altered behaviour, fever, and sore throat. His GP started him on a course of amoxicillin. His condition worsened and he was admitted to the Emergency Department with reduced consciousness and a rash over his torso that spread to his limbs together with neck stiffness. What is his likely diagnosis?

Options:

1.	Disseminating intravascular coagulopathy (DIC)
2.	HIV seroconversion
3.	Meningococcal sepsis
4.	Staphylococcus scalded skin syndrome
5.	Viral infection
A

Correct Answer:

3.	Meningococcal sepsis

Explanation:

The patient’s presentation is highly suggestive of meningococcal sepsis, which is a life-threatening condition caused by the bacterium Neisseria meningitidis. The key features pointing towards this diagnosis include:

•	Altered behaviour and reduced consciousness: Suggest central nervous system involvement.
•	Fever and sore throat: Common initial symptoms of an infection.
•	Rash: The rash described, which spreads from the torso to the limbs, is characteristic of the petechial or purpuric rash seen in meningococcal sepsis.
•	Neck stiffness: A sign of meningitis, which often accompanies meningococcal infection.

Other Options:

1.	Disseminating intravascular coagulopathy (DIC): A serious condition that involves widespread blood clotting and bleeding, but it does not typically present with the combination of symptoms described.
2.	HIV seroconversion: Can present with fever and rash, but the acute presentation of reduced consciousness and neck stiffness is not typical.
3.	Staphylococcus scalded skin syndrome: Characterized by widespread redness and skin peeling, typically without the severe systemic symptoms described.
4.	Viral infection: While it can present with fever and rash, the severity of symptoms and rapid progression is more indicative of a bacterial cause such as meningococcal sepsis.

Given the critical nature of meningococcal sepsis and the need for urgent treatment, this is the most likely diagnosis.

How well did you know this?
1
Not at all
2
3
4
5
Perfectly
75
Q

Question:

An 83-year-old woman is admitted to the hospital with bloody diarrhea, painful abdominal cramps, fever, and leucocytosis. She has recently been discharged from the hospital having been treated for community-acquired pneumonia. What is her likely diagnosis?

Options:

1.	Bowel cancer
2.	C. difficile
3.	Campylobacter jejuni
4.	Inflammatory bowel disease
5.	Peptic ulcer
A

Correct Answer:

2.	C. difficile

Explanation:

The patient’s presentation is highly suggestive of a Clostridium difficile (C. difficile) infection, which is a common cause of antibiotic-associated colitis. The key features include:

•	Recent antibiotic use: She was recently treated for pneumonia, likely with antibiotics, which is a major risk factor for developing C. difficile infection.
•	Bloody diarrhea and abdominal cramps: These are classic symptoms of C. difficile colitis.
•	Fever and leucocytosis: Indicate an infectious process, commonly seen in severe C. difficile infection.

Other Options:

1.	Bowel cancer: While it can cause changes in bowel habits and bleeding, the acute presentation with fever and leukocytosis is more indicative of an infectious cause.
2.	Campylobacter jejuni: Typically causes gastroenteritis with diarrhea, but the association with recent antibiotic use makes C. difficile more likely.
3.	Inflammatory bowel disease (IBD): Can present with bloody diarrhea and abdominal pain, but the acute onset after antibiotic use and presence of fever and leukocytosis point more towards an infectious etiology.
4.	Peptic ulcer: Usually presents with upper abdominal pain and possibly bleeding, but not with bloody diarrhea and systemic symptoms like fever and leukocytosis.

Given the patient’s history of recent hospitalization and antibiotic treatment, C. difficile infection is the most likely diagnosis.

How well did you know this?
1
Not at all
2
3
4
5
Perfectly
76
Q

Question:

A 45-year-old previously well man presents with a 3-day history of constant right groin pain and nausea. He has been limping for 2 days. Physical examination is unremarkable other than slight tenderness in the right groin on palpation. Urine dipstick is positive for blood and his white cells are raised. What is the likely diagnosis?

Options:

1.	Cystitis
2.	Incarcerated femoral hernia
3.	Psoas abscess
4.	Pyelonephritis
5.	Renal calculi
A

Correct Answer:

3.	Psoas abscess

Explanation:

While renal calculi can present with many of these symptoms, the correct diagnosis in this context is a psoas abscess. The key features supporting this diagnosis include:

•	Constant right groin pain and limping: The psoas muscle, located in the retroperitoneal space, can cause referred pain to the groin and hip, leading to limping.
•	Nausea: Can occur due to systemic infection and inflammation.
•	Tenderness in the right groin: Psoas abscess often presents with pain that can be elicited on palpation or during movements that stretch the psoas muscle, such as hip flexion.
•	Raised white cells: Indicative of an infection.
•	Positive urine dipstick for blood: Can occur due to adjacent structures being affected or due to irritation/inflammation from the abscess.

Other Options:

1.	Cystitis: Typically presents with urinary symptoms like dysuria, frequency, and urgency, which are not highlighted in this case.
2.	Incarcerated femoral hernia: Would likely present with a palpable mass and more acute, localized symptoms.
3.	Pyelonephritis: Usually presents with systemic symptoms like fever, flank pain, and dysuria, and less commonly with significant groin pain and limping.
4.	Renal calculi: Although possible, the presence of limping and the specific description of the groin pain suggests a muscular or soft tissue source, such as a psoas abscess.

Given the clinical presentation, a psoas abscess is the most appropriate diagnosis.

How well did you know this?
1
Not at all
2
3
4
5
Perfectly
77
Q

A 68-year-old male diabetic patient presents with foot pain. He has long-standing peripheral neuropathy and has been gardening wearing sandals. He reports 3 days of increasing pain and swelling of his calf. His legs are covered in blackened sores.

Which one of the following causes gas gangrene?

1) Clostridium perfringens

2) Staphylococcus aureus

3) Clostridium botulinum

4) Staphylococcus haemolyticus

5) Pseudomonas aeruginosa

A

Explanation

Clostridium perfringens

Gas gangrene can cause myonecrosis, gas production, and sepsis. It may progress to toxic shock rapidly. Features are large, blackened sores and crepitus caused by gas escaping the necrotic tissue. Clostridia are saprophytes; they live in soil and require a spore to protect them from dehydration. Clostridium perfringens, formerly C. welchii, can cause gas gangrene. There is gas production due to the proteolytic enzymes released by the organism. It also causes food poisoning at about 12–18 h after ingestion. This is caused by exotoxin production.

Staphylococcus aureus

Staphylococcus aureus is the commonest causative pathogen for cellulitis, however the case does not describe a simple cellulitis and another cause should be sought.

Clostridium botulinum

Also a member of the Clostridium genus, C. botulinum produces the exotoxin botulinum,which causes a flaccid paralysis as opposed to gas gangrene.

Staphylococcus haemolyticus

This bacteria is part of normal commensal skin flora, and is often the causative organism in opportunistic infections following a breach in the natural barrier of the skin, such as following cannulation.

Pseudomonas aeruginosa

Pseudomonas is an important pathogen with regard to hospital-acquired infections, multi-drug resistance and infections in immunocompromised patients. Diabetic patients with lower limb ulcers can often become colonised with Pseudomonas causing chronic non-healing ulcers, which are troublesome to treat. It does not however produce gas
gangrene.

How well did you know this?
1
Not at all
2
3
4
5
Perfectly
78
Q

A 35-year-old man presents to the Emergency department with a 24-h history of severevthrobbing pain in the finger tip
of his right index finger. He injured his hand while gardening 3 days before and thinks he cut his hand on a rose thorn. The fingertip is erythematous and swollen up to the distal interphalangeal joint (DIP) joint, but not proximal to it.

What is the most likely diagnosis?

1) Apical infection

2) Felon

3) Herpetic whitlow

4) Paronychia

5) Flexor tendon sheath infection

A

Explanation

Felon

A felon is an abscess in the compartments of the pulp. This is usually more painful than

a paronychia. The swelling does not extend proximal to the distal interphalangeal joint.

The infection often follows a penetrating trauma and is most common in the thumb and

index finger.

Apical infection

An apical infection involves the apical space, ie between the distal part of the nail and

the bone of the distal phalanx, infections of this space may occur when a splinter runs

under the nail. Tenderness is greatest at or just under the free edge of the nail.

Herpetic whitlow

Herpetic whitlow is caused by the herpes simplex virus and small clear vesicles would

be seen.

Paronychia

Paronychia is an infection beside or proximal to the nail, it usually begins as a simple

cellulitis before progressing into a definite abscess which requires incision and

drainage.

Flexor tendon sheath infection

Flexor tendon sheath infection is a surgical emergency with a sausage-shaped digit,

flexed position, tenderness over the flexor tendon sheath and pain on passive extension.

How well did you know this?
1
Not at all
2
3
4
5
Perfectly
79
Q

A 10-month-old boy presents with colicky abdominal pain, vomiting and redcurrant stools. On examination there is a
sausage-shaped mass palpable within an otherwise soft abdomen.

What is the most appropriate initial management?

1) Emergency laparotomy

2) Air enema

3) Lower gastrointestinal (GI) endoscopy

4) Rigid sigmoidoscopy

5) Stool softeners

A

Explanation

Air enema

The clinical picture is of intussusception which occurs in 0.4% of children. It is more common in boys, usually younger than 1 year old. Intussusception occurs when one section of the bowel invaginates into another, the most common form is ileocolic. Blood and mucus or ‘redcurrant jelly’ stool may be passed after the first 24 h as a late manifestation. Sausage-shaped mass may be felt in upper abdomen. Rectal examination is very important as occasionally the apex of intussusception is palpable. The majority of cases are successfully treated conservatively with an air enema.

Emergency laparotomy

Emergency operative management is only indicated in established peritonitis, perforation or failure of enema reduction.

Lower gastrointestinal (GI) endoscopy

Lower GI endoscopy would not be indicated given the clinical scenario, the initial management of choice would be an air enema.

Rigid sigmoidoscopy

Again, rigid sigmoidoscopy is not indicated.

Stool softeners

Stool softeners have no role in the treatment of intussusception.

How well did you know this?
1
Not at all
2
3
4
5
Perfectly
80
Q

You are examining an 18-month-old infant who is day 1 post-operation. The infant has a pulse of 150 beats/min, respiratory rate 30 breaths/min, urine output 1.6 ml/kg/h,
haemoglobin 11 g/dl and white-cell count 12 × 10 9 /l.

Which one of these options is abnormal for an infant of this age?

1) Haemoglobin

2) Pulse rate

3) Respiratory rate

4) Urine output

5) White-cell count

A

Explanation

Pulse rate

The normal ranges for physiological variables and blood test results vary with age in children. The normal value for pulse rate in a infant aged between 1–2 is 100–120 beats per minute, therefore, this infant is tachycardic, which warrants further investigation.

Haemoglobin

The normal range for haemoglobin is 10.5–13.5 g/dl.

Respiratory rate

A normal respiratory rate for an infant aged 1–2 years old is 25–35. This will reduce with age.

Urine output

Normal urine output would be more than 1.5 ml/kg/h.

White-cell count

A normal white-cell count will range from 6–15 × 10 9 /l.

How well did you know this?
1
Not at all
2
3
4
5
Perfectly
81
Q

A 1-day-old newborn girl has a distended abdomen and bilious vomiting. Per rectal (PR) examination reveals an empty rectum, and meconium has not been passed.

What is the most likely underlying associated condition that the patient may have?

1) Cerebral palsy

2) Cystic fibrosis

3) Down syndrome

4) Spina bifida

5) Turner’s syndrome

A

Explanation

Cystic fibrosis

The newborn has meconium ileus. Signs of this include a distended abdomen, bilious

vomiting and an empty rectum. The commonest association is with cystic fibrosis, and

15% of children with cystic fibrosis have meconium ileus.

Cerebral palsy

Cerebral palsy is a broad diagnosis covering a number of neurodevelopmental

conditions affecting movement and coordination. It is not associated with meconium

ileus.

Down syndrome

Down syndrome is a genetic disorder resulting from trisomy of chromosome 21. It is

associated with a number of conditions including congenital heart disease, epilepsy,

hypothyroidism and leukaemia.

Spina bifida

This is a birth defect resulting in failure of closure of the meninges. While there may be

bladder and bowel pathology due to failure of closure of the spinal cord, cystic fibrosis

has the commonest associated with meconium ileus.

Turner’s syndrome

This is a genetic disorder seen in women in which there complete, or partial loss, of the

X chromosome. It is not associated with meconium ileus.

How well did you know this?
1
Not at all
2
3
4
5
Perfectly
82
Q

An 8-year-old child presents with vague central abdominal pain and nausea. He is pyrexial (38.7 °C), and has a headache. On examination he has shifting abdominal tenderness and cervical lymphadenopathy.

What is the most likely diagnosis?

1) Acute appendicitis

2) Constipation

3) Crohn’s disease

4) Meckel’s diverticulum

5) Mesenteric adenitis

A

Explanation

Mesenteric adenitis

Mesenteric adenitis is an inflammation of the mesenteric lymph node accompanied by a

mild peritoneal reaction. It is most common between ages 5–10 years and often

follows/accompanies an URTI.

Acute appendicitis

Acute appendicitis can be distinguished from mesenteric adenitis by lack of cervical

lymphadenopathy, headache, mild abdominal pain, shifting tenderness and pyrexia

greater than 38 °C.

Constipation

Constipation would not cause pyrexia or cervical lymphadenopathy. There are a

number of causes of constipation in children including hypothyroidism, hypercalcaemia,

and Hirschsprung’s disease.

Crohn’s disease

The incidence of Crohn’s disease in children is increasing, presentation is similar to

adults, but children more commonly have extra-gastro-intestinal symptoms, none of

which has been described in the case history.

Meckel’s diverticulum

Meckel’s diverticulum can present with similar symptoms to appendicitis, or with

bleeding, perforation, intussusception, volvulus or obstruction.

How well did you know this?
1
Not at all
2
3
4
5
Perfectly
83
Q

A 9-month-old baby boy is brought to the Paediatric Surgical Emergency unit by his parents with a 24-hour history of intermittent episodes of crying, vomiting, and refusal to feed. The parents have noticed that the baby’s stools are mixed with blood. On examination, a ‘sausage-shaped’ mass is palpable over the right side of the abdomen. A per rectal examination reveals an empty rectum, but blood is noticed on the examining glove.

From the options below choose the one which you think is the most likely diagnosis in this patient:

1) Intussusception

2) Infantile hypertrophic pyloric stenosis

3) Duodenal atresia

4) Meconium ileus

5) Hirschsprung’s disease

A

The most likely diagnosis for this patient is:

Intussusception

Explanation:

1.	Intussusception: This is a condition where a part of the intestine telescopes into an adjacent part, causing a blockage. It typically presents with intermittent abdominal pain, vomiting, bloody stools (often described as “currant jelly” stools), and a palpable sausage-shaped mass in the abdomen. The presence of blood on rectal examination supports this diagnosis.
2.	Infantile Hypertrophic Pyloric Stenosis: This condition presents with projectile vomiting in infants but does not typically cause bloody stools or a palpable abdominal mass.
3.	Duodenal Atresia: This congenital condition causes bilious vomiting in newborns and is usually diagnosed shortly after birth. It does not typically present with bloody stools or a palpable mass.
4.	Meconium Ileus: Often associated with cystic fibrosis, this condition presents with bowel obstruction in newborns and meconium impaction but not with the acute symptoms described in this case.
5.	Hirschsprung’s Disease: This condition involves a lack of nerve cells in parts of the colon, leading to severe constipation and abdominal distension. It does not typically present with acute episodes of crying, vomiting, or bloody stools.

Given the clinical presentation, intussusception is the most likely diagnosis.

How well did you know this?
1
Not at all
2
3
4
5
Perfectly
84
Q

A mother presents her 1-year-old son to the hospital with a history of absence of faeces production for 48 hours. The infant has a distended abdomen with a palpable sausage-shaped mass. His mother thinks he is constipated. What is the most common cause of obstruction in the first 2 years of life?

1) Hirschsprung’s disease

2) Hyperthyroidism

3) Coeliac disease

4) Intussusception

5) Hypercalcaemi

A

The most common cause of obstruction in the first 2 years of life is:

Intussusception

Explanation:

1.	Intussusception: This is the most common cause of intestinal obstruction in young children, particularly between the ages of 6 months and 2 years. It occurs when a part of the intestine telescopes into an adjacent segment, leading to obstruction, reduced blood flow, and potentially severe complications. The classic presentation includes episodic crying, abdominal pain, vomiting, and the passage of “currant jelly” stools. A palpable sausage-shaped mass is a key clinical finding.
2.	Hirschsprung’s Disease: While this condition can cause chronic constipation and abdominal distension due to a lack of ganglion cells in the colon, it is not the most common cause of acute intestinal obstruction in this age group.
3.	Hyperthyroidism: This condition is rare in infants and does not typically present with acute intestinal obstruction.
4.	Coeliac Disease: This autoimmune disorder affects the small intestine due to gluten sensitivity and can cause chronic symptoms such as diarrhea, malabsorption, and failure to thrive, but it is not typically associated with acute intestinal obstruction in infants.
5.	Hypercalcaemia: Elevated calcium levels can lead to constipation but are not a common cause of intestinal obstruction in infants.

Given the clinical presentation of a distended abdomen, absence of stool production, and a palpable sausage-shaped mass, intussusception is the most likely diagnosis.

How well did you know this?
1
Not at all
2
3
4
5
Perfectly
85
Q

An 80-year-old man is referred to the Emergency Department with abdominal pain and vomiting. His pain
radiates to his back and slightly improves on leaning forwards. Examination reveals a soft abdomen with a very tender epigastrium . His past medical history consists of hypertension for which he takes bendroflumethiazide. Systolic blood pressure is 110 mmHg on both arms. X-rays are unremarkable apart from a prominent central loop.

What is the most likely diagnosis?

1) Aortic dissection

2) Biliary Colic

3) Pancreatitis

4) Renal Colic

5) Ruptured aortic aneurysm

A

Explanation

Pancreatitis

This patient is most likely to have pancreatitis possibly caused by thiazide diuretic use.

However, the most common cause is alcohol and gallstones in the UK and evidence of this must be sought on history and ultrasound scan. A significantly raised amylase should be apparent on the blood results, >1000 U/l (normal <200 U/l) is virtually diagnostic but beware that peritonitis and perforated duodenal ulcer for example can give similar appearances and a raised. Some units now use serum lipase which can be more sensitive. Pancreatitis can lead to vomiting but the X-ray appearance alludes to a sentinel loop – a paralytic ileus of duodenal or jejunal small bowel caused by local inflammation of the pancreas.

Aortic dissection

Of course in this age group, ruptured abdominal aortic aneurysm (AAA) and aortic dissection must be a consideration and vitally ruled out. Look for evidence of a pulsatile epigastric mass and difference in blood pressure in both arms. Computed tomography (CT) aortogram is suggested if there is any doubt, to rule out these other diagnoses.

Biliary Colic

Biliary colic more commonly presents with intermittent, colicky pain in the right upper quadrant and does not radiate through to the back.

Renal Colic

In renal colic, patients usually cannot get comfortable due to the spasmodic nature of the pain and urine dip may show blood.

Ruptured aortic aneurysm

As above, this is an important diagnosis to exclude, however, the case history is more consistent with acute pancreatitis.

How well did you know this?
1
Not at all
2
3
4
5
Perfectly
86
Q

At the request of the nursing staff, you agree to assess a patient who is being treated for a post-operative chest infection. You diagnose septic shock as per recent NICE guidance. On what parameters would you make that diagnosis?

1.	Temperature 37°C, pulse rate 110/min, 25 mmHg improvement in systolic blood pressure following fluid bolus
2.	Temperature greater than 38.3°C, pulse rate 100/min, blood pressure 134/82 mmHg without intravenous fluids
3.	Temperature greater than 38°C and urine output less than 30 ml/h
4.	Temperature 37.3°C and pulse rate 80/min
5.	Temperature 38.4°C, urine output less than 30 ml/h, no improvement in blood pressure despite fluid challenge
A

To diagnose septic shock according to the recent NICE guidelines, you should look for the following key parameters:

1.	Persistent hypotension requiring vasopressors to maintain mean arterial pressure (MAP) ≥ 65 mmHg despite adequate fluid resuscitation.
2.	Serum lactate level > 2 mmol/L despite adequate fluid resuscitation.

Given these criteria, the correct answer is:

5.	Temperature 38.4°C, urine output less than 30 ml/h, no improvement in blood pressure despite fluid challenge

Explanation:

•	A temperature of 38.4°C indicates a fever, which is often present in septic patients.
•	Urine output less than 30 ml/h indicates significant hypoperfusion, a common feature in septic shock.
•	No improvement in blood pressure despite a fluid challenge suggests persistent hypotension, which is critical for diagnosing septic shock.

This scenario matches the definition of septic shock as outlined by NICE guidelines, involving persistent hypotension and evidence of organ dysfunction despite fluid resuscitation.

How well did you know this?
1
Not at all
2
3
4
5
Perfectly
87
Q

Sepsis is a potentially life-threatening emergency that requires early recognition and management. Prompt intervention subsequently reduces future morbidity and
mortality. According to National Institute of Clinical Excellence (NICE) Guidance, which of the following is a ‘red flag’ for sepsis?

1) Systolic blood pressure <100 mmHg

2) Heart rate >100 beats/min

3) Respiratory rate >25 breaths/min

4) Lactate of 1.9 mmol

5) White blood-cell count of >12 × 10 9 /l

A

Explanation

Respiratory rate >25 breaths/min

A raised respiratory rate is one of the first parameters to change in the presence of physiological stress. As such, the National Institute of Clinical Excellence (NICE) guidance identifies a raised respiratory rate as an important red flag for sepsis. Recent National Institute of Clinical Excellence (NICE) Guidance was released in 2016 in response to a UK Parliamentary inquiry examining sepsis. The ‘Time to Act’ report found failures in the recognition, diagnosis, and early management of those who died from sepsis. Efforts are now being made to improve the recognition and diagnosis of
sepsis, and close monitoring and identification of abnormal physiological parameters is an important aspect of this.

Systolic blood pressure <100 mmHg

Systolic blood pressure <90 mmHg is identified as a red flag for sepsis. Consideration of baseline blood pressure is also an important factor and therefore a >40 mmHg fall from
patient’s baseline should also be considered as a red flag in the context of sepsis.

Heart rate >100 beats/min

Tachycardia is an important indicator of physiological stress, however, the National Institute of Clinical Excellence (NICE) states that a heart rate >130 beats/min is a red flag for sepsis.

Lactate of 1.9 mmol

An elevated lactate is an indicator of reduce tissue perfusion as a result of anaerobic respiration. A measured lactate of >2.0 mmol is a ‘red flag’ for sepsis.

White blood-cell count of >12 × 10 9 /l

Although this would form part of a detailed review of a patient with suspected sepsis, the red flags refer specifically to physiological parameters. If abnormal, these should then trigger an in-depth review including investigations such as full blood count, blood cultures, radiological imaging as required.

How well did you know this?
1
Not at all
2
3
4
5
Perfectly
88
Q

You are sitting in the endocrine surgery clinic and a patient sees you with moon facies, central obesity and peripheral muscle wasting. You suspect Cushing syndrome. Which one of the following options is true of Cushing syndrome?

1) The commonest cause of Cushing syndrome is iatrogenic

2) Cushing syndrome is due to an excess of aldosterone

3) True Cushing syndrome is due to an adrenal adenoma

4) Removal of a pituitary adenoma carries a high risk of Nelson syndrome

5) Cushing syndrome is seen more in men than in women

A

The correct option is:

1) The commonest cause of Cushing syndrome is iatrogenic

Explanation:

•	Iatrogenic Cushing syndrome is the most common cause of Cushing syndrome, resulting from the prolonged use of corticosteroid medications. These medications are often prescribed for conditions such as asthma, rheumatoid arthritis, and lupus.
•	Cushing syndrome is due to an excess of aldosterone: This statement is incorrect. Cushing syndrome is caused by an excess of cortisol, not aldosterone. Aldosterone excess is related to conditions such as Conn’s syndrome (primary hyperaldosteronism).
•	True Cushing syndrome is due to an adrenal adenoma: This statement is partially correct but not comprehensive. True Cushing syndrome can result from various sources, including pituitary adenomas (Cushing’s disease), ectopic ACTH production, or adrenal tumors (adenomas or carcinomas). However, adrenal adenomas are not the most common cause.
•	Removal of a pituitary adenoma carries a high risk of Nelson syndrome: This statement is incorrect. Nelson syndrome is a potential complication after bilateral adrenalectomy for Cushing’s disease, not after removal of a pituitary adenoma.
•	Cushing syndrome is seen more in men than in women: This statement is incorrect. Cushing syndrome is more commonly seen in women than in men, particularly in cases of Cushing’s disease (pituitary adenoma).

Thus, the most accurate statement regarding Cushing syndrome is that the commonest cause is iatrogenic, primarily due to the use of exogenous corticosteroids.

How well did you know this?
1
Not at all
2
3
4
5
Perfectly
89
Q

A 49-year-old man presented in the orthopaedic outpatient clinic with an X-ray that showed a compressed fracture at T10. He complained of increasing weakness over the past few months. Clinical examination revealed a blood pressure of 165/110 mmHg. Laboratory findings included a serum glucose of 8.6 mmol/l. He was mildly obese and had purplish striae on his thighs and abdomen. Which one of the following pathological lesions is most likely to explain these findings?

1.	Adrenal cortical carcinoma
2.	Anaplastic thyroid carcinoma
3.	Multinodular goiter
4.	Parathyroid adenoma
5.	Phaeochromocytoma
A

The patient’s presentation is consistent with Cushing’s syndrome, which is characterized by a combination of features including central obesity, hypertension, glucose intolerance (as indicated by elevated serum glucose), and purplish striae. The clinical scenario provided points towards an underlying pathological lesion that can cause Cushing’s syndrome.

Among the options listed, the most likely cause is:

1.	Adrenal cortical carcinoma

Explanation:
Adrenal cortical carcinoma can cause Cushing’s syndrome by producing excess cortisol. The symptoms described, such as obesity, hypertension, purplish striae, and glucose intolerance, align with Cushing’s syndrome. Other options, such as anaplastic thyroid carcinoma, multinodular goiter, parathyroid adenoma, and phaeochromocytoma, do not typically present with this specific combination of symptoms.

Thus, the correct answer is:

1.	Adrenal cortical carcinoma 

Adrenal cortical carcinoma is a rare cancer carrying a poor prognosis, with most cancers metastatic at diagnosis. The female to male ratio is 2–3:1, and there is a bimodal distribution, with peak incidence in the first decade of life and a second peak in the 4th–5th decade. Metastasis is most commonly to periadrenal tissue, regional lymph nodes, bone, liver, and lungs.

These are functioning tumors producing excess amounts of cortisol that are not under the control of pituitary adrenocorticotropic hormone (ACTH). Patients present with red flag symptoms such as fever, weight loss, back pain, and abdominal fullness, but also with symptoms relating to Cushing’s syndrome, as demonstrated in this scenario, and virilization syndromes.

Treatment is with adrenalectomy if the tumor is confined to the adrenal gland, or chemotherapy (mostly mitotane) and/or radiotherapy if there is extensive disease. If the tumor is confined to the adrenal gland and is treated, 5-year survival is 65%. This drops to 44% for tumors with local invasion to surrounding tissues and 7% for tumors with distant metastases at diagnosis.

Cushing’s syndrome involves chronic glucocorticoid excess, of which 90% are adrenocorticotropic hormone (ACTH)-dependent and 10% ACTH-independent. Causes of Cushing’s syndrome include Cushing’s disease and iatrogenic causes such as steroids. Signs of Cushing’s syndrome include:

Head and neck:

•	Frontal balding
•	Moon face
•	Acne
•	Gynaecomastia in males
•	Plethoric complexion

Trunk:

•	Kyphosis
•	Buffalo hump
•	Central obesity (due to altered fat distribution)
•	Purple striae on abdomen, breasts, and thighs
•	Thin, fragile skin
•	Predisposition to infection

Limbs:

•	Proximal myopathy
•	Pathological fractures
•	Ankle edema (salt and water retention due to excess cortisol)
•	Hirsutism
•	Hair growth on the forearms in particular

Anaplastic carcinoma of the thyroid is a rare entity, accounting for 1–2% of all thyroid cancers, a very aggressive cancer with poor prognosis. It is not known to cause symptoms relating to Cushing’s syndrome.

A multinodular goiter is associated with hyperthyroidism but not Cushing’s syndrome. A parathyroid adenoma causes excessive secretion of parathyroid hormone (PTH) and results in symptoms relating to hypercalcemia and hypophosphatemia but not Cushing’s syndrome.

Phaeochromocytoma is a rare tumor arising most commonly in the chromaffin cells of the adrenal medulla and resulting in increased secretion of catecholamines. Patients tend to present with severe hypertension (often resistant to multiple therapeutic agents), excessive sweating, anxiety/panic attacks, headaches, and pallor.

How well did you know this?
1
Not at all
2
3
4
5
Perfectly
90
Q

You were asked to do the endocrine surgery in the outpatient clinic by your consultant. On that particular day you only saw patients with endocrine malignancies. Based on your knowledge about the natural history of various endocrine malignancies, which one of the following neoplasms has the best prognosis?

1) Adrenal cortical carcinoma

2) Anaplastic carcinoma of the thyroid

3) Follicular carcinoma of the thyroid

4) Papillary carcinoma of the thyroid

5) Parathyroid carcinoma

A

Explanation

Papillary carcinoma of the thyroid

Papillary carcinoma of the thyroid is the most common form of thyroid cancer accounting for 80% of cases, and of the examples in these answers, is the most common.

It is a well differentiated tumour, most commonly presenting in wom en (female to male ratio of 3:1), aged 35–40. It is slow growing and arises from T4 and thyroglobulin- producing follicular cells. It can metastasise via lymphatics and can be multifocal at diagnosis. It can metastasise to the lungs. It most commonly presents as a solitary thyroid nodule.

Histologically it shows stalks of epithelial cells (papillae) and in 50% of cases psammomabodies, representing calcific collections, are present. Additionally, cells show a characteristic pattern of nuclear inclusions called ‘orphan Annie eye’.

Radiologically it appears as a single mass with irregular borders, found in the subscapular region. Treatment is with thyroidectomy and radioactive iodine administration for positive lymph nodes.

If the tumour is confined to the thyroid gland on diagnosis, it has an excellent prognosis and a 5 year survival rate of 95%.

Adrenal cortical carcinoma

Adrenal cortical carcinoma is a rare cancer and prognosis depends on the extent of tumour spread at diagnosis. If the tumour is confined to the adrenal gland and is treated 5-year survival is 65%. This number drops to 44% for tumours with local invasion to surrounding tissues and 7% for tumour with distant metastases at diagnosis.

Anaplastic carcinoma of the thyroid

Anaplastic carcinoma of the thyroid is a rare entity, accounting for 1–2% of all thyroid cancers. It is however the most aggressive tumour and carries the worst prognosis. It
most commonly affects women and patients tend to present at an older age than with other thyroid cancers, ie in the 6th–7th decade of life. Patients present with a rapidly enlarging, firm, fixed thyroid mass and symptoms of local invasion such as hoarse voice and dyspnoea or stridor. Median patient survival is 8.1 months, as anaplastic carcinoma is poorly responsive to treatment.

Follicular carcinoma of the thyroid

Follicular carcinoma of the thyroid is the second most common malignancy of the thyroid accounting for about 10% of all thyroid cancers. It is a well differentiated cancer, typically presenting in women aged 40–60. These tumours arise from the follicular cells of the thyroid. 5-year survival rate is over 80% for non-invasive disease.

Parathyroid carcinoma

Parathyroid carcinoma is a very rare malignancy, occurring equally in men and women, with men having a worse prognosis. It usually occurs after the age of 30. Disease confined to the parathyroid glands at diagnosis has a 5 -year survival rate of 88.5%.

How well did you know this?
1
Not at all
2
3
4
5
Perfectly
91
Q

A 25-year-old woman was seen in the Emergency Department complaining of vague abdominal pain and some pain in her extremities. On further enquiry she revealed that she had been feeling depressed for several months. Physical examination revealed no major findings. A chest X-ray was normal. Serum biochemistry revealed the following:

Calcium: 3.47 mmol/l (2.20–2.60 mmol/l)

Serum albumin: 38 g/l (35–55 g/l)

Phosphate: 0.64 mmol/l (0.70–1.40 mmol/l)

What is the most likely diagnosis in this patient?

1) Chronic renal failure

2) Metastatic carcinoma

3) Parathyroid adenoma

4) Pituitary adenoma

5) Thyroid carcinoma

A

Explanation

Parathyroid adenoma

The patient has hypercalcaemia and hypophosphataemia, pointing towards the diagnosis of primary parathyroid disease. This patient most likely has primary hyperparathyroidism, the most common cause of which is a single parathyroid adenoma (85%). Other causes include: parathyroid gland hyperplasia affecting all four parathyroid glands (10–15%), multiple parathyroid adenomas (3–5%) or a
parathyroid carcinoma (1%).

The adenoma produces an excess of parathyroid hormone leading to: excessive calcium resorption from bone, increased renal calcium reabsorption, increased renal phosphate excretion and increased production of 1,25-dihydroxyvitamin D3 .

Therefore, blood tests reveal: a raised PTH, hypercalcaemia, hypophosphataemia and a raised vitamin D3 .

Treatment of a parathyroid adenoma is by surgical removal of the parathyroid glands.

For patients not fit/suitable for parathyroidectomy options for treatment include:
-preventing vitamin D deficiency, bisphosphonates to protect the bones, calcimimetics to control hypercalcaemia if required.

Chronic renal failure
Chronic kidney failure leads to secondary hyperparathyroidism. The diseased kidney fails to convert an adequate level of vitamin D to its active form, which reduces the amount of calcium absorbed from the intestines, and fails to excrete phosphate leading to formation of calcium phosphate compounds in the circulation and a reduction in free circulating calcium. Therefore, in secondary hyperparathyroidism there is a low serum concentration of calcium and vitamin D, a high serum concentration of parathyroid hormone and a high serum phosphate.

Metastatic carcinoma

The two most common causes of hypercalcaemia are primary hy perparathyroidism and hypercalcaemia of malignancy. The two are generally quite difficult to differentiate. In both calcium is raised (with hypercalcaemia of malignancy causing a more dramatic hypercalcaemia) and phosphate is low. parathyroid hormone (PTH) in primary
hyperparathyroidism is raised whereas in hypercalcaemia of malignancy it can be low/normal/raised depending on the mechanism involved.

Given the patient’s age and the fact that the symptoms have been ongoing for several months make parathyroid adenoma a far more likely diagnosis than metastatic carcinoma. The most common cancers associated with hypercalcaemia are: breast, lung and myeloma.

Pituitary adenoma

A pituitary adenoma causes visual symptoms (bitemporal hemianopia), headaches and symptoms relating to the cellular origin of the adenoma. It does not commonly cause
hypercalcaemia.

Thyroid carcinoma

Thyroid carcinoma can lead to hypercalcaemia, but this is usually seen with advanced disease.

How well did you know this?
1
Not at all
2
3
4
5
Perfectly
92
Q

A 48-year-old man smoker presented to his GP complaining of a chronic dry cough and backache. The GP noticed truncal obesity, muscular weakness, hypertension, purplish abdominal striae and tenderness in the region of the lower thoracic spine. The patient was taking no medications.

Which one of the following diseases is most likely to be the cause for these findings?

1) 21-Hydroxylase enzyme deficiency

2) Extra-adrenal paraganglioma

3) Multiple endocrine neoplasia, type I

4) Small-cell anaplastic (oat cell) carcinoma

5) Tuberculosis

A

Explanation

Small-cell anaplastic (oat cell) carcinoma

The presenting signs in this scenario point to Cushing’s syndrome, secondary to excess circulating cortisol. Given that the patient is a smoker presenting with a dry cough and
lower back pain, it is most likely that the patient has small-cell lung cancer (also known as oat cell lung cancer) that has metastasised to the bones, and also secretes ectopic ACTH causing Cushing’s syndrome, as part of a paraneoplastic syndrome. This is very common with bronchogenic carcinoma.

Small-cell lung cancer is strongly associated with smoking and is a neuroendocrine tumour arising in the bronchial mucosa. It is fast growing, highly metastatic, rarely operable at diagnosis and associated with a 5-year survival of less than 20%.

CUSHing’s is where Cortisol is GUSHing.

Cushing’s syndrome: chronic glucocorticoid excess of which 90% are ACTH-dependent and 10% ACTH-independent. Causes of Cushing’s syndrome include Cushing’s disease
and iatrogenic causes such as steroids.

Signs of Cushing’s:

Head and neck

Trunk

Limbs

Frontal balding

Kyphosis

Proximal myopathy

Moon face

Buffalo hump

Pathological fractures

Acne

Gynaecomastia in males

Ankle oedema (salt and water rententiondue to excess cortisol) Plethoric complexion

Central obesity (due to altered fat distribution)

Hirsutism

Purple striae on abdomen, breasts and thigs

Thin, fragile skin

Hair growth of the foreaems in particular

Predisposition to infection

21-Hydroxylase enzyme deficiency

21-Hydroxylase enzyme deficiency, also known as congenital adrenal hyperplasia, is a condition commonly presenting at birth or early childhood with ambiguous genitalia.

Extra-adrenal paraganglioma

An extra-adrenal paraganglioma, also known as extra-adrenal phaeochromocytoma, are tumours producing catecholamines, such as adrenaline. The signs the patient presents with in this scenario relate to extra circulating cortisol and not catecholamines.

Multiple endocrine neoplasia, type I

Multiple endocrine neoplasia type I is a combination of parathyroid neoplasia, a pancreatic islet tumour and pituitary tumours. Presentation is most commonly with
symptoms associated with hypercalcaemia.

Other symptoms include: diarrhoea and abdominal pain in gastrinoma, hypoglycaemia

in insulinomas. Depending on the place of the pituitary tumour it can present with

Cushing’s disease due to over secretion of ACTH stimulating cortisol secretion.

MEN 1 is however an autosomal-dominant familial disorder, therefore there must be

some family history, presents usually in teenagers or patients in the 4th decade, but

does not tend to present with a cough.

MEN type I = Primary = pathology beginning with P

Parathyroid neoplasia

Pancreatic islet cell tumour (Zollinger–Ellison syndrome (50%), insulinoma (20%))

Pituitary tumours

Tuberculosis Tuberculosis can present with a cough and back pain but it is not known to cause a

Cushing’s syndrome.

How well did you know this?
1
Not at all
2
3
4
5
Perfectly
93
Q

A 37-year-old woman presents to your Endocrine Clinic with symptoms of anxiety, panic attacks and frequent headaches. Subsequent investigations have revealed her to have a
phaeochromocytoma. Regarding this condition, what is the
first-line diagnostic test?

1) Clonidine suppression test

2) Metaiodobenzylguanidine (MIBG) scintigraphy

3) Magnetic resonance imaging (MRI) scan

4) Plasma noradrenaline

5) 24-hour urinary vanillylmandelic acid (VMA)/homovanillic (HVA) secretion

A

Explanation

24-hour urinary vanillylmandelic acid (VMA)/homovanillic (HVA) secretion

Phaeochromocytomas are rare neuroendocrine tumours arising in the chromaffin cells

most commonly found in the adrenal medulla. They are associated with increased

catecholamine production and tend to present with severe hypertension (often

resistant to multiple therapeutic agents), excessive sweating, anxiety/panic

attacks, headaches andpallor. They are usually curable if detected and treated early. If,

however, they remain undiagnosed, they can lead to severe complications including

death.

First-line investigation for phaeochromocytoma includes a 24 h urine collection and

quantification of urinary vanillylmandelic and homovanillic acids. These are byproducts

of catecholamine production and are present at a low concentration in the urine of

unaffected individuals. In phaeochromocytoma, however, the concentration of these

molecules in the urine is significantly raised, indicating an increased production of

catecholamines.

Phaeochromocytomas are associated with conditions such as: multiple endocrine

neoplasias 2A and 2B, von Hippel–Lindau syndrome, neurofibromatosis.

Clonidine suppression test

Clonidine suppression test forms the second line test in the diagnosis of

phaeochromocytoma in cases where there is a high clinical suspicion but there was

borderline elevation in the catecholamines, not enough to qualify for a definitive

diagnosis.

The test relies on the fact that clonidine is a suppressor of physiological catecholamine

secretion, but has no effect over autonomous catecholamine

secretion, as in phaeochromocytoma. There should be collection of urine from 21:00 on

day 1 to 7:00 on day 2 occurs and catecholamines quantified. Then, on day 2, clonidine

is administered and collection of urine follows from 21:00 to 7:00 of day 3.

Catecholamines are quantified. In a normal individual the urine catecholamines from

the second collection, ie following clonidine administration, should be greatly reduced

when compared with the original collection. In patients with a

phaeochromocytoma, however, clonidine is unable to suppress catecholamine release,

and these remain in high concentrations in the urine. Metaiodobenzylguanidine (MIBG) scintigraphy

MIBG scintigraphy is a very specific test for detecting phaeochromocytoma.

Radioactive iodine-131-metaiodobenzylguanidine is administered to the patient. In a

normal patient, the adrenal medulla appears to uniformly uptake the radioactive

solution and its intensity is less than that of the liver uptake. For a patient with a

phaeochromocytoma there is a focal area in the ad renal medulla where there is very

strong and prolonged uptake of the solution. MIBG scintigraphy is not considered the

first-line investigation to detect phaeochromocytoma. MIBG is useful in detecting extra -

adrenal tumours or disease recurrence.

Magnetic resonance imaging (MRI) scan

Computed tomography (CT) and MRI scans are both sensitive in detecting

phaeochromocytomas, with MRI being superior to CT scans, but are not considered the

first-line investigation. These imaging modalities are used to localise t he tumour once

elevated urine catecholamines are detected.

Plasma noradrenaline

Elevated plasma noradrenaline is a sensitive test in detecting phaeochromocytoma. This

test has a high sensitivity of 96%, but a relatively low specificity of 85%, as plasma

noradrenaline can be elevated in other cases such as in patients

taking amitriptyline and some antipsychotic medication.

How well did you know this?
1
Not at all
2
3
4
5
Perfectly
94
Q

A 19-year-old woman presents to her GP with anxiety, sweating, heat intolerance,

weight loss and a tremor. Blood tests reveal Graves’ disease.

Which one of the following options concerning Graves’ disease is correct?

1) Carbimazole is drug of choice for treatment throughout pregnancy

2) Has equal sex incidence

3) In children has a high relapse rate when treated with antithyroid drugs

4) Is due to immunoglobulin M (IgM) thyroid-stimulating antibodies

5) Is a form of toxic multinodular goitre

A

Explanation

In children has a high relapse rate when treated with antithyroid drugs

About half of the children with Graves’ disease treated with antithyroid drug therapy

have a complete remission, but up to 1 in 3 children suffer a relapse. Features that

increase the risk of relapse include a large goitre, a high radioactive iodine uptake and a

high thyroid-stimulating immunoglobulin titre.

Graves’ disease: also known as Graves’ ophthalmopathy, thyroid-associated

ophthalmopathy, dysthyroid eye disease and infiltrative eye disease. In Graves’

disease the thyroid-stimulating auto-immunoglobulins are of the IgG class.

Carbimazole is drug of choice for treatment throughout pregnancy

Hyperthyroidism in pregnancy affects about 0.2% of the pregnant population (0.1 –

0.4%). The vast majority of these cases is caused by Grave’s disease. Carbimazole is the

medication used as first line in the treatment of Graves’ disease. It has however, been

reported to cause severe congenital defects if used during pregnancy. Therefore,

propylthiouracil is the medication of choice to treat Graves’ disease during the first

trimester of pregnancy. Following the first trimester, carbimazole use can resume for

the remainder of the pregnancy, as propylthiouracil is associated with hepatotoxicity.

Propylthiouracil inhibits the production of thyroid hormones by inhibiting the enzyme

thyroperoxidase.

Has equal sex incidence

Hyperthyroidism caused by Graves’ disease is more common in women, with an female

to male ratio of 7–8:1.

Is due to immunoglobulin M (IgM) thyroid-stimulating antibodies

Graves’ disease is the most common cause of hyperthyroidism and is an autoimmune

condition mediated by IgG thyroid-stimulating antibodies, which bind the thyroid-

stimulating hormone receptor, mimic the function of thyroid-stimulating hormone

leading to thyroid hyperplasia and an overproduction of thyroid hormones.

Is a form of toxic multinodular goitre In Graves’ disease the antibodies stimulating the thyroid -stimulating hormone receptor

lead to hyperplasia of the thyroid follicular cells of the thyroid gland leading to a toxic

diffuse goitre.

How well did you know this?
1
Not at all
2
3
4
5
Perfectly
95
Q

A 63-year-old woman is seen with an enlarging mass in her thyroid and difficulty

swallowing. She is later diagnosed with follicular carcinoma of the thyroid.

Which one of the statements is correct regarding follicular carcinoma of the

thyroid?

1) It can be diagnosed by fine-needle aspiration cytology (FNAC)

2) It can be monitored for recurrence by calcitonin levels

3) The most common presentation is a hoarse voice

4) It has a higher incidence in iodine-deficient regions of the world

5) It is most common at puberty

A

Explanation

It has a higher incidence in iodine-deficient regions of the world

Follicular carcinoma occurs more commonly in the Caucasian population, with an

incidence higher in areas of work where there is a higher incidenc e of iodine deficiency.

It can be diagnosed by fine-needle aspiration cytology (FNAC)

FNAC cannot differentiate between follicular carcinoma and follicular adenoma.

Therefore, histological diagnosis follows a thyroid lobectomy. If follicular carcinoma is

confirmed, a completion thyroidectomy should be performed.

It can be monitored for recurrence by calcitonin levels

In follicular carcinoma, up to 30% of patients may suffer a recurrence, which can occur

decades after the primary disease. Thyroglobulin seru m levels are tested every 6–12

months to detect recurrence. A rise in thyroglobulin levels indicates recurrence.

Calcitonin levels are used to monitor recurrence of medullary carcinoma.

The most common presentation is a hoarse voice

The most common presentation of a follicular carcinoma is a single, unilateral nodule or

a thyroid mass felt on palpation of the neck. Other patients present with symptoms of

difficulty swallowing, a hoarse voice or dyspnoea.

It is most common at puberty

Follicular carcinoma is common in all age groups, with cases reported from age 15 to

  1. It is, however, more common in adults aged 40 –60.
How well did you know this?
1
Not at all
2
3
4
5
Perfectly
96
Q

A 25-year-old woman presents to her general practitioner with a history of episodes of severe headaches and palpitations. On examination, it is noted she has a blood pressure of 180/100 mmHg, and further questioning reveals a family history of multiple endocrine neoplasia (MEN) 2a.

Which of the following conditions is found in MEN 2a?

1) Pancreatic tumour

2) Parathyroid hyperplasia

3) Pituitary tumour

4) Renal cell carcinoma

5) Small cell lung carcinoma

A

Explanation

Parathyroid hyperplasia

MEN is a group of conditions affecting the endocrine organs and presenting with tumours in multiple endocrine organs simultaneously. MEN 2 affects 1 in 35 000 people and, of the two subtypes, the most common is MEN 2a. It is inherited in an autosomal dominant pattern, and therefore, patients have family members who are affected.

MEN 2a is the association of medullary thyroid carcinoma, phaeochromocytoma and parathyroid hyperplasia. MEN 2b is the association of medullary thyroid carcinoma, mucosal neuroma and phaeochromocytoma.

Pancreatic tumour

Pancreatic islet tumours are seen in patients with MEN type 1. Pituitary tumour

A pituitary tumour forms part of MEN syndrome type 1.

Renal cell carcinoma

Renal cell carcinomas are not part of the tumours seen in patients with MEN syndromes.

MEN syndromes are characterised by the formation of tumours in multiple endocrine organs. Though non-endocrine tumours can coexist, they are not part of the description of the syndrome.

Small cell lung carcinoma

Small cell lung carcinoma is not part of the tumours seen in patients with MEN syndromes. MEN syndromes are characterised by the formation of tumours in multiple endocrine organs. Though tumours in non-endocrine organs can coexist, they are not part of the syndrome.

How well did you know this?
1
Not at all
2
3
4
5
Perfectly
97
Q

A 54-year-old woman presents with severe epigastric pain and vomiting. Blood tests show an elevated amylase level, and prior blood tests showed she has had hypercalcaemia which is thought may have precipitated acute pancreatitis.

Investigation of the hypercalcaemia shows she has primary hyperparathyroidism caused by a parathyroid adenoma.

What proportion of cases of primary hyperparathyroidism are caused by adenomas?

1) 1%

2) 10%

3) 30%

4) 5%

5) 85%

A

Explanation

85%

A single parathyroid adenoma is the cause of primary hyperparathyroidism in about 85% of cases.

Less common causes of primary hyperparathyroidism include: parathyroid gland hyperplasia involving all four glands or parathyroid carcinoma.

Excess parathyroid hormone causes hypercalcaemia.

1%

A parathyroid carcinoma can be the cause of primary hyperparathyroidism in about 1% of cases.

10%

In 10–15% of cases of primary hyperparathyroidism, the abnormalities are caused by parathyroid hyperplasia, involving all four parathyroid glands.

30%

Only 30% of patients with primary hyperparathyroidism are symptomatic. The remaining majority are usually asymptomatic, and the abnormalities associated with hyperparathyroidism are detected incidentally.

5%

Three to 5% of cases of primary hyperparathyroidism are due to a double adenoma.

How well did you know this?
1
Not at all
2
3
4
5
Perfectly
98
Q

A farmer presents with his wife as he has noted a lump in his lower neck which has increased in size substantially over the last few months. On further questioning, he admits to some difficulty swallowing and his wife thinks his voice has become hoarser. Investigations reveal anaplastic thyroid carcinoma.
In which of the following age groups is this most common?

1) 10-20

2) 30-40

3) 40-50

4) 60-70

5) 80-90

A

Explanation

60-70

Anaplastic thyroid carcinoma is rare (1–2% of thyroid malignancies) and the most aggressive form of primary thyroid cancer. Peak incidence is seen in adults aged 60–70
and carries a very bad prognosis, with only about 5% survival at 5 years. It is more common in women and is associated with iodine deficiency and a history of a multinodular goitre.

It can metastasise to the bone, brain and lung. Patients present with a rapidly growing neck mass, usually with symptoms of compression such as dyspnoea and dysphagia.
Treatment is usually palliative with doxorubicin and cisplatin but does not affect survival.

10-20

Thyroid cancer is rare in children. The rate of presentation peaks around the ages of

35–39. However, a significant amount of cases have been reported in the literature where a diagnosis of thyroid cancer is made in early adulthood.

30-40

Papillary thyroid carcinoma, which is the most common form of thyroid carcinoma, accounting for about 70% of cases, presents usually in patients aged 35 –40, with a female gender predilection.

40-50

Thyroid lymphomas most commonly affect women aged over 50 and there is an association of these cancers with Hashimoto’s thyroiditis.

80-90

Hurtle cell carcinoma, a rare form of thyroid carcinoma, has been reported in patients from the age of 20–85 years.

How well did you know this?
1
Not at all
2
3
4
5
Perfectly
99
Q

A 37-year-old woman presents with a 2-month history of progressively worsening hypothyroid symptoms. On examination, there is diffuse enlargement of her thyroid
gland, and after testing, she is diagnosed with Hashimoto’s thyroiditis.

In addition to anti-thyroglobulin antibodies, which other autoantibodies would be commonly found in Hashimoto’s thyroiditis?

1) Anti-centromere antibodies

2) Anti-microsomal antibodies

3) Anti-mitochondrial antibodiesc

4) ANCA (cytoplasmic Anti-Neutophil Cystoplasmic antibody)

5) pANCA (perinuclear ANCA)

A

Explanation

Anti-microsomal antibodies

Hashimoto’s thyroiditis is an autoimmune thyroid condition where the presence of

autoantibodies leads to cell destruction in the thyroid, reduced hormone output

and hypothyroidism.

It is the most common cause of hypothyroidism in areas where there is adequate iodine

intake. It is ten times more common in women and tends to present at ages of 30 –50;

however, it can present at any point during a patient’s lifetime.

The most common form of autoantibodies detected in Hashimoto’s thyroiditis is

antibodies against thyroid peroxidase. This is a microsomal component, an essential

enzyme involved in the production of thyroid hormones, and therefore these antibodies

are also referred to as anti-microsomal antibodies. Another group of commonly

detected autoantibodies is anti-thyroglobulin antibodies.

Hashimoto’s thyroiditis is associated with other autoimmune conditions such as

pernicious anaemia and coeliac disease. It also increases the risk for developing

lymphoma of the thyroid at a later stage in life.

Patients tend to present with symptoms (see mnemonic) of insidious onset, gradually

worsening, and with the presence of a diffuse goitre.

Treatment is usually with thyroid hormo ne supplementation. Surgery is not usually

required, unless there are compression symptoms from the goitre. Administration of

steroids can assist in shrinking of the goitre.

Anti-centromere antibodies

Anti-centromere antibodies are found in limited sclerod erma disease (CREST

syndrome).

Anti-mitochondrial antibodies

Anti-mitochondrial antibodies are associated with primary biliary sclerosis, an

autoimmune condition causing liver scarring.

cANCA (cytoplasmic Anti-Neutophil Cystoplasmic antibody)

c-ANCA is an autoantibody directed against proteinase 3 and is found in Wegener’s

granulomatosis. pANCA (perinuclear ANCA)

p-ANCA is an autoantibody directed against myeloperoxidase and is associated with

Churg–Strauss syndrome.

How well did you know this?
1
Not at all
2
3
4
5
Perfectly
100
Q

A 33-year-old woman has been listed for a laparoscopic adrenalectomy after being diagnosed with phaeochromocytoma due to episodic palpitations, hyperhidrosis and hypertension. During preoperative investigations, it is discovered she has multiple endocrine neoplasia 2a, and further investigation shows a medullary thyroid carcinoma.

What proportion of medullary thyroid carcinoma is familial ?

1) 1%

2) 5%

3) 20%

4) 50%

5) 75%

A

Explanation

20%

Medullary thyroid carcinoma is a malignancy of parafollicular C cells, of neural ectodermal origin, accounting for about 5% of total primary thyroid malignancies.

Medullary thyroid carcinoma is sporadic in 80% of the cases and familial in 20%. The familial form usually presents as part of the multiple endocrine neoplasia 2 syndrome (both a and b). The tumour is usually found in the upper two -thirds of the thyroid gland and can produce a variety of hormones such as calcitonin, prostaglandins and adrenocorticotrophic hormone (ACTH). Treatment is with thyroidectomy, with lymph node clearance as required.

The patient in this scenario has medullary carcinoma of the thyroid, presenting as part of multiple endocrine neoplasia type 2a.

1%

Medullary thyroid carcinoma can present with a thyroid nodule and local lymphadenopathy. It is important to note that metastatic medullary thyroid carcinoma presents with diarrhoea and flushes.

5%

Medullary thyroid carcinoma, a malignancy arising from parafollicular C cells , accounts for 5–8% of total primary thyroid malignancy cases.

50%

If a patient presents with metastatic medullary thyroid carcinoma that has spread to other parts of the body, the 10-year survival rate is < 50%, more specifically 20–40%.

75%

Up to 75% of medullary thyroid carcinoma are sporadic and therefore occur in patients with no family history.

How well did you know this?
1
Not at all
2
3
4
5
Perfectly
101
Q

A 54-year-old woman has undergone some blood tests, as part of an employment health screen. She reports she is in good health and, being very health-conscious, takes regular
vitamin and mineral supplements. She is taking bendroflumethiazide 2.5 mg for hypertension, and her blood pressure is 132/82 mmHg. The only abnormality is a serum calcium concentration of 2.94 mmol/l (normal 2.20–2.60 mmol/l).

Which of the following is the most likely cause?

1) Diuretic treatment

2) High dietary calcium intake

3) High dietary vitamin D intake

4) Occult malignancy

5) Primary hyperparathyroidism

A

Explanation

Primary hyperparathyroidism

Given the fact that the patient is asymptomatic, the most likely diagnosis is hypercalcaemia secondary to primary hyperparathyroidism. Primary hyperparathyroidism is, in the majority of cases (85%), the result of a single parathyroid adenoma. Other causes include: parathyroid gland hyperplasia affecting all four parathyroid glands (10–15%), multiple parathyroid adenomas (3–5%) or a parathyroid carcinoma (1%).

The adenoma produces an excess of parathyroid hormone, leading to: excessive calcium resorption from bone, increased renal calcium reabsorption, increased renal phosphate excretion and increased production of 1,25-dihydroxyvitamin D3 .

Therefore, blood tests reveal: a raised parathyroid hormone level, hypercalcaemia, hypophosphataemia and a raised vitamin D 3 level.

Treatment of a parathyroid adenoma is by surgical removal of the parathyroid glands.

For patients not fit/suitable for parathyroidectomy, options for treatment include: preventing vitamin D deficiency, bisphosphonates to protect the bones and calcimimetics to control hypercalcaemia if required.

Diuretic treatment

Thiazide diuretics, such as bendroflumethiazide mentioned in the above scenario, block the thiazide-sensitive Na + /Cl – cotransporter found on the apical membrane of the distal
convoluted tubules and therefore inhibit sodium and chloride reabsorption.

Common side-effects of thiazide diuretics include: hyponatraemia, hypokalaemia, hypercalcaemia, hyperglycaemia, hyperuricaemia, gout, postural hypotension and hypochloraemic alkalosis.

Hypercalcaemia caused by thiazide diuretics, however, is usually mild, and therefore unlikely to be the cause of the raised calcium concentration in this scenario.

High dietary calcium intake

Calcium absorption in the bowel is tightly regulated, and an increased dietary intake of calcium is not thought to be a cause of hypercalcaemia. High dietary vitamin D intake

Hypervitaminosis D, ie excess vitamin D intake, can lead to hypercalcaemia. It is usually the result of taking a large amount of vitamin D supplements. In this case , the patient is
only taking bendroflumethiazide and standard vitamin supplementation.

Occult malignancy

The two most common causes of hypercalcaemia are primary hyperparathyroidism and malignancy. Given the patient in this scenario is asymptomatic, it is more likely that the hypercalcaemia is the result of primary hyperparathyroidism.

How well did you know this?
1
Not at all
2
3
4
5
Perfectly
102
Q

A 42-year-old man with a past history of parathyroid adenoma presents with a mass in the neck and enlarged cervical lymph nodes. Fine-needle aspiration cytology confirms a diagnosis of medullary carcinoma of the thyroid.

Which one of the following forms of familial endocrine disease is this most likely to represent?

1) Cowden syndrome

2) Familial medullary carcinoma of the thyroid

3) Multiple endocrine neoplasia type 1

4) Multiple endocrine neoplasia type 2a

5) Multiple endocrine neoplasia type 2b

A

Explanation

Multiple endocrine neoplasia type 2a

This patient with a history of parathyroid adenoma presents with a medullary carcinoma of the thyroid. This represents an example of multiple endocrine neoplasia type 2a. This is described as the combination of: medullary thyroid carcinoma, phaeochromocytoma and hyperparathyroidism.

Cowden syndrome

Cowden syndrome is a very rare autosomal dominant condition, also referred to as multiple hamartoma syndrome, associated with the presence of multiple non -cancerous
growths called hamartomas. It increases the susceptibility of individuals affected to develop some forms of cancers such as thyroid, endometrial, colorectal and renal cancers and melanoma.

Familial medullary carcinoma of the thyroid

Familial medullary carcinoma of the thyroid is a subtype of multiple endocrine neoplasia type 2 where patients dev elop medullary carcinoma of the thyroid but have a lower probability of having a parathyroid adenoma or a phaeochromocytoma. This accounts for 10–20% of all multiple endocrine neoplasia type 2 cases.

Multiple endocrine neoplasia type 1

Multiple endocrine neoplasia type I describes the combination of parathyroid hyperplasia, pancreatic islet cell tumours and pituitary tumours.

Multiple endocrine neoplasia type 2b

Multiple endocrine neoplasia type 2b describes the combination of a medullary thyroid carcinoma, mucosal neuromas, phaeochromocytoma and marfanoid body habitus.

How well did you know this?
1
Not at all
2
3
4
5
Perfectly
103
Q

A woman patient presents with a moderately enlarged thyroid enlargement is symmetrical with a rubbery texture. She is euthyroid middle-aged woman gland. The Anti-thyroglobulin and anti-thyroid peroxidase/anti-microsomal antibodies are raised.

Which one of the following is the most likely diagnosis?

1) Acute suppurative thyroiditis

2) Autoimmune thyroiditis (Hashimoto’s thyroiditis)

3) Graves’ disease

4) Riedel’s thyroiditis

5) Subacute thyroiditis (De Quervain’s thyroiditis)

A

Explanation

Autoimmune thyroiditis (Hashimoto’s thyroiditis)

Hashimoto’s thyroiditis is an autoimmune thyroid condition where the presence of autoantibodies leads to cell destruction in the thyroid, reduced hormone output and hypothyroidism. It is the most common cause of hypothyroidism in areas where there is adequate iodine intake. It is ten times more common in women and tends to
present at ages 30–50; however, it can present at any point during a patient’s lifetime.

The most common form of autoantibodies detected in Hashimoto’s thyroiditis is antibodies against thyroid peroxidase. This is a microsomal component, an essential
enzyme involved in the production of thyroid hormones, and therefore these antibodies are also referred to as anti-microsomal antibodies. Another group of commonly detected autoantibodies is anti-thyroglobulin antibodies. Patients tend to present with symptoms of insidious onset, gradually worsening, and with the presence of a goitre. It
is a lymphocytic infiltration of the gland, with subsequent fibrosis, leading to diffuse enlargement. The goitre associated with Hashimoto’s thyroiditis is symmetrical,
rubbery in texture and painless. Hashimoto’s thyroiditis is associated with other autoimmune conditions such as pernicious anaemia and coeliac disease. It also increases the risk for developing lymphoma of the thyroid at a later stage in life.

Treatment is usually with thyroid hormone supplementation. Surgery is not usually required, unless there are compression symptoms from the goitre. Administration of steroids can assist in shrinking of the goitre.

Acute suppurative thyroiditis

Acute suppurative thyroiditis is a rare condition caused by bacterial infection of the thyroid gland that tends to affect patients with anatomical defects or concurrent thyroid
disease.

Graves’ disease

Graves’ disease presents with a toxic, diffuse goitre and is associated with thyroid - stimulating antibodies that bind to the thyroid -stimulating hormone receptors.

Riedel’s thyroiditis Riedel’s thyroiditis is a rare condition. It describes the chronic fibrosis of the thyroid gland, resulting in a rapidly growing, painful, hard and fixed thyroid mass. Fibrosis can extend outside the thyroid gland to surrounding structures.

Subacute thyroiditis (De Quervain’s thyroiditis)

De Quervain’s thyroiditis (or subacute thyroiditis) is usually viral in origin, and patients report a preceding viral illness such as mumps, measles, influenza, etc.

How well did you know this?
1
Not at all
2
3
4
5
Perfectly
104
Q

A 56-year-old woman presents with abdominal pain and constipation. She has no past medical history. Her blood tests show the following.

Result

Hb 15.3 g/dl. 11.5–15.5 g/dl

MCV. 95 fl. 76–98 fl

WCC. 10.3 x 10 9 /l. 4–11 x 10 9 /l

Serum corrected calcium. 2.95 mmol/l. 2.20–2.60 mmol/l

Serum phosphate. 0.7 mmol/l. 0.7–1.4 mmol/l

Serum alkaline phosphatase. 150 iu/l. 30–130 iu/l

What is the underlying diagnosis?

1) Primary hyperparathyroidism

2) Secondary hyperparathyroidism

3) Tertiary hyperparathyroidism

4) Osteoporosis

5) Paget’s disease

A

Explanation

Primary hyperparathyroidism

The most common causes of hypercalcaemia are primary hyperparathyroidism and malignancy. In this case, given the patient has mild hypercalcaemia, it is more likely that this is due to primary hyperparathyroidism.

Most primary hyperparathyroidism (85%) is caused by the presence of a single parathyroid adenoma. The adenoma produces an excess of parathyroid hormone (PTH), leading to: excessive calcium resorption from bone, increased renal calcium reabsorption, increased renal phosphate excretion and increased production of 1,25-dihydroxyvitamin D3 .

Therefore, blood tests reveal: a raised PTH level, hypercalcaemia, hypophosphataemia and a raised vitamin D 3 level. A mildly raised alkaline phosphatase level is also seen in primary hyperparathyroidism.

Treatment of a parathyroid adenoma is by surgical removal of the parathyroid glands. For patients not fit/suitable for parathyroidectomy, options for treatment include:
preventing vitamin D deficiency, bisphosphonates to protect the bones and calcimimetics to control hypercalcaemia if required.

Secondary hyperparathyroidism

In chronic renal disease, there is a reduction in the secretion of phosphate, with reduced estimated glomerular filtration rate (eGFR). The resulting hyperphosphataemia
stimulates PTH secretion and causes formation of calcium and phosphate crystals, leading to hypocalcaemia.

Therefore, in secondary hyperparathyroidism, biochemical findings include: raised PTH, low calcium, raised phosphate and raised alkaline phosphatase levels.

Tertiary hyperparathyroidism

Following a long-standing period of secondary hyperparathyroidism or following a renal transplant, tertiary hyperparathyroidism can arise. Following a period of
parathyroid gland stimulation, the parathyroid glands continue to secrete PTH autonomously and result in: raised PTH levels, hypercalcaemia and hyperphosphataemia.

Osteoporosis

Osteoporosis leaves calcium, phosphate and PTH levels unaffected. Alkaline phosphatase is normal.

Paget’s disease

Paget’s disease of the bone is associated with excessive bone breakdown and formation, with subsequent disorganised bone remodelling. This results in weakened bones
associated with pain and fractures.

In Paget’s disease, calcium, phosphate and PTH levels remain unaffected. Alkaline phosphatase level is markedly raised in Paget’s disease.

How well did you know this?
1
Not at all
2
3
4
5
Perfectly
105
Q

You are in the General Surgery Clinic and the general practitioner has referred a man with a breast lump. You examine the man and find he has gynaecomastia, with no
palpable mass of any concern.

Which of the following is a cause for this?

1) Liver cirrhosis

2) Low-dose, occasional cimetidine

3) Oral corticosteroids

4) Parathyroid gland tumours

5) Regular bromocriptine therapy

A

Explanation

Liver cirrhosis

Gynaecomastia, defined as ‘benign proliferation of male breast tissue’, is caused by a disruption in the oestrogen:testosterone ratio, in favour of oestrogen. It can be secondary to some medication therapy, most commonly spironolactone.

In liver disease, there is an increased activity of aromatase enzyme, leading to conversion of androgens to oestrogens. Additionally, the resulting high oestrogenic state increases the production of sex hormone-binding globulin (SHBG) in the liver, leading to a further decrease in serum free testosterone levels.

This tip in the balance in favour of oestrogen leads to gynaecomastia, as the oestrogens induce ductal hyperplasia and proliferation of breast tissue. Patients with alcoholic liver disease are at a higher risk for gynaecomastia because
phyto-oestrogens found in alcohol are direct inhibitors of testosterone.

Low-dose, occasional cimetidine

Cimetidine is a histamine receptor blocker used in the treatment of gastro-oesophageal reflux disease and in the prevention of gastric ulcers. Common side-effects of cimetidine include dizziness, drowsiness, diarrhoea and headache. It can rarely cause gynaecomastia or a breast lump, as it affects the cytochrome P450system in the liver and leads to raised serum oestrogens. This can lead to
gynaecomastia but is dose-dependent and seen in high doses and prolonged treatment.

Oral corticosteroids

Oral corticosteroids can lead to fat redistribution but are not known to cause gynaecomastia.

Parathyroid gland tumours

Parathyroid gland tumours present with signs relating to hypercalcaemia such as polyuria, polydipsia, renal stones, constipation and mental disturbances (depression).

Regular bromocriptine therapy Bromocriptine is an ergot derivative and a potent dopamine receptor agonist. It is used
in the treatment of Parkinson’s disease and also to suppress lactation in the treatment of galactorrhoea or in women following an intrauterine fetal death, by inhibiting secretion of prolactin. Known side-effects include: nausea, vomiting, constipation, hypotension, confusion and lethargy. It is not known to cause gynaecomastia.

How well did you know this?
1
Not at all
2
3
4
5
Perfectly
106
Q

A 42-year-old woman is referred to the Endocrine Clinic, as she was noted by the general practitioner to have a persistently raised blood pressure which has not responded to medication. She complains of heat intolerance, sweating and palpitations. She also reports a family history of phaeochromocytoma.

Which of the following options would be a good first-line investigation for phaeochromocytoma?

1) Clonidine suppression test

2) MIBG scintigraphy

3) Magnetic resonance imaging (MRI)

4) Plasma noradrenaline

5) 24-hour urinary vanillylmandelic acid (VMA)/homovanillic acid (HVA) secretion

A

Explanation

24-hour urinary vanillylmandelic acid (VMA)/homovanillic acid (HVA) secretion

Phaeochromocytomas are rare neuroendocrine tumours arising in the chromaffin cells,

most commonly found in the adrenal medulla. They are associated with increased

catecholamine production and tend to present with severe hypertension (often

resistant to multiple therapeutic agents), excessive sweating, anxiety/panic attacks,

headaches and pallor. They are usually curable if detected and treated appropriately. If,

however, they remain undiagnosed, they can lead to severe complications, including

death.

The first-line investigation for phaeochromocytomas includes a 24 -hour urine collection

and quantification of urinary VMA and HVA. These are by-products of catecholamine

production and are present in low concentrations in the urine of unaffected individuals.

In phaeochromocytoma, however, the concentration of these molecules in the urine is

raised, indicating an increased production of catecholam ines.

Phaeochromocytomas are associated with conditions such as multiple endocrine

neoplasia types 2a and 2b, von Hippel–Lindau syndrome and neurofibromatosis.

Clonidine suppression test

The clonidine suppression test forms the second -line test in the diagnosis of

phaeochromocytoma in cases where there is a high clinical suspicion but there is

borderline elevation in the catecholamines, not enough to qualify for a definitive

diagnosis.

The test relies on the fact that clonidine is a suppressor of physiologi cal catecholamine

secretion but has no effect over autonomous catecholamine secretion, like in the case of

phaeochromocytoma.

Collection of urine from 2100 h on day 1 to 0700 h on day 2 occurs and catecholamines

quantified. Then, on day 2, clonidine is administered and collection of urine follows

from 2100 h to 0700 h on day 3. Catecholamines are quantified. In a normal individual,

the urine catecholamines from the second collection, ie following clonidine

administration, should be greatly reduced when comp ared to the original collection. In patients with phaeochromocytoma, however, clonidine is unable to suppress

catecholamine release, and these remain in high concentrations in the urine.

MIBG scintigraphy

MIBG scintigraphy is a very specific test for detecting phaeochromocytoma. Radioactive

iodine-131-metaiodobenzylguanidine is administered to the patient. In a normal

patient, the adrenal medulla appears to uniformly uptake the radioactive solution and

its intensity is less than that of liver uptake. In the case of a patient with

phaeochromocytoma, there is a focal area in the adrenal medulla where there is very

strong and prolonged uptake of the solution. MIBG scintigraphy is not considered the

first-line investigation to detect phaeochromocytoma. MIBG is u seful in detecting extra-

adrenal tumours or disease recurrence.

Magnetic resonance imaging (MRI)

Computerised tomography (CT) and MRI scans are both sensitive in detecting

phaeochromocytomas, with MRI being superior to CT, but are not considered the first -

line investigation. These imaging modalities are used to localise the tumour once

elevated urine catecholamines are detected.

Plasma noradrenaline

Elevated plasma noradrenaline is a sensitive test in detecting phaeochromocytoma. This

test has a high sensitivity of 96%, but a relatively low specificity of 85%, as plasma

noradrenaline can be elevated in other cases such as in patients taking amitriptyline

and some antipsychotic medication.

How well did you know this?
1
Not at all
2
3
4
5
Perfectly
107
Q

A 60-year-old man is 2 weeks post-renal transplant, complaining of polyuria and polydypsia. His blood tests show the following:

Result

Hb. 10.1 g/dl. (13-18 g/dl)

MCV. 80 fl. (76–98 fl)

Ca. 2.95 mmol/l. (2.20–2.60 mmol/l)

Phosphate. 1.8 mmol/l. (0.7–1.4 mmol/l)

Alkaline phosphatase (ALP). 150 iu/l. (30–130 iu/l)

What is the underlying problem?

1) Primary hyperparathyroidism

2) Secondary hyperparathyroidism

3) Tertiary hyperparathyroidism

4) Osteoporosis

5) Paget’s disease

A

Explanation

Tertiary hyperparathyroidism

The patient presents with symptoms relating to hypercalcaemia that is caused by

tertiary hyperparathyroidism. Following a long -standing period of secondary

hyperparathyroidism, such as that occurring during renal failure, tertiary

hyperparathyroidism can arise. This is commonly seen following renal transplantation

to treat renal failure. Due to the prolonged stimulation that has occurred in the

parathyroid glands, in response to the relative serum hypocalcaemia caused by renal

failure, the glands continue to secrete PTH autonomously and inappropriately, even

after the correction of hypocalcaemia. As in the above scenario, this results in: raised

PTH levels, hypercalcaemia and hyperphosphataemia. Alkaline phosphatase level is

mildly raised in all forms of hyperparathyroidism but can also be normal.

Primary hyperparathyroidism

Primary hyperparathyroidism is most commonly due to a parathyroid adenoma and

results in raised parathyroid hormone (PTH) levels, hypercalcaemia (usually mild to

moderate), hypophosphataemia and raised vitamin D 3 levels. A mildly raised alkaline

phosphatase level is also seen in primary hyperparathyroidism. Secondary hyperparathyroidism

In chronic renal disease, there is a reduction in the secretion of phosphate, with reduced

estimated glomerular filtration rate (eGFR). The resulting hyperphosphataemia

stimulates PTH secretion and causes the formation of calcium and phosphate crystals,

leading to hypocalcaemia.

Therefore, in secondary hyperparathyroidism, biochemical findings include: raised PTH,

low calcium, raised phosphate and raised alkaline phosphatase levels.

Osteoporosis

Osteoporosis leaves calcium, phosphate and PTH levels unaffected. Alkaline

phosphatase level is normal.

Paget’s disease

Paget’s disease of the bone is associated with excessive bone breakdown and formation,

with subsequent disorganised bone remodelling. This results in weakened bones

associated with pain and fractures.

In Paget’s disease, calcium, phosphate and PTH levels remain unaffected. Alkaline

phosphatase level is markedly raised in Paget’s disease.

How well did you know this?
1
Not at all
2
3
4
5
Perfectly
108
Q

A patient is admitted to A&E with deranged biochemistry. Which of the following malignancies could be responsible for hypercalcaemia and low serum phosphate levels?

1.	Osteoclastoma
2.	Squamous cell carcinoma of the lung
3.	Prostate cancer
4.	Transitional cell carcinoma of the bladder
5.	Basal cell carcinoma
A

Squamous cell carcinoma of the lung is responsible for hypercalcaemia and low serum phosphate levels. Ten percent of all lung cancers produce hormones or peptides involved in paraneoplastic syndromes.

One of the most commonly encountered paraneoplastic syndromes is humoral hypercalcaemia of malignancy, seen with squamous cell carcinoma of the lung. This cancer produces parathyroid hormone-related peptide, which mimics the action of parathyroid hormone.

Consequently, it causes hypercalcaemia and hypophosphataemia, as parathyroid hormone stimulates calcium resorption from bone, renal calcium reabsorption, renal phosphate excretion, and increased production of 1,25-dihydroxyvitamin D3.

Osteoclastoma is normally a benign primary bone tumor and does not typically cause hypercalcaemia and hypophosphataemia.

Prostate cancer can metastasize to the bones and lead to the release of calcium and phosphate into the bloodstream, resulting in hypercalcaemia and hyperphosphataemia.

Transitional cell carcinoma of the bladder may metastasize to the bones. Metastatic cancerous bone lesions can result in the release of mineralized calcium and phosphate into the bloodstream, leading to hypercalcaemia and hyperphosphataemia.

Basal cell carcinoma rarely associates with hypercalcaemia. Hypercalcaemia is more commonly associated with squamous cell carcinoma of the skin and melanoma, but not with basal cell carcinoma.

How well did you know this?
1
Not at all
2
3
4
5
Perfectly
109
Q

A 70-year-old man complains of constantly feeling cold and lethargic. He presents to the outpatient clinic referred by the GP as possible malignancy. What is the most likely hormonal deficiency to account for this?

1.	Somatostatin
2.	Cholecystokinin
3.	Testosterone
4.	Thyroxine
5.	Insulin
A

Thyroxine deficiency, or hypothyroidism, is most likely to be affecting the patient in this scenario. It can be either primary (due to the thyroid gland not producing enough thyroxine) or secondary (where the thyroid gland is normal but does not receive enough TSH for stimulation of hormone production, suggesting a pituitary problem). Thyroxine is involved in the stimulation of cell metabolism and thermoregulation. Deficiency presents with a variety of symptoms relating to metabolic and mental slowness. If hypothyroidism is confirmed, it is treated with hormone replacement with a synthetic T4 molecule and monitored by regular TSH serum levels.

Somatostatin is a peptide hormone produced in the delta cells of the islets of Langerhans in the pancreas and the ventromedial nucleus of the hypothalamus. It inhibits the production of glucagon and insulin in the pancreas and the secretion of gastric acid, gastrin, and secretin. In the pituitary, it inhibits the secretion of multiple hormones such as growth hormone and thyroid-stimulating hormone (TSH). Somatostatin overproduction can result in diabetes, gallstones, fat intolerance in the diet, steatorrhea, and diarrhea. Somatostatin deficiency is very rare; only a few cases have been reported in the literature. However, decreased cerebrospinal fluid levels of somatostatin have been documented in patients with Alzheimer’s disease.

Cholecystokinin is a hormone produced by the I cells of the duodenum, which has multiple functions. It is released in response to the presence of fat in the duodenum and inhibits gastric emptying, promotes pancreatic hormone secretion, and promotes contraction of the gall bladder and relaxation of the sphincter of Oddi. Deficiency of cholecystokinin is extremely rare and usually manifests as part of a polyglandular syndrome, resulting in malabsorption.

Testosterone deficiency presents with loss of libido, erectile dysfunction, low semen volume, fatigue, hair loss, decrease in bone and muscle mass, with a corresponding increase in body fat, as well as mood changes.

Insulin deficiency presents with features of diabetes mellitus such as polyuria, polydipsia, increased thirst, and loss of weight.

Remember, hypothyroidism is ten times more common in women than men and occurs mainly in middle life.

How well did you know this?
1
Not at all
2
3
4
5
Perfectly
110
Q

A 41-year-old woman presents with renal colic. Biochemistry results reveal a plasma calcium level of 3.3 mmol/l (2.20–2.60 mmol/l), with an albumin level of 45 g/l (35 –55 g/l). Her medical history includes dyspepsia and depression.

What is the most likely underlying diagnosis?

1) Actinomycosis infection

2) Ectopic parathyroid hormone

3) Hyperparathyroidism

4) Hypoparathyroidism

5) Vitamin D deficiency

A

Explanation

Hyperparathyroidism

The patient presents symptomatic of hypercalcaemia with a renal stone, dyspepsia and depression. Given the patient is young and fit, the most likely cause of the hypercalcaemia seen in this scenario is primary hyperparathyroidism. However, as hypercalcaemia is commonly due to malignancy, this needs to be excluded.

Primary hyperparathyroidism is, in the majority of cases (85%), the result of a single parathyroid adenoma. Other causes include: parathyroid gland hyperplasia affecting all
four parathyroid glands (10–15%), multiple parathyroid adenomas (3–5%) or a parathyroid carcinoma (1%).

The adenoma produces an excess of parathyroid hormone, leading to excessive calcium resorption from bone, increased renal calcium reabsorption, increased renal phosphate
excretion and increased production of 1,25-dihydroxyvitamin D3 .

Therefore, blood tests reveal: a raised parathyroid hormone level, hypercalcaemia, hypophosphataemia and a raised vitamin D 3 level.

Treatment of a parathyroid adenoma is by surgical removal of the parathyroid glands (parathyroidectomy). For patients not fit/suitable for parathyroidectomy, options for treatment include: preventing vitamin D deficiency, bisphosphonates to protect the bones and calcimimetics to control hypercalcaemia if required.

Actinomycosis infection

Actinomycosis infection is a bacterial infection characterised by a suppurative granulomatous inflammation and formation of multiple abscesses, discharging sulfur - containing granules. It commonly affects the thorax, abdomen or face and neck (cervicofacial). Abdominal actinomycosis presents with sensation of a mass, fever,
weight loss, fatigue, nausea and vomiting. Blood biochemistry is usually normal, with the exception of a raised alkaline phosphatase level in the case of hepatic actinomycosis.

Ectopic parathyroid hormone

Ectopic parathyroid hormone secretion is usually secondary to malignancy as part of a paraneoplastic syndrome. Even though hypercalcaemia of malign ancy is a common cause of hypercalcaemia and needs to be excluded in this case, given the patient is young and fit, the most likely cause of the biochemical picture seen is primary hyperparathyroidism. Hypoparathyroidism

Hypoparathyroidism results in parathyroid hormone deficiency and subsequently in hypocalcaemia.

Vitamin D deficiency

Vitamin D deficiency would result in hypocalcaemia, as vitamin D is essential for the absorption of dietary calcium in the gastrointestinal system.

How well did you know this?
1
Not at all
2
3
4
5
Perfectly
111
Q

A patient is seen in the clinic with exopthalmous.

Which of the following thyroid function tests (TFTs) is suggestive of Graves’

disease?

1) Raised thyroid-stimulating hormone (TSH), raised free T4, raised free T3

2) Normal TSH, raised free T4, decreased T3

3) Decreased TSH, raised free T4, raised free T3

4) Decreased TSH, decreased free T4, decreased free T3

5) Raised TSH, normal free T4, normal free T3

A

Explanation

Decreased TSH, raised free T4, raised free T3

Graves’ disease is an autoimmune condition leading to a diffuse enlargement of the

thyroid, hyperstimulation and increased production of thyroid hormones. In Graves’

disease, there are thyrotropin receptor antibodies (immunoglobulin G (IgG) antibodies)

circulating in the blood, stimulating the TSH receptor and promoting the production of

thyroid hormones.

There is therefore increased free T3 and T4; this acts as a negative feedback on the

pituitary, suppressing the production of endogenous TSH, which is why, in Graves’

disease, TSH is low.

Raised thyroid-stimulating hormone (TSH), raised free T4, raised free T3

This can be seen in the presence of a TSH-secreting pituitary tumour, amiodarone

therapy or acute psychiatric illness.

Normal TSH, raised free T4, decreased T3

Beta blocker therapy interferes with the conversion of T4 to T3 and can therefore cause

the above picture.

Decreased TSH, decreased free T4, decreased free T3

This clinical scenario describes patients who have recently been treated for

hyperthyroidism or patients with congenital TSH or thyrotropin-releasing hormone

deficiency.

Raised TSH, normal free T4, normal free T3

This is the presentation of subclinical hypothyroidism.

How well did you know this?
1
Not at all
2
3
4
5
Perfectly
112
Q

A 49-year-old woman presents to the Outpatient Clinic with a corrected calcium level of 1.9 mmol/l. Clinical examination is unremarkable.

What is the most likely cause for her clinical problem?

1) Acute renal failure

2) Multiple endocrine neoplasia type 1 (MEN 1)

3) Parathyroid adenoma

4) Pregnancy

5) Vitamin D deficiency

A

Explanation

Vitamin D deficiency

Vitamin D is essential for the absorption of calcium in the gastrointestinal tract.

Therefore, a deficiency in vitamin D can cause reduced absorbed calcium from the diet and lead to hypocalcaemia.

Secondary hyperparathyroidism is a condition that results in stimulation of PTH secretion due to a reduction in serum calcium concentration. It is most commonly seen in chronic renal failure where there is reduced vitamin D activation and reduced phosphate excretion. The result is a reduction in calcium absorption from the gastrointestinal tract, but also binding of the free calcium to phosphate, causing the
formation of calcium phosphate compounds. These both lead to hypocalcaemia and stimulation of the parathyroid glands to produce PTH, in an effort to increase calcium
concentration in the serum.

Acute renal failure

Hypocalcaemia is often seen with chronic renal failure, as part of secondary hyperparathyroidism.

Multiple endocrine neoplasia type 1 (MEN 1)

MEN 1 is associated with a parathyroid adenoma, leading to hypercalcaemia through the mechanism of primary hyperparathyroidism, a pancreatic islet cell tumour and a
pituitary tumour.

Parathyroid adenoma

A parathyroid adenoma leads to primary hyperparathyroidism, through excess

secretion of parathyroid hormone (PTH) from the adenoma. Biochemical features of

primary hyperparathyroidism include: hypercalcaemia, hypophosphataemia, raised

PTH level and raised vitamin D 3 level. There is sometimes raised alkaline phosphatase

level as well.

Pregnancy

Pregnancy is a cause of primary hyperparathyroidism, resulting from hyperplasia of the

parathyroid glands.

How well did you know this?
1
Not at all
2
3
4
5
Perfectly
113
Q

A 23-year-old woman is commenced on

long-term

prednisolone for adrenal

insufficiency. She is currently otherwise fairly fit and well

On which of the side-effects should you counsel her?

1) Avascular necrosis of bone

2) Hepatotoxicity

3) Hyperkalaemia

4) Bone marrow suppression

5) Hypotension

A

Explanation

Avascular necrosis of bone

Avascular necrosis of bone and osteoporosis are well -recognised side-effects of long-

term corticosteroid therapy, irrespective of its mode of administration.

This is a multifactorial process, characterised by interruption of the vascular supply to

the bone, leading to marrow, medullary bone and cortex necrosis. Most commonly,

affected bones are those with a terminal blood supply, and necrosis occurs in the

epiphysis. These include the femur, humerus, carpals and talus.

Factors that increase the risk include: female gender, high daily dose and prolonged

therapy. It is thought that concurrent autoimmune disease also increases the risk of

developing avascular necrosis with corticosteroid therapy.

Hepatotoxicity

Glucocorticoid therapy is not known to have toxic effects on the liver.

Hyperkalaemia

Glucocorticoids exert several effects on the kidneys. They increase water diuresis and

glomerular filtration rate, while causing hypernatraemia and hypokalaemia, due to an

increase in sodium retention and potassium secretion.

Bone marrow suppression

Initiating corticosteroid therapy can cause a refl ex leukocytosis, predominantly

affecting polymorphonuclear leukocytes, ie neutrophils.

Prolonged therapy can lead to suppression of the immune system and increase the risk

for infection. Steroid therapy is not, however, a common cause of bone marrow

suppression, with common causes being cytotoxic chemotherapy or

immunosuppressants such as azathioprine.

Hypotension

Glucocorticoid therapy is associated with hypertension, and not hypotension. Steroids

stimulate the mineralocorticoid receptor in the kidney, resulting in sodium and water

resorption, increase in intravascular volume and therefore increase in blood pressure.

How well did you know this?
1
Not at all
2
3
4
5
Perfectly
114
Q

A 44-year-old woman is seen in the pre-assessment clinic before inguinal hernia repair. Her chest radiograph is normal and her blood results are as follows:

Ca. 3.0 mmol/l. 2.20–2.60 mmol/l

Na. 136 mmol/l. 135–145 mmol/l

K. 3.7 mmol/l. 3.5–5.0 mmol/l

Urea. 5.5 mmol/l. 2.5–6.5 mmol/l

Cr. 50 μmol/l. 50–120 μmol/l

What is the most likely cause of the hypercalcaemia?

1) Addison’s disease

2) Chronic renal failure

3) Diabetes mellitus

4) Sarcoidosis

5) Thyrotoxicosis

A

Explanation

Thyrotoxicosis

Thyrotoxicosis is associated with hypercalcaemia. Thyroid hormones directly stimulate bone resorption leading to decalcification and demineralisation leading to an increase in
serum calcium. This mechanism is not associated with hyperparathyroidism. However, there is a significant proportion of patients with hyperthyroidism that have concomitant primary hyperparathyroidism, leading to hypercalcaemia.

It is estimated that hypercalcaemia is seen in 25% of patients with proven hyperthyroidism. The patient will need other tests such as: thyroid-stimulating hormone (TSH), parathyroid hormone (PTH) as well as free T4 / T 3 to confirm the diagnosis. Toxic goitres tend to most commonly present around the 4 th -5 th decade of life, fitting with the age of the patient in this scenario.

Addison’s disease

Addison’s disease is a known cause of hypercalcaemia. It is due to adrenocortical insufficiency leading to glucocorticoid and mineralocorticoid deficiency. As a result of the mineralocorticoid (aldosterone) deficiency there is sodium loss in the kidneys and potassium retention.

Therefore Addison’s disease is associated with hyponatraemia an hyperkalaemia. With a normal sodium and potassium in this scenario the cause of the hypercalcaemia is not Addison’s disease.

Chronic renal failure

In this scenario the patient’s renal function is completely normal, with a normal urea and creatinine, ruling out chronic renal failure. Chronic renal failure is a common cause of secondary hyperparathyroidism presenting with a raised parathyro id hormone, hypocalcaemia and hyperphosphataemia. This is because in chronic renal disease there is a reduction in the excretion of phosphate and a low glomerular filtration rate. The retained phosphate creates phosphate–calcium compounds in the blood leading to a low free serum calcium.

Diabetes mellitus

Patients with diabetes mellitus are more likely to develop hypocalcaemia, as a result of diabetic renal disease, rather than hypercalcaemia. However, in some cases there is co -
existence of hyperparathyroidism and diabetes mellitus.

Sarcoidosis

Even though sarcoidosis is a cause of hypercalcaemia, it is unlikely in this scenario given the mention of a normal chest radiograph. Sarcoidosis presents with hilar and mediastinal node enlargement as well as parench ymal lung disease, both evident on chest radiography.

How well did you know this?
1
Not at all
2
3
4
5
Perfectly
115
Q

A 53-year-old woman undergoes

total thyroidectomy

for a benign multinodular goitre.

Her post-operative calcium is

1.8 mmol/l

and she complains of

tingling

in her fingers.

What is the most likely diagnosis?

1) Hypercalcaemia and overall reduced osteoclast activity

2) Hypocalcaemia and overall increased bone resorption and reduced parathyroid

hormone secretion

3) Hypocalcaemia and reduced conversion of vitamin D to 1,25-

dihydroxycholecalciferol

4) Hypocalcaemia and reduced renal reabsorption of calcium through the action of

1,25-dihdroxycholecalciferol

5) Hypocalcaemia and increased bone resorption

A

Explanation

Hypocalcaemia and reduced conversion of vitamin D to 1,25-dihydroxycholecalciferol

Post-operative hypocalcaemia is the most common complication encountered following

a thyroidectomy. This is caused by acute parathyroid insufficiency from the reduction in

the parathyroid parenchymal tissue, as often some or all of the parathyroid glands are

removed during a total thyroidectomy. This results in a reduced level of synthesis of

parathyroid hormone and leads to a reduction in bone resorption, reduced conversion

of vitamin D to 1,25-dihydroxycholecalciferol in the kidney are a

resulting hypocalcaemia due to a reduced absorption of dietary calcium in the

gastrointestinal tract. The resulting hypocalcaemia presents with paraesthesia in the

extremities and peri-orally (circumoral), abdominal cramps, tetany and posi tive

Chvostek’s and Trousseau’s signs.

Hypercalcaemia and overall reduced osteoclast activity

The patient presents with hypocalcaemia.

Hypocalcaemia and overall increased bone resorption and reduced parathyroid

hormone secretion

Increased bone resorption is the result of increased action of the parathyroid hormone

(PTH) and leads to hypercalcaemia. In hypoparathyroidism, there is a reduced level of

parathyroid hormone, with an overall decrease in bone resorption and hypocalcaemia.

Hypocalcaemia and reduced renal reabsorption of calcium through the action of 1,25 -

dihdroxycholecalciferol

Absorption of calcium through the action of 1,25 -dihydroxycholecalciferol occurs in the

gastrointestinal tract and not in the renal tract.

Hypocalcaemia and increased bone resorption Increased bone resorption is the result of increased action of parathyroid hormone

(PTH) and leads to hypercalcaemia and not hypocalcaemia.

How well did you know this?
1
Not at all
2
3
4
5
Perfectly
116
Q

You review a 75-year-old woman in fracture clinic who has a left distal radial fracture,

following a low energy injury. She has previously had a fractured neck of femur and two

vertebral wedge fractures. You discuss osteoporosis with the patient. They ask you what

can cause poor bone calcification.

Which one of the following is a cause of poor bone calcification?

1) Hypothyroidism

2) Bisphosphonates

3) Calcitonin

4) Hormone replacement therapy (HRT)

5) 1,25-Dihydroxyvitamin D deficiency

A

Explanation

1,25-Dihydroxyvitamin D deficiency

This patient presents with fragility fractures secondary to osteoporosis. Osteoporosis, is

a disease characterised by reduced bone strength leading to increased risk of fractures

and it most commonly occurs in post-menopausal women due to a variety of causes that

favour bone catabolism rather than bone mass increases and build up. Osteoporosis

presents with unusual fractures at low impact eg vertebral, neck of femur and Colles’

fractures. Vitamin D deficiency is one of the common causes leading to osteoporosis and

fragility fractures. Inadequate 1,25-dihydroxyvitamin D (1,25-dihydroxycholecalciferol)

can result either from inadequate exposure to sunlight, inadequate intake of vitamin D.

Vitamin D deficiency in children results in rickets, and in adults in osteomalacia, both of

which are characterised by a reduced mineral density of the bones making bones brittle

and fragile. Additionally, in the absence of vitamin D causes a reduced absorption of

calcium from the gastrointestinal system, leading to serum hypocalcaemia which is

detected by the parathyroid glands that stimulate production of parathyroid hormone.

The latter leads to an increase in bone resorption. Bones therefore lose their calcium

and become demineralised and brittle. Management of osteoporosis is multifaceted. It

includes lifestyle changes such as adequate calcium and vitamin D intake, smoking

cessation, introduction of exercise in daily activities, reduction in alcohol consumption

and falls prevention strategies. Pharmacological treatments include calcium and vitamin

D supplementation, bisphosphonates and calcitonin.

Hypothyroidism

In hyperthyroidism, the excess circulating thyroxine, increases the rate of bone

metabolism and resorption. Hypothyroidism however does not have a similar effect.

Bisphosphonates

Bisphosphonates are used in the treatment of osteoporosis as they act by promoting

osteoclast apoptosis and slowing bone resorption. They reduce the risk of fragility

fractures in post-menopausal women with osteoporosis. The most commonly used

bisphosphonate is alendronic acid.

Calcitonin Calcitonin, a hormone produced by the C-cells of the thyroid gland, in response to

hypercalcaemia reduces the serum calcium level and opposes the effect of parathyroid

hormone. It inhibits the resorption of bone by inhibiting osteoclast activity. Therefore, it

helps in maintaining adequate bone calcification and is being used as a treatment for

patients with osteoporosis.

Hormone replacement therapy (HRT)

Hormone replacement therapy, in the form of oestrogen only or oestrogen –

progesterone, slows bone metabolism and increases bone mineral density and reduces

the risk of a bone fragility fracture. However, once HRT is discontinued, there is

accelerated bone metabolism with a rapid decrease in bone mineral density and the

protective function of HRT that has preceded is lost. Osteoporosis is more common in

post-menopausal women due to the loss of the oestrogenic protective factors.

How well did you know this?
1
Not at all
2
3
4
5
Perfectly
117
Q

You are treating a patient pre-operatively who is going to undergo resection of a phaeochromocytoma. The patient is exhibiting features of high levels of circulating adrenaline.

What signs and symptoms would you expect to see?

1) Decreased sweat production, paroxysmal hypertension and decreased metabolic rate

2) Sweating, paroxysmal hypertension and decreased metabolic rate

3) Sweating, paroxysmal hypertension and increased metabolic rate

4) Sweating, paroxysmal hypotension and decreased metabolic rate

5) Sweating, paroxysmal hypotension and increased metabolic rate

A

Explanation

Sweating, paroxysmal hypertension and increased metabolic rate

Phaeochromocytomas are rare tumours, most commonly arising in the chromaffin cells of the adrenal medulla and resulting in increased secretion of catecholamines (adrenaline, noradrenaline, dopamine, occasionally). Adrenaline is a potent stimulator of the sympathetic system and increases the metabolic rate causing sweating. It binds to a- and b-adrenoceptors. Binding to a-adrenergic receptors brings about effects such as vasoconstriction, elevation in blood pressure, glycogenolysis and gluconeogenesis.

Binding to b-adrenergic receptors brings about tachycardia and increased contractility of the heart. Phaeochromocytoma is treated by adrenalectomy, however, adequate a- and b-adrenoceptor blockage is necessary before surgery. Alpha-receptor blockage controlsblood pressure and prevents a hypertensive crisis, whereas b -receptor blockage controls the heart rate. Adequate a-adrenoceptor blockage is necessary first, as unopposed alpha adrenergic activity that can occur during b-blockade may precipitate a hypertensive crisis.

Decreased sweat production, paroxysmal hypertension and decreased metabolic rate

Phaeochromocytoma is associated with increased sweat production, paroxysmal hypertension and increased metabolic rate.

Sweating, paroxysmal hypertension and decreased metabolic rate

Although phaeochromocytomas can occur in any age their peak incidence has been reported in patients between the age of 30–50.

Sweating, paroxysmal hypotension and decreased metabolic rate

Occasionally patients with phaeochromocytoma can have postural hypotension during an episode due to the high volume of fluid lost from excessive sweating.

Sweating, paroxysmal hypotension and increased metabolic rate Phaeochromocytomas typically present with a tetrad of headaches, palpitations, severe hypertension and excessive sweating. It is important to note that the symptoms are not
constant and are episodic, varying in frequency and duration over the course of the day.

As the tumour size increases, episodes become more frequent and prolonged.

How well did you know this?
1
Not at all
2
3
4
5
Perfectly
118
Q

An elderly man with a history of prostatism presents with acute retention of urine. His serum creatinine concentration is 520 μmol/l (50–120 μmol/l).

Which one of the following additional abnormal serum biochemistry test results is most suggestive of a chronic component to his renal failure?

1) Hyperkalaemia

2) Hyperuricaemia

3) Hypocalcaemia

4) Hyponatraemia

5) Low serum bicarbonate concentration

A

Explanation

Hypocalcaemia

Hypocalcaemia suggests that there is at least in part a longstanding element to the renal failure seen in this patient. This is due to two mechanisms: firstly there is reduced renal
synthesis of 1,25-dihydrocholecalciferol, the active form of vitamin D, necessary in the absorption of dietary calcium in the gastrointestinal system. Secondly, as the glomerular
filtration rate (GFR) drops, and the filtering ability of the kidney deteriorates, less phosphate is excreted in the urine. The circulating phosphate forms calcium-phosphate
compounds, reducing the amount of free circulating calcium. These two mechanisms lead to hypocalcaemia.

Hyperkalaemia

Hyperkalaemia can be the result of both acute and chronic renal failure and occurs due to reduced excretion of potassium and accumulation in the body.

Hyperuricaemia

Hyperuricaemia is the result of both acute and chronic renal failure and occurs secondary to the reduced ability of the kidney to filter and excrete waste products from the blood, leading to accumulation of these in the blood.

Hyponatraemia

Hyponatraemia can be a feature of both acute and chronic renal failure and occurs mainly due to a continued fluid intake while there has been a reduced ability to excrete
this fluid.

Low serum bicarbonate concentration

Low serum bicarbonate is a feature of metabolic acidosis that can precipitate both acute and chronic renal failure.

How well did you know this?
1
Not at all
2
3
4
5
Perfectly
119
Q

A patient is seen in the surgical endocrine clinic. They have developed systemic signs

and symptoms.

Which one of the following hormones is secreted by the

anterior

pituitary?

1) Testosterone

2) Oxytocin

3) TSH

4) CRH

5) ADH

A

Explanation

TSH

Thyroid-stimulating hormone (TSH) is produced by the thyrotrope cells in the anterior

pituitary. The anterior pituitary gland produces six hormones.

Testosterone

Testosterone is produced in the Leydig cells of the testicles, under the influence of

luteinising hormone (LH) and follicle-stimulating hormone (FSH) produced from the

anterior pituitary gland.

Oxytocin

Oxytocin is produced by the posterior pituitary gland and causes uterine contraction,

milk ejection and labour.

CRH

Corticotrophin releasing hormone (CRH) is produced by cells in the paraventricular

nucleus of the hypothalamus and stimulates corticotropes in the anterior pituitary to

produce ACTH.

ADH

Antidiuretic hormone (ADH), also known as vasopressin, is secreted form the posterior

pituitary gland and increases water permeability of the renal collecting ducts and distal

convoluted tubules leading to increased water reabsorption in the kidneys.

How well did you know this?
1
Not at all
2
3
4
5
Perfectly
120
Q

A patient presents with failure to tolerate cold and lethargy. Following thyroid function tests a diagnosis of hypothyroidism of the thyroid gland is made. Hypothyroidism is associated with increased plasma level of which one of the following?

1) Cholesterol

2) Albumin

3) rT3

4) Iodide

5) Thyroid stimulating immunoglobin

A

Explanation

Cholesterol

Hypothyroidism is the second most common cause of hypercholesterolaemia after diet. Thyroid hormones are inducers of HMG –CoA reductase enzyme, the first step in cholesterol synthesis, but also T 3 upregulates low-density lipoprotein (LDL) receptors in
the liver, where LDL-cholesterol binds and catabolism occurs.

Therefore, in hypothyroidism, even though there is a reduction in cholesterol synthesis there is also a downregulation of LDL receptors and catabolism of cholesterol in the liver, leading to an overall increase in the serum level of cholesterol. Hypothyroid patients are at an increased risk of developing cardiovascular disease and their lipid profile should be carefully monitored.

Albumin

Thyroid hormones promote albumin catabolism, while some of the circulating thyroid molecules travel in the blood bound to albumin. Therefore in hypothyroidism, there is decreased albumin catabolism, leading to more albumin available for glycosylation. It is the glycated albumin level that is raised in hypothyroid patients. Similarly in
hyperthyroidism, the glycated albumin levels are reduced as albumin catabolism is induced.

rT3

rT3 , is reverse tri-iodothyronine and is an inactive molecule formed by the deiodination of thyroxine. It differs from tri-iodothyronine in the positions that the iodine atoms are attached to the aromatic rings. The concentration of rT 3 in the serum tends to follow that of T4 . It is therefore reduced in hypothyroidism and raised in hyperthyroidism.

Iodide

Iodide is reduced in hypothyroid disease.

Thyroid stimulating immunoglobin

Thyroid-stimulating immunoglobulin is an antibody that binds the thyrotropin (TSH) receptor on the thyroid gland, stimulating thyroid hormone production. These are diagnostic of Grave’s disease, a form of hyperthyroidism.

How well did you know this?
1
Not at all
2
3
4
5
Perfectly
121
Q

A patient presents to the A&E with pneumonia. Incidentally, significant biochemical

abnormalities are found.

A parathyroid adenoma will be most likely to cause which one of the following?

1) Decreased osteoclastic activity

2) Decreased urinary phosphate excretion

3) Hypocalcaemia

4) Increased osteoblastic activity

5) Increased osteoclastic activity

A

Explanation

Increased osteoclastic activity

A parathyroid adenoma leads to increased production of parathyroid hormone (PTH)

the actions of which aim to increase serum calcium. This is done by acting on the bone,

gastrointestinal system and the kidney. In the bone PTH stimulates the activity of

osteoclasts in the bone leading to increased bone resorption. In the kidney it promotes

the conversion of 25-hydroxycholecalciferol to 1,25-dihydroxycholecalciferol, the active

form of vitamin D, necessary to absorb dietary calcium from the gut. In the kidney PTH

also promotes excretion of phosphate in the urine at the renal proximal convoluted

tubule and calcium reabsorption in the distal convoluted tubule.

Decreased osteoclastic activity

A parathyroid adenoma produces parathyroid hormone (PTH) that stimulates

osteoclastic activity.

Decreased urinary phosphate excretion

A parathyroid adenoma producing parathyroid hormone (PTH) causes increased

urinary phosphate excretion in the proximal convoluted tubule.

Hypocalcaemia

A parathyroid adenoma produces parathyroid hormone (PTH) that causes

hypercalcaemia, hyperchloraemia and hypophosphataemia.

Increased osteoblastic activity

A parathyroid adenoma produces parathyroid hormone (PTH) that promotes the action

of osteoclasts, leading to bone resorption.

How well did you know this?
1
Not at all
2
3
4
5
Perfectly
122
Q

A 38-year-old woman presents with a plasma thyroid -stimulating hormone (TSH)

of

12.5 μ/L

(normal 0.17–3.2 μ/l) and a T 3 resin uptake of

19% (normal 25–35%).

Which one of the following clinical symptoms and signs would you expect in this

patient?

1) Anxiety

2) Increased body temperature

3) Palpitations

4) Periorbital swelling and lethargy

5) Tachycardia

A

Explanation

Periorbital swelling and lethargy

A raised TSH combined with a low T 3 resin uptake, and a low T4 , are characteristics of

hypothyroidism. Hypothyroidism is associated with a dull facial expression, periorbital

swelling secondary to mucopolysaccharide infiltration, decreased adrenergic drive –

causing bradycardia/cold intolerance/lethargy, as well as reduced basal metabolic rate

leading to decreased appetite and weight gain. Patients also experience mental

slowness, depression and forgetfulness. Periorbital swelling in hypothyroidism needs to

be distinguished to exophthalmos in Grave’s disease which is proptosis due to increased

retro-orbital tissue.

Anxiety

Anxiety is seen in hyperthyroid patients. The results of the thyroid function tests above

indicate that the patient is in a hypothyroid state.

Increased body temperature

Increased body temperature is a sign seen in hyperthyroid patients. The results of the

thyroid function tests above indicate that the patient is in a hypothyroid state.

Palpitations

Palpitations are experienced by hyperthyroid patients. The results of the thyroid

function tests above indicate that the patient is in a hypothyroid state.

Tachycardia

Tachycardia is a sign associated with hyperthyroidism. The results of

the thyroid function tests above indicate that the patient is in a hypothyroid state.

How well did you know this?
1
Not at all
2
3
4
5
Perfectly
123
Q

A 13-year-old girl develops

exophthalmos

, nervousness, diarrhoea and weight loss

following the death of her mother. Her blood pressure is

170/90 mmHg

(normal value

<120/80 mmHg).

What is the most likely diagnosis?

1) Multiple endocrine neoplasia 2 (MEN type 2)

2) Phaeochromocytoma

3) Graves’ disease

4) Retro-orbital pseudotumour

5) Acute appendicitis

A

Explanation

Graves’ disease

Graves’ disease is the most common cause of hyperthyroidism and is an autoimmune

condition mediated by thyroid-stimulating hormone antibodies. These bind the thyroid-

stimulating hormone receptor and mimic the action of endogenous TSH, leading to

hyperplasia of the follicular cells of the thyroid and excess hormone production. It can

present with a toxic diffuse goitre. It is m ore common in women, with a female-to-male

ratio of 7–8:1. The typical age range of presentation is 20 –40, but there have been

reports in teenage patients. The patient in this scenario presents symptomatic of

hyperthyroidism: nervousness, hypertension, diarrhoea and weight loss. Other

symptoms include: sweating, heat intolerance, tachycardia, an increased appetite and

emotional lability.

This patient also exhibits signs of Graves’ ophthalmopathy with exophthalmos and

proptosis from increased retro-orbital tissue. It is also associated with upper eyelid

retraction and lid lag. Treatment of Graves’ disease involves antithyroid drugs such as

carbimazole and propylthiouracil. Given the patient’s age propylthiouracil should be

avoided due to the high risk of hepatotoxicity especially in the paediatric population.

Other treatments involve radioactive iodine and thyroidectomy.

Multiple endocrine neoplasia 2 (MEN type 2)

Multiple endocrine neoplasia type 2 is associated with tumours of the endocrine system

and typically involves phaeochromocytoma and medullary thyroid cancer. Type 2A also

involves hyperparathyroidism, while those with type 2B may demonstrate a Marfanoid

body habitus. Even though the patient exhibits some signs associated with

phaeochromocytoma, it is very unlikely in this age group. Additionally, exophthalmos is

mostly associated with Graves’ disease, making hyperthyroidism a more likely

diagnosis.

Phaeochromocytoma

Even though the patient exhibits some signs associated with phaeochromocytoma

(hypertension, weight loss, diarrhoea, anxiety), it is an unlikely diagnosis in this age group. Additionally, exophthalmos is mostly associated with Graves’ disease, making

hyperthyroidism a more likely diagnosis.

Retro-orbital pseudotumour

A retro-orbital pseudotumour is an inflammatory condition involving the extraocular

muscles that presents with painful unilateral exophthalmos and diplopia. It is not

associated with diarrhoea or weight loss.

Acute appendicitis

Even though acute appendicitis can present with diarrhoea as well as hypertension and

anxiety relating to pain, it is not associated with exophthalmos or weight loss as in the

scenario above.

How well did you know this?
1
Not at all
2
3
4
5
Perfectly
124
Q

A patient is referred with refractory hypertension. Despite basic medical

treatment their blood pressure remains 180/110mmHg.

Which one of the following is correct regarding adrenals and surgical

hypertension?

1) Renovascular disease is a rare cause of secondary hypertension

2) A phaeochromocytoma can be shown by metaiodobenzylguanidine (MIBG)

scintigraphy

3) Most adrenal incidentalomas are hormone secreting

4) A raised midnight cortisol is diagnostic of Cushing’s syndrome

5) 30% of phaeochromocytomas are extra-adrenal

A

Explanation

A phaeochromocytoma can be shown by metaiodobenzylguanidine (MIBG) scintigraphy

MIBG scintigraphy is a very specific test for detecting phaeochromocytoma. Radioactive

iodine-131-metaiodobenzylguanidine is administered to the patient and localises to

adrenergic tissue. In the case of a patient with a phaeochromocytoma there is a focal

area in the adrenal medulla where there is very strong and prolonged uptake of the

solution. MIBG scintigraphy is not considered the first line investigation to detect

phaeochromocytoma. MIBG is useful in detecting extra-adrenal tumours or disease

recurrence.

Renovascular disease is a rare cause of secondary hypertension

Secondary hypertension accounts for 5–10% of hypertensive patients. Renovascular

disease is the most common cause of secondary hypertension, occu rring due to

narrowing of the renal artery (renal artery stenosis). It typically presents in young

women (<30 years) and older men (>50 years). Other causes of secondary hypertension

include: Conn syndrome, Cushing syndrome, phaeochromocytoma, hyperthyroid ism,

congenital adrenal hyperplasia, use of oral contraceptives, and pregnancy.

Most adrenal incidentalomas are hormone secreting

Adrenal incidentalomas are described as adrenal masses found incidentally. Of these,

85% are non-functional, ie do not secrete any hormones and the remaining 15% are

either hormone-secreting or malignant.

A raised midnight cortisol is diagnostic of Cushing’s syndrome

Cortisol levels are secreted in a diurnal rhythm. They peak in the morning between

0700–0900 h and then gradually reduce throughout the day, reaching a nadir at around

midnight. In Cushing syndrome there is loss of the diurnal rhythm and a raised midnight

cortisol is thought to be the earliest sign of Cushing syndrome with a very high

sensitivity. However, other factors can cause a midnight cortisol surge such as stress

resulting from physical illness, being awake at midnight to perform the test and depression. As a result, for the diagnosis of Cushing syndrome the dexamethasone

suppression test is used.

30% of phaeochromocytomas are extra-adrenal

About 10% of phaechromocytomas are extra-adrenal. It is important to remember that

phaeochromocytoma follows the rule of 10s: 10% are malignant, 10% are bilateral, 10%

are found in children, 10% of patients have multiple tumours, 10% are extra -adrenal

(found in the thorax, neck, bladder, kidney, scrotum) and 10% are familial (von Hippel –

Lindau syndrome).

How well did you know this?
1
Not at all
2
3
4
5
Perfectly
125
Q

You are treating a patient with hyperthyroidism. They want to know what the options

for ongoing management are.

Which one of the following statements is correct?

1) With antithyroid drugs patients have a high recurrence rate when therapy is

discontinued

2) Surgery is the most rapid method of permanent control and results in

euthyroidism in about 50% of cases at 5 years

3) Surgery has no permanent complications

4) Radio-iodine therapy gives permanent control and h as a low rate of

hypothyroidism

5) All other treatments, except surgery, are contraindicated in pregnancy

A

Explanation

With antithyroid drugs patients have a high recurrence rate when therapy is

discontinued

Antithyroid drugs do not offer permanent control as seen with surgery or radio-iodine.

If the patients discontinue their antithyroid medication, hyperthyroidism recurs in the

vast majority of cases. Antithyroid medication such as propylthiouracil and carbimazole

work as hormone antagonists therefore following cessation the disease can recur.

Combining antithyroid medication and radio -iodine therapy increases the rate of

achieving permanent control.

Surgery is the most rapid method of permanent control and results in euthyroidism in

about 50% of cases at 5 years

Thyroid surgery can lead to hypothyroidism, and occurs in 90% of patients post -

operatively at 5 years and this rises to almost 100% in 20 years.

Surgery has no permanent complications

Thyroid surgery is not without complications. These are grouped in to immediate, early

and late complications.

Radio-iodine therapy gives permanent control and has a low rate of hypothyroidism

Radio-iodine therapy yields good results and generally has a lower rate of recurrence of

hyperthyroidism when compared to medical therapy only. Return to a euthyroid state

takes about 2 months following treatment. Hypothyroidism is however a common

complication following radio-iodine therapy, affecting patients at a rate of up to 90% in

the first year.

All other treatments, except surgery, are contraindicated in pregnancy

Hyperthyroidism in pregnancy affects about 0.2% of pregnancies with Graves’ disease

being the most common cause, followed by gestational hyperthyroidism. During

pregnancy, the treatment options for hyperthyroidism ar e limited to antithyroid

medication and surgery. Radioactive iodine is contraindicated during pregnancy. During early pregnancy, propylthiouracil should be used, as carbimazole has a risk of

teratogenic effects. After the first trimester, the use of carbim azole can resume for the

remainder of the pregnancy, as propylthiouracil is associated with hepatotoxicity.

Surgery in pregnancy is reserved for cases of malignant disease where there is

uncontrolled hyperthyroidism or airway compromise. Otherwise, it is g enerally advised

for the operation to be postponed until after delivery due to the risk of general

anaesthesia on the fetus.

How well did you know this?
1
Not at all
2
3
4
5
Perfectly
126
Q

A 34-year-old woman has her blood pressure checked by her GP and it is found to be

180/100 mmHg (normal value <120/80 mmHg). She is diagnosed with a

phaeochromocytoma causing secondary hypertension.

Which one of the following is most likely regarding this patient’s

phaeochromocytoma?

1) It is usually bilateral

2) It is usually extra-adrenal

3) It is usually malignant

4) It may be screened for by urinary catecholamines

5) It is best treated medically

A

Explanation

It may be screened for by urinary catecholamines

Phaeochromocytoma is a rare neuroendocrine tumour, arising in the adre nals in 90% of

the cases, producing an excess amount of catecholamines. In patients where a

phaeochromocytoma is suspected, quantifying urinary total catecholamines

and metanephrines in a 24-hour urine sample collection, has the highest sensitivity and

specificity. Assessing the level of vanillylmandelic acid in a 24 -hour urine sample is not

as accurate.

It is usually bilateral

Phaeochromocytoma follows the rule of 10s. Only 10% of cases are bilateral.

It is usually extra-adrenal

Phaeochromocytoma follows the rule of 10s. Only 10% of cases are extra-adrenal and

have been described in the thorax, neck, bladder, kidney and scrotum.

It is usually malignant

Phaeochromocytoma follows the rule of 10s. Only 10% of cases are malignant.

It is best treated medically

There is no effective medical treatment for phaeochromocytoma. The patient is

adequately prepared pre-operatively with a-blockers and then b-blockers before

surgical removal of the phaeochromocytoma.

How well did you know this?
1
Not at all
2
3
4
5
Perfectly
127
Q

A 45-year-old man is being screened for secondary hypertension. He complains of muscle weakness and his blood results show hypernatraemia, hypokalaemia and

metabolic

alkalosis.

What is the most likely diagnosis?

1) Addison’s disease

2) Coarctation of the aorta

3) Conn syndrome

4) Incidental findings in patient with essential hypertension

5) Phaeochromocytoma

A

Explanation

Conn syndrome

The patient shows signs of Conn syndrome, with secondary hypertension,

hypernatraemia, hypokalaemia and a metabolic alkalosis. Conn syndrome is

primary hyperaldosteronism, ie excess secretion of aldosterone. Aldosterone is

produced in the zona glomerulosa of the cortex of the adrenals. It is a mineralocorticoid

hormone that acts on the mineralocorticoid receptors, found mainly in the distal

convoluted tubule and collecting ducts of the nephron, activating the basolateral

Na + /K + ion channels to reabsorb sodium and water and excrete potassium. It increases

the permeability of the collecting duct to sodium and stimulates the secretion of H + ions

into the urine. Through conservation of sodium and water, aldosterone increases blood

pressure. In hyperaldosteronism, there is excessive release of aldosterone, typically

from either adrenal hyperplasia or an adenoma. Conn syndrome most commonly affects

women, with a female to male ratio of 2:1, aged 30–50. Presenting features of Conn

syndrome include: hypertension, abdominal distension, weakness, ileus or features

related to complications of hypertension such as headaches, proteinuria. Treatment

incudes spironolactone (an aldosterone antagonist) or surgery with adrenalectomy.

Addison’s disease

Addison’s disease is the result of adrenocortical insufficiency and may present with

hypotension. It is associated with biochemical disturbances which include

hyponatraemia, hyperkalaemia and hypercalcaemia.

Coarctation of the aorta

Coarctation of the aorta is a known cause of secondary hypertension but does not lead

to the biochemical disturbances evident in the above scenario.

Incidental findings in patient with essential hypertension

Essential hypertension does not have a known cause and should not be associated with

biochemical abnormalities of hypernatraemia, hypokalaemia and metabolic alkalosis.

Phaeochromocytoma Phaeochromocytoma is a known cause of secondary hypertension, but does not lead to

the biochemical abnormalities evident in the scenario.

How well did you know this?
1
Not at all
2
3
4
5
Perfectly
128
Q

A 68-year-old man presents to his GP with weight gain, bruising and generalised weakness and is diagnosed with Cushing syndrome. He is found to be hyperglycaemic and serum adrenocorticotropic hormone (ACTH) levels are very high Overnight high-dose dexamethasone suppression test shows a raised morning cortisol level.

Which one of the following is most likely to be found?

1) Adrenocortical adenoma

2) Adrenocortical carcinoma

3) Basophil pituitary adenoma

4) Small-cell bronchial carcinoma

5) Bronchial carcinoid

A

Explanation

Small-cell bronchial carcinoma

The patient presents with signs relating to Cushing syndrome, characterised by the excess production of cortisol in the body. The raised ACTH excludes an adrenal cause. The high-dose overnight dexamethasone suppression test is used to differentiate ACTH -dependent causes of Cushing syndrome. If the cause is a pituitar y adenoma then high-dose dexamethasone suppresses the production of ACTH from the adenoma cells and therefore causes a low morning cortisol.

As the cortisol level remained unaffected, the source of ACTH is not the pituitary and is therefore an ectopic sour ce. From the options available, small-cell bronchial carcinoma, accounting for 15% of all primary lung tumours, is the most common cause of ectopic ACTH production. This is a paraneoplastic syndrome that can occur with the following tumours: small -cell lung tumours, bronchial carcinoid tumours, islet-cell tumours of the pancreas, medullary carcinoma of the thyroid and thymic tumours. Signs of Cushing syndrome are summarised in the table below:

Head and neck

Trunk

Limbs

Frontal balding

Kyphosis

Proximal myopathy

Moon face

Buffalo hump

Pathological fractures

Acne

Gynaecomastia in men

Ankle oedema (salt and water retention due to excess cortisol)

Plethoric complexion

Central obesity (due to altered fat distribution) Hirsutism

Purple striae on abdomen, breasts, thighs

Pigmentation in ACTH-dependent cases, in areas exposed to sunlight

Thin skin

Hair growth of forearms in particular

Tissue wasting

Predisposition to infection

Bad wound healing

Adrenocortical adenoma

When Cushing syndrome is the result of an adrenal cause, such as an adrenocortical adenoma or an adrenocortical carcinoma, the excess circulating cortisol suppresses the hypothalamus–pituitary axis and leads to a low corticotrophin releasing hormone (CRH) and ACTH. In this scenario ACTH is high as is morning cortisol.

Adrenocortical carcinoma

Where Cushing syndrome is the result of an adrenal cause, such as an adrenocortical adenoma or an adrenocortical carcinoma, the excess circulating cortisol suppresses the hypothalamus–pituitary axis and leads to a low CRH and ACTH. In this scenario ACTH is high as is morning cortisol.

Basophil pituitary adenoma

Cushing’s disease is caused by a pituitary adenoma, arising in either the basophil or chromophobe cells. However, in this scenario the cause of the patient’s presentation is not a pituitary adenoma. In the case of a pituitary adenoma, cortisol level should be suppressed, following a high-dose overnight dexamethasone suppression test.

Bronchial carcinoid

Both bronchial carcinoid and small-cell bronchial carcinoma can produce a Cushing syndrome by ectopic secretion of ACTH. Small -cell bronchial carcinoma is more common that bronchial carcinoid. Bronchial carcinoid is a rare entity, accounting for 1–6% of primary lung tumours and tends to present between the 4th and 5th decade of life.

How well did you know this?
1
Not at all
2
3
4
5
Perfectly
129
Q

A 43-year-old woman has a history of medullary thyroid carcinoma and parathyroid hyperplasia. She is concerned about symptoms to be aware of in her children. Which other condition would be associated with multiple endocrine neoplasia type 2A (MEN2A)?

1) Pituitary adenoma

2) Gastrinoma

3) Marfanoid habitus

4) Neurofibromatosis

5) Phaeochromocytoma

A

Explanation

Phaeochromocytoma

Phaeochromocytoma is found in patients with both MEN2A and MEN2B syndromes. Patients with MEN2A have the following combination of conditions: medullary thyroid carcinoma, phaeochromocytoma and hyperparathyroidism (parathyroid hyperplasia).

MEN2 is an autosomal-dominant condition associated with the RET proto-oncogene. There is also a third variation of MEN2 called familial MEN2, presenting with a medullary thyroid carcinoma and no other manifestations. MEN2A (Sipple syndrome):
type 2 = secondary = Sipple.

Pituitary adenoma

Pituitary adenomas are associated with multiple endocrine neoplasia type 1. This condition is described as the combination of: parathyroid hyperplasia, pancreatic islet - cell tumours and a pituitary adenoma.

Gastrinoma

A gastrinoma is a gastrin-secreting tumour most commonly found in the duodenum or pancreas and can lead to Zollinger–Ellison syndrome – peptic ulceration because of excess. Up to 50% of patients with Zollinger–Ellison syndrome are found to have MEN1.

Marfanoid habitus

Marfanoid habitus is seen in patients with multiple endocrine neoplasia type 2B. This condition describes the combination of medullary thyroid carcinoma, phaeochromocytoma, mucosal neuromas and Marfanoid body habitus.

Neurofibromatosis

Neurofibromatosis is not part of multiple endocrine neoplasia.

How well did you know this?
1
Not at all
2
3
4
5
Perfectly
130
Q

A 26-year-old woman presents to the pre-assessment clinic with elevated serum calcium levels. She tells you she has familial hypocalciuric hypercalcaemia. Which one of the following statements is correct regarding familial hypocalciuric hypercalcaemia?

1) Hypermagnesaemia may be seen

2) It is autosomal-recessive

3) It is associated with increased urinary calcium excretion – more than 200 mg per 24 h

4) It is associated with low parathyroid hormone (PTH) levels

5) The condition responds to parathyroidectomy

A

Explanation

Hypermagnesaemia may be seen

Familial hypocalciuric hypercalcaemia (FHH) is a rare autosomal -dominant condition with high penetrance, associated with loss-of-function mutations in the calcium sensing receptor (CaSR), a G-coupled protein receptor which is found in the kidney and parathyroid tissue. In the parathyroid gland its function is to sense serum calcium levels and regulate parathyroid hormone (PTH) release, whereas in the kidney it inhibits reabsorption of calcium. In FHH, there is loss of the negative feedback system on PTH release when serum calcium is elevated due to a reduction in the receptor’s sensitivity, leading to sustained mild hypercalcaemia. In the kidney there is reduced excretion leading to hypocalciuria. In most cases there is also hypermagn esaemia. Patients are usually asymptomatic. FHH is confirmed with genetic testing.

It is autosomal-recessive

Familial hypocalciuric hypercalcaemia is inherited in an autosomal -dominant fashion with a high penetrance.

It is associated with increased urinary calcium excretion – more than 200 mg per 24 h Familial hypocalciuric hypercalcaemia is characterised by raised serum calcium levels, low urinary calcium levels, normal or slightly elevated parathyroid hormone level and commonly associated with hypermagnesaemia.

It is associated with low parathyroid hormone (PTH) levels Familial hypocalciuric hypercalcaemia is characterised by normal or slightly raised levels of parathyroid hormone.

The condition responds to parathyroidectomy

In patients that are asymptomatic, treatment is not necessary. Familial hypocalciuric hypercalcaemia should not be treated with parathyroidectomy and does not respond to diuretic or bisphosphonate treatment. Calcimimetics have been used for symptomatic cases as well as pamidronate. Parathyroidectomy is reserved for patients with recurrent pancreatitis.

How well did you know this?
1
Not at all
2
3
4
5
Perfectly
131
Q

A 52-year-old woman with longstanding rheumatoid arthritis is seen in a pre-assessment clinic before a planned left total knee replacement. On examination, she is noted to be
overweight and struggles to rise from a chair. She is noted to be on long-term steroids
for rheumatoid arthritis and is found to have a high blood pressure ,raised fasting plasma glucose and striae seen in her skin.

Which one of the following conditions is she most likely to have developed secondary to her medication?

1) Addison’s disease

2) Congenital adrenal hyperplasia

3) Conn syndrome

4) Cushing syndrome

5) Phaeochromocytoma

A

Explanation

Cushing syndrome

Cushing syndrome is a condition related to excess cortisol. The commonest cause is
iatrogenic, resulting from exogenous administration of glucocorticoid medication for
chronic conditions such as asthma, chronic obstructive pulmonary disease (COPD),
arthritis etc. Endogenous causes of Cushing inclu de: a pituitary adenoma producing an
excess of adrenocorticotropic hormone (ACTH), adrenocortical adenoma or carcinoma
and a functional tumour producing ectopic adrenocorticotropic hormone (ACTH) such
as an oat-cell lung carcinoma. Cushing syndrome relates to a collection of signs and
symptoms that occur as the result of excess cortisol. These are summarised in the table
below:

Head and neck

Trunk

Limbs

Frontal balding

Kyphosis

Proximal myopathy

Moon face

Buffalo hump

Pathological features

Acne

Gynaecomastia in men

Ankle oedema (salt and water retention due to excess cortisol)

Plethoric complexion

Central obesity (due to altered fat distribution)

Hirsutism

Purple striae on abdomen, breasts, thighs

Pigmentation in ACTH-dependent cases, in areas exposed to sunlight

Thin skin

Hair growth of forearms in particular

Tissue wasting

Predisposition to infection

Bad wound healing

Addison’s disease

Addison’s disease is the result of adrenocortical insufficiency and can present with hypotension. It is associated with biochemical disturbances as well: hyponatraemia,
hyperkalaemia and hypercalcaemia.

Congenital adrenal hyperplasia

Congenital adrenal hyperplasia is an autosomal-recessive condition associated with
deficiency in the enzyme 21α-hydroxylase, involved in the synthesis of cortisol and
aldosterone. It tends to present at birth or early infancy.

Conn syndrome

Conn syndrome is a condition associated with hyperaldosteronism. It is mineralocorticoid and not glucocorticoid excess. Even though patients with Conn syndrome present with hypertension, the remaining signs described in the scenario above relate to glucocorticoid excess. Patients with Conn syndrome present with hypernatraemia and hypokalaemia.

Phaeochromocytoma

Phaeochromocytoma is a rare neuroendocrine tumour, arising in the chromaffin cells of
the adrenals in 90% of the cases, producing an excess amount of catecholamines. Even
though it is associated with hypertension, the remaining signs described in the scenario
relate to corticosteroid excess. Patients with phaeochromocytoma present with severe
hypertension associated with headaches, palpitations and diaphoresis.

How well did you know this?
1
Not at all
2
3
4
5
Perfectly
132
Q

A 35-year-old woman is found at her preoperative assessment to be

tachycardic

, with a

heart rate of 120 beats/min, extremely

anxious

and to have a smoothly

enlarged

goitre

in her neck. A diagnosis of

Graves’ disease

is confirmed.

What are her blood tests likely to show?

1) Low thyroxine; which regulates carbohydrate metabolism

2) Raised thyroxine; secreted by the colloid cells of the thyroid gland

3) Raised thyroxine level; mostly bound to albumin in the blood

4) Raised thyroxine-stimulating hormone (TSH) from the posterior pituitary

5) Thyroid-stimulating hormone receptor immunoglobulins

A

Explanation

Thyroid-stimulating hormone receptor immunoglobulins

Graves’ disease is a form of hyperthyroidism of autoimmune origin. It is associated with

the presence of circulating TSH receptor immunoglobulins (antibodies), that bind to the

TSH receptor in the thyroid. These antibodies mimic the function of TSH, leading to

stimulation of the thyroid gland to produce more triiodothyronine (T3) and thyroxine

(T4). The elevated serum-circulating hormones activate the negative feedback

mechanism and suppress release of TSH from the anterior pituitary gland.

Low thyroxine; which regulates carbohydrate metabolism

Thyroxine (T4) is the hormone produced from the thyroid gland. Graves’ disease is a

state of hyperthyroidism. Therefore, serum thyroxine level s would be

raised. Thyroxine stimulates lipid and carbohydrate metabolism.

Raised thyroxine; secreted by the colloid cells of the thyroid gland

Raised thyroxine is found in patients with Graves’ disease as this is a form of

hyperthyroidism, however thyroxine is secreted by the follicular and not the colloid

cells of the thyroid gland.

Raised thyroxine level; mostly bound to albumin in the blood

In overt hyperthyroidism there is a raised free (unbound) thyroxine level (free T4).

Raised thyroxine-stimulating hormone (TSH) from the posterior pituitary

TSH is secreted from the anterior pituitary. However, the cause of Graves’ disease is not

raised TSH but the presence of TSH receptor immunoglobulins, that mimic the function

of TSH by binding to the TSH receptor and stimulating the thyroid gland to release

hormone molecules. TSH is suppressed via a negative feedback mechanism and is

therefore low in patients with Graves’ disease.

How well did you know this?
1
Not at all
2
3
4
5
Perfectly
133
Q

A 45-year-old woman presents with generalised weakness and fatigue. She has a history of renal colic. Her calcium is found to be raised at 3.1. An inferior parathyroid adenoma

is suspected.

Which one of the following statements is most likely to be true?

1) This parathyroid gland would have developed fro m the second branchial pouch

2) This would be an unusual case, as the majority of primary hyperparathyroidism

is due to diffuse hypertrophy of the parathyroids

3) The adenoma could be localised by a combination of ultrasound and sestamibi

radionucleotide scan

4) This woman’s weakness and fatigue are incidental to the hypercalcaemia

5) Parathyroid tissue cannot be implanted elsewhere in the body after total

parathyroid excision

A

Explanation

The adenoma could be localised by a combination of ultrasound and sestamibi

radionucleotide scan

Pre-operative parathyroid adenoma localisation can be done by either ultrasound,

sestamibi (methoxyisobutylisonitrile (MIBI), uses technetium) radionucleotide scan or

technetium/thallium scan. The inferior parathyroids are much more variable in their

position.

This parathyroid gland would have developed from the second branchial pouch

The superior and inferior parathyroids develop from the fourth and third branchial

pouches, respectively.

This would be an unusual case, as the majority o f primary hyperparathyroidism is due

to diffuse hypertrophy of the parathyroids

Primary hyperparathyroidism is usually due to a solitary secreting parathyroid

adenoma. Hyperparathyroidism presents as:

‘bones’ (arthralgia and osteoporosis)

‘stones’ (renal)

abdominal ‘groans’ (pancreatitis, peptic ulcers and constipation)

‘moans’ (depression, fatigue and weakness).

This woman’s weakness and fatigue are incidental to the hypercalcaemia

This woman’s weakness and fatigue are almost definitely as a result of her

hypercalcaemia. Parathyroid tissue cannot be implanted elsewhere in the body after total parathyroid

excision

Tertiary hyperparathyroidism occurs in renal failure, in which the parathyroids

function autonomously. In such cases, subcutaneous reimplantation of half a

parathyroid in the forearm allows easy access if further resection is required.

How well did you know this?
1
Not at all
2
3
4
5
Perfectly
134
Q

A 55-year-old woman with

poorly-controlled

polymyalgia rheumatica (PMR) complains

of weight gain, muscle weakness and easy bruising. She is found to have increased

abdominal girth, striae, bruising and raised serum glucose levels. A diagnosis of

Cushing’s syndrome is made.

What is most likely to be the cause of her Cushing syndrome?

1) Iatrogenic steroids

2) Medullary carcinoma of the thyroid

3) Pituitary tumour

4) Lung tumour secreting adrenocorticotropic hormone (ACTH)

5) Polymyalgia rheumatica

A

Explanation

Iatrogenic steroids

This woman has Cushing syndrome and the commonest cause of this is iatrogenic

through the administration of steroids. In this case, this woman has poorly controlled

polymyalgia rheumatica (PMR) and is likely to have been on prolonged steroid

treatment. Endogenous cases are caused by primar y pituitary disease ie pituitary

adenoma (70%, referred to as Cushing’s disease), primary adrenal tumours (15%) and

ectopic adrenocorticotropic hormone (ACTH)-secreting tumours (15%).

The diagnosis of Cushing syndrome depends on the loss of normal circadia n rhythm as

well as persistent elevation of cortisol. It is necessary to take at least three serum

samples to demonstrate the loss of rhythm. The mean 24 -h cortisol levels can be

estimated by collecting and measuring the 24-h urinary free cortisol, which gives an

integrated measure of cortisol production. As a screening test it has 95% specificity.

Medullary carcinoma of the thyroid

Medullary carcinomas of the thyroid can rarely produce ectopic adrenocorticotropic

hormone (ACTH), however iatrogenic administration of steroids is more likely in this

case given the history of PMR.

Pituitary tumour

Pituitary tumours can produce adrenocorticotropic hormone (ACTH), however, the

history of polymyalgia rheumatica (PMR) means iatrogenic steroid administration is

more likely.

Lung tumour secreting adrenocorticotropic hormone (ACTH)

ACTH secreting tumours, of which bronchial carcinoma is the most common, can cause

Cushing syndrome, however there is no history indicative of lung cancer.

Polymyalgia rheumatica

Administration of steroids to treat polymyalgia rheumatica (PMR), not PMR per se,

causes Cushing syndrome.

How well did you know this?
1
Not at all
2
3
4
5
Perfectly
135
Q

A 65-year old man is diagnosed with a parathyroid

adenoma

. He attends the

Outpatient’s Clinic for advice on management options.

What is the best option for treatment?

1) Only surgery is curative

2) Bisphosphonates and calcitonin result in good long -term control

3) Octreotide results in good control of parathyroid hormone (PTH) levels

4) Surgery is reserved for those not improving on bisphosphonate treatment

5) Reducing dietary calcium intake can be curative

A

Explanation

Only surgery is curative

Parathyroidectomy is the only curative treatment for primary hyperparathyroidism

caused by a parathyroid adenoma.

Bisphosphonates and calcitonin result in good long-term control

Bisphosphonates, calcitonin and octreotide do not have a role in treatment.

Octreotide results in good control of parathyroid hormone (PTH) levels

Octreotide is used in nuclear imaging. Parathyroid tumours expressing somato statin

may show octreotide uptake, which may indicate a possibility for the use of

somatostatin analogues for medical treatment when surgery is not possible. Octreotide

does not have a role in the treatment of primary hyperparathyroidism.

Surgery is reserved for those not improving on bisphosphonate treatment

Surgery is the only curative treatment of primary hyperparathyroidism secondary to a

parathyroid adenoma. Bisphosphonates do not have a role in treatment of primary

hyperparathyroidism.

Reducing dietary calcium intake can be curative

Low calcium intake stimulates the parathyroids and high intake accentuates

hypercalcaemia, so the recommendation is moderate intake followed up with curative

surgery by removal of the parathyroid glands.

How well did you know this?
1
Not at all
2
3
4
5
Perfectly
136
Q

A 48-year old woman is found to have incidental deranged biochemistry. She is

subsequently diagnosed with

primary hyperparathyroidism

.

Which one of the following is likely to be correct?

1) She will also have diabetes mellitus

2) She will probably be asymptomatic

3) She will have reduced parathyroid hormone (PTH) levels

4) She will have a raised phosphate

5) She will have MEN2b syndrome

A

Explanation

She will probably be asymptomatic

Asymptomatic primary hyperparathyroidism is found in greater than 50 % of patients.

In 85% of cases, it is caused by a single parathyroid adenoma, but can be due to multiple

gland adenomas/hyperplasia and rarely even parathyroid carcinoma.

She will also have diabetes mellitus

Patients with primary hyperparathyroidism are very slightly more likely to also have

diabetes and vice versa. It is not certain therefore that this patient would have diabetes.

She will have reduced parathyroid hormone (PTH) levels

Patients will have a raised serum calcium and parathyroid hormone and a reduced

phosphate (secondary to renal excretion, causing phosphaturia).

She will have a raised phosphate

As mentioned, the patient will have a low phosphate due to renal excretion of phosphate

in response to raised serum calcium and parathyroid hormone.

She will have MEN2b syndrome

Hyperparathyroidism is found in MEN 1 and MEN 2a syndromes, but not in MEN 2b.

How well did you know this?
1
Not at all
2
3
4
5
Perfectly
137
Q

A 40-year-old fit and well Polish woman presents to the Ear, Nose and Throat (ENT)

clinic with a

solitary

thyroid nodule. She is

asymptomatic

.

What is the most useful first line investigation?

1) Computed tomography (CT) scan

2) MRI scan

3) Fine-needle aspiration cytology

4) Technetium radio-isotope scan

5) Core biopsy

A

Explanation

Fine-needle aspiration cytology

Thyroid nodules are common, and the majority are within multinodular goitres. The

most useful initial investigations are ultrasound and fine-needle aspiration cytology of

any dominant nodules.

Thyroid function tests in patients with multinodular goitre or a solitary thyroid nodule

are usually normal. These are therefore usually only necessary if you suspect a hot

nodule in a symptomatic patient. Only around 10 –20% of cold nodules in adults and

50% in children are malignant.

Computed tomography (CT) scan

Further imaging is only necessary if you suspect a very large goitre causing tracheal

compression or retrosternal extension. In such cases, CT scan of the neck and thorax is

useful.

MRI scan

MRI scanning does not have a role in the investigation of a simple thyroid nodule.

Technetium radio-isotope scan

Technetium radio-isotope scan is useful for the assessment of parathyroid adenomas

and the myocardium, but is not used in the assessment of thyroid nodules.

Core biopsy

Fine-needle aspiration, not core biopsy, is sufficient in gaining samples from nodules for

histological examination. Furthermore, fine-needle aspiration is less invasive than core

biopsy.

138
Q

A 54-year-old man presents to his GP with a 2-week history of sweating, headache,

constipation and itchy lesions over his back. On examination his blood pressure is

162/94 mmHg and his pulse rate is 102/min. Twenty-four hour urinary

catecholamines, metanephrines and vanillylmandelic acid are fo und to be elevated. A

computed tomography (CT) and a 131I-meta-iodo-benzyl-guanidine scan confirms

a

phaeochromocytoma

. He is subsequently found to have a

medullary carcinoma

of the

thyroid.

Which one of the following do you think is the most likely diagnosis in this

patient?

1) Multiple Endocrine Neoplasia (MEN) I

2) Secondary hyperparathyroidism

3) Multiple Endocrine Neoplasia IIB

4) Carcinoid tumour

5) Multiple Endocrine Neoplasia IIA

A

Explanation

Multiple Endocrine Neoplasia IIA

Multiple Endocrine Neoplasia II (MEN II) is an autosomal-dominant disorder caused by

mutations in the RET proto-oncogene. MEN II has three distinct subtypes – MEN

IIA, MEN IIB, and familial medullary thyroid carcinoma only. MEN IIA describes the

association of medullary thyroid carcinoma, phaeochromocytomas and parathyroid

tumours. Patients with pheochromocytomas may present with hypertension,

tachycardia, sweating and headaches. Cutaneous lichen amyloidosis in patients with

MEN IIA manifests as multiple pruritic scaly skin lesions in the scapular area of the

back.

Multiple Endocrine Neoplasia (MEN) I

MEN I is characterised by carcinomas of the pituitary, pancreas and parathyroid. This

patient has symptoms of a phaeochromocytoma, which is then confirmed on

investigation. Phaeochromocytomas are not present in MEN I.

Secondary hyperparathyroidism

This is most commonly seen in patients with chronic kidney disease causing

hypocalcaemia and hyperphosphataemia which then stimulates the parathyroid glands.

Multiple Endocrine Neoplasia IIB

MEN IIB can be distinguished from MEN IIA due to the presence of marfanoid features

and mucosal neuromas, neither of which are described in the case history. In MEN IIB,

the medullary cancer is very aggressive with most patients dying before developing

either a phaeochromocytoma or hyperparathyroidism.

Carcinoid tumour Carcinoid tumours most commonly secrete ectopic serotonin, excess levels of which

would not produce the clinical picture described in the case history. Instead they are

likely to present with flushing, diarrhoea, abdominal cramps and wheezing.

139
Q

A 30-year-old woman presents with a lump in the non-tender and has rapidly increased in size. Her right lobe of lymph nodes the thyroid. It is hard , are enlarged. She has previously had radiotherapy to her neck. Histology shows Orphan Annie nuclei.

The most likely tumour is?

1) Follicular adenoma

2) Anaplastic carcinoma

3) Medullary carcinoma

4) Papillary carcinoma

5) Follicular carcinoma

A

Explanation

Papillary carcinoma

Papillary carcinoma is the commonest malignant thyroid tumour. This is more common

in younger patients, women and those with a past history of head and neck irradiation.

It often spreads to local lymph nodes. Orphan Annie nuclei are characteristic,

psammoma bodies may also be seen. It is extremely responsive to treatment and

prognosis is excellent.

Follicular adenoma

Follicular adenomas of the thyroid are benign and therefore would not metastasise to

the regional lymph nodes as described in the case history.

Anaplastic carcinoma

Anaplastic carcinoma is an extremely aggressive thyroid cancer and is usually

unresponsive to treatment. It more commonly presents in elderly patients.

Medullary carcinoma

Medullary carcinoma only accounts for around 3% of thyroid cancer and therefore it is

more likely the patient described has a papillary carcinoma.

Follicular carcinoma

Follicular carcinoma more commonly presents in women over the age of 50, accounts

for around 15% of all thyroid cancer, and has a good prognosis.

140
Q

A 22-year-

hormone (

old woman has puffy eyes TSH) concentration is low

and

but

hoarse voice. Her plasma thyroid -stimulating increases markedly when she is given

thyrotrophin-releasing hormone (

TRH).

She probably has which one of the following conditions?

1) Hyperthyroidism due to a thyroid tumour

2) Hyperthyroidism due to a primary abnormality in the hypothalamus

3) Hypothyroidism due to a primary abnormality in the hypothalamus

4) Hypothyroidism due to a primary abnormality in the pituitary gland

5) Hypothyroidism due to a primary abnormality in the thyroid gland

A

Explanation

Hypothyroidism due to a primary abnormality in the hypothalamus

Tertiary hypothyroidism, also called hypothalamic -pituitary axis hypothyroidism,

results when the hypothalamus fails to instruct the pituitary to produce sufficient TSH.

This young woman’s hypothyroidism improves with TRH administration, confirming

the diagnosis of tertiary hypothyroidism.

Hyperthyroidism due to a thyroid tumour

The patient has no signs or symptoms of thyroid malignancy, which often presents with

a firm mass noticed by the patient which arises from the thyroid.

Hyperthyroidism due to a primary abnormality in the hypothalamus

The patient has symptoms of hypothyroidism, not hyperthyroidism. Hyperthyroidism

can present with palpitations, tremor, weight loss (despite increased appetite), fatigue,

hair loss and heat intolerance.

Hypothyroidism due to a primary abnormality in the pituitary gland

As her TSH level increases in response to TRH, this suggests there is no abnormality in

the pituitary gland.

Hypothyroidism due to a primary abnormality in the thyroid gland

The administration of TRH and subsequent increase in TSH suggests that the primary

abnormality is due to an abnormality with the hypothalamus.

141
Q

A young patient undergoes OGD and they are found to have multiple ulcers in the stomachand duodenum. They are promptly started on PPI and undergo further

investigations.

What is the cause of gastric hypersecretion in Zollinger–Ellison (ZE) syndrome?

1) Histamine antagonism

2) Increased gastrin secretion

3) Somatostatin secretion

4) Vagal inhibition

5) Vagal stimulation

A

Explanation

Increased gastrin secretion

Zollinger–Ellison syndrome is a result of an endocrine tumour of the pancreatic G -cells

(gastrinoma) and results in gastrin hypersecretion with resultant severe gastric

ulceration. The normal physiological stimulation of gastric secretion has three phases:

cephalic

gastric

intestinal.

The cephalic phase is vagally mediated and stimulated by the sight and smell of food.

The parasympathetic input causes gastrin release from antral G -cells, which causes acid

and pepsin secretion. Vagal input also directly stimulates gastric glands and rel eases

histamine from mast cells which activated parietal cells to produce acid. The gastric

phase begins with food entering the stomach, distending it, causing vagal stimulation.

This accounts for ~60% of gastric acid secretion.

The intestinal phase only accounts for ~5% of secretion and gastrin secretion is

mediated by the presence of food in the duodenum.

Histamine antagonism

Histamine agonism, not antagonism, would result in gastric hypersecretion.

Somatostatin secretion

Somatostatin is an inhibitory hormone and acts directly on parietal cells to reduce acid

secretion.

Vagal inhibition

Vagal inhibition would reduce gastric secretion while ZE is characterised by severe

gastric ulceration secondary to increased gastric secretion.

Vagal stimulation

The vagal nerve does play a role in the normal physiological stimulation of gastric

secretion, most notably due to food entering the stomach causing vagal stimulation and

subsequently gastric secretion. However, in ZE syndrome, excessive gastric secretion is

due to increased gastrin secretion.

142
Q

A 67-year old man presents with diarrhoea and abdominal pain. He is found to

have raised levels of urinary 5-HIAA.

Which one of the following symptoms or signs, in addition to his presentation, is

most likely to go with a diagnosis of carcinoid syndrome?

1) Alternating constipation

2) Dysmenorrhoea

3) Flushing

4) Mitral regurgitation

5) Nausea

A

Explanation

Flushing

Carcinoid syndrome occurs secondary to carcinoid tumours which are neuroendocrine

neoplasia. These tumours often secrete serotonin and can produce the following

symptoms;

flushing

telangiectasia

diarrhoea

bronchospasm

right heart valve lesions (tricuspid regurgitation and stenosis, pulmonary stenosis)

pellagra (rarely).

Alternating constipation

Diarrhoea, not constipation, is associated with carcinoid syndrome.

Dysmenorrhoea

Dysmenorrhoea refers to pain during menstruation, and as the patient is male this is

very unlikely indeed!

Mitral regurgitation

The tricuspid valve is more likely to be affected in carcinoid syndrome, and flushing is

more likely than either of these.

Nausea

Nausea is a very non-specific symptom and is present in many conditions. Flushing is

far more characteristic of carcinoid syndrome.

143
Q

A 43-year-old man presents with abdominal pain and is found on

oesophagogastroduodenoscopy (OGD) to have a duodenal ulcer. He is treated with a

proton pump inhibitor and Helicobacter pylori eradication treatment, but the

ulcer is

persistent

.

Gastrin levels

are checked and found to be

raised

.

What syndrome does this man have?

1) Multiple endocrine neoplasia type I

2) Multiple endocrine neoplasia type IIa

3) Multiple endocrine neoplasia type IIb

4) Zollinger–Ellison syndrome

5) None of the above

A

Explanation

Zollinger–Ellison syndrome

Zollinger–Ellison syndrome results from a gastrin-secreting tumour of the islet cells of

the pancreas or rarely of the duodenum or gastric antrum. This causes an increase in

the levels of circulating gastrin. It often presents as intractable duodenal ulceration

with a high incidence of bleeding and perforation. If the ulcer perforates, the patient

may present with the classical signs and symptoms of peritonitis.

Approximately 25% of patients with Zollinger –Ellison syndrome are associated with

multiple endocrine neoplasia type 1, with an associated adenoma of the parathyroid in

approximately 25% of patients and hyperplasia of the adrenal and thyroid in 10% of

patients. Proton pump inhibitors that suppress acid production and promote healing

comprise first-line treatment. H-2 antagonists may also be used, but are less effective

than proton pump inhibitors in reducing acid production. Surgery to remove the

tumour is only undertaken if the ulcers are resistant to the above medical treatment

options.

Multiple endocrine neoplasia type I

MEN I can be associated with Zollinger–Ellison syndrome in around 25% of patients,

however there are no other features of MEN I, for example, hyperparathyroidism.

Multiple endocrine neoplasia type IIa

MEN II is characterised by parathyroid hyperplasia, medullary thyroid carcinoma

and phaeochromocytoma. This condition would not cause raised gastrin levels or

persistent gastric ulceration.

Multiple endocrine neoplasia type IIb

Again, MEN II would not cause persistent ulceration. MEN IIb is differentiat ed from

MEN IIa by the presence of mucosal neuromas. None of the above

The patient’s ulceration has not improved despite appropriate therapy,

and he has been subsequently found to have raised gastrin levels, which is indicative of

Zollinger–Ellison syndrome.

144
Q

Acute hypokalaemia can lead to muscle weakness and direct renal tubular cell injury.

The causes can be classified according to the presence or absence of hypertension with

reference also to the plasma renin activity and urinary potassium excretion.

Which one of the following is a cause of hypokalaemia

without hypertension

but

with high plasma renin activity?

1) Liddle syndrome

2) Cushing’s syndrome

3) Liquorice excess

4) Renovascular disease

5) Gitelman’s syndrome

A

Explanation

Gitelman’s syndrome

Gitelman’s syndrome can be either autosomal recessive or dominant and is

characterised by hypokalaemic metabolic alkalosis and with hypocalciuria and

hypomagnesaemia. Blood pressure is low or normal.

Liddle syndrome

Causes of hypokalaemia with hypertension (potassium excretion usually

> 30 mmol/day) and low plasma renin activity include Liddle’s syndrome, which is

an autosomal-dominantsyndrome of hypertension and variable degrees of

hypokalaemic metabolic alkalosis.

Cushing’s syndrome

In Cushing’s syndrome (sustained overproduction of cortisol) patients will exhibit

hypertension and low plasma renin activity.

Liquorice excess

With liquorice excess, 11-β-hydroxysteroid dehydrogenase metabolises co rtisol and

prevents it from binding to the mineralocorticoid receptor. Acquired or congenital

conditions in which this enzyme is inhibited have a Conn’s phenotype and this includes

patients taking liquorice in excess and renovascular disease.

Renovascular disease

As mentioned, patients with acquired or congenital conditions in which 11-β-

hydroxysteroid dehydrogenase is inhibited have a Conn’s phenotype and this includes

patients taking liquorice in excess and with renovascular disease.

145
Q

A patient admitted under your care with appendicitis underwent successful surgery and

is ready for discharge. You notice their blood glucose has been high throughout their

admission.

The 2011 WHO criterion for diagnosis of diabetes mellitus is whi ch of the

following?

1) Fasting plasma glucose >6.0 mmol/l

2) Random plasma glucose >12.0 mmol/l

3) Plasma glucose <7.8 mmol/l at 2 h after glucose-tolerance test

4) Plasma HbA1c >6.5%

5) Glucose-tolerance test is required for confirmation of diagnosis

A

Explanation

Plasma HbA1c >6.5%

The 2011 WHO recommendations for diagnosis of type II diabetes mellitus suggests

that Hb A 1c can be used as a diagnostic test for diabetes providing that stringent quality

assurance tests are in place and assays are standardised to criteria al igned to the

international reference values, and there are no conditions present that preclude its

accurate measurement.

An Hb A 1c of 48 mmol/mol (6.5%) is recommended as the cut point for diagnosing

diabetes. A value of less than 48 mmol/mol (6.5%) does not exclude diabetes diagnosed

using glucose tests.

Situations when Hb A 1c is not appropriate for diagnosis of diabetes include:

children and young people

patients suspected of having type 1 diabetes

pregnancy

patients with symptoms of diabetes for less tha n 2 months

patients at high diabetes risk who are acutely ill

patients taking medication that may cause rapid glucose rise, eg steroids,

antipsychotics

patients with acute pancreatic damage, including pancreatic surgery

presence of other factors that influence HbA 1c and its measurement (such as

haemoglobinopathies, haemolytic disorders, alcohol excess or chronic kidney

disease).

Fasting plasma glucose >6.0 mmol/l

Diabetes may be diagnosed with a fasting plasma glucose >7.0 mmol/l and associated

diabetic symptoms such as increased thirst/urination, recurrent infections or weight loss. It must be remembered, however, that compliance with the fasting period is often

poor.

Random plasma glucose >12.0 mmol/l

A random plasma glucose of >11.1 mmol/l is sufficient for the diagnosis of diabetes.

Again, symptoms of diabetes must be present.

Plasma glucose <7.8 mmol/l at 2 h after glucose-tolerance test

The glucose-tolerance test can be used to diagnose diabetes. Here, 75 g of glucose is

administered to the patient and if their venous glucose is > 11.1 mmol/l at 2 h following

administration then diabetes can be diagnosed.

Glucose-tolerance test is required for confirmation of diagnosis

The glucose-tolerance test is only one of the several available diagnostic tests that can

be utilised.

146
Q

A 55-year-old woman presents with features consistent with Cushing’s syndrome. She is taking no medication. Her basal cortisol and plasma adrenocorticotrophic hormone

(ACTH

)levels are

significantly raised

. A high-dose dexamethasone suppression test

results in

no decrease

in morning cortisol serum concentration.

What is the most likely diagnosis?

1) Adrenal tumour

2) Carney syndrome

3) Cushing’s disease

4) Depression

5) Ectopic ACTH-secreting tumour

A

Explanation

Ectopic ACTH-secreting tumour

A raised ACTH level with a raised cortisol implies that the problem is caused by excess

ACTH production, otherwise negative feedback would suppress ACTH. Although failure

of cortisol suppression after low-dose dexamethasone may be seen with both pituitary

tumours (ieCushing’s disease) and ectopic ACTH-secreting tumours, high-dose

dexamethasone often results in cortisol suppression for pituitary tumours, as they

retain a degree of negative feedback. A persistently raised cortisol level after high-dose

dexamethasone suppression therefore implies that an ectopic ACTH-secreting tumour is

the most likely pathology.

Adrenal tumour

An adrenal adenoma can produce excess cortisol, however this would have the effect of

suppressing ACTH production due to a negative feedback loop.

Carney syndrome

Carney syndrome comprises atrial myxoma and freckles with high cortisol leve ls

independent of ACTH.

Cushing’s disease

Cushing’s disease refers to a pituitary adenoma that secretes excess ACTH.

Given that the morning cortisol level remains high despite the high-dose

dexamethasone suppression test suggests there is ectopic ACTH production, as even

pituitary tumours retain some negative feedback.

Depression

Depression does not result in persistently raised cortisol and ACTH levels.

147
Q

A patient with known cancer is noted to have high concentrations of calcium in their

blood. Hypercalcaemia can be present in malignant disease secondary to the secretion

of

parathyroid hormone-related peptide

by a tumour.

It is most frequently associated with which one of the following?

1) Carcinoid tumours

2) Lymphoma

3) Multiple myeloma

4) Small-cell carcinoma of the bronchus

5) Squamous-cell carcinoma of the bronchus

A

Explanation

Squamous-cell carcinoma of the bronchus

Hypercalcaemia is a frequent complication of cancer, and is most usually the result of

the secretion of parathyroid hormone-related peptide (paraneoplastic syndrome).

Squamous-cell carcinomas are particularly frequently responsible, but hypercalcaemia

is uncommon with other bronchogenic cancers.

Carcinoid tumours

Carcinoid tumours sometimes secrete vasopressin or adrenocorticotrophic hormone

(ACTH), but rarely cause hypercalcaemia.

Lymphoma

In lymphomas, unregulated extrarenal production of calcitriol can be responsible.

Multiple myeloma

In myeloma, the secretion of osteoclast-activating cytokines is the usual cause of

hypercalcaemia.

Small-cell carcinoma of the bronchus

Small-cell carcinoma of the bronchus is more commonly associated with ectopic ACTH

and antidiuretic hormone (ADH) secretion [the latter leading to syndrome of

inappropriate antidiuretic hormone secretion (SIADH)], as well as Lambert–Eaton

myasthenic syndrome.

148
Q

A 72-year-old

Afro-Caribbean

woman is found to have a serum calcium concentration

of

3.12 mmol/l

. They undergo investigations to delineate the cause.

Which one of the following clinical features, if present, would most direct you

towards a specific cause?

1) Bone pain

2) Hilar lymphadenopathy

3) Polyuria

4) Short QT interval

5) Ureteric colic

A

Explanation

Hilar lymphadenopathy

The presence of hilar lymphadenopathy in a patient (especially an Afro-Caribbean

patient) with hypercalcaemia should raise a suspicion that the latter is due to sarcoid

(in which the granulomas secrete calcitriol, 1,25 -dihydroxycholecalciferol).

Bone pain

Bone pain can occur with hypercalcaemia secondary t o malignancy or

hyperparathyroidism.

Polyuria

Polyuria is a feature of severe hypercalcaemia, irrespective of the cause.

Short QT interval

A short QT interval is also a non-specific feature of hypercalcaemia.

Ureteric colic

Ureteric colic is particularly associated with primary hyperparathyroidism, but is not

specific to this cause.

149
Q

On examination, he has a pre-adolescent body habitus and no evidence of development

of secondary sexual characteristics. Serum testosterone, luteinising hormone (LH) and

follicle-stimulating hormone (FSH) concentrations are all in the prepubertal range. The

LH and FSH concentrations increase only slightly following a single injection

of

gonadotrophin-releasing hormone (GnRH)

, but a

normal response is elicited after

GnRH is given

daily for 7consecutive days.

Which one of the following is most likely to be the cause of his delayed puberty?

1) A hypothalamic disorder

2) A pituitary disorder

3) Klinefelter’s syndrome

4) Mumps orchitis in childhood

5) Seminiferous tubular dysfunction

A

Explanation

A hypothalamic disorder

The combination of low testosterone and low gonadotrophin concentration suggests a

defect at the level of the hypothalamus.

A pituitary disorder

The lack of response to a single injection of GnRH, but normal response following

chronic stimulation, suggests that the mechanism for the secretion of FSH and LH by the

pituitary is intact. Chronic understimulatio n as a result of a lack of GnRH causes

decreased pituitary responsiveness to the hypothalamic hormone, which can be

restored by chronic stimulation.

Klinefelter’s syndrome

In Klinefelter’s syndrome (47XXY) and testicular failure due to mumps orchitis, the lack

of negative feedback leads to increased gonadotrophin concentrations.

Mumps orchitis in childhood

As mentioned, due to testicular failure secondary to mumps, the lack of negative

feedback leads to increased gonadotrophin concentrations.

Seminiferous tubular dysfunction

In seminiferous tubule dysfunction, the plasma FSH concentration tends to be elevated;

spermatogenesis is reduced or absent but masculinisation is normal.

150
Q

A 64-year-old woman presents with a painless lump she has noticed on the front

neck which moves on swallowing. On examination, she is noted to have a

thyroid

of her

nodule

and cervical

lymphadenopathy

. A fine-needle aspiration (FNA) of the thyroid

nodule reveals

psammoma bodies

.

What is the most likely diagnosis?

1) Anaplastic thyroid carcinoma

2) Follicular thyroid carcinoma

3) Lymphoma

4) Medullary thyroid carcinoma

5) Papillary thyroid carcinoma

A

Explanation

Papillary thyroid carcinoma

Papillary carcinoma of the thyroid gland accounts for around two-thirds of thyroid

malignancies. The tumour spreads via the lymphatics and at presentation over half of

patients will have cervical lymph node involvement. Histologically psammoma bodies

are typical.

Anaplastic thyroid carcinoma

Anaplastic carcinoma is a very aggressive tumour type that is most commonly found in

the elderly. It has a very poor prognosis with few patients surviving beyond 1 year.

Follicular thyroid carcinoma

Follicular thyroid carcinoma accounts for 20% of thyroid tumours and classically

spreads via the bloodstream. Therefore lymph node involvement at presentation is very

uncommon.

Lymphoma

Thyroid lymphoma is very rare affecting predominantly women over the age of 70

years, many of whom have a diagnosis of Hashimoto’s thyroiditis.

Medullary thyroid carcinoma

Medullary thyroid carcinoma occurs in 5 –10% of thyroid malignancies. It is derived

from C-cells and therefore produces calcitonin, which can be measured in the blood.

151
Q

A 42-year-old man is referred to his GP because of problems in his workplace linked to his mood swings. On taking a detailed history he reports episodes of light-headedness,

disorientation

with an associated

aggressive

temper. These

resolve

following

the

consumption of food

. He has previously been investigated and was found to have a

blood sugar of

1.8

mm/l during an attack.

What is the most likely diagnosis?

1) Gastrinoma

2) Glucagonoma

3) Insulinoma

4) Phaeochromocytoma

5) Zollinger-Ellison syndrome

A

Explanation

Insulinoma

Insulinomas are not common and occur in around 1 in a million of the population. They

arise from β islet cells and are mostly benign. They often cause symptoms relating to

hypoglycaemia, eg light-headedness, mood swings and in some cases unconsciousness.

The diagnosis should be suspected when Whipple’s triad is present – attacks brought on

by fasting, hypoglycaemia (<2.0 mmol/l) during an episode and symptoms then

improving with glucose administration.

Gastrinoma

A gastronome is a tumour found in the pancreas or duodenum that secretes gastrin. It is

the most common cause of Zollinger–Ellison syndrome.

Glucagonoma

A glucagonoma is a rare tumour of the alpha cells within the pancreas causing excess

levels of glucagon resulting in a high plasma glucose.

Phaeochromocytoma

Phaeochromocytoma is a neuroendocrine tumour of the adrenal gland that secretes

excess catecholamines.

Zollinger-Ellison syndrome

Zollinger–Ellison syndrome is characterised by severe gastric ulceration secondary to

tumour secreting gastrin.

152
Q

A 55-year-old woman is referred to a general surgery clinic after a dermatologist has diagnosed a migratory necrolytic erythema. The patient also has a history of diabetes

mellitus and weight loss.

What is the most likely diagnosis?

1) Cushing’s syndrome

2) Gastrinoma

3) Glucagonoma

4) Insulinoma

5) Phaeochromocytoma

A

Explanation

Glucagonoma

Glucagonomas are very rare tumours with an incidence of around 1 in 20 million. There

are often metastases present when the patient presents. Classic features of

glucagonomas are migratory necrolytic erythema (bright red, papular and blister-type

rash that can affect any part of the body, especially around the mouth), diabetes mellitus

and weight loss (somewhat paradoxical). The rash is very characteristic and may be

sufficient evidence to raise the concern itself. It is secondary to a zinc deficiency in the

skin.

Cushing’s syndrome

Cushing’s syndrome refers to prolonged exposure to cortisol and has several

aetiologies, including iatrogenic steroid administration and pituitary tumours. Weight

gain, not weight loss, is associated with Cushing’s syndrome. Skin changes, such as

abdominal striae, may be apparent.

Gastrinoma

A gastrinoma is a tumour found in the pancreas or duodenum that secretes gastrin. It is

the most common cause of Zollinger–Ellison syndrome.

Insulinoma

Insulinomas are not common and occur in around 1 in a million of the population. They

arise from β islet cells and are mostly benign. They often cause symptoms relating to

hypoglycaemia, eg light-headedness, mood swings and in some cases unconsciousness.

The diagnosis should be suspected when Whipple’s triad is present – attacks brought on

by fasting, hypoglycaemia (<2.0 mmol/l) during an episode and symptoms then

improving with glucose administration.

Phaeochromocytoma

Phaeochromocytoma is a neuroendocrine tumour of the adrenal gland that secretes

excess catecholamines.

153
Q

A 35-year-old woman

it emerges that this is

is referred by her GP with hyperhidrosis. On intermittent in nature and is precipitated by

further questioning exercise and

overeating

. In addition to this she reports severe headaches and

palpitations. On a

previous, occasion her blood pressure was measured and was found to be

190/110

mmHg

.

What is the most likely diagnosis?

1) Cushing syndrome

2) Gastrinoma

3) Neuroblastoma

4) Phaeochromocytoma

5) Zollinger-Ellison syndrome

A

Explanation

Phaeochromocytoma

Phaeochromocytomas are rare catecholamine-producing tumours that arise from

sympathetic paraganglia cells that are collections of adrenaline-secreting chromaffin

cells. They occur in around one in a million people and are the underlying cause for less

than 1% of hypertension. Around 10% are bilateral, 10% are malignant, 10% are

familial and 10% are extra-adrenal (the ‘rule of 10s’). The most common extra-adrenal

site is the organ of Zuckerkandl, which is by the aortic bifurcation. The most common

symptom is sweating, and other symptoms include paroxysmal headaches,

palpitations and precordial pain. Pre-operative management with Alpha- and Beta-

blockers is crucial before anaesthetic induction to minimise morbidity and mortality.

Cushing syndrome

Cushing syndrome refers to prolonged exposure to cortisol and has several aetiologies,

including iatrogenic steroid administration and pituitary tumours. While it cause

hypertension, headaches, palpitations and hyperhidrosis are not recognised symptoms

of Cushing syndrome.

Gastrinoma

A gastrinoma is a tumour found in the pancreas or duodenum that secretes gastrin. It is

the most common cause of Zollinger–Ellison syndrome.

Neuroblastoma

Neuroblastoma is a malignant tumour of nerve cells and can commonly arise in the

adrenal glands, spinal cord and brain, however it would not account for the symptoms

and signs described in the case history.

Zollinger-Ellison syndrome

Zollinger–Ellison syndrome is characterised by severe or recurrent peptic

ulcerationsecondary to a gastrin-secreting tumour

154
Q

A 27-year-old man is referred to an endocrine clinic as he has been complaining of excessive thirst and polyuria. Random and fasting plasma glucose levels are normal

,

and his urine is noted to have a

low specific gravity

and osmolality.

Which of the following is the most likely cause?

1) Diabetes insipidus

2) Glucagonoma

3) Insulinoma

4) Type 1 diabetes mellitus

5) Type 2 diabetes mellitus

A

Explanation

Diabetes insipidus

Diabetes insipidus (DI) is an uncommon condition with excessive production of urine

and thirst. It is classified according to the source of the problem as cranial DI [deficiency

in antidiuretic hormone (ADH) production] or nephrogenic DI (renal tubules

unresponsive to ADH). The urine has a low specific gravity and osmolality.

Glucagonoma

A glucagonoma is a rare tumour of the alpha cells within the pancreas, causing excess

levels of glucagon resulting in a high plasma glucose.

Insulinoma

Insulinomas are not common and occur in around one in a million of the population.

They arise from β islet cells and are mostly benign. They often cause symptoms relating

to hypoglycaemia, eg light-headedness, mood swings and in some cases

unconsciousness.

Type 1 diabetes mellitus

As the random and fasting glucose levels are within the normal range, this makes a

diagnosis of diabetes mellitus (DM) less likely Furthermore, type 1 DM is often

diagnosed at an earlier age than this patient.

Type 2 diabetes mellitus

Again, as the plasma glucose levels are normal, this makes a diagnosis of diabetes

mellitus less likely.

155
Q

A 39-year-old man is admitted to the intensive care unit (ICU) following a road traffic

accident for which he has sustained multiple injuries. One of these is a skull base

fracture and he is being investigated for possible pituitary dysfunction.

Which one of the following hormones is secreted by the

posterior

pituitary gland?

1) Follicle-stimulating hormone (FSH)

2) Growth hormone

3) Prolactin

4) Thyroid-stimulating hormone

5) Vasopressin

A

Explanation

Vasopressin

The pituitary gland is divided into two parts - the anterior pituitary gland

(adenohypophysis) and posterior (neurohypophysis). These have different

embryological origins and different functions. The neurohypophysis (posterior

pituitary) secretes oxytocin and vasopressin (ADH).

Follicle-stimulating hormone (FSH)

The adenohypophysis produces gonadotrophins (LH and FSH), growth hormone,

prolactin, thyroid-stimulating hormone and adrenocorticotrophic hormone (ACTH).

Growth hormone

The anterior pituitary produces growth hormone.

Prolactin

Prolactin is produced by the anterior pituitary.

Thyroid-stimulating hormone

The anterior pituitary produces thyroid -stimulating hormone (TSH).

156
Q

A final-year medical student is on his elective in

South America

and is intrigued to find a

series of patients with very large goitres who are otherwise well. Blood tests reveal

these patients are euthyroid.

What is the most likely cause for the goitres?

1) Autoimmune hypothyroidism

2) Hashimoto’s thyroiditis

3) Iodine deficiency

4) Myxoedema coma

5) Toxic multinodular goitre

A

Explanation

Iodine deficiency

Iodine deficiency can cause a goitre and is still found in mountainous areas, for example

the Alps, Himalayas, and Andes. The thyroid function may be euthyroid or hypothyroid

depending on the severity of the iodine deficiency. Iodine deficiency causes a

generalised hyperplasia of the thyroid gland resulting in a large goitre (the so-called

endemic goitre).

Autoimmune hypothyroidism

Autoimmune thyroiditis can present with a goitre and the patient may be either hyper -

or hypothyroid. There is a familial element to autoimmune hypothyroidism, and it can

also occur in the context of high iodine consumption.

Hashimoto’s thyroiditis

Hashimoto’s thyroiditis is an autoimmune condition that leads to gradual destruction of

the thyroid gland resulting in hypothyroidism and a large painless goitre.

Myxoedema coma

This occurs in the context of severe hypothyroidism – the patients in this case history

are euthyroid.

Toxic multinodular goitre

A toxic multinodular goitre is a common cause of hyperthyroidism, second only to

Grave’s disease.

157
Q

A 47-year-old man is referred to an

Endocrine Clinic

for further investigation as he has

been complaining of a headache, sweating and his GP thinks his appearance

has

changed

since his last appointment 10 years ago. On examination he has

a

prominent

supraorbital ridge,

prognathism

and

interdental separation

.

Which one of the following tests are of most diagnostic value in confirming

acromegaly?

1) Growth hormone (GH) levels

2) Insulin-like growth factor-1 (IGF-1) levels

3) Oral glucose-tolerance test with GH levels

4) Prolactin levels

5) Visual field testing

A

Explanation

Oral glucose-tolerance test with GH levels

In normal subjects an oral glucose-tolerance test will suppress GH to <2 mU/l, in

patients with acromegaly, it is not suppressed, and increases in 50%.

Growth hormone (GH) levels

Serum GH levels are often elevated, but levels can fluctuate and a normal level

does notexclude a diagnosis.

Insulin-like growth factor-1 (IGF-1) levels

IGF-1 levels are often raised with acromegaly.

Prolactin levels

Prolactin levels are raised in 30% of patients, but again does not confirm the diagnosis.

Visual field testing

Although visual field defects are common, this does not confirm the diagnosis.

158
Q

A 57-year-old woman is referred to an Endocrine Clinic with an increase in proximal myopathy and thin skin. She is thought to have Cushing syndrome weight, after a low-
dose dexamethasone suppression test shows that the serum cortisol concentration
remains high . A further high-dose dexamethasone suppression test shows the serum cortisol level does decrease after 48 h of dexamethasone.
What is the most likely source for her Cushing’s syndrome?

1) Adrenal adenoma

2) Adrenal carcinoma

3) Ectopic ACTH

4) Iatrogenic cause

5) Pituitary disease

A

Explanation

Pituitary disease

Most patients with pituitary-dependent Cushing’s syndrome, ie Cushing’s disease will

suppress plasma cortisol following a high -dose dexamethasone test as illustrated in this

case.

Adrenal adenoma

Adrenal adenomas may produce cortisol, however this would result in suppression

of adrenocorticotrophic hormone (ACTH) due to the negative feedback loop to the

pituitary.

Adrenal carcinoma

Adrenal carcinoma are extremely rare and may produce cortisol, although again this

would result in suppression of ACTH.

Ectopic ACTH

A failure to suppress ACTH would suggest an ectopic source of ACTH or an adrenal

tumour.

Iatrogenic cause

Although the most common cause for Cushing’s syndrome is exogenous steroid

administration, this would have been excluded in the history for this patient.

159
Q

A patient who is being followed up for a bilateral

adrenalectomy

for Cushing syndrome

is noted to have increased pigmentation and complains of headaches in his follow-up

appointment. Visual field testing also demonstrates

bitemporal hemianopia

which was

not present pre-operatively.

What complication has the patient developed?

1) Acromegaly

2) Addison’s disease

3) Diabetes insipidus

4) Nelson’s syndrome

5) Waterhouse–Friderichsen syndrome

A

Explanation

Nelson’s syndrome

Nelson’s syndrome involves a combination of skin hyperpigmentation and an expanding

pituitary tumour. It mostly occurs following an adrenalectomy for Cushing’s disease and

is caused by the reduction in circulating cortisol causing reduced negative feedback on

the pituitary gland, and therefore and increased ACTH production. This increase in

ACTH levels causes hyperpigmentation as it is physically related to and

resembles melanocyte-stimulating hormone (MSH).

Acromegaly

Acromegaly is a disorder, often of the pituitary gland, resulting in excess production of

growth hormone. It can present with visual field defects, however, given the history of

the adrenalectomy, Nelson syndrome is more likely.

Addison’s disease

Addison’s disease is an autoimmune disorder resulting in primary adrenal insufficiency.

Diabetes insipidus

Diabetes insipidus is a condition whereby either the kidneys are not responsive

to antidiuretic hormone (ADH) or there is a deficiency in production of ADH. This

patient does not have excess thirst and micturition. Furthermore, visual field defects

would not be expected.

Waterhouse–Friderichsen syndrome

This syndrome refers to haemorrhage within the adrenal glands, most commonly

secondary to Neisseria meningitidis infection. These patients are often desperately

unwell with overwhelming sepsis, for example as seen with meningitis.

160
Q

The paediatric surgical team are asked to review a neonate with ambiguous external

genitalia. Further testing reveals a female genotype and it is thought she

has pseudohermaphroditism due to congenital adrenal hyperplasia (CAH).

What is the most common enzyme deficiency in patients with CAH?

1) 3ß-hydroxysteroid dehydrogenase

2) 11β-Hydroxylase

3) 11α-Hydroxylase

4) 17-Hydroxylase

5) 21-Hydroxylase

A

Explanation

21-Hydroxylase

The most common cause for CAH is an enzyme deficiency of 21 -hydroxylase (in 90% of

cases). This is essential in cortisol synthesis, and reduced cortisol levels cause an

increase in ACTH which results in increased precursors of cortisol which are instead

switched to testosterone production. In women this can manifest

as pseudohermaphroditism and in males as precocious sexual maturity.

3ß-hydroxysteroid dehydrogenase

This enzyme has a role in the production of progesterone and therefore is not

implicated in CAH.

11β-Hydroxylase

This enzyme is involved in the final steps of synthesis of cortisol. Deficiencies can cause

CAH, however it is less common than 21-hydroxylase.

11α-Hydroxylase

This enzyme is involved in the hydroxylation of steroids such as cortisol and does not

result in CAH.

17-Hydroxylase

Deficiency in this hormone can result in CAH, however it is uncommon.

161
Q

A 54-year-old woman presents to the GP with palpitations and heat intolerance. Further

questioning reveals an increased appetite, weight loss and persistent diarrhoea. It is

thought she may have thyrotoxicosis.

What is the most common cause of thyrotoxicosis?

1) Congenital

2) Graves’ disease

3) Multinodular toxic goiter

4) Riedel’s thyroiditis

5) Toxic solitary nodule

A

Explanation

Graves’ disease

Thyrotoxicosis affects almost 2% of the population and in 95% of cases is the result of

one of three causes: Graves’ disease, multinodular toxic goitre and a toxic solitary

nodule. The commonest of these is Graves’ disease, which is most common in women

between 20 and 40 years. Grave’s disease is also associated with eye changes

(exophthalmos, ophthalmoplegia) and pretibial myxoedema, in addition to the other

signs and symptoms of hyperthyroidism.

Congenital

Congenital abnormalities can affect the thyroid gland, however this is more commonly

hypothyroidism and would present at birth or shortly after.

Multinodular toxic goitre

Multinodular toxic goitre can cause thyrotoxicosis, however Graves’ disease is a more

common cause.

Riedel’s thyroiditis

Riedel’s thyroiditis (woody hard thyroid) is rare and results in painless, chronic fibrosis

of the thyroid gland. Some patients can become hypothyroid, however, very few become

hyperthyroid.

Toxic solitary nodule

Again, a toxic solitary nodule may cause thyrotoxicosis, however it is much less common

than either Graves’ disease or multi nodular toxic goitre.

162
Q

A 33-year-old woman is noted to have a raised serum calcium when being investigated

following renal calculi. Further investigation reveals raised serum parathyroid hormone

(PTH) levels and she is diagnosed with primary hyperparathyroidism.

Approximately what proportion of primary hyperparathyroidism is caused by a

parathyroid

carcinoma

?

1) 1%

2) 5%

3) 10%

4) 30%

5) 50%

A

Explanation

1%

Primary hyperparathyroidism is caused by a solitary adenoma in 85 –90% of patients,

hyperplasia in around 15–10% and a carcinoma in less than 1%. It may be diagnosed

through routine blood tests on asymptomatic patients who are noted to have

hypercalcaemia, or patients may develop symptoms. These symptoms mostly affect the

kidneys and bone, and include nephrocalcinosis and renal calculi, and demineralisation

of bone.

5%

Parathyroid carcinoma is rare and causes primary hyperparathyroidism and is the

underlying pathology in less than 1% of cases. Typically, the serum PTH concentration

is extremely high, unlike for a parathyroid adenoma or hyperplasia, in

which the PTH concentration is increased, but not massively.

10%

Parathyroid carcinomas are rare, accounting for less than 1% of cases of primary

hyperparathyroidism. Parathyroid gland hyperplasia, however, accounts for 10–15% of

cases of hypercalcaemia with raised PTH levels.

30%

Parathyroid carcinomas are rare, accounting for less than 1% of cases of primary

hyperparathyroidism.

50%

Parathyroid adenomas are the most common cause for primary hyperparathyroid ism.

Parathyroid carcinomas are rare.

163
Q

A 42-year-

successful

old man is noted to have raised plasma calcium levels 6 months after a renal transplantation for end-stage renal failure secondary to polycystic

kidney disease. He has had no problems with his graft and no longer requires dialysis.

Further blood tests reveal

raised parathyroid hormone (PTH)

levels.

What is the most likely diagnosis in this patient?

1) Hypoparathyroidism

2) Primary hyperparathyroidism

3) Pseudohypoparathyroidism

4) Secondary hyperparathyroidism

5) Tertiary hyperparathyroidism

A

Explanation

Tertiary hyperparathyroidism

Tertiary hyperparathyroidism develops when the excessive PTH secretion in secondary

hyperparathyroidism (eg due to chronic hypocalcaemia in renal failure) becomes

autonomous. This often occurs when the underlying cause for chronic hypocalcaemia is

corrected, eg following a renal transplant. Management options for tertiary

hyperparathyroidism include total parathyroidectomy, subtotal parathyroidectom y or

autotransplantation of parathyroid tissue into an arm muscle (where it can be easily

located if problems persist). Patients require calcium and vitamin D replacement to

correct bone disease and reduce the risk of recurrence.

Hypoparathyroidism

Hypoparathyroidism would present with low levels of PTH.

Primary hyperparathyroidism

Primary hyperparathyroidism is usually as a result of parathyroid adenomas. Given the

history of chronic kidney disease and subsequent hyperparathyroidism, tertiary

hyperparathyroidism is the likely cause.

Pseudohypoparathyroidism

This is an extremely rare condition in which there is resistance to PTH, resulting in

hypocalcaemia and appropriately high PTH levels. Given the history of renal transplant

and rarity of pseudohypoparathyroidism, tertiary hyperparathyroidism is more likely.

Secondary hyperparathyroidism

Secondary hyperparathyroidism occurs in the context of chronic kidney disease

resulting in hypocalcaemia, this stimulates the parathyroid glands to increase

production of PTH.

164
Q

You are assisting your consultant in the ENT clinic. You see a patient in clinic with a

malignant thyroid swelling.

Which one of the following is correct regarding the types of thyroid cancer?

1) Anaplastic cancer has a good prognosis

2) Anaplastic cancer is commoner in younger patients

3) Follicular carcinoma cannot be diagnosed on fine-needle aspiration (FNA)

4) Follicular carcinoma is the most common type

5) Medullary cancer only occurs in patients with multiple endocrine neoplasia type

II (MEN II)

A

Explanation

Follicular carcinoma cannot be diagnosed on fine-needle aspiration (FNA)

Follicular carcinoma cannot be differentiated from follicular adenoma by FNA.

Anaplastic cancer has a good prognosis

Anaplastic cancer has a very poor prognosis, and is usually seen in older patients.

Anaplastic cancer is commoner in younger patients

As mentioned, it is more commonly in the elderly, particularly women over the age of 70

years.

Follicular carcinoma is the most common type

Papillary adenocarcinoma is the commonest type (70%). Female to male ratio for

thyroid cancer is 3:1.

Medullary cancer only occurs in patients with multiple endocrine neoplasia type II

(MEN II)

Medullary cancer of the thyroid is associated with the MEN II syndrome but most

patients who have medullary cancer will not have MEN.

165
Q

A 55-year-old man with a history of

secondary to alcohol is being managed conservatively as an outpatient. It is decided to assess the exocrine function

chronic pancreatitis

of

his pancreas before nutritional interventions.

Which one of the following is the optimal technique for assessing the exocrine

status of the pancreas?

1) Albumin levels

2) Barium meal

3) CT

4) Lundh meal

5) Ultrasound

A

Explanation

Lundh meal

The Lundh meal is a combination of skimmed milk powder mixed with corn oil and

dextrose used to assess exocrine pancreatic function. Following administration, serum

lipase measurements can provide a good indicator of exocrine pancreatic status.

The faecal elastase test is increasingly used for detecting exocrine insufficiency and is

more acceptable to patients.

Albumin levels

Serum albumin level may be decreased for many reasons eg inadequate diet, sepsis,

nephrotic syndrome. It is more commonly used to assess the synthetic function of the

liver.

Barium meal

A barium meal will not provide a functional assessment of the exocrine status of the

pancreas.

CT

While CT can be very good for imaging the pancreas and assessing anatomical

abnormalities and gland atrophy, it does not provide any information about the

exocrine status of the pancreas.

Ultrasound

Again, ultrasound imaging will not provide a function assessment of the exocrine

function of the pancreas.

166
Q

A 20-year-old medical student presents with a history of epigastric pain and family

history of multiple endocrine neoplasia (

MEN) syndrome. He undergoes endoscopy

which shows

gastric erosions

. He is a non-smoker.

Which one of the following statements is most likely to be correct?

1) Blood tests show a high serum gastrin level

2) Family history is unlikely to be relevant

3) No further tests are required

4) Proton pump inhibitors are the mainstay of management

5) The aetiology is likely to be alcohol related

A

Explanation

Blood tests show a high serum gastrin level

This patient has Zollinger–Ellison syndrome. This syndrome is rare and can be linked

with MEN type 1. It is characterised by high serum gastrin levels as a result of a gastrin -

secreting neuroendocrine tumour usually situated in the pancreas. Treatment involves

proton pump inhibitors and surgical excision of tumour or chemotherapy.

Family history is unlikely to be relevant

Family history is important when considering MEN syndrome, which is associated with

Zollinger–Ellison syndrome.

No further tests are required

The patient has gastric erosions at a young age and warrants further investigation,

particularly in light of the family history of MEN syndrome.

Proton pump inhibitors are the mainstay of management

Proton pump inhibitors are an important aspect of treatment in Zollinger–

Ellison syndrome, however, the patient is likely to require surgical excision of the

tumour or chemotherapy.

The aetiology is likely to be alcohol related

Alcohol can increase the risk of gastritis and associated erosions, and while medical

students often have a high alcohol consumption there is no mention of excessive alcohol

intake in this case history.

167
Q

A 27-year-old woman presents with

pain in the front of her neck which radiates to her jaw and ears. She reports she has been generally run down and lethargic for a few days.

On examination she has a slight swelling in the anterior neck which elevates on swallowing and which exacerbates her pain. Blood tests show slightly elevated T4 and T3 levels.

What is the most likely diagnosis?

1) De Quervain’s thyroiditis

2) Graves’ disease

3) Hashimoto’s thyroiditis

4) Post-partum thyroiditis

5) Riedel’s thyroiditis

A

The most likely diagnosis for the 27-year-old woman presenting with these symptoms is 1) De Quervain’s thyroiditis.

Here are the key points supporting this diagnosis:

  • Neck Pain and Radiating Symptoms: The patient reports pain in the front of her neck that radiates to her jaw and ears, which is characteristic of De Quervain’s thyroiditis.
  • General Malaise and Lethargy: She has been feeling run down and lethargic, which aligns with the systemic symptoms often associated with De Quervain’s thyroiditis, such as myalgia, malaise, and fatigue.
  • Thyroid Swelling and Pain: The slight swelling in the anterior neck that exacerbates her pain on swallowing is consistent with De Quervain’s thyroiditis, which typically presents with a tender and swollen thyroid gland.
  • Elevated T4 and T3 Levels: The blood tests showing slightly elevated T4 and T3 levels are indicative of the initial hyperthyroid phase seen in De Quervain’s thyroiditis.

These features collectively point towards De Quervain’s thyroiditis as the most likely diagnosis.

168
Q

A patient presents with tiredness. High titres of antithyroid microsomal and

antithyroglobulin

antibodies

are found.

Which one of the following diagnoses is suggested?

1) Hashimoto’s thyroiditis

2) Reidel’s thyroiditis

3) Grave’s disease

4) Hypoparathyroidism

5) Idiopathic hypothyroidism

A

Explanation

Hashimoto’s thyroiditis

The condition associated with antithyroid microsomal antibodies (antithyroid

peroxidase antibodies) and antithyroglobulin antibodies, presenting with a complaint of

tiredness is Hashimoto’s thyroiditis. Hashimoto’s thyroiditis is an autoimmune thyroid

condition, in which the presence of autoantibodies leads to cell d estruction in the

thyroid, reduced hormone output and hypothyroidism. It is the most common cause

of hypothyroidism in areas where there is adequate iodine intake. It is 10 times more

common in women and tends to present in the ages of 30 –50, however it can present at

any point during a patient’s lifetime. Hashimoto’s thyroiditis is associated with other

autoimmune conditions such as pernicious anaemia and coeliac disease. It also

increases the risk of developing lymphoma of the thyroid at a later stage in life. Patients

tend to present with symptoms as described in the mnemonic below, of insidious onset,

gradually worsening and the presence of a diffuse goitre. Treatment is with thyroid

hormone supplementation. Surgery is not usually required unless there a re

compression symptoms from the goitre. Administration of steroids can assist in the

shrinking of the goitre pre-operatively. Hypothyroidism is 10 times more common in

women than men and occurs mainly in middle life.

Reidel’s thyroiditis

Riedel’s thyroiditis is a rare condition. It describes the chronic fibrosis of the thyroid

gland resulting in a rapidly growing, painful, hard and fixed thyroid mass. Fibrosis can

extend outside the thyroid gland to surrounding structures.

Grave’s disease

Grave’s disease presents is a form of hyperthyroidism, commonly presenting with a

toxic goitre. It is associated with the presence of IgG thyroid -stimulating

immunoglobulins, that bind to the thyroid-stimulating hormone receptors and stimulate

the production of thyroid hormones.

Hypoparathyroidism

Hypoparathyroidism is most commonly secondary to total thyroidectomy. It can also be

idiopathic or due to inability of the bones or kidneys to respond to parathyroid

hormone. In any case, it is not characterised by the presence of autoantibodies. Patients

present with symptoms relating to hypocalcaemia such as paraesthesia in the

extremities and peri-orally (circumoral), abdominal cramps and tetany. Idiopathic hypothyroidism

The term idiopathic is used to describe disease where the cause is unknown therefore it

does not apply in this case.

169
Q

A 35-year-old woman presents with a neck lump which is hard and, on examination,

moves up with swallowing, and palpable cervical lymphadenopathy is present. Her only

other history of note is that she is from the Ukraine and grew up

near Chernobyl

and

informs you her older sister had a similar problem.

Which of the following thyroid tumours are

associated with radiation exposure

?

1) Anaplastic

2) Follicular

3) Lymphoma

4) Medullary

5) Papillary

A

Explanation

Papillary

Papillary carcinoma is the most common thyroid carcinoma, accounting for about 80%

of thyroid malignancies. It is a well-differentiated tumour, most commonly presenting in

women aged 35–40. It can be familial and is associated with exposure to radiation, most

significantly if this occurred in childhood.

It most commonly presents as a solitary thyroid nodule that radiologically appears as a

mass with irregular borders and is non-capsulated. It metastasises to the cervical lymph

nodes and rarely to the lungs.

Treatment is with surgical thyroidectomy ± regional lymph node clearance and

radioactive iodine administration for positive lymph nodes.

If the tumour is confined to the thyroid gland on diagnosis, it has an excellent prognosis

and a 5-year survival rate of 95%.

Anaplastic

Anaplastic thyroid carcinoma is rare (1% of all thyroid malignancies), is highly

malignant and has a very poor prognosis.

Follicular

Follicular carcinoma is the second most common type of thyroid malignancies and is

more common in areas where there is a higher incidence of iodine deficiency.

Lymphoma

Primary thyroid lymphoma is a non-Hodgkin’s B-cell lymphoma associated with

Hashimoto’s thyroiditis.

Medullary

Medullary thyroid carcinoma accounts for about 5% of all thyroid malignancies an d is

associated with multiple endocrine neoplasia type 2.

170
Q

A

25-year-old

woman was seen in the Emergency Department complaining

of

vagueabdominal pain and some pain in her extremities. On further enquiry she

revealed that she had been feeling

depressed

for several months. Physical examination

revealed no major findings. A chest X-ray was normal. Serum biochemistry revealed the

following:

Result

Normal

Calcium

3.47 mmol/l

2.20–2.60 mmol/l

Serum albumin

38 g/l

35–55 g/l

Phosphate

0.64 mmol/l

0.70–1.40 mmol/l

What is the most likely diagnosis in this patient?

1) Chronic renal failure

2) Metastatic carcinoma

3) Parathyroid adenoma

4) Pituitary adenoma

5) Thyroid carcinoma

A

Explanation

Parathyroid adenoma

The patient has hypercalcaemia and hypophosphataemia, pointing towards the

diagnosis of primary parathyroid disease. This patient most likely has primary

hyperparathyroidism, the most common cause of which is a single parathyroid

adenoma (85%). Other causes include: parathyroid gland hyper plasia affecting

all four parathyroid glands (10–15%), multiple parathyroid adenomas (3–5%) or a

parathyroid carcinoma (1%).

The adenoma produces an excess of parathyroid hormone leading to: excessive calcium

resorption from bone, increased renal calcium reabsorption, increased renal phosphate

excretion and increased production of 1,25-dihydroxyvitamin D3 .

Therefore, blood tests reveal: a raised PTH, hypercalcaemia, hypophosphataemia and a

raised vitamin D3 .

Treatment of a parathyroid adenoma is by surgical removal of the parathyroid glands.

For patients not fit/suitable for parathyroidectomy options for treatment include:

preventing vitamin D deficiency, bisphosphonates to protect the bones, calcimimetics to

control hypercalcaemia if required.

Chronic renal failure Chronic kidney failure leads to secondary hyperparathyroidism. The diseased kidney

fails to convert an adequate level of vitamin D to its active form, which reduces the

amount of calcium absorbed from the intestines, and fails to excrete phosphate leading

to formation of calcium phosphate compounds in the circulation and a reduction in free

circulating calcium. Therefore, in secondary hyperparathyroidism there is a low serum

concentration of calcium and vitamin D, a high serum concentration of para thyroid

hormone and a high serum phosphate.

Metastatic carcinoma

The two most common causes of hypercalcaemia are primary hyperparathyroidism and

hypercalcaemia of malignancy. The two are generally quite difficult to differentiate. In

both calcium is raised (with hypercalcaemia of malignancy causing a more dramatic

hypercalcaemia) and phosphate is low. parathyroid hormone (PTH) in primary

hyperparathyroidism is raised whereas in hypercalcaemia of malignancy it can be

low/normal/raised depending on the mechanism involved.

Given the patient’s age and the fact that the symptoms have been ongoing for several

months make parathyroid adenoma a far more likely diagnosis than metastatic

carcinoma. The most common cancers associated with hypercalcaemia are: breast, l ung

and myeloma.

Pituitary adenoma

A pituitary adenoma causes visual symptoms (bitemporal hemianopia), headaches and

symptoms relating to the cellular origin of the adenoma. It does not commonly cause

hypercalcaemia.

Thyroid carcinoma

Thyroid carcinoma can lead to hypercalcaemia, but this is usually seen with advanced

disease.

171
Q

A 3-week -old boy presents with non-bilious vomiting, hypochloraemic metabolic alkalosis , hypokalaemia. On examination there is an epigastric mass. What is the most likely diagnosis?

1) Bacterial gastroenteritis

2) Gastro-oesophageal reflux disease

3) Obstructed umbilical hernia

4) Pyloric stenosis

5) Wilms’ tumour

A

Diagnosis of the 3-Week-Old Boy with Symptoms

To determine the most likely diagnosis for the 3-week-old boy presenting with non-bilious vomiting, hypochloraemic metabolic alkalosis, hypokalaemia, and an epigastric mass, we need to analyze each symptom and correlate it with the potential diagnoses provided.

Symptom Analysis

Non-bilious vomiting: This type of vomiting is often associated with conditions affecting gastric outflow. In infants, this can indicate a blockage or narrowing in the pylorus (the opening from the stomach to the small intestine).

Hypochloraemic metabolic alkalosis: This condition arises when there is a loss of hydrochloric acid (HCl) from the stomach due to persistent vomiting. The loss of chloride leads to low levels of potassium (hypokalaemia) as well.

Hypokalaemia: Low potassium levels are commonly seen in cases of prolonged vomiting or conditions that lead to significant fluid loss.

Epigastric mass: The presence of a palpable mass in the epigastric region is particularly indicative. In infants, this could suggest hypertrophy of the pyloric muscle, which is characteristic of pyloric stenosis.

Differential Diagnosis Evaluation

Now let’s evaluate each option:

Bacterial gastroenteritis: Typically presents with diarrhea rather than isolated vomiting and would not usually cause an epigastric mass.

Gastro-oesophageal reflux disease (GERD): While it can cause vomiting in infants, it does not typically present with an epigastric mass or significant electrolyte imbalances like hypochloraemic metabolic alkalosis.

Obstructed umbilical hernia: This condition may cause vomiting but would more likely present with signs of bowel obstruction and pain rather than a specific epigastric mass.

Pyloric stenosis: This condition is characterized by non-bilious projectile vomiting due to gastric outlet obstruction caused by hypertrophy of the pyloric sphincter muscle. The classic presentation includes electrolyte imbalances such as hypochloraemic metabolic alkalosis and hypokalaemia, along with a palpable “olive-like” mass in the right upper quadrant or epigastric area.

Wilms’ tumour: Although it can present as an abdominal mass in children, it typically does not cause non-bilious vomiting or significant electrolyte disturbances like those described here.

Conclusion

Given all these considerations, the symptoms presented by this infant—non-bilious vomiting, hypochloraemic metabolic alkalosis, hypokalaemia, and an epigastric mass—are most consistent with:

Pyloric stenosis

172
Q

A 3-week-old boy presents with non-bilious vomiting, hypokalaemia and hypochloraemic

alkalosis. On examination he has an epigastric mass. He is diagnosed with

pyloric stenosis

and is

taken to theatre to undergo

Ramstedt’s pyloromyotomy.

To which level should the incision be made?

1) Circular muscle layer

2) Longitudinal muscle layer

3) Mucosa

4) Serosa

5) Submucosa

A

Explanation

Mucosa

For Ramstedt’s procedure to be effective, a longitudinal incision is made through the

hypertrophied muscle as far as the mucosa; the mucosa is left intact and is seen bulging into the

incision. If the mucosa is split during the procedure then it should be re-sutured.

Circular muscle layer

The incision is a sero -muscular incision as deep as the mucosa. An incomplete incision will

result in continued gastric outflow obstruction.

Longitudinal muscle layer

The incision is a sero -muscular incision as deep as the mucosa.

Serosa

The incision is a sero -muscular incision as deep as the mucosa.

Submucosa

The incision is a sero -muscular incision as deep as the mucosa.

173
Q

A 56-year-old nervous,

obese

woman presents to the Emergency Department with

mild

upper

abdominal pain and symptoms of an acidic taste in the mouth occurring several times per day

over the past 2 years. She states her pain is worse after eating. She is

haemodynamically

stable

and has normal observations.

Which one of the following most accurately describes this woman’s probable condition?

1) It is usually treated by highly selective vagotomy in cases where trials of medication have

failed

2) It is usually worsened by smoking which acts to contract the lower oesophageal sphincter

and increase oesophageal acidity

3) In cases such as this lifestyle modifying advice often has the desired effect

4) Prokinetic drugs such as metoclopramide should be used as the treatment of choice in this

condition

5) It leads to the development of Barrett’s metaplasia in 30% of cases

A

Explanation

In cases such as this lifestyle modifying advice often has the desired effect

In patients with mild symptoms, the first option should be lifestyle modifications like losing

weight, giving up smoking, not eating within 2 h of lying flat and raising the head of the bed by

6–8 inches.

It is usually treated by highly selective vagotomy in cases where trials of medication have failed

If surgical intervention is needed then the treatment of choice is a Nissen’s fundoplication.

Vagotomy has largely been superseded by proton pump inhibitors.

It is usually worsened by smoking which acts to contract the lower oesophageal sphincter and

increase oesophageal acidity

Insufficient clearance of acid can contribute to the symptoms of gastro -oesophageal reflux

disease (GORD), but this is not the most common problem which is incompetence of the lower

oesophageal sphincter plus failure of the anatomical valve at the angle of His. It is worsened by

smoking, the main action of which is to relax the lower oesophageal sphincter allowing stomach

acid to enter the oesophagus. Other risk factors include obesity and alcohol, which also result in

decreased competency of the lower oesophageal sphincter.

Prokinetic drugs such as metoclopramide should be used as the treatment of choice in this

condition

If simple lifestyle modifications do not work then proton pump inhibitors and H 2 blockers should

be tried before prokinetics.

It leads to the development of Barrett’s metaplasia in 30% of cases

Barrett’s oesophagus is seen in approximately 10% of patients undergoing endoscopy for

investigation of GORD.

174
Q

A 63-year-old obese man has been referred to your clinic with symptoms of acid reflux, upper

abdominal pain and a new onset nocturnal cough. He has tried maximum dose proton pump

inhibitor treatment for 6 months with little relief. Which one of the following statements seems the most appropriate?

1) The oesophagus is a muscular tube approximately 25 cm long comprised of an inner

longitudinal muscle and an outer circular muscle coat which overlap in the middle part.

2) The lower oesophageal sphincter is a distinct anatomical condensation of muscle which

together with the angle of His acts to present reflu x of gastric acid back into the distal

oesophagus.

3) The best way of investigating gastro -oesophageal reflux disease is ambulatory pH

monitoring in which a probe is placed within the oesophagus itself and provides a

continuous reading of the level of oesophageal acidity

4) Barrett’s oesophagus is diagnosed on the anatomical endoscopic appearance of the Z line.

5) There are three predictable sites of oesophageal constriction which relate to the

cricopharyngeal sphincter (15 cm from incisors), where it is crossed by the aortic arch

(22 cm) and where it passes through the diaphragm (38 cm)

A

Explanation

The best way of investigating gastro -oesophageal reflux disease is ambulatory pH monitoring in

which a probe is placed within the oesophagus itself and provides a continu ous reading of the

level of oesophageal acidity

Twenty-four hour ambulatory monitoring was introduced in 1969 and has become the best way

for monitoring the pattern of exposure of acid in the oesophagus, and the gold standard for

gastro-oesophageal reflux disease. The monitor is placed so that the pH probe lies 5 cm above

the oesophageal sphincter, therefore being able to record the presence of acid within the

oesophagus itself.

The oesophagus is a muscular tube approximately 25 cm long comprised of an inne r longitudinal

muscle and an outer circular muscle coat which overlap in the middle part.

The oesophagus is approximately 25 cm long, but comprises an inner circular muscle coat and an

outer longitudinal coat, which is striated muscle in the upper part and smooth muscle in the

lower part with considerable overlap in the middle.

The lower oesophageal sphincter is a distinct anatomical condensation of muscle which together

with the angle of His acts to present reflux of gastric acid back into the distal oesophagus.

The lower oesophageal sphincter is a physiological rather than an anatomical sphincter and there

is no distinct condensation of muscle here.

Barrett’s oesophagus is diagnosed on the anatomical endoscopic appearance of the Z line.

Barrett’s oesophagus needs to be diagnosed by endoscopy, with multiple biopsies to show

metaplasia of the normal oesophageal squamous epithelium to columnar epithelium with goblet

cells – it cannot be diagnosed from pH monitoring or the position of the Z line alone.

There are three predictable sites of oesophageal constriction which relate to the cricopharyngeal

sphincter (15 cm from incisors), where it is crossed by the aortic arch (22 cm) and where it

passes through the diaphragm (38 cm) In addition to the three levels of constriction described there is a fourth which lies at 27 cm from

the incisors and corresponds to the left principle bronchus.

175
Q

A 67-year-old man presents with a long history of vague upper gastrointestinal symptoms

including dysphagia and heartburn. An oesophago -gastro-duodenoscopy (OGD) reveals the

presence of a para-oesophageal hernia.

Which one of these statements most accurately describes this condition?

1) Can cause palpitations due to a relative increase in pressure on the underside of the

thoracic diaphragm

2) Are more common than sliding herniae

3) Occurs in a younger age group than sliding herniae 4) Occurs when a part of the stomach herniates through the oesophageal hiatus with

subsequent herniation of the gastro oesophageal junction

5) Occurs more commonly in female patients

A

Explanation

Occurs more commonly in female patients

Para-oesophageal hiatus herniae have an established preponderance of women affected.

Can cause palpitations due to a relat ive increase in pressure on the underside of the thoracic

diaphragm

Palpitations can occur but this is though pressure on the vagus nerve. Symptoms usually relate to

the mechanical effects of additional compression on the lower oesophagus, and patients

commonly complain of dysphagia. Gastric volvulus is a recognised complication, but because of

the elderly age group, surgery may be high risk.

Are more common than sliding herniae

Sliding hiatus herniae are the commonest type (95%).

Occurs in a younger age group than sliding herniae

Para-oesophageal hernias occur when a part of the stomach herniates through the oesophageal

hiatus to lie next to the oesophago -gastric (OG) junction without herniation of the junction. The

more common sliding hiatus hernia occur when the oesophagogastric junction herniates through

the hiatus.

Occurs when a part of the stomach herniates through the oesophageal hiatus with subsequent

herniation of the gastro oesophageal junction

Para-oesophageal hernias occur when a part of the stoma ch herniates through the oesophageal

hiatus to lie next to the oesophago -gastric (OG) junction without herniation of the junction. The

more common sliding hiatus hernia occur when the oesophagogastric junction herniates through

the hiatus.

176
Q

An elderly woman presents to the Emergency Department with severe dysphagia. Her symptoms

have been progressing and there is a vague suggestion of a gastric bubble above the diaphragm

on the chest radiograph, raising the possibility of a hiatus hernia.

Which one of the following is correct regarding hiatus herniae?

1) The majority of patients with hiatus herniae are symptomatic, and usually experience acid

reflux

2) As with gastro-oesophageal reflux disease, the most appropriate initial diagnostic

modality is ambulatory pH monitoring 3) When GORD is associated with a proven sliding hiatus hernia, surgical management in

the form of a Nissen fundoplication is the treatment of choice

4) Despite the high fibre diet, paradoxically hiatus herniae are found more commonly in

rural African communities

5) Sliding herniae are primarily associated with symptoms of GORD, whilst rolling herniae

can cause gastric strangulation and necrosis and gangrene

A

Explanation

Sliding herniae are primarily associated with symptoms of GORD, whilst rolling herniae can

cause gastric strangulation and necrosis and gangrene

Complications include incarceration, gangrene and gastric volvulus. Oesophagitis is more

commonly associated with sliding hiatus hernias.

The majority of patients with hiatus herniae are symptomatic, and usually experience acid reflux

The majority of hiatus herniae are sliding or axial in nature, are usually asymptomatic but can be

associated with oesophagitis, stricture formation, dysphagia, chronic anaemia and inhal ational

pneumonitis. Rolling herniae or para-oesophageal hiatal herniae usually affect elderly patients

who present with intermittent dysphagia, pain after eating due to distension of the intrathoracic

part of the stomach, cardiac symptoms such as palpitat ions due to pressure effects on the vagus

nerve, and hiccups due to phrenic nerve irritation.

As with gastro-oesophageal reflux disease, the most appropriate initial diagnostic modality is

ambulatory pH monitoring

Ambulatory pH monitoring is the gold standard for diagnosis of GORD, however, it is not

usually necessary. The most appropriate investigation is more frequently upper GI endoscopy,

upper GI contrast series or upper GI manometry to disprove other possible diagnoses (eg peptic

ulcer disease), and to monitor chronic patients for signs of Barrett’s oesophagus.

When GORD is associated with a proven sliding hiatus hernia, surgical management in the form

of a Nissen fundoplication is the treatment of choice

In severe cases of hiatus herniae, a Nissen fundoplication may be necessary but in the vast

majority of patients symptoms can be managed by lifestyle modification and medications such as

proton pump inhibitors and H 2 blockers.

Despite the high fibre diet, paradoxically hiatus herniae are found more commonly in rural

African communities

The low fibre diet of westernised society, and also the prolonged sitting position for defecation,

results in a higher incidence of hiatus herniae.

177
Q

A 76-year-old woman presents to the Emergency Department with profuse fresh rectal bleeding with clots. She has a pulse rate of 90 beats per min (bpm) and a blood pressure (BP) of 140/86.

She had a similar episode 2 years ago, has not lost any weight and has normal bowel habits. She is also on omeprazole for long standing gastro -oesophageal reflux disease.

Which one of the following statements best describes this clinical scenario?

1) The commonest cause of massive rectal bleeding is diverticulitis and this is usually
painless

2) The passage of bright red blood is never due to duodenal ulcer

3) It is usually controlled by catheter-directed infusion of vasopressin

4) Following resuscitation an urgent OGD is a diagnostic option

5) It may be localised by angiography if <0.5 ml/ min

A

Explanation

Following resuscitation an urgent OGD is a diagnostic option

In a stable patient an urgent OGD is a relatively simple diagnostic test that can also be

therapeutic if a bleeding source is identified. However, usually a computed tomography (C T)

angiogram and selective embolization if the bleeding point can be identified would be indicated

in this patient.

The commonest cause of massive rectal bleeding is diverticulitis and this is usually painless

Diverticular disease with erosion of small vessels in the bowel wall is the most common form of

significant lower gastrointestinal haemorrhage and is usually painless. Diverticulitis however

often presents with left iliac fossa (LIF) pain and pyrexia but is rarely associated with bleeding.

The passage of bright red blood is never due to duodenal ulcer

Fresh per rectal (PR) bleeding is usually from the lower GI tract, however, if bleeding is brisk, it

may come from as high up as a peptic or duodenal ulcer, with the presence of blood promoting a

fast transit time.

It is usually controlled by catheter-directed infusion of vasopressin

Vasopressin has little role in the management of this scenario – if they do not settle

conservatively, then patients will need surgery.

It may be localised by angiography if <0.5 ml/min

Angiographic techniques can pick up bleeding at the rate of >0.5 ml/min in experienced hands.

178
Q

A 32-year-old man is referred to your general surgical clinic complaining of a 3 -month history of dysphagia starting to both solids and liquids. In addition he complains of retrosternal chest pain and 1 stone of weight loss over the last 3 months. He is a moderate drinker and has a 20 pack -year smoking history.

Which one of the following statements best represents this man’s likely condition?

1) The most likely differential diagnosis in this man is oesophageal carcinoma

2) In oesophageal carcinoma, dysphagia is progressive like achalasia

3) Oesophageal achalasia which can be treated by dilating the lower oesophagus and associated sphincter

4) Achalasia which is characterised by incomplete lower oesophageal sphincter relaxation, decreased LOS tone and increased peristalsis of the oesophagus

5) Although achalasia can be primary (affecting 1/100,000 individuals per year) it is much more commonly associated with other conditions

A

Explanation Oesophageal achalasia which can be treated by dilating the lower oesophagus and associated

sphincter

Treatment of achalasia should initially be by lifestyle modification and medications to reduce

lower oesophageal sphincter (LOS) pressure (eg nifedipine). Invasive treatments include balloon

dilatation and also Heller’s cardiomyotomy together with a partial fundoplication to prevent

excessive reflux.

The most likely differential diagnosis in this man is oesophageal carcinoma

In carcino ma, difficulties begin with swallowing solids and progress to include liquids. Normally

this dysphagia is painless.

In oesophageal carcinoma, dysphagia is progressive like achalasia

While in carcinoma, difficulties begin with swallowing solids and progress to include liquids, in

achalasia patients have equal difficulty in swallowing both liquids and solids.

Achalasia which is characterised by incomplete lower oesophageal sphincter relaxation,

decreased LOS tone and increased peristalsis of the oesophagus

It is characterised by incomplete LOS relaxation, increased LOS tone and lack of oesophageal

peristalsis. This results in dysphagia to both liquids and solids, chest pain that can be worse after

eating, and unintentional weight loss.

Although achalasia can be primary (affecting 1/100,000 individuals per year) it is much more

commonly associated with other conditions

Achalasia is a condition caused by failure of development of the lower oesophageal inhibitory

neurons and involves the smooth muscle layers of the oesophagus and LOS. The most common

form is primary achalasia but it also occurs associated with other conditions such as cancer or

Chagas’ disease but much more infrequently.

179
Q

A 56-year-old woman has been referred to your surgical clinic with symptoms of waterbrash and

a new onset night cough. She has tried maximal medical management with her GP and a

diagnosis of reflux oesophagitis has been made on oesophago -gastro-duodenoscopy (OGD).

Which one of the following statements is true regarding th is condition?

1) Does not require more than alteration of lifestyle to treat

2) If untreated may cause stricturing of the oesophagus

3) Is always present with hiatus hernia

4) Is preferably treated with surgery

5) Is treated surgically principally by attempting to narrow the gastro-oesophageal junction

A

Explanation

If untreated may cause stricturing of the oesophagus

If GORD goes untreated then a common eventual complication is oesophageal stricture. Does not require more than alteration of lifestyle to treat

Often lifestyle alterations are successful, but should they not be medical management with

proton pump inhibitor (PPI) or H 2 antagonists are frequently required. Further steps in

management include gastric motility agents, then surgery.

Is always present with hiatus hernia

The majority of hiatus herniae are asymptomatic.

Is preferably treated with surgery

The initial management of mild cases is with lifestyle changes – these include weight loss,

reducing smoking and alcohol intake, and avoiding food in the 2 h before be d. Should these fail

or symptoms be more severe medical management with PPI or H 2 antagonists can be tried,

followed by gastric motility agents. Surgery is only usually considered if symptoms remain

following failure of lifestyle and medical management.

Is treated surgically principally by attempting to narrow the gastro -oesophageal junction

Surgical management is with a Nissen’s fundoplication in which the fundus is wrapped around

the distal oesophagus with plication of the oesophageal hiatus. The gastro-oesophageal

junction (GOJ) itself is not touched. This procedure can be performed open or laparoscopically.

180
Q

A 78-year-old woman presents to the Emergency Department 3 years following a Hartmann’s

procedure for perforated diverticular disease. She has never wanted a reversal of her stoma but

has recently noticed a tender lump adjacent to the stoma. A diagnosis of a parastomal hernia has

been made.

Which one of the following statements best reflects this condition?

1) Parastomal herniae are commonly asymptomatic

2) Is more common when the stoma is brought through the rectus muscle

3) Occurs in 30% of colostomies

4) Should always be repaired

5) Should be repaired using a mesh

A

Explanation

Parastomal herniae are commonly asymptomatic

Parastomal herniae are usually asymptomatic.

Is more common when the stoma is brought through the rectus muscle

Parastomal herniae occur in approximately 10% of colostomies and are usually asymptomatic.

They rarely lead to complications and therefore usually do not need treating surgically. Factors that may increase the risk of developing a parasternal hernia include obesity, increasing age, post

operative infection, malnutrition, and site of the stoma (stoma being placed outside of the rectus

abdominus). Approximately 80 percent of patients with parastomal hernia remain asymptomatic

and do not need further intervention.

Occurs in 30% of colostomies

Parastomal herniae occur in 10% of colostomies.

Should always be repaired

Since parastomal herniae are usually asymptomatic and rarely lead to complications treatment is

not usually necessary.

Should be repaired using a mesh

Mesh repair can be undertaken, but the presence of foreign material around a stoma site leads to

a high risk of infection, so this procedure is rarely carried out.

181
Q

A 26-year-old woman presents to the Emergency Department with

right iliac fossa peritonism,

pyrexia and vomiting

. A diagnostic laparoscopy reveals appendicitis which is duly resected.

Regarding the Alvarado score, which one of the following is correct?

1) Appendicitis is associated with a right shift in the neutrophil profile

2) Appendicitis is associated with migratory right iliac fossa pain

3) Appendicitis is unlikely with a score over 5

4) Does not count rebound tenderness in the marking scheme

5) Is associated with a higher mortality if the score is over 13

A

Explanation

Appendicitis is associated with migratory right iliac fossa pain

This feature of the history is part of the scoring process.

Appendicitis is associated with a right shift in the neutrophil profile

A left shift in the neutrophil profile is part of the scoring process, a right shift is not.

Appendicitis is unlikely with a score over 5

If the score is less than 5 then appendicitis is unlikely, 5 –6 is equivocal, 7 or greater is likely.

Does not count rebound tenderness in the marking scheme

Rebound tenderness is an important part of the scoring process. Is associated with a higher mortality if the score is over 13

The score dose not predict mortalit y.

Acute appendicitis has an associated mortality of 0.25% and a morbidity of 7.7%. It classically

presents with periumbilical pain that then localises to the right iliac fossa, anorexia, nausea and

vomiting. The patient will likely be tender in the right iliac fossa (RIF) and have signs of

localised peritonism. The presentation is, however, variable because of the variability in position

of the appendix and a high retrocaecal appendix may mimic cholecystitis, and appendicitis itself

may be caused by a caecal carcinoma which occludes the appendix lumen. The peak incidence is

towards 30 years of age with a second peak at 70 years. It has an overall incidence of 10% of the

population. It is marginally more common in men (1.6 times).

The Alvarado score is a clinical and laboratory based scoring system marked out of 10 which

assesses the likelihood of the diagnosis of appendicitis:

if <5 appendicitis is unlikely

5–6 is equivocal and may require scanning

> 7 is strongly predictive.

Points are awarded as below:

Migratory RIF pain (1 point), anorexia (1 point), nausea and vomiting (1point), RIF tenderness

(2 points), rebound tenderness (1 point), fever (1 point), leucocytosis (2 points), left shift in

neutrophils (1 point).

182
Q

You are sitting in on a general surgical clinic and your consultant is discussing rarer causes of

small bowl obstruction. The next patient previously had an intussuscepted bowel.

Intussusception is most common in which one of the following?

1) Women in the summer months

2) Women in the winter months

3) Women regardless of the time of year

4) Men in the summer months

5) Men in the winter months

A

Explanation

Men in the winter months

It is more common in the winter months and in men. Children with this condition other have flu-

like symptoms, and this effect, together with the predisposition to winter months, suggests virus

involvement in the development of the condition.

Women in the summer months

Intussusception is less common in women and the summer months.

Women in the winter months

Intussusception is more common in men than in women, but is indeed more common in the

winter months. Women regardless of the time of year

Intussusception is more common in men than women.

Men in the summer months

While intussusception is more common in men, it is still more common in winter than in

summer.

Intussusception is the telescoping of one part of the intestine into the other and is most common

in infants aged 4 months to 1 year, with peak incidence between 6 –9 months. Presenting features

are abdominal pain, fever, ‘redcurrant jelly’ stool, and obstruction. Other risk factors include

previous intussusception, or family history of the condition, and intestinal malrotation. An

ultrasound scan (USS) or a barium or a ir enema may detect the point of intussusception. The

enema may also relieve the intussusception, otherwise surgery may be required.

183
Q

A 23-year-old man presents with severe perianal discomfort and fever. A magnetic resonance

imaging scan of the pelvis is performed and the radiologist reports a collection of pus

inferior

and lateral to levator ani.

What is the most likely diagnosis?

1) Horseshoe abscess

2) Intersphincteric abscess

3) Ischiorectal abscess

4) Perianal abscess

5) Supralevator abscess

A

Explanation

Ischiorectal abscess

The ischiorectal (or ischioanal) fossae are prism-shaped fascia-lined potential spaces adjacent to

the rectum. They are bordered by the levator ani muscle (superiorly and medially), the anal canal

(medially), the perineal skin (inferiorly), and the ischial tuberosity (laterally). Ischiorectal

abscesses are the second most common type of anorectal sepsis.

Horseshoe abscess

This option is an ischiorectal abscess that extends to both sides of the anal canal so forming a

horseshoe shape.

Intersphincteric abscess

Intersphincteric abscesses sit between the internal and external sphincters. They may not be

visible as they can be completely contained within the anal canal. In this instance either digital

rectal examination or examination under anaesthetic (EUA) is required for diagnosis.

Perianal abscess These are most common. They are superficial, do not transverse the external sphincter but sit just

under the skin.

Supralevator abscess

These are the least common, they form from an upward extension of an intersphincteric abscess

into the supralevator space. They may also form from an intra -abdominal inflammation (eg

appendicitis) tracking down.

184
Q

A 70-year-old woman attends the clinic with a history of right upper quadrant pain. She has an

ultrasound scan of the abdomen that reveals a normal liver, a common bile duct diameter of 5

mm and

multiple gallstones

within a contracted

thick-walled gall-bladder

.

Which one of the following statements is true regarding this patient?

1) A laparoscopic cholecystectomy should be recommended

2) Gallbladder carcinoma can be excluded

3) She has acute cholecystitis

4) She has Mirizzi’s syndrome which is a gallstone that is eroding through the gallbladder

wall into the ileum

5) The common bile duct is dilated

A

Explanation

A laparoscopic cholecystectomy should be recommended

Given the patient’s ongoing symptoms and thick-walled appearance to the gall-bladder,

cholecystectomy should be considered. Differentials for a thic k-walled gall-bladder include the

normal physiological state after food, cholecystitis (acute and chronic), infiltrative gall -bladder

cancer, and liver conditions such as cirrhosis.

Gallbladder carcinoma can be excluded

This condition cannot be fully excluded without biopsy.

She has acute cholecystitis

Acute cholecystitis presents with right upper quadrant pain, fever and raised inflammatory

markers. Murphy’s sign is usually positive (a positive Murphy’s sign is when pressure under the

right costal margin o n inspiration results in pain and the patient ‘catching’ their breath). There

can be fluid surrounding the gall-bladder as part of the inflammatory process.

She has Mirizzi’s syndrome which is a gallstone that is eroding through the gallbladder wall into

the ileum Mirizzi’s syndrome is obstructive jaundice caused by a large gallstone in Hartmann’s pouch

compressing the common bile duct. Mirizzi’s syndrome is one of the indications to convert from

laparoscopic cholecystectomy to an open procedure, often leav ing Hartmann’s pouch behind to

fibrose.

The common bile duct is dilated

Six millimetres is the upper limit of normal for common bile duct diameter, and allowances of a

few millimetres more may be made in elderly patients.

185
Q

A 30-year-old man with

ulcerative colitis

attends the coloproctology clinic following a

Gastroenterology referral for restorative proctocolectomy.

Which one of the following is most accurate regarding restorative proctocolectomy?

1) Patients with both ulcerative colitis and primary sclerosing cholangitis are at higher risk

of requiring proctocolectomy than patients with ulcerative colitis alone

2) Pouch surveillance is mandatory in all cases of ulcerative colitis

3) Pouchitis is common with restorative procotocolectomy

4) Restorative proctolcolectomy is a three-stage procedure

5) Restorative proctocolectomy is contra-indicated in patients who have an internal anal

sphincter defect

A

Explanation

Patients with both ulcerative colitis and primary sclerosing cholang itis are at higher risk of

requiring proctocolectomy than patients with ulcerative colitis alone

Indications for proctocolectomy in ulcerative colitis (UC) include (non-exhaustive list):

Acute/emergency – toxic mega-colon, fulminant UC uncontrolled by medical treatment,

perforation, uncontrolled bleeding.

Elective – chronic steroid dependency or systemic side-effects from treatment, dysplasia or

adenocarcinoma on biopsy, refractory disease despite medical management.

PSC, in particular, is associated with an increased risk of developing colorectal cancer so, in

line with the above indications, is more likely to require restorative proctocolectomy.

Pouch surveillance is mandatory in all cases of ulcerative colitis Pouch surveillance is recommended for tho se who have colonic dysplasia or neoplasm, as there

are rare case reports of neoplasm developing within the pouch. It is not mandatory in all cases of

UC.

Pouchitis is common with restorative procotocolectomy

After 10 years around 50% of patients will have had their first episode of pouchitis. The

incidence of pouch failure is 5–15% per year and reasons include pelvic sepsis, poor function

and pouchitis.

Restorative proctolcolectomy is a three-stage procedure

Reoperative pelvic surgery (RPC) should be a one stage or two stage operation (if a

defunctioning stoma is employed). Further descriptive evidence may be found via the links

below.

Restorative proctocolectomy is contra-indicated in patients who have an internal anal sphincter

defect

Contraindications fo r restorative proctocolectomy pouch formation include small bowel disease

involvement (eg in Crohns) and anal disease involvement (eg in Crohns, or low rectal cancers).

Ulcerative colitis starts as a proctitis and ascends in a continuous fashion affecting up to the

entire colon, but typically not the small bowel. However, incompetent ileocaecal valves are not

uncommon and can lead to a ‘backwash’ ileitis, making distinction from Crohn’s or a mixed

colitis difficult. UC has many extra-intestinal manifestations, such as uveitis, arthritis, primary

sclerosing cholangitis and erythema nodosum. It is associated to the HLA-B27 phenotype.

Restorative proctocolectomy is a procedure by which a pouch from terminal ileum is fashioned

and anastomosed to the anus, 1–2 cm above the dentate line. The pouch is usually J in shape and

measures 15 cm in length. RPC is most commonly recommended in ulcerative colitis and

familial adenomatous polyposis where the colon needs to be removed to eliminate the risk of

colon cancer. Ideal pouch function is approximately 4–6 times per day and two times at night.

The benefit of the pouch is to eliminate urgency from a simple ileo -anal anastomosis, and avoid

a permanent ileostomy.

186
Q

A 70-year-old woman presents to

the clinic with a lump in

the groin. The lump is reducible and

has a cough impulse. It is located

above

and

medial to the

pubic tubercle

.

Which one of the following is true about this type of hernia?

1) In women a direct hernia is more likely than an indirect one

2) Open repair requires fixation of mesh to the reflected edge of the Inguinal ligament

3) Repair requires preservation of the round ligament

4) The diagnosis is that of an femoral hernia which is more common than an inguinal hern ia

in women

5) The hernia can be repaired with a local anaesthetic block to L2 and L3

A

Explanation

Open repair requires fixation of mesh to the reflected edge of the Inguinal ligament

An inguinal hernia emerges above and medial to the pubic tubercle, so in this patient we suspect

an inguinal hernia. The inguinal canal in women contains the round ligament of the uterus and a

remnant of the processus vaginalis. A persistent processus vaginalis in women forms the canal of

Nuck. The floor of the inguinal canal is formed by the reflected edge of the inguinal ligament. As

part of the Lichtenstein procedure the lower part of the mesh is fixed to this reflected edge of the

inguinal ligament with the mesh above this positioned to reinforce the posterior wall of the canal.

In women a direct hernia is more likely than an indirect one

Direct hernias are rare in women (man to woman ratio 10:1), they result from an acquired

weakness of the wall of the inguinal canal formed by the transversalis fascia. Indirect hernias

occur through the internal inguinal ring where there is an inherent weakness. Direct hernias

occur usually in the elderly population, whereas indirect hernias can occur at any age and can be

congenital.

Repair requires preservation of the round ligament

During repair the round ligament can be divided.

The diagnosis is that of an femoral hernia which is more common than an inguinal hernia in

women

Femoral hernia accounts for 20% of hernias in women and 5% of hernias in men. The man to

woman ratio of inguinal hernia is 10 to 1. The hernia can be repaired with a local anaesthetic block to L2 and L3

Inguinal hernia can be repaired by a local anaesthetic block that requires infiltration of the

iliohypogastric and ilioinguinal nerves (T12 and L1), 2 cm medial from the ant erior superior iliac

spine, the genitofemoral nerve (L1 and L2), 1–2 cm lateral to the pubic tubercle and local

infiltration of the subcutaneous tissues along the line of incision.

187
Q

A 55-year old man with

cirrhosis

of the liver due to chronic alcohol use presents to the acute

surgical take. He has had fresh per rectum

(PR) bleeding

and is found to have a

mass

in the low

rectum on PR.

What would be the next step in his management?

1) Computerized tomography scan of the abdomen and pelvis

2) Capsule endoscopy

3) Colonoscopy

4) Low anterior resection

5) Magnetic resonance imaging (MRI) of the pelvis

A

Explanation

Colonoscopy

The differential here is rectal neoplasm, polyp and varices. The best investigation would be an

endoluminal one, ie colonoscopy. Biopsies should only be considered if the mass is not of a

vascular nature as attempting to biopsy a varices would likely lead to heavy bleeding. In patients

that are unfit for colonoscopy axial imaging may be considered.

Computerized tomography scan of the abdomen and pelvis

This is not the best initial diagnostic investigation.

Capsule endoscopy

Capsule endoscopy is a useful investigation to image the small intestines, however for a low

colonic mass a different endoluminal investigation would be far more useful.

Low anterior resection

This may be the treatment of choice, for example if a malignancy is diagnosed. It is not,

however, a diagnostic investigation.

Magnetic resonance imaging (MRI) of the pelvis

This is part of the imaging required for the local staging of rectal cancers, it is not the best initial

diagnostic investigation. Along with the differentials of rectal neoplasm or polyp, this patient’s history of cirrhosis means

that varices must also be considered on the differential list. Common sites for varices are as

follows:

In the gastro-oesophageal region, the oesophageal tributaries of the left gastric vein

anastomose with the oesophageal veins, which empty into the azygos vein.

In the paraumbilical region the paraumbilical veins in the falciform ligament anastomose

with the subcutaneous veins in the anterior abdominal wall.

In the anorectal region the superior rectal vein anastomoses with the middle and inferior

rectal veins, which are anastomoses of the internal iliac and internal pudendal veins.

188
Q

A 2-week-old boy presents with projectile vomiting, palpable epigastric mass and failure to thrive. He is diagnosed with hypertrophic pyloric stenosis and undergoes Ramstedt’s
pyloromyotomy.

Which one is the deepest layer of the pyloric wall to be divided?

1) Circular muscle

2) Longitudinal muscle

3) Mucosa

4) Parietal peritnoneum

5) Serosal surface

A

Explanation

Circular muscle

The circular muscle is the last layer to be divided.

Longitudinal muscle

The longitudinal muscle sits outside the circular muscle.

Mucosa

The mucosa is left intact.

Parietal peritnoneum

The peritoneum is divided to gain access to the greater curvature of the stomach.

Serosal surface

The serosa is cut through before the muscular layers being divided. Ramstedt’s pyloromyotomy is indicated in hypertrophic pyloric stenosis. An incision is made
along the length of the hypertrophic muscle, and the layers of longitudinal and circular muscle are divided until the mucosa is exposed.

189
Q

A 25-year-old woman attends the Coloproctology Clinic with a 4-month history of

intermittent,

painful, fresh per rectum (PR) bleeding. She is found to have anal

spasm

and a

chronic

anal fissure

.

What will be the most appropriate first line management?

1) Advancement flap

2) Examination under anaesthesia and Botulinum A toxin

3) Flexible sigmoidoscopy and high fibre diet

4) Lateral sphincterotomy

5) 2% Diltiazem ointment

A

Explanation

2% Diltiazem ointment

The accepted treatment protocol for anal fissures is firstly topical 0.2% glyceryl trinitrate(GTN)

or 2% Diltiazem followed by Botox and then lateral sphincterotomy or advancement flap. The

latter is considered in women with low pressure anal fissure. Anterior anal fissures are

commoner in women due to the lack of support to the anal canal anteriorly, bu t posterior fissures

are most common overall.

Advancement flap

An advancement flap is considered in low pressure anal fissure, but is not first -line treatment.

Examination under anaesthesia and Botulinum A toxin

Examination under anaesthesia is not usually indicated for primary anal fissure – it may,

however, be necessary in elderly patients or other groups in whom a secondary cause is

suspected. Additionally, in the paediatric group if abuse is considered, a possible cause

examination under anaesthesia (EUA) may be indicated. Botulinum toxin is a useful treatment

causing relaxation of the sphincter, improving blood flow and therefore healing time, but is

usually considered after other topical treatments have failed.

Flexible sigmoidoscopy and high fibre diet

Whilst a high fibre diet is important in both prevention and treatment of primary anal fissure it is

not thought to offer much benefit in the treatment of chronic anal fissures. Additionally a flexible

sigmoidoscopy is not usually a first -line measure in young patients when primary anal fissure is

suspected. However, in elderly patients, secondary causes such as anal or rectal cancer are more

likely and further investigation may be indicated.

Lateral sphincterotomy Lateral sphincterotomy can be considered should topical treatments or Botox management not

prove successful. The aim is to reduce the pressure of the sphincter, which aids blood flow to the

area and therefore healing.

190
Q

A 56-year -old woman presents to casualty with severe upper abdominal pain. On questioning she admits to several similar episodes of upper abdominal pain radiating to her back over the past 4 years that have all required admission. On examination she is in in severe pain and looks thin and unkempt. A diagnosis of chronic pancreatitis is made.

Which one of the following statements is the most likely aetiology in this woman?

1) Alcohol

2) Cholelithiasis

3) Hypocalcaemia

4) Hypothermia

5) Mumps

A

Explanation

Alcohol

The most common cause of chronic pancreatitis is chronic excess alcohol.

Cholelithiasis

Cholelithiasis (gallstones) and choledocholithiasis (gallstones in the common bile duct) are common causes of acute pancreatitis, which with alcohol accounts for approximately 80% of cases.

Hypocalcaemia

Hypocalcaemia is a complication of acute pancreatitis, as opposed to hypercalcaemia which is a cause.

Hypothermia

Hypothermia is a recognised cause of acute pancreatitis but alcohol is the most likely cause in this scenario.

Mumps

Viral infections such as mumps are also well recognised causes of acute pancreatitis, but alcohol is the most likely cause in this scenario.

191
Q

A 36-year-old woman presents to casualty with

profuse hematemesis

. She is a heavy drinker and

has been for several years. On examination she appears confused and has obvious ascites.

Which one of the following procedures should be completed first in her management?

1) Administration of vasoactive drugs such as somatostatin or octreotide

2) Balloon tamponade using a Sengstaken–Blakemore tube

3) Performing TIPS procedure as an emergency

4) Performing an urgent oesophago -gastroduodenoscopy (OGD) and endoscopic band

ligation

5) Securing IV access

A

Explanation

Securing IV access

The clinical vignette suggests a patient with decompensated liver disease (ascites, confusion) as a

result of heavy drinking. The suspicion must therefore be that this patient has presented with a

variceal haemorrhage. Oesophageal varices may present with profuse haematemesis and this

patient should be initially approached with an AB CDE assessment. As part of this, IV access

should be secured and bloods sent including an urgent crossmatch.

Administration of vasoactive drugs such as somatostatin or octreotide

If urgent endoscopy is available then this should occur as soon as possible in a patient with

active variceal bleeding. If not, then vasoactive drugs such as somatostatin, octreotide or

Glypressin should be considered. These would require an IV line to be placed to be administered.

Balloon tamponade using a Sengstaken–Blakemore tube

Balloon tamponade is considered normally only if band ligation or sclerotherapy fails to

adequately control the patient’s variceal haemorrhage, or in an emergency setting when

endoscopy is not available.

Performing TIPS procedure as an emergency

Transjugular intrahepatic portosystemic shunt (TIPS) procedure can be used to reduce portal

pressure by diverting portal blood flow into the hepatic vein and may be used if other methods

firstly cannot control bleeding.

Performing an urgent oesophago -gastroduodenoscopy (OGD) and endoscopic band ligation In patients with variceal bleeding this procedure is the most important immediate definitive

management, however it does not supersede a basic ABCDE (Airway maintenance with cervical

spine protection, Breathing and ventilation, Circulation with haemorrhage control, Disability:

neurological status, Exposure/environmental control) assessment and resuscitation of the patient.

192
Q

You are seeing a

patient in clinic with

complete

rectal prolapse, and she asks about the potential

complications of

conservative treatment

of this condition. Which one of the following statements would you tell her is the commonest?

1) Incarceration

2) Incontinence

3) Rectal carcinoma

4) Strangulation

5) Ulceration

A

Explanation

Incontinence

The commonest of the correct options listed here is faecal incontinence (75%), which may

actually be the patient’s presenting symptom. Alternatively constipation can very commonly be

either the presenting symptom or a complication. Rectal prolapse is a condition that most

commonly affects older women, but can occur in any age. Partial rectal prolapse (prolapse of the

mucosal layer only) occurs more commonly in children while complete rectal prolapse (prolapse

of all layers of the rectum) is typical in elderly women. Patients may present because they have

felt/noticed the prolapse, often while straining to pass stool. Additionally they may present with

symptoms such as incontinence, constipation, per rectal (PR) ble eding or pain. Diagnosis may be

simple with the prolapse visualised either at rest or on straining/performing the Valsalva

manoeuvre. Imaging may be required in some patients, for example an evacuating proctogram.

Incarceration

Incarceration is a complicat ion of rectal prolapse, however, it is not the commonest of the correct

options listed here.

Rectal carcinoma

Occasionally a prolapse can be a presenting feature of rectal carcinoma.

Strangulation

This is a complication of rectal prolapse that requires emergency surgery. It presents with a

painful non-reducible prolapse. It is not the most common of the options listed.

Ulceration

Ulceration and necrosis of the rectal wall are common complications, albeit not the most

common listed option.

193
Q

An 18-month-old child is brought to see you in clinic as his mother has noted mucus and blood

staining on

nappies, and has seen a bright red mass intermittently protruding from his anus. You

diagnose a

partial

rectal prolapse.

Which one of the following statements is true regarding partial rectal prolapses? 1) It involves all layers of the rectal wall

2) It is associated with a significant increased risk of rectal carcinoma

3) It is more common in adults than children

4) It should be treated with Delorme’s procedure

5) The majority resolves with conservative treatment

A

Explanation

The majority resolves with conservative treatment

Most cases of partial rectal prolapse in children resolve with conservative treatment.

It involves all layers of the rectal wall

Complete rectal prolapse involves all layers of the rectal wall and is more commonly seen in

elderly women. Partial rectal prolapse is more common in children and involves the mucosal

layer only.

It is associated with a significant increased risk of r ectal carcinoma

There is an associated risk of rectal carcinoma in adults with complete rectal prolapse, but this

risk has not been reported in children with partial rectal prolapse; it may be more commonly

associated with cystic fibrosis, Ehlers–Danlos or Hirschsprung’s disease.

It is more common in adults than children

In adults complete rectal prolapse is more common, in children partial rectal prolapse is more

common.

It should be treated with Delorme’s procedure

Delorme’s procedure is excision of the mucosal component of a prolapse with plication of the

muscle from below, and is generally used in adults.

194
Q

A 25-year

-old man from Uganda presents with a high

perianal fistula. He is a non-smoker who

has had a

chronic cough

for 3 months and has noticed

weight loss

of 2 stone in 6 months.

What is the most likely associated condition leading to this high anal fistula?

1) Carcinoma

2) Crohn’s disease 3) Diverticular disease

4) Tuberculosis

5) Ulcerative colitis

A

Explanation

Tuberculosis

The patient’s country of origin, Uganda, where tuberculosis (TB) is endemic, along with the

history of chronic cough and weight loss are highly suspicious for tuberculosis infection. Low

and high fistula are defined by the level at which they pass through the anal sphincters. A low

fistula’s internal orifice is below puborectalis. A high fistula’s orifice begins above this. The

former are common, relatively easy to manage and do not usually threaten the patient’s

sphincteric muscle function (and therefore continence). High fistulae are infrequent, complex to

manage and can lead to incontinence.

Carcinoma

This option is a possibility, and should certainly be considered given the weight loss red flag. It

is unlikely in a patient of this age, and does not explain h is cough and therefore is not the most

likely option.

Crohn’s disease

All the options listed are possible causes of high fistula. Crohn’s disease is a recognised cause in

this age group and the weight loss would fit with this diagnosis. Crohn’s disease is, however,

more common in smokers and this option does not explain the chronic cough.

Diverticular disease

All the options listed are possible causes of high fistula. Diverticular disease is possible, but

uncommon in younger patients.

Ulcerative colitis

Ulcerative colitis is a recognised cause in this age group and the weight loss would fit with this

diagnosis, however, this option also does not explain the chronic cough from which the patient

has been suffering.

195
Q

An

88-year-old

woman presents to the Emergency Department with

recurrent

painless abdominal

distension and vomiting. She has not opened her bowels for 5 days. Examination reveals a

distended, soft, mildly tender abdomen with reduced bowel sounds. Plain radiog raphs

demonstrate distended loops of large bowel that appear

unchanged

from those on previous

admissions for similar presentations. A diagnosis of colonic pseudo -obstruction is made.

Which one of the following statements is true? 1) A barium enema is likely to demonstrate a mechanical cause

2) The condition is not commonly associated with physical trauma

3) The condition is rarely associated with electrolyte disturbances

4) Institutionalised and immobile elderly patients are at increased risk

5) The condition may successfully be treated by gastroscopy

A

Explanation

Institutionalised and immobile elderly patients are at increased risk

This is correct, other risk factors include cerebral palsy, Parkinson’s disease or other

neurological disorders; chronic renal, pulmonary or cardiac disease; prolonged immobility; and

prolonged use of narcotic drugs and other anticholinergic drugs that reduce bowel motility.

Chronic colonic pseudo-obstruction is characterised by symptoms of recurrent bowel

obstruction, abdominal bloating and/or pain, evidence of bowel obstruction on radiographic

imaging, and no evidence of anatomic/structural abnormality. Possible complica tions include

diarrhoea (and associated electrolyte disturbances), intestinal perforation, nutritional deficiencies

and weight loss.

A barium enema is likely to demonstrate a mechanical cause

Pseudo-obstruction is not caused by a mechanical obstruction, instead it is a result of ineffective

peristalsis of the intestine.

The condition is not commonly associated with physical trauma

Physical trauma is a common cause of pseudo -obstruction.

The condition is rarely associated with electrolyte disturbances

Weight loss and electrolyte disturbances are common occurring as a result of loss of appetite,

frequent vomiting and poor nutritional intake. Additionally, the causative conditions in

secondary pseudo-obstruction (eg Parkinson’s disease, cerebral palsy, immobility, or chronic

renal failure) may impair the patient’s ability to take on adequate/appropriate nutrition or may

directly lead to electrolyte disturbances themselves.

The condition may successfully be treated by gastroscopy

Colonoscopy is often successful in decompressing the colon, although symptomatic relief from

this is often temporary.

196
Q

A 47-year-old man with a

long history of alcohol abuse

is admitted to hospital with acute

abdominal pain and is diagnosed clinically as having acute pancreatitis. This is confirmed by

finding a high serum amylase activity and by the results of an ultrasound scan. He has not had a

similar illness before. His serum is seen to be

lipaemic: serum triglyceride concentration is 26

mmol/l (normal <1.7 mmol/l)

Which one of the following statements is the most likely cause of the

hypertriglyceridaemia?

1) Decreased pancreatic lipase secretion

2) Deficiency of apolipoprotein C-II 3) Deficiency of LDL (low-density lipoprotein) receptors

4) Deficiency of lipoprotein lipase

5) Increased synthesis of VLDL (very-low-density lipoprotein)

A

Explanation

Increased synthesis of VLDL (very-low-density lipoprotein)

Hypertriglyceridaemia occurs frequently in association with a high alcohol intak e, and is due to

the increased synthesis of triglyceride for incorporation into VLDL in the liver. This is thought to

be as a result of the altered redox state as a consequence of alcohol metabolism, which favours

fat synthesis

Decreased pancreatic lipase secretion

Pancreatic lipase is normally secreted into the gut and higher plasma levels are found

in pancreatitis

Deficiency of apolipoprotein C-II

Apolipoprotein C-II activates lipoprotein lipase, which is responsible for the clearance of very-

low-density lipoprotein (VLDL) and chylomicrons from the plasma. Both lipase deficiency and

apolipoprotein C-II deficiency are rare, inherited causes of severe hypertriglyceridaemia, which

usually present in childhood.

Deficiency of LDL (low-density lipoprotein) recep tors

LDL receptors are involved in the transport of cholesterol from the plasma to the interior of cells:

deficiency is the cause of familial hypercholesterolaemia.

Deficiency of lipoprotein lipase

Lipoprotein lipase deficiency is a rare inherited cause of severe hypertriglyceridaemia. The

condition usually presents in childhood. Lipoprotein lipase is responsible for the clearance of

very-low-density lipoprotein (VLDL) and chylomicrons from the plasma.

197
Q

An infant should pass meconium at or soon after bir th. It results from materials ingested by the

neonate whilst it remains in utero.

The commonest underlying cause for meconium ileus in a newborn is?

1) Hirschsprung’s disease

2) Intussusceptions

3) Duodenal atresia

4) Imperforate anus

5) Cystic fibrosis

A

Explanation

Cystic fibrosis

Most newborn babies with meconium ileus have cystic fibrosis. Cystic fibrosis (CF) is an

autosomal-recessive disorder associated with the production of abnormal mucous with high salt

content. The Guthrie tests done in the first days of life include a spot test for CF, however, a

sweat chloride test is used to confirm the diagnosis. Meconium ileus occurs in CF as a result of

the following: the pancreas undergoes dysplasia, so with the loss of pancreatic enzymes and

presence of abnormal mucous, thick meconium forms. This is thick enough to causes

intraluminal intestinal obstruction usually at the level of the terminal ileum. Affected bowel will

be dilated and filled with heavy meconium making it prone to volvulus.

Hirschsprung’s disease

Hirschsprung’s disease results from abnormal development of the intrinsic nerves of the colon

and results in megacolon, rather than meconium ileus.

Intussusceptions

Intussusception is a common paediatric cause of obstruction, classically presenting with passing

of a ‘red currant jelly’ stool and signs of obstruction. It results from part of the bowel ‘folding’

into an adjacent section of bowel such that it becomes trapped within the lumen of the adjacent

section.

Duodenal atresia

This condition is the congenital a bsence of the duodenal lumen. It may be detected antenatally as

it is a cause of polyhydramnios, in newborns it will present with features of intestinal

obstruction.

Imperforate anus

This is a congenital absence or obstruction of the anal passage, which wi ll be quickly detected as

part of the routine newborn screening examination.

198
Q

Oesophageal cancer is a severe diagnosis requiring risky surgery where oncological success is

not always assured. Benign lesions are rare.

The commonest benign tumour of the oesophagus is?

1) Fibroma

2) Pseudopolyps

3) Adenoma

4) Leiomyoma

5) Fibrous polyps

A

Leiomyoma

Leiomyoma is the commonest benign tumour that occurs commonly in the lower part of the

oesophagus, although it is still a relatively rare growth. Overall benign tumours of the

oesophagus make up approximately 1% of all oesophageal tumours, and leiomyomas are around

two-thirds of this, or 0.6% of all oesophageal tumours. The majority of these are asymptomatic.

The commonest symptomatic presentation is dysphagia. The tumours are well encapsulated.

Endoscopic resection for small pedunculated lesions and localised resection for larger lesions is

the treatment if symptomatic.

Fibroma

Fibromas do occur, but are very rare and certainly not as common as leiomyomas.

Pseudopolyps

Pseudopolyps can occur, these are typically inflammatory in nature as a result of repetitive

inflammation/ulceration followed by deposition of granular tissue/regrowth of norma l tissue.

Adenoma

Adenomas do occur, but are very rare and certainly not as common as leiomyomas.

Fibrous polyps

The second commonest benign tumours are fibrous or fibrovascular polyps that occur commonly

in the upper third of the oesophagus.

Most of the tumours of the oesophagus are malignant and symptomatic benign tumours account

for less than 1% of oesophageal tumours.

199
Q

Crohn’s disease has distinct pathological features. This differentiates from other gastrointestinal

pathologies

Which one of the following is not a pathological characteristic of Crohn’s disease?

1) There are abnormal areas intervening between normal mucosa ‘skip lesions’

2) Usually limited to mucosa

3) Ulceration in the bowel is usually deep

4) Non-caseating granuloma

5) May affect any part of the alimentary tract

A

Explanation

Usually limited to mucosa Crohn’s is typically a transmural inflammatory process

There are abnormal areas intervening between normal mucosa ‘skip lesions’

In Crohn’s disease there are often abnormal areas intervening between normal mucosa, these are

called ‘skip lesions’.

Ulceration in the bowel is usually deep

The inflammation is transmural and ulceration are typically deep. This situation leads to a

predisposition to fistulae formation, perforations, abs cesses and strictures.

Non-caseating granuloma

Granuloma are a histological feature of Crohn’s disease.

May affect any part of the alimentary tract

In ulcerative colitis the rectum is always affected, however in Crohn’s disease any part of the

alimentary tract may be affected. Perianal disease is common in Crohn’s disease presenting with

features such as fissures, abscesses and fistulae. Mouth ulcers can also be a presenting feature.

Crohn’s disease is an inflammatory bowel disease that affects any part of the alimentary tract.

Patients are typically diagnosed before the age of 30, and it affects any ethnic group, but mostly

white patients and Ashkenazi Jews. Family history of the disease is a risk factor, and current

cigarette smoking is a very strong risk factor and also associated with more severe disease and an

increased likelihood of needing surgery. Presenting symptoms of gastrointestinal disease (ie not

extra-intestinal manifestations) frequently include weight loss, loss of appetite, abdominal pain,

diarrhoea with blood in stool. Additionally, mouth ulcers and perianal disease are common in

Crohn’s presenting with features such as fissures, abscesses and fistulae. The inflammation is

transmural affecting the whole thickness of the affected bowel and no n-caveating granulomas are

typically found. The terminal ileum is affected in nearly 75% of the cases. As the inflammation

is transmural and the ulcers associated with the disease are deep, fistulae formation, perforations,

abscesses and strictures are not unusual. Inflammatory bowel diseases are associated with extra -

intestinal manifestations that affect the skin (pyoderma gangrenosum, erythema nodosum), eyes

(iritis), joints (sacroiliitis, ankylosing spondylitis), biliary tree (sclerosing cholangitis, chr onic

active hepatitis).

200
Q

Blockage of biliary outflow from the liver results in dilated intrahepatic ducts. There is

interruption of the enterohepatic circulation and concentration of bilirubin increases in the blood.

For a patient with obstructive jaundice, which one of the following statements is true?

1) Clinically detectable jaundice does not occur until the bilirubin is above 60 µmol

2) Bile salt deposition in the skin can cause intense itching

3) Most biliary strictures are due to inflammatory scarring after cholecystitis

4) Cholangiocarcinoma is the commonest neoplasm causing obstructive jaundice

5) Primary sclerosing cholangitis (PSC) is more commonly seen in Crohn’s disease than in

Ulcerative colitis

A

Explanation

Bile salt deposition in the skin can cause intense itching

Biliary obstruction leads to accumulation of bile salts with backflow into the liver and systemic

circulation, raising their blood concentration and leading to skin deposition and pruritus. Clinically detectable jaundice does not occur until the bilirubin is above 60 µmol

Clinically detectable jaundice occurs above 30 µmol/l, at 60 µmol/l the jaundice should be

obvious.

Most biliary strictures are due to inflammatory scarring after chole cystitis

Most biliary strictures are iatrogenic after surgery, only small numbers are post inflammatory

disease.

Cholangiocarcinoma is the commonest neoplasm causing obstructive jaundice

Adenocarcinoma of the head of the pancreas is the commonest neoplasm leading to obstructive

jaundice. Ninety per cent of pancreatic carcinomas are adenocarcinomas derived from the ductal

cells of the exocrine pancreas; these have only a 5% 5 -year survival rate.

Cholangiocarcinoma is a rare tumour arising from biliary epithe lium. Unlike adenocarcinoma of

the pancreas these tumours are slow growing and often metastasise late so that complete

resection is often possible – nevertheless they have a poor prognosis.

Primary sclerosing cholangitis (PSC) is more commonly seen in Croh n’s disease than in

Ulcerative colitis

Primary sclerosing cholangitis is a rare condition thought to be of autoimmune origin; it results

in progressive fibrosis of the biliary system. PSC may arise spontaneously but is usually in

association with long-standing UC, rather than Crohn’s disease. Areas of short localised disease

can be treated with an indwelling stent.

201
Q

A senior house officer (SHO) working in a surgical ward

developed hepatitis, which resolved in

2 weeks. His blood tests showed that he had hepatitis A.

Six months after

this episode a liver

biopsy is performed for this SHO.

What is the biopsy most likely to show?

1) Central necrosis

2) Chronic septal fibrosis

3) Lobular fibrosis

4) Normal architecture

5) Periportal fibrosis

A

Explanation

Normal architecture

Hepatitis A virus has no known chronic carrier state and does not lead to chronic hepatitis or

cirrhosis. If a liver biopsy is performed after the patient has completely recovered from this

infection it will therefore show normal architecture.

Central necrosis

Central necrosis is a finding of acute hepatitis. Chronic septal fibrosis

This option a part of the histological findings of chronic hepatitis.

Lobular fibrosis

This option is a part of the histological findings of chronic hepatitis.

Periportal fibrosis

This option is a part of the histological findings of chronic hepatitis. It is also found in

schistosomiasis.

Hepatitis A virus is a single-stranded RNA picornavirus. This virus is the most common cause of

acute viral hepatitis and is particularly commo n in children and young adults. In some countries

more than 75% of adults have been exposed. Hepatitis A virus spreads primarily by faecal–oral

contact and so can occur in areas of poor hygiene. Water -borne and food-borne epidemics occur,

especially in underdeveloped countries. Eating contaminated raw shellfish is sometimes

responsible. Sporadic cases are also common, usually as a result of person-to-person contact.

Faecal shedding of the virus occurs before symptoms develop and usually ceases a few days after

symptoms begin. This means that infectivity often has already ceased when the hepatitis

becomes clinically evident. Hepatitis A virus has no known chronic carrier state and does not

lead to chronic hepatitis or cirrhosis. If a liver biopsy is performed after the patient has

completely recovered from this infection it will therefore show normal architecture.

202
Q

A

25-year-old

man with jaundice had laboratory investigations performed which showed: total

bilirubin

7.0 mg/dL

, (normal 0.3 to 1.0 mg/dL), direct (conjugated) bilirubin

0.5 mg/dl

(normal <

0.2 mg/dL) and

anaemia

.

What is the most likely cause of his jaundice?

1) Fibrosis of the common bile duct

2) Haemolysis

3) Hepatitis

4) Sclerosing cholangitis

5) Schistosomiasis

A

Explanation

Haemolysis

Haemolysis results in the destruction of red cells therefore anaemia. Destruction of red cells also

leads to the increased production of unconjugated bilirubin.

Fibrosis of the common bile duct

This option would cause an obstructive pattern of jaundice, the patient’s direct (conjugated)

bilirubin would be expected to be significantly more elevated than here.

Hepatitis Hepatitis typically causes a non-obstructive pattern of jaundice, as this patient’s blood tests

suggest. However, it does not explain the anaemia.

Sclerosing cholangitis

This option would cause an obstructive pattern of jaundice, the patient’s direct (conjugated)

bilirubin would be expected to be significantly more elevated than here.

Schistosomiasis

Schistosomiasis typically causes a non-obstructive pattern of jaundice, as this patient’s blood

tests suggest. However, it does not explain the anaemia.

Hyperbilirubinaemia results from increased bilirubin production, decreased liver uptake or

conjugation, or decreased biliary excretion. Total bilirubin normally is mostly unconjugated,

with values of <1.2 mg/dl ( <20 μmol/l). Fractionation can measure the proportion of bilirubin

that is conjugated (or direct, ie measured directly). Fractionation is r equired only in neonatal

jaundice or if the bilirubin is elevated, but the other liver test results are normal, suggesting that

hepatobiliary disease is not the cause. Unconjugated hyperbilirubinaemia (indirect bilirubin

fraction > 85%) reflects increased bilirubin production (for example when haemolysis is the case,

as in this scenario) or defective liver uptake or conjugation (eg in hepatitis, or Gilbert’s

syndrome). Such increases in unconjugated bilirubin are generally less than five -fold ( <6 mg/dl

or <100 μmol/l) unless there is concurrent liver disease. Conjugated hyperbilirubinaemia

typically suggests decreased biliary excretion, also known as obstructive jaundice. Common

causes of this condition include gall stones, cancers (such as head of pancreas tumours), chronic

inflammatory diseases such as primary sclerosis cholangitis, and iatrogenically following

procedures such as ERCP.

203
Q

A 43-year-old man has developed

sclerosing cholangitis

and iron deficiency anaemia secondary

to

chronic bloody diarrhoea

. They undergo colonoscopy as an outpatient.

A colonic biopsy would be most likely to show which one of the following?

1) Colonic adenocarcinoma

2) Diverticulitis

3) Granulomatous inflammation

4) Pseudopolyps

5) Villous adenoma

A

Explanation

Pseudopolyps

Pseudopolyps are a classic finding of ulcerative colitis. This diagnosis is suggested by presence

of sclerosing cholangitis (an extra-intestinal manifestation) along with the bloody diarrhoea in

the scenario above.

Colonic adenocarcinoma

This answer is not correct, it does not explain the sclerosis cholangitis.

Diverticulitis

Diverticulitis is not usually a chronic complaint, it is an acute inflammation of diverticula

typically in the distal colon. It presents typically with left iliac fossa tenderness, bloody

diarrhoea, and fever. Granulomatous inflammation

Granulomatous inflammation suggests Crohn’s disease, however, Crohn’s is not classically

associated with sclerosing cholangitis.

Villous adenoma

A villous adenoma is a non-malignant polyp that is most commonly found within the colon.

They can result in diarrhoea, typically not bloody diarrhoea though. They have considerable

potential to develop into malignant adenocarcinoma.

Ulcerative colitis affects the mucosa and submucosa of the rectum and colon, with a sharp border

between normal and affected tissue. Only in severe disease is the muscularis layer involved. In

early cases, the mucous membrane is erythaematous, finely granular and friable, with loss of the

normal vascular pattern and often with scattered haemorrhagic areas. Large mucosal ulcers with

copious purulent exudate are characteristic of severe disease. Islands of relatively normal or

hyperplastic inflammatory mucosa project above areas of ulcerated mucosa – these are name

pseudopolyps. Fistulas and abscesses do not occur, unlike in Crohn’s disease.

204
Q

A 38-year-old man with ulcerative colitis develops pruritus and fatigue. His alkaline phosphatase is elevated. The biliary tree appears to have a ‘beaded’ appearance on barium radiography.

Which one of the following is the most likely diagnosis?

1) Acute cholecystitis

2) Chronic cholelithiasis

3) Cholesterosis

4) Gallstone ileus

5) Sclerosing cholangitis

A

Explanation

Sclerosing cholangitis

Primary sclerosing cholangitis is a chronic cholestatic syndrome characterised by patchy

inflammation, fibrosis and strictures of the intrahepatic and extrahepatic bile ducts. These patchy

affects are what led to the classic ‘beaded’ appearance mentioned in the scenario above. Around

80% of patients with PSC have inflammatory bowel disease, often ulcerative colitis. Symptoms

of fatigue and pruritus develop late. The diagnosis is based on contrast cholangiography (with

endoscopic retrograde cholangiopancreatography) or magnetic resonance

cholangiopancreatography. The disease leads to eventual obliteration of the bile ducts, with

cirrhosis, hepatic failure and sometimes cholangiocarcinoma.

Acute cholecystitis

Cholecystitis presents with Murphy’s positive right upper quadrant pain and signs of infection.

On imaging the gall-bladder may appear thick-walled and with some surrounding fluid.

Chronic cholelithiasis

Chronic cholelithiasis may well be asymptomatic, or may present with symptoms of biliary colic.

Imaging will reveal gallstones within the gall-bladder. Cholesterosis

Otherwise known as ‘strawberry gall-bladder’, this condition gives the wall of the gall-bladder a

strawberry-like colour and visual texture as a result of hig h cholesterol.

Gallstone ileus

Gallstone ileus is mechanical obstruction of the bowel due to a gallstone.

205
Q

You are asked to examine a 42-yearof pain in the upper central abdomen

old man in the surgical out -patient clinic who gives a history that occurs 2–3 h after meals. He also tells you that he

often

wakes

up during the night with a similar pain.

Based on this history, what do you feel is the most likely cause of the pain?

1) Duodenal ulcer

2) Gastritis

3) Gastric ulcer

4) Pyloric stenosis

5) Zollinger-Ellison syndrome

A

Explanation

Duodenal ulcer

Duodenal ulcers tend to be associated with a fairly consistent pattern of pain. Pain is absent when

the patient awakens but appears mid-morning, is relieved by food, but recurs 2–3 h after a meal.

Pain that awakens a patient at night is common and is highly suggestive of duodenal ulcer.

Duodenal ulcers are more common than gastric ulcers.

Gastritis

Gastritis pain is usually intermittent and described as indigestion or ‘burning’ pain, it is typically

worsened shortly after eating food.

Gastric ulcer

Gastric ulcer pain also tends to be relieved by food, and come back a few hours later. Pain at

night being common. They are not, however, as common as duodenal ulcers.

Pyloric stenosis

Pyloric stenosis is extremely rare in adults. It presents with vomiting after meals and weight loss

as patients are not usually able to adequately nourish themselves. Zollinger-Ellison syndrome

This is a rare condition. Gastrinomas within either the pancreas or duodenum secret the hormone

gastrin, this hormone results in increased acid production within the stomach uncontrolled by

normal feedback loops. Excessive acid production then results in large numbers of gastric and

duodenal ulcers.

206
Q

A 67-year-old woman presenting with symptoms of severe gastro-oesophageal reflux
disease has recently had an upper gastrointestinal (GI) endoscopy. She has now been given a diagnosis of Barrett’s oesophagus.

Which one of the following statements most accurately describes this condition?

1) Occurs in approximately 40% of patients with gastro-oesophageal reflux disease (GORD)

in the long term

2) Is caused by chronic reflux of gastric acid into the lower oesophagus causing metaplasia of the normal squamous epithelium of the lower oesophagus to columnar and goblet cell epithelium.

3) Presents with malignancy in one-third of cases

4) Is diagnosed by the position of the z line which rises from its normal position

5) Histamine H2 blockers are the treatment of choice

A

Explanation

Is caused by chronic reflux of gastric acid into the lower oesophagus causing metaplasia of the

normal squamous epithelium of the lower oesophagus to columnar and goblet cell epithelium.

The chronic reflux of gastric acid into the lower oesophagus causes chronic inflammation. As a

result of this inflammation there is metaplasia of the affected oesophageal epithelium from

squamous epithelium to columnar epithelium with goblet cells. Later dysplasia can occur and

eventually this can develop into malignancy.

Occurs in approximately 40% of patients with gastro-oesophageal reflux disease (GORD) in the

long term

Around 5–15% of patients with gastro -oesophageal reflux disease (GORD) will develop

Barrett’s oesophagus (metaplasia), which in turn is associated with an increased risk of

oesophageal malignancy (neoplasia).

Presents with malignancy in one-third of cases

The risk of developing oesophageal cancer in people with Barrett’s is 0.13 %. About 1 in 860

patients with Barrett’s go on to develop cancer (Cancer Research UK).

Is diagnosed by the position of the z line which rises from its normal position The histological changes (from squamous to columnar epithelium with goblet cells) together

with elevation of the Z line on oesophago -gastro-duodenoscopy (OGD) is diagnostic. The Z line

usually lies 3 cm from the lower oesophageal sphincter and is identified by a change in colour

from salmon pink to a deeper red.

Histamine H2 blockers are the treatment of choice

The treatment of choice is with proton pump inhibitors and close follow up.

207
Q

A 56-year-old man presents to the acute surgical team with tenderness and pain in the left lower

quadrant, fever, nausea, diarrhoea and an increase white cell count. You suspect colonic

diverticular disease.

Which one of the following is correct regarding colonic diverticular disease?

1) It is frequently associated with electrolyte disturbances

2) It is a premalignant disease

3) It is usually an inherited condition

4) It may cause a vesicovaginal fistula

5) It may present with large bowel obstruction

A

Explanation

It may present with large bowel obstruction

Obstruction is unusual but well recognised. Often this is a result of fibrous tissue development

and stricturing following an episode, or several episodes, of diverticulitis. Diverticular

perforation is a common and important complication of diverticular disease and can be managed

medically or surgically depending on the Hinchey Classification. Fistulae to bladder, skin,

vagina and other loops of bowel are also common complications.

It is frequently associated with electrolyte d isturbances

Diverticulae are very common and often asymptomatic. Presenting features can include

persistent pain, irregular bowel habit including diarrhoea or constipation (or alternating).

Complications are also common presenting features, most commonly diverticulitis.

It is a premalignant disease

Colonic diverticular disease is not related to colon cancer but shares common dietary factors with

colorectal carcinoma.

It is usually an inherited condition

Colonic diverticular disease is not inherited. The biggest risk factor is lack of dietary fibre and it

usually occurs in older patients.

It may cause a vesicovaginal fistula

Fistula to bladder, skin, vagina and other loops of bowel are common but not vesicovaginal

fistulae.

208
Q

A 78-year-old woman presents with weight loss, nausea, vomiting and jaundice and is diagnosed

with carcinoma of the gall-bladder. She has a history of gallstones.

Which one of the following would suggest a better prognosis in this patient?

1) Age

2) Female

3) Presence of gallstones

4) TNM stage I disease

5) TNM of T4, any N, M0

A

Explanation

TNM stage I disease

While most gall-bladder carcinoma presents at a late stage and has a poor survival, if detected

early prognosis is much better. At stage 1A (T1, N0, M0) – simple cholecystectomy, should be

curative. At stage IB (T2, N0, M0) extended cholecystectomy is performed conveying a 5 -year

survival rate of 70–90%. The tumour is usually an adenocarcinoma, although in 10% of patients

squamous cell carcinoma may occur. Adenocar cinoma, particularly papillary adenocarcinoma, is

associated with the best prognosis.

Age

Age is a recognised risk factor for carcinoma of the gall-bladder, with the incidence rising with

age. It is most commonly diagnosed in those greater than 65 years of age.

Female

Female gender is a recognised risk factor for carcinoma of the gall-bladder, with a 2:1 female to

male ratio. This may be due to the increased frequency of cholelithiasis in women.

Presence of gallstones

Over 75% of patients with gall-bladder cancer have cholelithiasis. Other important predisposing

conditions include chronic cholecystitis, porcelain gall-bladder, gall-bladder polyps and

choledochal cysts.

TNM of T4, any N, M0

Gall-bladder carcinoma usually presents at a late stage and stage III disease (T4, any N, M0) is

generally not surgically curable (a 1-year survival rate for advanced gall-bladder cancer is less

than 5% with a median survival of 2–4 months).

209
Q

A 55-year-old man presents with

jaundice

, abdominal

ascites

, loss of weight and abdominal pain.

You suspect hepatocellular carcinoma.

Which one of the following would you expect in hepatocellular carcinoma?

1) Women are affected more than men

2) In hepatitis B virus endemic areas it usually presents above the age of 50

3) It is associated with HIV

4) It is associated with alcoholic liver disease

5) Serum a-fetoprotein levels (AFP) are normal

A

Explanation

It is associated with alcoholic liver disease

Alcoholic liver disease is strongly associated with hepatocellular carcinoma. I t is considered a

causative factor in up to 45% of hepatocellular carcinoma cases (in American and Italian studies.

Alcohol is causative through two possible mechanisms – direct genotoxicity and because

excessive chronic alcohol consumption leads to cirrho sis.

Women are affected more than men

Men are more commonly affected than women. Worldwide this ratio varies between 2:1 and 4:1.

In hepatitis B virus endemic areas it usually presents above the age of 50

In the UK the average age of presentation is 66, in countries with endemic hepatitis B there are

peaks in the age distribution at 45 and 65.

It is associated with HIV

Common viral risk factors include hepatitis B and C viruses (HBV and HCV). In Europe and

North America anti-HCV antibodies are present in around 70% of patients with hepatocellular

carcinoma (HCC). In countries with endemic hepatitis B, 90% of patients with HCC are positive

for hepatitis B. HIV is not an independent risk factor for HCC although co -infection with HCV

and HIV may lead to more rapid progression to HCC.

Serum a-fetoprotein levels (AFP) are normal

AFP levels are usually raised. Patients with cirrhosis due to HBV, HCV, alcohol, primary biliary

cirrhosis or haemochromatosis should be offered a 6 -monthly abdominal ultrasound scan and

serum AFP test as screening for hepatocellular carcinoma.

Clinical features of hepatocellular carcinoma include weight loss, anorexia, fever, a right upper

quadrant pain and ascites. Chemotherapy and radiotherapy are generally unhelpful. Survival,

except in very selected groups, is seldom more than 6 months.

210
Q

A 64-year-old man presents with a large mass in his anus, which is ulcerated and bleeds easily.

He is concerned and seeking your advice about treatment options.

Which one of the following statements regarding squamous cell carcinoma (SCC) of the

anal canal is correct?

1) It characteristically presents with faecal incontinence

2) It does not metastasise

3) It is relatively radioresistant

4) It is related to increased dietary fat intake

5) It is associated with human papillomavirus infection

A

Explanation

It is associated with human papillomavirus infection

The human papillomavirus (usually type 16) may cause intraepithelial neoplasia, which with

time progresses to anal carcinoma. This virus is found in approximately 70% of patients. Other

risk factors for anal carcinoma include a history of cervical or vaginal cancer, history of sexually

transmitted disease and immunosuppression following solid organ transplantation.

It characteristically presents with faecal incontinence

Rectal bleeding is the most common initial symptom (45% of cases); 30% of patients complain

of pain or a sensation of a mass.

It does not metastasise

Anal cancer can metastasize. The nodal metastatic locations depend on the lymphat ic drainage,

which is dependent on where the anal cancer originates: above the dentate line, flows to the

perirectal and paravertebral nodes; below the dentate line, drainage is through the inguinal and

femoral nodes. The most common site of distal metasta sis is the liver.

It is relatively radioresistant

Anal carcinoma is relatively radiosensitive. Presently, combination radiotherapy and

chemotherapy [fluorouracil (5-FU) ± mitomycin or 5-FU + cisplatin] is used as initial standard

first-line treatment. Abdo minoperineal (AP) resection is reserved for recurrent or resistant

tumours.

It is related to increased dietary fat intake

Increased dietary fat intake is thought to be a risk factor for colorectal cancer, not for anal

cancer.

211
Q

A 58-year-old man presents with ongoing indigestion, abdominal pain and loss of appetite. An

oesophago-gastroduodenoscopy (OGD) reveals gastric cancer.

Which one of the following statements regarding incidence of gastric cancer is correct?

1) Incidence is increasing in Europe

2) Incidence is lower in people with H. pylori infection

3) Incidence shows no preference for diets high or low in fresh fruit

4) Incidence is higher in those with blood group B or O

5) Incidence is higher in patients with pernicious anaemia

A

Explanation

Incidence is higher in patients with pernicious anaemia

Risk factors for developing gastric cancer include the presence of H. pylori, blood group A,

pernicious anaemia, previous gastric surgery and the presence of gastric polyps.

Incidence is increasing in Europe

The incidence of gastric cancer is decreasing in Europe.

Incidence is lower in people with H. pylori infection

Risk factors for developing gastric cancer include the presence of H. pylori, blood group A,

pernicious anaemia, previous gastric surgery and the presence of gastric polyps.

Incidence shows no preference for diets high or low in fresh fruit

Fresh fruit and vitamin C may be protective, but there is no consistent evidence to support it.

Incidence is higher in those with blood group B o r O

Blood group A may be associated with higher incidence, not groups B or O.

Gastric cancer in Western Europe often presents in patients over 55 years of age with new -onset

dyspepsia/reflux and upper abdominal pain. Other red flags include upper abdominal mass and

dysphagia, – these patients should be considered for urgent gastroscopy. Patients may also have

nausea, vomiting, anaemia, treatment -resistant dyspepsia or weight loss.

212
Q

A 21-year-old man presents with multiple colorectal polyps. He has a family history of familial

adenomatous polyposis (FAP). At colonoscopy he has multiple colorectal polyps.

Which one of the following statements is correct? 1) FAP is inherited as an autosomal recessive trait

2) FAP has an approximate 60% risk of progression to adenocarcinoma

3) FAP is diagnosed by computed tomography (CT) scan

4) FAP can present with other extra-colonic connective tissue abnormalities

5) The APC gene is an oncogene

A

Explanation

FAP can present with other extra-colonic connective tissue abnormalities

Gardner’s syndrome is a variant of FAP and can present with extra-colonic abnormalities

including multiple osteomas, epidermal cysts, desmoid tumours, lipomas and other connective

tissue abnormalities. Additionally it is associated with small bowel cancer, thyroid cancer,

pancreatic cancer, biliary cancer and hepatoblastoma.

FAP is inherited as an autosomal recessive trait

FAP is an inherited disease transmitted as an autosomal dominant trait.

FAP has an approximate 60% risk of progression to adenocarcinoma

In FAP, the risk of progression of an adenomatous polyp to adenocarcinoma is virtually 100%.

FAP is diagnosed by computed tomography (CT) scan

FAP is normally diagnosed at colonoscopy when a patient is found to have more than 100 polyps

alongside a strong family history and genetic testing.

The APC gene is an oncogene

An oncogene is a gene that is causative in cancer – either through activation or higher expression

levels. APC is a tumour suppressor gene – mutation results in inactivation. It is responsible for

the production of adenomatous polyposis coli.

FAP is an autosomal dominant inherited disease resulting in multiple adenomatous polyps within

the colon and a near 100% chance of progression to adenocarcinoma. It is estimated to have an

incidence at between 1:7000 to 1:22 000. Less than 1% of colorectal ca ncers is due to FAP.

Patients with known FAP should have yearly endoscopic screening at a minimum and will

usually require a colectomy at some point in their disease progression (as the number of polyps

increases, the risk of cancer increases). Even after colectomy, surveillance is still required at

between 6-monthly and 4-yearly testing depending on how much rectal tissue is left.

213
Q

A 57-year-old woman presents with hypergastrinaemia, duodenal ulcers and diarrhoea. She is

subsequently diagnosed with a gastrinoma neuroendocrine tumour.

Which one of the following statements concerning gastrinoma is correct?

1) They are malignant in 10% of cases

2) They occur at extrapancreatic sites in 30% of cases

3) They originate from H cells

4) They are associated with multiple endocrine neoplasia (MEN)

5) They secrete hydrochloric acid

A

Explanation

They are associated with multiple endocrine neoplasia (MEN)

Gastrinoma may occur as one of the MEN syndromes (ie typically MEN 1):

MEN type

Features

MEN1

Parathyroid

Pancreas

Pituitary

MEN2a

Parathyroid

Phaeochromocytoma

Medullary thyroid cancer

MEN2b

Marfanoid/mucosal neuroma

Phaeochromocytoma

Medullary thyroid cancer

They are malignant in 10% of cases

Around 60% of cases are malignant.

They occur at extrapancreatic sites in 30% of cases

Although gastrin is usually produced by gastric antral G cells, tumours of G cells most

commonly originate in the pancreas. Around 10% occur in extrapancreatic sites, eg in the

duodenum.

They originate from H cells

They originate from G cells.

They secrete hydrochloric acid

G cells secrete gastrin. Gastrin is responsible for stimulating hydrochloric acid secretion from the

parietal cells of the stomach. Excess gastrin production results in Zollinger–Ellison syndrome,

resulting in widespread peptic ulceration.

214
Q

You are discussing a patient in the

colorectal

multi-disciplinary team (

MDT

). This young man

has presented with profuse bloody diarrhoea and progr essive weight loss. Colonic biopsies are

suggestive of inflammatory bowel disease.

Which one of the following is more suggestive of ulcerative colitis than Crohn’s disease?

1) Cancer

2) Fistula formation

3) Obstruction

4) Perianal involvement

5) Stricture formation

A

Explanation

Cancer

The risk of developing cancer is higher in ulcerative colitis (UC) than in Crohn’s disease – the

main associated risks are duration of disease and extent of disease (ie total colitis is greater than

partial colitis). After 10 years of total colitis the risk of cancer is approximately 4–5%.

Fistula formation

Fistula formation, as well as intra-abdominal or peri-anal abscesses, are a frequent complication

of Crohn’s disease.

Obstruction

Obstruction is a frequent complication of Crohn’s disease.

Perianal involvement

Perianal problems (fissures, fistulae, skin tags) are much more commonly seen in Crohn’s

disease than in UC.

Stricture formation

Strictures are a frequent complication of Crohn’s disease.

Crohn’s disease is a granulomatous inflammatory disorder involving the whole thickness of the

bowel wall (any part of the gastrointestinal tract from mouth to anus). Ulcerative colitis is a non -

granuloma process that only involves the mucosa. This difference may explain the higher

incidence of stricture, obstruction and fistula formation in Crohn’s disease. Perianal problems

(fissures, fistulae, skin tags) are much more commonly seen in Crohn’s disease than in UC. UC

invariably involves the rectum and extends proximally towards the caecum, and can sometimes

cause a ‘backwash ileitis’.

Crohn’s disease

Ulcerative colitis

Segments affected

Any part of the alimentary tract from mouth to anus, most commonly the terminal ileum

Affects the rectum, extending contiguously proximally a variable amount but limited to the colon

Pattern of disease

Skip lesions

Continuous

Macroscopic findings

Cobblestones, aphthous and linear ulcers, abscesses, luminal narrowing

Pseudopolyps, very friable mucosa, toxic megacolon

Microscopic

Transmural inflammation, granulomas

Submucosal or mucosal findings

inflammation, crypt abscesses

Common symptoms

Abdominal pain, diarrhoea, weight loss, mouth ulcers, perianal disease

Bloody diarrhoea, weight loss, abdominal pain

Extraintestinal features

Perianal disease (fissures, fist ulae, tags, abscesses), mouth ulcers, oral aphthous ulcers, erythema nodosum, arthritis and spondyloarthropathies, uveitis, episcleritis

PSC, arthritis and spondyloarthropathies, uveitis, episcleritis

215
Q

You are performing a laparoscopic appendicectomy and notice an inflamed structure

approximately 60 cm from the terminal ileum. The appendix appears normal. You diagnose an

inflamed Meckel’s diverticulum.

Which one of the following statements is true of a Meckel’s’ diverticulum?

1) It is a common cause of malabsorption

2) It is found in over 20% of the general population

3) It is invariably found in the jejunum

4) It may lead to macrocytic anaemia

5) It may present with rectal bleeding

A

Explanation

It may present with rectal bleeding

Some cases of Meckel’s diverticulum are asymptomatic, but 1 in 2 contain either heterotopic

gastric or pancreatic mucosa. Most symptomatic cases will present before the age of 2 years.

Complications of Meckel’s diverticulum include diverticulitis, intussusception, ulceration and gastrointestinal bleeding, intestinal obstruction and perforation. Meckel’s diverticulum, the rule

of 2s:

occurs in 2% of the population

2:1 male:female ratio

approximately 2 inches long

found 2 feet from the ileocaecal junction

1 in 2 will contain ectopic tissue (gastric or pancreatic)

only 2% are symptomatic

important cause of rectal bleeding in the under 2s.

It is a common cause of malabsorption

Most cases of Meckel’s diverticulum are asymptomatic, but 20% contain heterotopic gastric or

pancreatic mucosa. Complications of Meckel’s diverticulum include infection/inflammation,

intussusception, ulceration and gastrointestinal bleeding, intestinal obstruction and perforation.

It is found in over 20% of the general population

Meckel’s diverticulum occurs in 2% of the population and affects men twice as often as women.

It is invariably found in the jejunum

It is classically found in the distal ileum, 60 cm from the ileocaecal valve on the antimesenteric

border of the small intestine and is approximately 5 cm long.

It may lead to macrocytic anaemia

The common causes of macrocytic anaemia include vitamin B12 or folate deficiency, alcohol,

liver disease or reticulocytosis – Meckel’s diverticulum does not cause a macrocytic anaemia.

Meckel’s diverticulum is a true diverticulum with a mucous membrane and a muscular coat and

may be connected to the umbilicus by either a fibrous band or a complete fistula – remnants of

the vitellointestinal duct. If a Meckel’s diverticulum is found incidentally, it should be left alone,

especially if it is non-inflamed with a wide neck.

216
Q

You are assessing a middle-aged woman in your clinic. She initially presented to you via

casualty with progressive weight loss and abdominal pain together with intermittent diarrhoea

and constipation. Her computed tomography (CT) scan reveals a stenosing lesion in the sigmoid

colon.

Which one of the following statements is true regarding colorectal carcinoma?

1) Irritable bowel syndrome is associated with an increased incidence of colorectal

carcinoma

2) Left sided colonic tumours usually present with anaemia

3) More than 80% of colonic carcinomas arise within adenomatous polyps

4) Left-sided colonic tumours tend to be exophytic

5) The commonest genetic abnormality in colonic carcinoma is p53

A

Explanation

More than 80% of colonic carcinomas arise within adenomatous polyps

More than 80% of colorectal cancers arise from pre-existing polyps, which makes the disease

suitable for screening. The initial screening test used in the UK is the faecal occult blood test,

given to 60–74-year-olds every 2 years. In patients with a positive test or symptoms suggestive

of colorectal cancer, endoscopy with colonoscopy or flexible sigmoidoscopy and biopsy is the

diagnostic investigation of choice. Irritable bowel syndrome is associated with an increased incidence of colorectal carcinoma

Colorectal carcinoma is associated with inflammatory bowel disease, especially ulcerative

colitis, rather than irritable bowel syndrome.

Left sided colonic tumours usually present with anaemia

Right-sided colonic tumours tend to be exophytic and often present with anaemia as the caecum

is of a large calibre so rarely obstructs. In contrast, left-sided cancers tend to be stenotic.

Left-sided colonic tumours tend to be exophytic

Right-sided colonic tumours tend to be exophytic and often present with anaemia as the caecum

is of a large calibre so rarely obstructs. In contrast, left -sided cancers tend to be stenotic.

The commonest genetic abnormality in colonic carcinoma is p53

The commonest genetic abnormality is in the APC gene – it is a tumour suppressor gene.

Mutations in the TP53 gene, which codes for the p53 protein – another tumour suppressor gene –

are also common. There are also several syndromes that have a strong association with this

cancer [Gardener’s syndrome, hereditary nonpolyposis colorectal cancer (HNPCC) and FAP].

217
Q

You are admitting a young man who has presented with 1 week of se vere abdominal pain

following a binge drinking session on holiday. A diagnosis of acute pancreatitis is made.

Which one of the following are correct regarding complications of acute pancreatitis?

1) Gastrointestinal haemorrhage with resultant haematemesis and melaena is usually the

result of rupture of the posterior duodenal artery due to stress

2) Hypocalcaemia is relatively common and is caused by fat saponification

3) Pancreatic pseudocysts are formed in the greater sac of the abdomen

4) Relative hypoxia in acute pancreatitis is due to multiple sub-clinical pulmonary emboli

due to hypercoagulability

5) Splenic rupture is a common complication

A

Explanation

Hypocalcaemia is relatively common and is caused by fat saponification

Acute pancreatitis is an inflammatory process that is caused by the release of pancreatic enzymes

from the pancreatic acini. This causes both localised and systemic damage. The release of

pancreatic enzymes into the body’s circulation results in fat necrosis and saponification – the free

fatty acids released by this process bind calcium and result in systemic hypocalcaemia.

Gastrointestinal haemorrhage with resultant haematemesis and melaena is usually the result of

rupture of the posterior duodenal artery due to stress

Stress-induced gastric erosions frequently occur – these can result in haemorrhage and resultant

haematemesis and melaena. This is the most common cause of haemorrhage. Proton pump inhibitors (PPIs) or H 2 antagonists are prophylactically used to prevent stress-induced gastric

erosions.

Pancreatic pseudocysts are formed in the greater sac of the abdomen

Pancreatic pseudocysts are enzyme rich collections, which are formed in the lesser sac of the

abdomen following acute and/or chronic pancreatitis.

Relative hypoxia in acute pancreatitis is due to multiple sub-clinical pulmonary emboli due to

hypercoagulability

Hypoxia is usually due to hypo -ventilation due to pain, however, ARDS can also occur due to

release of inflammatory mediators.

Splenic rupture is a common complication

Splenic rupture is a very rare complication of acute pancreatitis. Common complications include:

pancreatic pseudocysts, pancreatic abscess, stress-induced gastric erosions with haematemesis or

melaena, acute renal failure, toxic psychosis, multiple organ failure, hypocalcaemia and chronic

pancreatitis.

218
Q

You are trying to give the family of a young woman who has endoscopic retrograde

cholangiopancreatography (ERCP)-related pancreatitis, some indication of her disease severity.

You decide to perform additional tests so that you can determine the modified Glasgow score

(Imrie criteria).

Which one of the following thresholds scores a point in the modified Glasgow criteria

(Imrie criteria) for acute pancreatitis?

1) Age >40

2) Alkaline phosphatase (ALP) > 200 IU/l

3) Corrected calcium < 2 mmol/l

4) Haematocrit <.45

5) Pa (O2 ) < 10

A

Explanation

Corrected calcium < 2 mmol/l

Prognosis of pancreatitis (Ranson’s criteria)

One point is given for each criterion present, severe pancreatitis = three or more points.

Age >40

Age > 55 would score a point.

Alkaline phosphatase (ALP) > 200 IU/l

ALP is not part of the scoring system. Haematocrit <.45

A haematocrit count is not part of the scoring system.

Pa (O2 ) < 10

Pa (O2 ) < 8 would score a point.

The modified Glasgow criteria enable early diagnosis of severe acute pancreatitis. The following

parameters are part of the scoring system:

age > 55

blood glucose > 10 mmol/l – no history of diabetes

white cell count (WCC) > 15.5 × 10 9 /l

lactate dehydrogenase (LDH) > 600 IU/l

aspartate aminotransferase (AST) > 200 U/l

corrected calcium < 2 mmol/l

urea > 16 mmol/l

Pa (O2 ) < 8 kPa (60 mmHg)

albumin < 32 g/l.

The mortality per positive criterion:

0–2 < 5% mortality

3–4 20% mortality

5–6 40% mortality

7–8 100% mortality.

The modified Glasgow criteria does not include serum amylase, haematocrit or pH.

219
Q

You are examining a

63-year-old

man with abdominal pain, ascites and jaundice. An abdominal

ultrasound scan (USS) shows signs suspicious of a hepatocellular carcinoma.

Which one of the following is correct regarding hepatocellular carcinoma (HCC)?

1) Women are affected more than men

2) It is associated with hepatitis A viruses

3) It is associated with type 1 diabetes

4) Serum a-fetoprotein (AFP) levels are in the normal range

5) Usually presents above the age of 50

A

Explanation

Usually presents above the age of 50

Liver cancer incidence is strongly related to age, with the highest incidence rates being in older

males and females. In the UK in 2012-2014, on average each year more than 4 in 10 (44%) cases

were diagnosed in people aged 75 and over.

Women are affected more than men

Men are more commonly affected than women, world-wide this varies between 2:1 and 4:1.

It is associated with hepatitis A viruses

Common viral risk factors include hepatitis B and C viruses (HBV and HCV). In Europe and

North America anti-HCV antibodies are present in around 70% of patients with hepatocellular

carcinoma. While in countries with endemic hepatitis B 90% of patients with HC C are positive for hepatitis B. Alone, HIV is not a risk factor for HCC although co -infection with HCV and

HIV may lead to more rapid progression to HCC.

It is associated with type 1 diabetes

Type 2 diabetes with poor insulin control may be associated with hepatocellular carcinoma.

Serum a-fetoprotein (AFP) levels are in the normal range

Serum a-fetoprotein (AFP) levels are usually raised. Patients with cirrhosis due to HBV, HCV,

alcohol, primary biliary cirrhosis or haemochromatosis should be offered a 6 -monthly abdominal

ultrasound scan and serum AFP test as screening for hepatocellular carcinoma.

Clinical features include weight loss, anorexia, fever, a right upper quadrant pain, and ascites.

Serum a-fetoprotein is usually raised. Surgical resection is occasionally possible. Chemotherapy

and radiotherapy are unhelpful. Survival, except in very selected groups, is seldom more than 6

months.

220
Q

You are assisting your consultant with a laparoscopic cholecystectomy. During the operation he

asks you questions to test your knowledge.

Which one of the following statements is correct regarding biliary pathology?

1) Pigment stones area the commonest type of gallstones

2) Sickle-cell disease causes largely acalculous cholecystitis

3) 80% of gallstones are symptomatic

4) Murphy’s sign is right upper quadrant (RUQ) pain on palpation, causing sudden cessation

of inspiration and forcing the patient to catch breath

5) A common infecting agent in cholecystitis is Salmonella

A

Explanation

Murphy’s sign is right upper quadrant (RUQ) pain on palpation, causing sudden cessation of

inspiration and forcing the patient to catch breath

Murphy’s sign is an indication of acute cholecystitis. The patient catches breath due to pain on

inspiration while the right hypochondrium is deeply palpated, but not when the left

hypochondrium is deeply palpated, due to downward movement of the inflamed gall-bladder

onto the examining hand during inspiration.

Pigment stones area the commonest type of gallstones

Cholesterol gallstones are the most common but with haemolytic conditions, such as sickle -cell

anaemia, pigment stones are also common.

Sickle-cell disease causes largely acalculous cholecystitis

Sickle-cell disease results in increased red-cell haemolysis and turnover, leading to a

preponderance to develop pigment stones.

80% of gallstones are symptomatic

Approximately 70% of gallstones are thought to be asymptomatic and are found on routine

ultrasound examination. A common infecting agent in cholecystitis is Salmonella

Secondary infection within stagnant bile converts cholelithiasis to cholecystitis. Common

organisms are E. coli, Klebsiella, Enterococcus faecalis and Bacteroides.

221
Q

You are seeing an elderly man in the Emergency Department with gross abdominal distension and prominent superficial abdominal veins. He has shifting dullness and you diagnose ascites secondary to chronic liver disease.

Which one of the following statements is correct regarding ascites?

1) Malignancy causing ascites is a transudate

2) When associated with oedema this is usually non-pitting

3) A high serum-ascites albumin gradient (>1.1) indicates a transudate such as pancreatitis

4) A low serum-ascites albumin gradient (<1.1) indicates an exudate such as hereditary

angioedema

5) Spontaneous bacterial peritonitis is due to the introduction of sepsis following an ascitic

tap

A

Understanding Ascites and Its Causes

Ascites is the accumulation of fluid in the peritoneal cavity, often associated with liver disease, malignancy, heart failure, or infections. The nature of the fluid can be classified into transudate and exudate based on the serum-ascites albumin gradient (SAAG). This classification helps determine the underlying cause of ascites.

  1. Malignancy causing ascites is a transudate

This statement is incorrect. Malignant ascites typically presents as an exudate due to increased capillary permeability associated with tumor involvement in the peritoneum. Therefore, malignancy-related ascites usually has a low serum-ascites albumin gradient (<1.1).

  1. When associated with oedema this is usually non-pitting

This statement is misleading. Edema associated with liver disease and ascites can be either pitting or non-pitting depending on various factors such as the duration and severity of fluid retention. In cases of significant liver dysfunction leading to hypoalbuminemia, edema may present as pitting.

  1. A high serum-ascites albumin gradient (>1.1) indicates a transudate such as pancreatitis

This statement is incorrect. A high SAAG (>1.1) indicates that the ascitic fluid is likely a transudate, which is commonly seen in conditions like cirrhosis or heart failure rather than pancreatitis specifically, which can lead to exudative ascites.

  1. A low serum-ascites albumin gradient (<1.1) indicates an exudate such as hereditary angioedema

This statement is also incorrect in its association; while a low SAAG (<1.1) does indicate an exudative process, hereditary angioedema does not typically cause ascites. Exudative causes are more commonly linked to infections (like tuberculosis), malignancies, or inflammatory processes.

  1. Spontaneous bacterial peritonitis is due to the introduction of sepsis following an ascitic tap

This statement contains some truth but lacks clarity regarding spontaneous bacterial peritonitis (SBP). SBP occurs without any identifiable source of infection and is primarily due to bacterial translocation from the gut flora into the ascitic fluid rather than being directly caused by an invasive procedure like an ascitic tap.

Given these evaluations:

The correct answer regarding ascites from the options provided would be none of them are entirely accurate, but if we were to select one that aligns closest with common clinical understanding regarding SBP’s pathophysiology without misinterpretation, it would be option 5 concerning SBP’s association with infection risk post-tap.

However, since none of these statements accurately reflect established medical knowledge about ascites comprehensively:

Answer: None of the statements are correct regarding ascites in their entirety; however, option 5 has elements related to SBP risk post-tap but misrepresents its primary causation mechanism.

222
Q

A patient with cirrhosis requires a laparotomy. The surgical team are making a decision on the

risks and benefits of surgical vs conservative treatment.

The Child–Pugh classification of the severity of

the following?

chronic liver disease

includes which one of

1) Ascites

2) Aspartate transaminase

3) Bleeding varices

4) Cholesterol

5) Clotting time

A

Explanation

Ascites

See overall explanation for tables showing the full scoring system and prognosis depending on

score.

Aspartate transaminase

See overall explanation for tables showing the full scoring system and prognosis depending on

score.

Bleeding varices

See overall explanation for tables showing the full scoring system and prognosis depending on

score.

Cholesterol

See overall explanation for tables showing the full scoring system and prognosis depending on

score.

Clotting time

See overall explanation for tables showing the full scoring system and prognosis depending on

score.

The Child-Pugh classification takes into account bilirubin level, albumin level, ascites,

encephalopathy, and prothrombin time.

A

B

C Bilirubin

<34

34-51

> 51

Albumin

> 35

28-35

<28

Ascites

None

Controlled

Refractory

Encephalopathy

None

Minimal

Advanced

Prothombin

<4

4-6

> 6

8

223
Q

A 28-year-old man presents with severe right -sided abdominal pain together with peritonism in

the right iliac fossa. His clinical picture and haematological profile are consistent with

appendicitis but at operation an inflamed Meckel’s diverticulum is identified.

Which one of the following statements is true regarding Meckel’s diverticulum?

1) It is a partial persistence of the median umbilical ligament

2) It arises on the mesenteric border of the small bowel

3) It typically lies about 60 cm from the ileocaecal valve

4) It is present in 0.2% of individuals

5) It contains heterotopic pancreatic tissue in 40% of cases

A

Explanation

It typically lies about 60 cm from the ileocaecal valve

It is classically found in the distal ileum, within 100 cm (typically 60 cm) from the ileocaecal

valve on the antimesenteric border of the small intestine and is approximately 5 cm long.

Meckel’s diverticulum is a true diverticulum with a mucous membrane and a muscular coat and

may be connected to the umbilicus by either a fibrous band or a complete fistula – remnants of

the vitellointestinal duct. If a Meckel’s diverticulum is found incidentally, it should be left alone,

especially if it is non-inflamed with a wide neck. Meckel’s diverticulum, the rule of 2s:

occurs in 2% of the population

2:1 male:female ratio

approximately 2 inches long

found 2 feet from the ileocaecal junction

1 in 2 will contain ectopic tissue (gastric or pancreatic)

only 2% are symptomatic

important cause of rectal bleeding in the under 2s.

It is a partial persistence of the median umbilical ligament

A Meckel’s diverticulum is a partial persistence of the vitellointestinal duct and there may be a

fibrous connection to the umbilicus. The median umbilical ligament or ‘urachus ’ is a connection

between the umbilicus and the bladder.

It arises on the mesenteric border of the small bowel

Meckel’s diverticulum arises on the antimesenteric border of the small bowel. It is present in 0.2% of individuals

It is present in 2% of individuals and affects men twice as often as women.

It contains heterotopic pancreatic tissue in 40% of cases

Some cases of Meckel’s diverticulum are asymptomatic, but 50% contain heterotopic gastric or

pancreatic mucosa. Complications of Meckel’s diverticulum include intussusception, ulceration

and gastrointestinal bleeding, intestinal obstruction and perforation.

224
Q

An elderly man presents with progressive dysphagia and weight loss. He is a lifetime heavy smoker and his GP is concerned. Which one of the following statements best represents his diagnosis of oesophageal carcinoma?

1) It has wide geographical variation in incidence rates and is increasing in frequency in the UK

2) Squamous-cell carcinomas of the oesophagus usually occur in the upper two -thirds of the oesophagus and are associated with GORD

3) The development of a hoarse or husky voice is a concerning feature likely to be due to
direct invasion of the aryepiglottic folds

4) Palliation of symptoms is seldom necessary as most patients die soon after developing
dysphagia

5) Diagnosis is usually established by a barium swallow alone

A

Explanation

It has wide geographical variation in incidence rates and is increasing in frequency in the UK

Oesophageal carcinoma in the UK is increasing in incidence. There is a great deal of

geographical variation in the incidence, with the highest risk areas being central Asia and

South/East Africa. Common risk factors include smoking, alcohol, ingestion of caustic agent s

(including very hot drinks), age, obesity and previous radiotherapy.

Squamous-cell carcinomas of the oesophagus usually occur in the upper two -thirds of the

oesophagus and are associated with GORD

If found in the lower one-third of the oesophagus, it is usually of the adenocarcinoma type and

associated with gastro -oesophageal reflux disease (GORD). If found in the upper two -thirds of

the oesophagus is of similar origin as head and neck cancers and is of squamous -cell origin. This

type, however, is usually associated with smoking not GORD as the above answer states.

The development of a hoarse or husky voice is a concerning feature likely to be due to direct

invasion of the aryepiglottic folds

A hoarse voice is a worrying feature but is usually due to involvement of the recurrent laryngeal

nerve.

Palliation of symptoms is seldom necessary as most patients die soon after developing dysphagia

Palliation is an important aim of all treatment for oesophageal cancers when cure is not possible.

Overall survival for oesophageal cancer is 40% at 1 year, 15% at 5 years (cancer research UK

statistics). However, survival varies by type (squamous-cell carcinoma or adenocarcinoma) and

by stage of presentation. Early cancers may be fully resectable with oesophagectomy or

endoscopic mucosal resection for very early cancer. Adjuvant and neoadjuvant radiotherapy and

chemotherapy are also used in oesophageal cancer.

Diagnosis is usually established by a barium swallow alone Diagnosis may be suggested by a barium swallow but definitive diagnosis requires histology

obtained via an oesophago -gastro- duodenoscopy (OGD).

225
Q

A 68-year-old woman presents with symptoms of gastro -oesophageal reflux disease. She has

tested positive for Helicobacter pylori infection and been started on proton pump inhibitor (PPI)

and eradication therapy.

Which one of the following tests, in common usage, has the highest combined sensitivity

and specificity when used to detect Helicobacter pylori infection?

1) Culture of biopsy material

2) McNemar’s test

3) The biopsy urease test

4) The urea breath test

5) Serology/stool testing

A

Explanation

The urea breath test

The urea breath test detects radiolabelled carbon dioxide in the breath after ingesting

radiolabelled urea (sensitivity 95%, specificity 96%).

Culture of biopsy material

Culture of biopsies may yield the presence of H. pylori, but this is not routinely used

McNemar’s test

McNemar’s test is a non-parametrical statistical test and nothing to do with H. pylori.

The biopsy urease test

Biopsy of the stomach wall is the basis of the urease test (‘Clo -test’), which takes up to 30 min to

get a result and depends on the presence of urease in the bacterium to alter the substrate from

orange to pink. Sensitivity is the highest of the presented options (98%), but it is not highly

specific (82%).

Serology/stool testing

Serology testing is used, but serological testing (sensitivity 92%, specificity 83%) performs less

well when compared with breath testing, which is the most sensitive and specific (sensitivity

95%, specificity 96%) and stool testing (sensitivity 95%, specificity 94%)

226
Q

A 76-year-old lifelong smoker presented to the Emergency Department with haematemesis.

Following stabilisation, an oesophago -gastro- duodenoscopy (OGD) was performed with the

presumed diagnosis of a gastric ulcer. A large polypoid mass was found in the distal oesophagus

adjacent to the gastro -oesophageal junction with evidence of a co -existent hiatus hernia. Biopsy

confirmed the diagnosis of

oesophageal adenocarcinoma

.

Which one of the following confers the highest relative risk of developing this condition?

1) Barrett’s oesophagus

2) Human papillomavirus (HPV) infection

3) Alcohol

4) Gastro-oesophageal reflux disease

5) Helicobacter pylori infection

A

Explanation

Barrett’s oesophagus

Barrett’s oesophagus is considered a pre-malignant lesion associated with a risk of 6–14% of

developing oesophageal adenocarcinoma and requires regular endoscopic surveillance. T he

relative risk of oesophageal cancer is approximately 11.

Human papillomavirus (HPV) infection

The relative risk of oesophageal squamous-cell carcinoma is between 2.4 and 3.6 in human

papillomavirus (HPV) infected patients.

Alcohol

Alcohol is a recognised risk factor for squamous carcinoma, the risk increases with volume

drunk with relative risk of between 1.8–7.4.

Gastro-oesophageal reflux disease

The relative risk of adenocarcinoma is between 4.9 for patients who have weekly symptoms, to

7.4 for patient s who have daily symptoms.

Helicobacter pylori infection

This infection is associated with gastric cancer rather than oesophageal cancer.

The most common presenting symptom of oesophageal cancer is dysphagia, usually this will

progressively worsening and symptoms will start with dysphagia to food and progress on to

dysphagia for liquids. Cancer in the distal third of the oesophagus is most frequently

adenocarcinoma, which is most commonly associated with gastro -oesophageal reflux disease and

especially Barrett’s oesophagus. Cancers of the upper two -thirds of the oesophagus are

classically squamous-cell carcinomas.

884 Intussusception is much more common in the paediatric population. Often a lesion in the bowel

will cause the pathological process to take place.

227
Q

Which one of the following is the most common pathological lead point in intussusception?

1) Dance’s sign

2) Intraluminal haematoma

3) Meckel’s diverticulum

4) Hypertrophic Peyer’s patch

5) Polyps

A

Explanation

Hypertrophic Peyer’s patch

Peyer’s patches in the ileum are the pathological lead patch in up to 90% of cases.

Dance’s sign

Dance’s sign, (a feeling of emptiness on palpation), with viscera absent in the right lower

quadrant on abdominal X-ray is seen in 15% of cases. It is not, however, a ‘lead point’ in

intussusception.

Intraluminal haematoma

An intraluminal haematoma in Henoch–Schönlein purpura may trigger intussusception.

Meckel’s diverticulum

Meckel’s diverticulum and polyps are identified as a pathological lead point in 10% of cases.

Polyps

Meckel’s diverticulum and polyps are identified as a pathological lead point in 10% of cases.

Intussusception is a condition where a part of the bowel, normally small bowel, telescopes inside

an adjacent part of the bowel. It is most common in infants aged 4 months to 1 year, with peak

incidence between 6–9 months. Risk factors, aside from age, include previous intussusception, or

family history of the condition, and intestinal malrotation. It presents with abdominal pain,

‘redcurrant jelly’ stool and signs of obstruction and sepsis. Diagnosis can be made on ultrasound

scan (USS) or on air or contrast enema. The enema can serve a dual purpose to also decompress

the intussuscepted segment. If this fails surgery may be required. Pathologically the theory is th at

a ‘lead point’ is usually to blame that instigates the telescoping – it is thought that up to 90% of

cases are as a result of Peyer’s patches.

228
Q

You are reviewing a 1-day-old with abdominal distension and bile stained vomiting . He has not yet passed meconium. You suspect Hirschsprung’s disease.
Which one of the following is true?

1) There is a congenital ganglion absence in the rectum

2) There will be dilation of the affected segment

3) This affects both sexes equally

4) This is diagnosed by cold-cup rectal biopsy

5) This usually presents in patients greater than 10-years of age

A

Explanation

There is a congenital ganglion absence in the rectum

In Hirschsprung’s disease there is a defective neural crest -cell migration resulting in a congenital absence of the ganglia that normally make up Auerbach (myenteric) plexus and Meissner (submucosal) plexus to part of the colon. The result of this absence is increased tone in the affected part of the colon, and an absence of appropriate peristalsis that causes a functional (rather than mechanical) obstruction.

There will be dilation of the affected segment

Dilatation occurs proximal to the affected segment. The affected segment is hypertonic and therefore stenotic so faeces accumulate proximal to this.

This affects both sexes equally

Hirschsprung’s disease occurs in 1 in 5000 births and has an M:F ratio of 5:1.

This is diagnosed by cold-cup rectal biopsy

Suction biopsy of the narrowed segment is first line, this is examined for absence of Auerbach and Meissner plexus. Full-thickness rectal biopsy is required if suction biopsy is inconclusive.

This usually presents in patients greater than 10-years of age

Most commonly Hirschsprung’s disease is diagnosed shortly after birth because of failure to pass meconium within 48 h of deliver y. Other symptoms can include signs of obstruction such as abdominal distension and vomiting. Additionally diagnosis can be delayed, presenting later in childhood with chronic constipation.

229
Q

A patient presents

with jaundice. On closer examination

the presence of Kayser–Fleischer rings

is noted.

This is pathognomonic of which one of the following?

1) Autoimmune hepatitis 2) Sclerosing cholangitis

3) Primary biliary cirrhosis (PBC)

4) Haemochromatosis

5) Wilson’s disease

A

Explanation

Wilson’s disease

In Wilson’s disease, the presence of Kayser –Fleischer rings indicates neurological impairment. It

consists of copper deposition in the cornea and may be seen as a greenish or golden brown ring

around the cornea on slit lamp examination. Symptoms mostly relate to deposition of copper in

the liver (causing liver disease) and brain (causing neuropsychiatric symptoms, such as

parkinsonism due to deposition in the basal ganglia).

Autoimmune hepatitis

Autoimmune hepatitis has no specific associat ed eye signs, it can present as an acute or chronic

hepatitis.

Sclerosing cholangitis

This condition is frequently associated with inflammatory bowel disease, especially ulcerative

colitis. It may present before or after diagnosis of inflammatory bowel disease. Symptoms are

mostly related to liver disease, there are no specific eye signs.

Primary biliary cirrhosis (PBC)

Patients with PBC frequently develop xerostomia and dry eyes. It typically presents in middle -

aged women with intense itching and signs of liver disease such as jaundice.

Haemochromatosis

There are no specific eye signs associated with this condition. Presentation can include liver

disease, cardiac failure, arthritis, diabetes, tray-bronze skin and other endocrine complaints.

230
Q

A 32-year-old woman who had undergone an appendicectomy 2 weeks ago presents to her General Practitioner with a 24 h history of right upper quadrant abdominal pain, fever with chills

and rigors, and shortness of breath. She says that the pain is radiating to her right

shoulder

tip

.

On examination, her temperature is 38.9°C, pulse rate is 106/min and she is acutely

tender

over

the right hypochondric region. Chest X-ray reveals a right -sided basal

atelectasis

and

mild

pleural effusion

. From the options below choose the one option that you think is the most likely diagnosis in

this patient.

1) Emphysema of right lung

2) Acute pancreatitis

3) Sclerosing cholangitis

4) Subphrenic abscess

5) Pyonephrosis

A

Explanation

Subphrenic abscess

Subphrenic abscess usually arises 3 to 6 weeks following abdominal surgery, mainly to the

biliary tract, duodenum or stomach, or following a perforated viscus or anastomotic leakage.

The subphrenic space is in direct contact with the para-colic gutter thereby allowing peritoneal

contamination such as bile, blood or bowel contents to spread. Subphrenic abscess is right -sided

in about 50%, left -sided in 25% and bilateral in 25% of patients. Clinical features

include pyrexia with chills and rigors, anorexia, loss of appetite and loss of weight.

Diaphragmatic irritation may affect the lung, resulting in chest pain, dyspnoea and non -

productive cough. Basal atelectasis, pneumonia and pleural effusion are recognised

complications of this condition. Additionally, diaphragmatic irritation can cause referred pain to

the shoulder because the phrenic nerve (which innervates the diaphragm) shares nerve roots with

the dermatomes overlying the shoulder (ie roots C3 –5). Ultrasound scan is the investigation of

choice to diagnose subphrenic abscess, and, if an abscess is identified, ultrasound guided

percutaneous drainage catheter may be placed at the same time.

Emphysema of right lung

Pulmonary emphysema is a chronic lung condition that, along with chronic bronchitis, is now

more commonly known clinically as chronic obstructive pulmonary disease (COPD). In

emphysema there is dilation of the airways distal to the terminal bronchioles due to destruction

of elastase within the supporting connective tissue of the distal airways, most commonly it is

associated with smoking although a less predisposing factor is alpha 1 -antitrypsin deficiency.

Patients with chronic obstructive pulmonary disease (COPD) are predisposed to lower

respiratory tract infections which would explain the shortness of breath, fevers and c hest X-ray

changes, but would not normally cause RUQ tenderness with pain radiating to the shoulder.

Acute pancreatitis

Acute pancreatitis is an important differential of the acute abdomen, however, this patient’s

recent surgery and confirmed appendicitis, combined with atypical signs for pancreatitis (ie RUQ

pain) means this diagnosis is not the most likely. It rarely presents with rigors.

Sclerosing cholangitis

Sclerosing cholangitis is a chronic inflammatory condition, usually found in the presence of

inflammatory bowel disease. It can lead to chronic liver disease through obstruction of the biliary

ducts. It is not a likely diagnosis in this acute scenario especially without evidence of liver

disease (eg jaundice). Pyonephrosis

Pyonephrosis is a collection within the renal pelvis. It is a recognised complication of

pyelonephritis, renal stones or hydronephrosis but can also occur as a primary condition. You

would expect the patient’s pain to be over the renal angle more that the right hypochondrium,

urinary symptoms may be present, and chest signs would not be typical.

231
Q

An 87-year-old woman who lives in residential care is brought to the Accident and Emergency

Department with a 24 h history

of abdominal pain of

sudden onset

. On examination, she is pale,

cold and appears to be in

shock

. Her blood

and

mildly tender

pressure is 102/68 mmHg and her pulse is 92/min and

irregular. Abdomen is soft

over the umbilical

region. Bowel sounds are

absent

.

She was noticed to pass a few episodes of passing dark (

bloody) coloured motions during the

day. Analysis of arterial blood gas reveals:

Result

Normal

pH

7.24

7.35–7.45 Bicarbonate

20 mmol

22–26 mmol

Base excess

−10 mmol

−2 to +2 mmol

Which of the following is the most likely diagnosis in this patient?

1) Sigmoid volvulus

2) Acute small bowel obstruction

3) Toxic megacolon

4) Leaking abdominal aortic aneurysm

5) Mesenteric infarction (Ischaemic bowel disease)

A

Explanation

Mesenteric infarction (Ischaemic bowel disease)

The history, signs and symptoms are classical of mesenteric infarction, which is part of the

spectrum of ischaemic bowel disease. While ischaemic bowel disease is a spectrum of acute and

chronic conditions resulting in inadequate blood flow to the bowel (either large or small),

mesenteric infarction specifically implies an obstruction to blood flow of the small bowel (which

is supplied by the mesenteric vessels) that has resulted in cell death. The risk factors include age,

atrial fibrillation, previous myocardial infarction, heart failure and thrombophilia – the ischaemia

then caused by an embolic event. Mesenteric infarction typically presents with persistent and

generalised abdominal pain. Vomiting may or may not be present. Some patients may pre sent

with shock, with the shock being out of proportion to the clinical symptoms. The infarcted bowel

may shed the ‘non-viable mucosa’, which mixed with mucus, results in dark-coloured (also

known as plum-coloured) stools. The inflammatory markers such as the white cell count and C-

reactive protein may be elevated. Arterial blood gas analysis is a very useful investigation, which

may reveal a metabolic acidosis. This condition is a surgical emergency as the patient rapidly

become septic and may die from septic shock unless the infarcted bowel (‘dead gut’) is removed.

Sigmoid volvulus

This condition typically presents with abdominal distension and failure to pass stool or flatus

(absolute constipation), lower abdominal pain is also a frequent symptom. Their a bdomen will be

clinically distended but may not be tender. Complications include ischaemia of the section

affected, perforation and peritonitis. Risk factors include being elderly, chronic constipation and

male gender.

Acute small bowel obstruction

Acute small bowel obstruction normally presents with colicky abdominal pain, nausea and

vomiting failure to pass stool/flatus occurring after this as the obstruction may be relatively

proximal. Mostly commonly it is secondary to adhesions, hernias or malignancy. Other causes of

small bowel obstruction include Crohn’s disease, volvulus, and foreign bodies (including

gallstone ileus and swallowed foreign bodies). Toxic megacolon

Toxic megacolon presents as a complication to conditions such as inflammatory bowel dise ase

or Clostridium difficile infection most commonly. It presents with severe abdominal pain,

distention and signs of sepsis. With no antecedent history of a causative condition this option is

not the most likely.

Leaking abdominal aortic aneurysm

A leaking abdominal aortic aneurysm may present acutely with haemorrhagic shock, or

subacutely less commonly. Additionally the rupture may be contained within the retroperitoneum

or uncontained – the latter allowing for much greater and faster blood loss. Classic signs include

abdominal pain, back pain, hypotension and a palpable expansile mass in the abdomen. This

diagnosis could account for the patient’s relatively shocked state, but does not explain the fever

or dark stool.

232
Q

A

74-year-old man

presents to his General

Practitioner

with a 4-month history of

vague

lower

abdominal pain, abdominal distension and

constipation. On further questioning he says that he

has noticed a

change in his bowel habits

, a sense of

incomplete

evacuation

of the bowel and his

stools to be mixed with

blood

. He has

lost

more than a stone in

weight

during this period. On

examination, the abdomen is soft but a

mass

is felt over the left iliac fossa. Plain abdominal X-

ray reveals a markedly

dilated colon.

From the options below choose the one that you think is the most likely diagnosis in this

patient.

1) Toxic megacolon

2) Acute colonic pseudo-obstruction

3) Sigmoid volvulus

4) Colonic carcinoma

5) Diverticular disease

A

Colonic carcinoma Elderly patients with per rectal bleeding, change in bowel habit, and weight loss should be

considered to have colonic cancer unless proven otherwise.

Toxic megacolon

Toxic megacolon normally presents secondary to a predisposing condition, such as ulcerative

colitis or C. difficile infection. It is an acute presentation, with abdominal pain and distension and

signs of sepsis.

Acute colonic pseudo-obstruction

The chronicity of the history makes acute colonic pseudo -obstruction unlikely.

Sigmoid volvulus

Sigmoid volvulus usually presents with a rapid onset abdominal distension and absolute

constipation. The chronic history to this presentation makes a sigmoid volvulus unlikely, it

would also not explain the per rectum (PR) bleeding nor the weight loss.

Diverticular disease

While diverticular disease can explain the patient’s change in bowel habit and PR bleeding, it

does not explain the patient’s weight loss.

The clinical presentation of patients with colonic malignancy depends on the site of the tumour.

Right-sided colonic carcinoma commonly presents with anaemia, tiredness, malaise, pallor and

loss of weight, while the left -sided colonic carcinoma presents with change in the bowel habits,

bleeding per rectum and intestinal obstruction. Rectal carcinoma, in addition to the features seen

in left-sided colonic carcinoma, is associated with a sense of incomplete evacuation of the bowel

(tenesmus). Increase in age is a risk factor for developing colonic cancer. The other risk factors,

among others, include a family history of colon cancer, familial adenomatous polyposis, diet rich

in red meat and long standing ulcerative colitis or Crohn’s disease. The investigation of choice

for suspected colonic malignancy is a colonoscopy with biopsy of suspicious lesions. If this is

not possible, usually due to patient’s lack of fitness for the examination, then flexible

sigmoidoscopy or computed tomography (CT) colonoscopy is usually the next

line. Carcinoembryonic antigen (CEA) is the commonly used tumour marker to diagnose colonic

malignancy and subsequently to assess the progress, including recurrence.

Obviously, in a case of acute obstruction due to suspected malignancy a different initial set of

investigations will be required, likely to include basic bloods and plain abd ominal X-ray (AXR),

with CT scan likely required acutely if surgery is contemplated. Dukes’ classification for

colorectal cancers is useful to be aware of, however, in clinical practice the TNM classification is

now more frequently used.

Classification

Disease extent

Duke A

Limited to muscular propria

Duke B

Extending beyond muscularis propria Duke C

Lymph node involvement

Duke D

Distant Metastatic spread

233
Q

A 33-year-old company executive presents to his General Practitioner with a 6-week history

of

upper

abdominal pain and diarrhoea. He also says that he has had a few episodes of

‘dark-

coloured’ vomitus

during this period. General examination is unremarkable. Endoscopic

examination

of his upper gastrointestinal region reveals

multiple ulcers

in the

stomach

and

duodenum.

From the options below choose the ONE that you think is the most likely diagnosis in this

patient

1) Carcinoid tumour

2) Gastrinoma

3) Somatostatinoma

4) Vasoactive intestinal peptide tumour

5) ACTH secreting tumour

A

Explanation

Gastrinoma

Gastrinomas, which occur primarily in the pancreas and duodenum, are malignant in nearly two -

thirds of cases. The patients may present with upper abdominal/epigastric pain and vomiting; the

vomitus may be ‘coffee-ground’ due to bleeding from the ulcers. Ninety per cent of patients with gastrinomas develop peptic ulceration due to excess gastrin release by the tumour – gastrin

stimulates gastric acid hypersection, which causes ulceration. Gastrinomas may eit her arise

sporadically or as part of Zollinger–Ellison syndrome (peptic ulceration, gastric acid

hypersecretion and islet -cell tumour of the pancreas). Sporadic Zollinger –Ellison occurs most

frequently in the fifth decade of life. Approximately 20% of patients with Zollinger–Ellison

syndrome have MEN type I syndrome. An elevated basal gastric acid output >15 mEq/h and a

serum gastrin >1000 pg/ml are suggestive of a gastrinoma. If it is difficult to make a diagnosis, a

secretin stimulation test may be indicated. Lesions are localised by somatostatin-receptor

scintography. A CT scan may be indicated to exclude metastases. The treatment of this condition

is either conservative (high dose proton pump inhibitors) or surgical. Surgical resection may be

aided by intra-operative ultrasound and/or intra-operative endoscopy.

Carcinoid tumour

This is a neuroendocrine tumour most commonly found in the small bowel. It can cause local

symptoms (eg in the bowel it can cause bowel obstruction, bleeding or diarrhoea), and sym ptoms

as a result of release of hormones such as serotonin into the bloodstream (carcinoid syndrome –

diarrhoea, abdominal pain, flushing, tachycardia, wheeze). They can be sporadic, but are also

part of MEN-1. Diagnosis is by 24-h urinary collection to detect unusually high levels of urinary

carcinoid breakdown products.

Somatostatinoma

This is a tumour of the delta cells (the cells responsible for production of somatostatin) of the

pancreas. The clinical result of this is a result of the inhibitory actions of somatostatin on various

pancreatic and gastrointestinal hormones: suppression of insulin – diabetes, suppression of CCK

and secretin – steatorrhoea and gallstones, suppression of gastrin – hypochlorhydria.

Vasoactive intestinal peptide tumour

VIPomas develop from pancreatic islet cells. Release of VIP results in sometimes profound

watery diarrhoea, hypokalaemia and associated symptoms, abdominal pain, weight loss.

ACTH secreting tumour

ACTH tumours are normally pituitary tumours, however, exogenous ACT H production can

result from other tumours (notably certain lung cancers). Excess ACTH production results in

excess stimulation of cortisol production in the adrenals and therefore Cushing’s syndrome.

234
Q

A 28-year-old hair stylist presents to her General Practitioner with a 3-month history of crampy lower abdominal pain, diarrhoea (12–15 times/day), mouth ulcers and loss of appetite. She reckons that she has lost nearly a stone in weight during this period. She smokes 20 cigarettes/day.

Abdominal examination reveals a mildly tender mass over her right iliac fossa. A few abscesses with sinus formation are noticed in the perianal region.

Which of the following is the most likely diagnosis in this patient?

1) Pelvic inflammatory disease (PID)

2) Pseudomembranous colitis

3) Ulcerative colitis

4) Crohn’s disease

5) Carcinoma of the colon

A

Step-by-Step Analysis of the Symptoms and Diagnosis

Patient Presentation: The patient is a 28-year-old female hair stylist presenting with a 3-month history of:

Crampy lower abdominal pain
Diarrhoea (12–15 times/day)
Mouth ulcers
Loss of appetite
Significant weight loss (nearly a stone)
Physical Examination Findings:

Mildly tender mass over the right iliac fossa
Presence of abscesses with sinus formation in the perianal region
Differential Diagnosis Consideration:

Pelvic Inflammatory Disease (PID):

Typically presents with lower abdominal pain, fever, and abnormal vaginal discharge.
Less likely due to the absence of fever and specific gastrointestinal symptoms.
Pseudomembranous Colitis:

Often caused by Clostridium difficile infection, leading to diarrhea and abdominal pain.
Usually associated with recent antibiotic use, which is not mentioned in this case.
Ulcerative Colitis:

A form of inflammatory bowel disease (IBD) characterized by continuous inflammation of the colon.
Symptoms include bloody diarrhea, abdominal pain, and urgency but does not typically present with perianal abscesses or sinus formation.
Crohn’s Disease:

Another type of IBD that can affect any part of the gastrointestinal tract.
Commonly presents with abdominal pain, diarrhea (which may be bloody), weight loss, and complications such as strictures or fistulas.
The presence of perianal abscesses is particularly indicative of Crohn’s disease.
Carcinoma of the Colon:

Generally presents later in life with changes in bowel habits, weight loss, and possibly obstruction.
Less likely given the patient’s age and symptom profile.
Conclusion: Considering all symptoms presented—particularly the combination of crampy abdominal pain, frequent diarrhea, mouth ulcers, significant weight loss, and especially the presence of perianal abscesses—the most fitting diagnosis for this patient is Crohn’s disease. This condition aligns well with her clinical presentation and physical examination findings.

Thus, based on this thorough analysis:

Answer: Crohn’s disease

235
Q

A 44-year-old woman presents to the Surgical Emergency Assessment Unit with a 72 h history of abdominal pain, vomiting and being generally unwell. She has also noticed pale stools and

dark urine

. On examination, she is

jaundiced

and is tender over the

right upper quadrant

. An

ultrasound reveals a

dilated

proximal common bile duct with intra-hepatic duct

dilatation. A

magnetic resonance (MR) cholangiopancreatogram confirms a

fistula

between the gall-bladder

and the common bile duct, and a

large calculus

is found in the

common bile duct

just distal to the

fistula.

From the options below choose the one that you think is the most likely diagnosis in this

patient.

1) Carcinoma of the head of the pancreas

2) Cholangiocarcinoma

3) Mirizzi’s syndrome

4) Hepatocellular carcinoma

5) Carcinoma of the ampulla of Vater

A

cholecystocholedochal fistula formation due to direct pressure necrosis of adjacent duct

walls from large impacted stones.

Patients may present with pain over the right upper quadrant of the abdomen, vomiting, fever,

recurrent cholangitis, cholecystitis or pancreatitis. Pale stools and dark urine result from

obstruction of the flow of bile into the intestine. Treatment of this condition is exploration of the

common bile duct by either open or laparoscopic cholecystectomy and placement of a T-tube.

Carcinoma of the head of the pancreas

This is the commonest malignant cause of obstructive jaundice classically causing ‘painless

jaundice’ and presenting late. The acute history and tenderness is not typical for this

presentation, additionally the imaging findings are not in keeping with this diagnosis.

Cholangiocarcinoma

This is a relatively uncommon malignant cause of obstructive jaundice. The acute history and

tenderness is not typical for this presentation, additionally the imag ing findings are not in

keeping with this diagnosis.

Hepatocellular carcinoma

Hepatocellular carcinoma (HCC) does not typically cause obstructive jaundice. In the UK it is

normally found in patient’s with underlying liver disease or cirrhosis, for example due to

hepatitis B, C or alcohol. Aflatoxin ingestion is a notable cause in South-East Asia and sub-

Saharan Africa – this is a toxin produced by fungal growth in certain grains. Presentation of

HCC may be on routine screening of patients with known liver d isease, with local physical

symptoms such as right upper quadrant pain, or with signs of liver failure – either due to

decompensation of underlying liver disease or massive infiltration of the liver by the carcinoma

cells.

Carcinoma of the ampulla of Vater

This is a rare cancer arising from the ampulla of Vater. It most commonly presents with

obstructive jaundice, however, the acute history and tenderness is not typical for this

presentation, additionally the imaging findings are not in keeping with this diagnosis.

236
Q

A 56-year-old man, who drinks about

60–70 units

of alcohol per week, presents to the

Emergency Department with severe abdominal pain and 2 –3 episodes of

vomiting blood

. On

examination, he appears pale with a pulse rate of 110/min. Abdominal examination

reveals

dilated veins in the anterior abdominal wall

, mild

ascites, a

large

spleen and a

small

nodular liver

.

Which of the following is the most likely diagnosis in this patient?

1) Chronic pancreatitis

2) Hepatocellular carcinoma (HCC)

3) Amoebic hepatitis

4) Alcoholic liver cirrhosis

5) Myeloproliferative disorder

A

Explanation

Alcoholic liver cirrhosis

Alcoholic liver cirrhosis can cause portal hypertension; which causes the stigmata this patient

presents with. Portal hypertension is defined as an increase in the portal vein pressure of more

than 10 mmHg (the normal portal vein pressure is in the range of 5 –10 mmHg). Among other

causes, cirrhosis of the liver is one of the important causes of portal hyperte nsion and currently

accounts for up to 90% of cases in the UK. As a result of the portal hypertension collateral

channels develop between the portal system and systemic circulation – common sites for this

include the lower end of oesophagus (resulting in o esophageal varices), distal rectum, and

anterior abdominal wall (resulting in dilated tortuous veins in the anterior abdominal wall known

as caput medusae). Bleeding from varies can be profuse and life-threatening – bleeding from

oesophageal varies is part icularly common particularly oesophageal varices leading to

haematemesis and/or melaena. Treatment requires urgent resuscitation, definitive management is

usually endoscopic. Some of the signs and symptoms of portal hypertension include abdominal pain, ascites, jaundice, splenomegaly and signs of cirrhosis (such as spider naevi, gynaecomastia,

palmar erythema and testicular atrophy).

Chronic pancreatitis

Chronic pancreatitis can present either due to chronic insufficiency of it is exocrine function

(causing steatorrhoea, fat-soluble vitamin deficiency and weight loss), endocrine function (most

notably causing insulin-dependent diabetes mellitus) or due to distinct episodes of acute -on-

chronic inflammation presenting with severe epigastric abdominal pain and often vomiting.

Alcohol is the commonest cause of this condition in the UK.

Hepatocellular carcinoma (HCC)

In the UK this condition is normally found in patient’s with underlying liver disease or cirrhosis,

for example due to hepatitis B, C or alcohol. Aflatoxin ingestion is a notable cause in South-East

Asia and sub-Saharan Africa – this is a toxin produced by fungal growth in certain grains.

Presentation of HCC may be on routine screening of patients with known liver disease, with

local physical symptoms such as right upper quadrant pain, or with signs of liver failure – either

due to decompensation of underlying liver disease or massive infiltration of the liver by the

carcinoma cells.

Amoebic hepatitis

Entamoeba histolytica can cause localised abscesses or a diffuse hepatitis. Presentation normally

includes right upper quadrant tenderness and hepatomegaly, fever, and possibly jaundice. Bloods

will show evidence of hepatitis. Liver infection will normally follow an episode of intestinal

amebiasis/amoebic d ysentery. Risk factors for this condition include old age, recent tropical

travel and immunosuppression (including chronic alcohol use, steroids, HIV, etc.).

Myeloproliferative disorder

Myeloproliferative disorders encompass chronic myeloid leukaemia (CML) , essential

thrombocythaemia, polycythaemia (rubra) vera and myelofibrosis. They can all cause

hepatosplenomegaly and portal hypertension (which explains this patient’s presentation) –

however, in a patient drinking as much alcohol as this portal hypertens ion caused by liver

cirrhosis is the most likely cause.

237
Q

Gallbladder pathology commonly presents on the acute general surgical take. A spectrum of

diseases exist.

Which one of the following clinical conditions is most likely to be associated with

identification of Aschoff–Rokitansky sinuses?

1) Acute cholecystitis

2) Cholesterosis of the gall-bladder (strawberry gall-bladder)

3) Chronic cholecystitis

4) Mucocele of the gall-bladder

5) Xanthogranulomatous cholecystit is

A

Explanation

Chronic cholecystitis

Chronic cholecystitis is associated with a fibrotic thick walled gall-bladder. Within the thickened

wall are Aschoff–Rokitansky sinuses, mucosal herniations often containing inspissated bile.

Acute cholecystitis

Acute cholecystitis is associated with increased vascular permeability and infiltration with acute

inflammatory cells.

Cholesterosis of the gall-bladder (strawberry gall-bladder)

Cholesterol-laden macrophages in the lamina propria of the gall-bladder mucosa are a feature of

cholesterosis.

Mucocele of the gall-bladder

Mucocele of the gall-bladder normally occurs in a thin walled non-inflamed gall-bladder.

Xanthogranulomatous cholecystitis

Xanthogranulomatous cholecystitis is a rare form of chronic cholecystitis characterised by huge

numbers of lipid-laden macrophages and giant -cells. Xanthogranulomatous cholecystitis can

easily be mistaken for carcinoma.

238
Q

Inflammatory bowel diseases are systemic conditions. The significance o f other signs and

symptoms can point more clearly to the underlying disease process.

Which one of the following associated extra-alimentary conditions is far more suggestive of

ulcerative colitis than Crohn’s disease?

1) Ankylosing spondylitis

2) Arthropathy

3) Erythema nodosum

4) Primary sclerosing cholangitis

5) Pyoderma gangrenosum

A

Explanation

Primary sclerosing cholangitis

Primary sclerosing cholangitis is far more common in ulcerative colitis than in Crohn’sdisease. It

is characterised by a progressive fibrous inf lammatory reaction within the biliary tree leading to

irregularity with multiple stenosis and biliary obstruction. There is no apparent relationship

between the duration or severity of ulcerative colitis and ultimate progression to liver failure.

Ankylosing spondylitis

Arthropathy and ankylosing spondylitis are both features of ulcerative colitis and Crohn’s

disease. Ankylosing spondylitis is slightly more common in ulcerative colitis.

Arthropathy

Arthropathy and ankylosing spondylitis are both features of ulcerative colitis and Crohn’s

disease.

Erythema nodosum

Erythema nodosum is the commonest cutaneous manifestation of inflammatory bowel disease

and occurs slightly more commonly in Crohn’s disease.

Pyoderma gangrenosum

Pyoderma gangrenosum is slightly more prevalent in ulcerative colitis than Crohn’s disease,

however, primary sclerosing cholangitis is far more common still in ulcerative colitis.

The following table summarises several key differences between ulcerative colitis and Crohn’s

disease. Crohn’s disease

Ulcerative colitis

Segments affected

Any part of the alimentary tract from mouth to anus, most commonly the terminal ileum

Affects the rectum, extending contiguously proximally a variable amount but limited to the colon

Pattern of disease

Skip lesions

Continuous

Macroscopic findings

Cobblestones, aphthous and linear ulcers, abscesses, luminal narrowing

Pseudopolyps, very friable mucosa, toxic megacolon

Microscopic findings

Transmural inflammation, granulomas

Submucosal or mucosal inflammation, crypt abscesses

Common symptoms

Abdominal pain, diarrhoea, weight loss, mouth ulcers, perianal disease

Bloody diarrhoea, weight loss, abdominal pain

Extraintestinal features

Perianal disease (fissures, fistulae, tags, abscesses), mouth ulcers, oral aphthous ulcers, erythema nodosum, arthritis and spondyloarthropathies, uveitis, episcleritis

PSC, pyoderma gangrenosum, arthritis and spondyloarthropathies, uveitis, episcleritis

239
Q

A 52-year-old woman with chronic abdominal pain was noticed to be pale at the time of

assessment

in the surgical out -patient clinic. Investigations showed that

she had a

decreased

serum iron

and

total iron-binding capacity (TIBC) in association with a

normal serum ferritin

.

These findings are most indicative of which one of the following?

1) Anaemia of chronic disease

2) Autoimmune haemolytic anaemia

3) Chronic blood loss

4) Malabsorption

5) Vitamin B 12 deficiency

A

Explanation

Anaemia of chronic disease

The serum iron, iron-binding capacity and saturation are all low, but there is plenty of storage

iron that is not being utilised so ferritin is normal. These symptoms are typical of anaemia of

chronic disease – you would also expect to find a normal mean corpuscular volume (MCV)

(normocytic).

Autoimmune haemolytic anaemia

In haemolytic anaemia the haemolysed cells release haemoglobin that is usually not lost from the

body and is recycled, so the iron studies are usually normal. The patient’s MCV will normally be

increased due to reticulocytosis (macrocytic), while LDH and bilirubin will be increased and

haptoglobin decreased.

Chronic blood loss

Chronic blood loss can lead to iron deficiency because of the loss of iron, with a low serum iron,

high TIBC and low ferritin. The MCV is typically decreased (microcytic).

Malabsorption

Malabsorption, particularly in disease processes involving the duodenum, can occasionally

explain iron deficiency anaemia, which has a high TIBC and low ferritin. Malabsorption

involving the distal ileum can produce a megalobla stic anaemia through malabsorption of

vitamin B 12 (macrocytic).

Vitamin B 12 deficiency Vitamin B 12 and folate deficiencies lead to a megaloblastic anaemia that is not associated with

abnormalities in iron metabolism. The bone marrow contains megaloblastic cells, while

peripheral blood contains large red blood cells (macrocytosis) and hypersegmented neutrophils.

The following table explains the expected results of iron studies in several conditions:

Iron

TIBC

TIBC saturation

Ferritin

MCV

Iron deficiency

Low

High

Low

Low

Decreased

Anaemia of chronic disease

Low

Low

Low

Normal

Normal

Acute phase response

Low

Low

Low

High

Normal

Haemochromatosis/iron overload

High

Low

High

High

Normal

240
Q

A 38-

year-old woman has been diagnosed with atrophic gastritis. You are reviewing her

OGD

report

.

Which one of the following findings is most likely in this patient?

1) Decreased mean corpuscular volume in the red blood cells

2) Decreased serum ferritin

3) Decreased serum folate

4) Increased neutrophil segmentation

5) Increased reticulocyte count

A

Explanation

Increased neutrophil segmentation

This is a sign of megaloblastic anaemia, known as hypersegmented neutrophils, which is a

symptom of atrophic gastritis due to vitamin B-12 deficiency. Other signs include macrocytosis

(high MCV), and megaloblasts within the bone marrow.

Decreased mean corpuscular volume in the red blood cells

Decreased microcytosis (MCV) is classic of iron deficiency anaemia, as well as thalassaemia and

sideroblastic anaemia.

Decreased serum ferritin

Decreased serum ferritin represents a lack of iron stores in the body, for example in iron

deficiency anaemia.

Decreased serum folate

Folate deficiency leads to a megaloblastic anaemia. Its absorption is not affected by atrophic

gastritis.

Increased reticulocyte count

Increased reticulocyte count is a classic sign of haemolytic anaemia.

241
Q

A

40-year-old

man is diagnosed with right -sided

synchronous

colonic tumours,

which

are

mucinous

and

poorly differentiated

. His mother had died at an

early age of

ovarian cancer

and

his maternal grandfather had died aged approximately 40 years of

advanced caecal carcinoma

.

From the following list of inherited syndromes which one is most likely to be preval ent in

this family?

1) Cowden’s syndrome

2) Familial adenomatous polyposis

3) Hereditary non-polyposis colorectal cancer

4) Juvenile polyposis

5) Peutz-Jeghers syndrome

A

Explanation

Hereditary non-polyposis colorectal cancer

Hereditary non-polyposis colorectal cancer (HNPCC) is an inherited condition – there are

various genes that may be involved but they are all characteristically involved in DNA mismatch

repair. Colonic tumours associated with HNPCC tend to have certain distinguishing pathological

features. They occur on average 20 years before the peak incidence of sporadic tumours and are

frequently synchronous and metachronous with a predilection for the proximal colon. They tend

to be mucinous, poorly differentiated and ‘signet -ring’ in appearance. Ovarian cancer is

associated with HNPCC along with cancers of the endometrium and stomach.

Cowden’s syndrome

This is an autosomal-dominant condition associated with multiple hamartomas (especially in the

breast and thyroid tissue, skin and mucosal membranes), and an incr eased incidence of certain

cancers (breast and endometrial in women, as well as thyroid and renal cancers).

Familial adenomatous polyposis

This condition is inherited, resulting in the development of 100s of polyps in the large bowel,

with an associated increased risk of colorectal cancer. Additionally, patients are at increased risk

of other cancers including pancreatic and stomach, and have other characteristic findings such as

osteomata, sebaceous cysts and fibromas. A specific collection of signs/sympto ms in familial

adenomatous polyposis (FAP) is known as Gardner syndrome.

Juvenile polyposis

Juvenile polyposis is a genetic condition in which multiple hamartomas develop within the

gastro intestinal tract (GI) tract with an associated increased risk in ca ncer of the GI tract (from

the stomach to rectum).

Peutz-Jeghers syndrome

This autosomal-dominant condition is characterised by multiple patches of hyperpigmentation on

the lips/oral mucosa and hands and feet, and hamartomas within the GI tract. Patients a re also at

increased risk of cancer in the pancreas, liver and lung, in men also in the testicles and in women

the breast, uterus and ovaries.

242
Q

A 26-year-old woman presents

with right upper quadrant discomfort and elevated serum

transaminases. A diagnosis of

hepatitis C virus infection

is made following detection of

antihepatitis C virus antibodies.

Which one of the following represents the most likely subsequent event?

1) Complete recover y 2) Development of chronic hepatitis C

3) End stage liver disease

4) Transmission to her monogamous partner

5) Vertical transmission to potential children

A

Explanation

Development of chronic hepatitis C

In total, 50–80% of those with acute hepatitis C will develop a chronic infection.

Complete recovery

In total, 25–50% of patients with acute hepatitis C will completely clear the virus and therefore

make a complete recovery.

End stage liver disease

In total, 20% of those individuals affected by chronic hepatitis C will develop progressive

cirrhosis and resultant end-stage liver failure. Additionally, hepatocellular carcinoma occurs in

1–4% of those with associated cirrhosis per year.

Transmission to her monogamous partner

Transmission to monogamous partners and vertical transmission is uncommon and accounts for

the minority of new cases of hepatitis C virus infection. Hepatitis C virus is transmitted by

parenteral or per mucosal exposure to infected blood or body fluids. Many patients have a history

of intravenous drug abuse or transmission of blood products before the implementation of

antihepatitis C virus screening of blood donors in 1992.

Vertical transmission to potential children

Transmission to monogamous partners and vertical transmission is uncommon and accounts for

the minority of new cases of hepatitis C virus infection. Hepatitis C virus is transmitted by

parenteral or per mucosal exposure to infected blood or body fluids. Many patients have a history

of intravenous drug abuse or transmission of blood products before the implementation of

antihepatitis C virus screening of blood donors in 1992.

Hepatitis C virus has risk factors for infection in the UK patient population including intravenous

drug use with sharing of paraphernalia, blood product transfusion (before screening for HCV in

1992), tattoos or piercings, while sexual and vertical transmission are also routes they are less

common. Acute hepatitis C is defined as being the first 6 months of infection after the incubation

period, while chronic hepatitis C is an infection longer than this period. Most patients are

asymptomatic during this period – however, testing for hepatitis C should be considered in

patients with clinical features of hepatitis (RUQ pain, jaundice, flu -like symptoms, diarrhoea and

vomiting), risk factors, or deranged liver function tests. The presence of HCV antibodies show

whether a patient has ever been infected with HCV, while the presence of HCV RNA shows an

active infection.

243
Q

You are in the general surgical clinic seeing a patient with histology results after biopsy of a

lesion at the anal verge. The tissue sample is in keeping with a diagnosis of anal cancer.

Which one of the following statements is correct regardi ng anal cancer?

1) First-line treatment is by radical surgery

2) It accounts for 50% of anorectal malignancies

3) Most anal malignancies are squamous cell carcinomas

4) Perianal papilloma is not related to the development of anal cancer 5) Tumours above the dentate line spread to the para-aortic lymph nodes

A

Explanation

Most anal malignancies are squamous cell carcinomas

Of anal cancers, 80% of them are squamous-cell carcinomas. Rectal bleeding is the most

common initial symptom (45% of cases); 30% of patients complain of pain or sensation of a

mass.

First-line treatment is by radical surgery

Anal carcinoma is relatively radiosensitive. Presently, combination radiotherapy and

chemotherapy [fluorouracil (5-FU) ± mitomycin or 5-FU + cisplatin] is used as initial standard

first-line treatment. Abdominoperineal (AP) resection is reserved for recurrent or resistant

tumours or in combination with neoadjuvant chemo -radiotherapy.

It accounts for 50% of anorectal malignancies

Anal cancers account for approximately 10% of anorectal ma lignancies.

Perianal papilloma is not related to the development of anal cancer

Perianal papillomas have been shown to predispose to the development of anal cancer. The

human papillomavirus (usually type 16) may cause papillomas/intraepithelial neoplasia, which

with time progresses to anal carcinoma. This virus is found in approximately 70% of patients.

Other risk factors for anal carcinoma include a history of cervical or vaginal cancer, history of

sexually transmitted disease and immunosuppression following solid organ transplantation.

Tumours above the dentate line spread to the para-aortic lymph nodes

The nodal metastatic locations depend on the lymphatic drainage, which is dependent on where

the anal cancer originates: above the dentate line, flows to t he perirectal and internal iliac nodes;

below the dentate line, drainage is through the inguinal and femoral nodes. The most common

site of distal metastasis is the liver.

244
Q

You

are in the general surgery clinic reviewing a 27-year-old patient referred with an

abdominal

mass

and you suspect colorectal carcinoma. They are subsequently referred for urgent

colonoscopy.

Which one of the following statements is correct regarding colorectal carcinoma?

1) It is associated with a high fibre, low fat diet

2) It is associated primarily with Krukenberg’s tumours

3) It is best treated by abdominal-perineal resection if the tumour is up to 15 cm from the

anal canal 4) It has a palpable mass on digital rectal examination in over 60% of cases of r ectal

tumours

5) It is found in the rectum in 10% of cases

A

Explanation

It has a palpable mass on digital rectal examination in over 60% of cases of rectal tumours

Colorectal cancer presents as a palpable rectal mass in 60% of cases of rectal tumours. Other

common symptoms of a rectal cancer include peri-rectal (PR) bleeding, change in bowel habit,

tenesmus, or abdominal discomfort.

It is associated with a high fibre, low fat diet

The opposite is true, a high fat and low fibre diet predisposes to colorectal c arcinoma. Other risk

factors include diet high in red/processed meat, obesity, increasing age, alcohol and tobacco use.

Additionally other conditions such as inflammatory bowel disease, and inherited conditions such

as familial adenomatous polyposis syndro me and hereditary non-polyposis colorectal cancer

(HNPCC) syndrome are significant risk factors.

It is associated primarily with Krukenberg’s tumours

Krukenberg’s tumours are malignant metastases within the ovaries – their primary site is

normally gastrointestinal (GI), primarily it is associated with stomach carcinomas spreading to

ovaries.

It is best treated by abdominal-perineal resection if the tumour is up to 15 cm from the anal canal

Abdominal–perineal resections are used for low rectal tumours, where tumours are within 8 cm

from the anal canal. Left -sided tumours above this point may undergo left hemi-colectomy,

while transverse and right -sided hemi-colectomies may be used in these respective areas. Right -

sided hemi-colectomy is also the operation of choice for caecal tumours. Radiotherapy is

frequently used in rectal cancers too – either as neoadjuvant or adjuvant therapy, but not

commonly in other cancers of the colon. Chemotherapy may be used in all sites of colorectal

cancer, depending on type/stage.

It is found in the rectum in 10% of cases

Approximately 45% of colorectal tumours are found in the rectum.

It is useful to be aware of the Dukes’ staging system for colorectal cancer (although this system

has largely been replaced by the TNM staging system clinically it is a frequent exam question!).

Please see the following link for a comprehensive description of colorectal cancer staging by the

American Joint Committee on Cancer (AJCC, 7th edition). Modified Astler –Coller staging

system:

Stage A: Limited to mucosa

Stage B1: Extending into the muscularis propria but not penetrating through it; nodes are

not involved.

Stage B2: Penetrating through the muscularis propria; nodes are not involved. 

Stage C1: Extending into the muscularis propria but not penet rating through it. Nodes are

involved.

Stage C2: Penetrating through the muscularis propria. Nodes are involved.

Stage D: Distant metastatic spread.

245
Q

You are reviewing the results of a colonoscopy with a patient in the general clinic. They are

concerned about their future risk of significant disease.

Which one of these conditions puts the patient at increased risk of colorectal cancer?

1) Haemorrhoids

2) Hamartoma

3) Angiodysplasia

4) Solitary polyp

5) Ulcerative colitis

A

Explanation

Ulcerative colitis

There is malignant potential in ulcerative colitis – after 10 years the risk is around 2%, then 8%

at 20 years and 18% at 30 years. Those individuals with ulcerative colitis for 10 years or longer

are offered surveillance at between 1–5-yearly intervals depending on whether they are deemed

low, intermediate or high risk of developing colorectal cancer.

Haemorrhoids

Haemorrhoids are swollen or inflamed vascular structures within the anal canal. They can

present with pain, itching and per rectal (PR) bleeding – the major risk factor for these is

constipation or straining at stool, as well as being common in pregnancy. They do not increase

the risk of colorectal cancer.

Hamartoma

Hamartomas are benign focal growths within an organ comprised of cells/tissue that normal ly

arise from that organ but have developed into an abnormally structured growth. They resemble

neoplasms but are do not have malignant potential. There are, however, syndromes in which

multiple hamartomas are present that can result in increased risk of malignancy. Cowden’s

syndrome is characterized by multiple hamartomas throughout the gastrointestinal (GI) tract,

skin, thyroid and other organs, and while the hamartomas themselves are benign, the patient has

an increased risk of malignancy due to mutations in the PTEN tumour suppressor gene.

Angiodysplasia

Angiodysplasia are arteriovenous malformations within the gastrointestinal tract which may be

single or multiple. They are usually asymptomatic but can present with acute episodes of

painless bleeding or malena (depending on their position within the GI tract). Chronic bleeding

may lead to anaemia. Solitary polyp

Solitary juvenile polyps are benign hamartomatous polyps, most commonly found in the rectum

and occurring before the age of 20 (although they can occur at any age). The term juvenile

describes the polyp, not the age of the patient. They may present with rectal bleeding, but do not

predispose to cancer. This is in contrast to juvenile polyposis syndrome where multiple mostly

hamartomatous polyps are found either within the rectum and colon, or throughout the GI tract –

with an increased risk of adenocarcinoma.

246
Q

A 74-year-old

Asian man

presents with a 6-month history of weight loss and epigastric pain. An

oesophago-gastro-duodenoscopy (OGD) shows an ulceration lesion in the fundus of the stomach.

Which one of the following is true regarding the incidence of gastric carcinoma?

1) It is decreased in patients with chronic atrophic gastritis

2) It is higher among women

3) It is highest in India

4) It is reduced in patients with blood group A

5) The incidence is declining world-wide

A

Explanation The incidence is declining world-wide

This statement is correct, the incidence of gastric cancer is declining world-wide. The incidence

of gastric cancer in the UK is between 6500–7000 cases a year and is also declining, the highest

risk age group being 85–90-year-olds.

It is decreased in patients with chronic atrophic gastritis

Chronic atrophic gastritis is a condition where chronic inflammation of the gastric mucosa leads

to a gastric glandular atrophy and loss of parietal cells (therefore a decrease in production of

peptic acid and intrinsic factor), replaced by intestinal metaplasia and fibrous tissue deposition.

This condition can present with chronic stomach pain, nausea, vomiting and weight loss, and

with a megaloblastic anaemia due to lack of vitamin B 12 absorption. The commonest predisposing

conditions are H. pylori infection and autoimmune gastritis. Chronic atrophic gastritis and

resultant intestinal metaplasia both lead to increased gastric cancer risk.

It is higher among women

The male to female ratio is approximately 2:1.

It is highest in India

The highest rates of gastric cancer occur in Japan, Mongolia and Korea, while most cases world -

wide are in developing countries.

It is reduced in patients with blood group A

The incidence of stomach cancer has been shown to be 20% higher in those with blood group A

compared to those with blood group O.

247
Q

A

6-month

-old male infant presents with fresh

rectal bleeding (resembling

redcurrant jelly

),

vomiting, abdominal pain and a

palpable mass

. The parents want to know whether or not their

son will need an operation

What is the most likely cause?

1) Anal fissure

2) Hamartomatous polyp

3) Inflammatory bowel disease

4) Intussusception

5) Rectal ulceration

A

Explanation

Intussusception Intussusception is a condition in which a portion of the bowel telescopes within an adjacent

segment of bowel. Most commonly this is the ileum entering the caecum. Sometimes an

anatomical lead point is identified as causing the intussusception – this may be a Meckel’s

diverticulum, polyp or a hyper plastic Peyer’s patch (ie from a rece nt viral infection). It

classically presents with the triad of rectal bleeding, vomiting and abdominal pain. The rectal

bleeding is often described as like red currant jelly. An abdominal mass (the intussuscepted

segment) may be palpable.

Ultrasound is the investigation of choice initially, and treatment is with either barium or water -

soluble enema (which will usually both visualise and reduce the intussusception). If this fails

surgical reduction may be required.

Anal fissure

Anal fissures present with pain and fresh per rectal (PR) bleeding on passing stool, often noticed

most on wiping rather than in the stool itself. The main risk factor is constipation. The pain is felt

at the site of the fissure rather than intra-abdominally (although this may be difficult to determine

in a 6-month-old infant. This condition would not explain the vomiting, ‘redcurrant jelly’ or the

abdominal mass.

Hamartomatous polyp

Hamartomas are benign focal growths within an organ comprised of cells/tissue that normally

arise from that organ but have developed into an abnormally structured growth. They resemble

neoplasms but do not have malignant potential. Within the gastrointestinal (GI) tract they may

present with GI bleeding, but the description of ‘red currant jelly stool’ is classical for

intussusception.

Inflammatory bowel disease

Inflammatory bowel disease (IBD) could explain this patient’s symptoms (Crohn’s disease, for

example, can present with an inflammatory right iliac fossa mass), however, it would be unusual

to present in such a young child and usually presents between the ages of 15 –25. It is by no

means the most likely diagnosis offered here.

Rectal ulceration

This option is a rare, benign condition in childhood that usually presents with per rectal (PR)

bleeding and mucous, lower abdominal and perineal pain, and constipation. It would not explain

the patient’s abdominal mass or vomiting. The most likely diagnosis in this scenario is

intussusception.

Other causes of rectal bleeding in children not considered above inclu de Meckel’s diverticulum,

vascular malformations, and Henoch–Schönlein purpura.

248
Q

A 20-year-old diabetic man presents to the surgical take with anal pain, pruritus ani and

discharge. He has had repeat admissions for anal abscesses and is awaiting an examination under

anaesthesia (EUA) for anal fistula.

Of the following statements regarding anal fistula, which one is correct?

1) They always start from an abscess

2) They are always associated with Crohn’s disease

3) They are initially managed with antibiotics

4) In the case of complex fistulae the investigation of choice is a contrast CT

5) When following Goodsall’s rule, in the case of an anterior fistula, the internal opening follows a

direct path

A

Explanation

When following Goodsall’s rule, in the case of an anterior fistula, the internal opening follows a direct

path Goodsall’s rule states that an external opening lying anterior to Goodsall’s line (the transverse

anal line) is usually associated with a straight tr act, whereas an external opening lying posterior

to it may follow a curving tract.

They always start from an abscess

A fistula is an abnormal communication between two epithelialized surfaces. Up to 80% of anal

fistulas are due to an anorectal abscesses. Other causes include post-surgical complications,

trauma, penetrating injury, tuberculosis (TB) infection or Crohn’s disease.

They are always associated with Crohn’s disease

They can be caused by Crohn’s disease. Most, however, start from an abscess, while others

develop insidiously or from causes such as haemorrhoidectomy, trauma, penetrating injury, TB

infection or tumours

They are initially managed with antibiotics

Antibiotics are used in cases complicated by cellulitis or sepsis, or if the patient is

immunocompromised. The treatment of the fistula itself in simple cases is with fistulotomy,

while in other cases other techniques may be required/indicated such as the Ligation of the

Intersphincteric Fistula Tract (LIFT) procedure or Seton sutures.

In the case of complex fistulae the investigation of choice is a contrast CT

Complex fistulae are best imaged by a magnetic resonance imaging (MRI) scan.

249
Q

A 22-year-old

diabetic

man presents to the surgical take with

anal pain, pruritus ani and

discharge. He is awaiting an EUA for

anal fistula

.

When looking back through his medical history what are you most likely to see of

particular note?

1) Carcinoma

2) Ulcerative colitis

3) Recurrent abscess

4) Trauma

5) Tuberculosis

A

Explanation

Recurrent abscess A fistula is an abnormal communication between two epithelialized surfaces. Up to 80% of anal

fistulas are due to an anorectal abscesses. Other causes include post -surgical complications,

trauma, penetrating injury, tuberculosis (TB) infection or Crohn’s disease.

Carcinoma

This option is a cause, but not the most common.

Ulcerative colitis

Fistula are common in Crohn’s disease, however, because inflammation in ulcerative colitis is

confined to the mucosa and submucosa (rather than being transmural as in Crohn’s) fistula are

not classically seen in ulcerative colitis.

Trauma

This option is a cause, but not the most common.

Tuberculosis

This option is a cause, but not the most common.

250
Q

A 24-year-old

woman

presents with a recurrent cyst over her natal cleft. She has had multiple

previous incisions and drainage and has exc essive scarring as a result. She is obese and hirsute.

With reference to the description of the patient above, what is most unusual given her

diagnosis of pilonidal sinus?

1) Age of patient

2) Hirsutism

3) Female sex

4) Recurrence of condition following surgery

5) Obesity

A

Explanation

Female sex Men are affected three to four times more often than women.

Age of patient

Pilonidal sinus is a condition predominantly affecting young adults between the ages of 20 and

40 years.

Hirsutism

Risk factors include hirsutism, obesity, prolonged sitting as part of normal daily routine, family

history, male sex, and deep natal cleft.

Recurrence of condition following surgery

If an abscess forms then treatment is incision and drainage, while chronic discharging sinuses or

recurrent abscesses may require removal of the tract or laying it open and allowing it to heal by

secondary intention. Reported recurrence rates vary but may be up to 50%.

Obesity

Patients may frequently be described as hairy and may also be obese.

A pilonidal sinus is usually found in the midline in the natal cleft. The cause is unclear although

a loose hair piercing the skin is thought to be attributable. These may be asymptomatic, or

present acutely with abscess formation, or chronically with ongoing discharge.

251
Q

A patient is listed on the day case endoscopy list for surveillance colonoscopy for familial

adenomatous polyposis (FAP). Your consultant quizzes you on this disease.

How is FAP most usually diagnosed in children of affected individu als?

1) DNA screening - gene present on short arm of chromosome 15

2) Male children only of known FAP patients need will be screened

3) Ophthalmoscopy or DNA screening

4) Sigmoidoscopy and biopsy after adenomas become symptomatic

5) Such children are most likely to present with carcinoma of the colon or rectum

A

Explanation

Ophthalmoscopy or DNA screening

DNA screening is 95% predictive. In addition, it is important to note that many patients with

FAP are found to have bilateral lesions of congenital hypertrophy of the retinal pigment

epithelium (CHRPE) on ophthalmoscopy.

DNA screening - gene present on short arm of chromosome 15 The gene is present on the short arm of chromosome 5.

Male children only of known FAP patients need will be screened

FAP, due to mutation of t he APC gene, is an autosomal-dominant condition that affects men and

women equally.

Sigmoidoscopy and biopsy after adenomas become symptomatic

Diagnosis in new patients with new mutations and no family history is usually by sigmoidoscopy

and biopsy after t he adenomas become symptomatic. If 100 or more adenomas are present, FAP

is presumed.

Such children are most likely to present with carcinoma of the colon or rectum

The risk of malignancy in FAP is almost 100%. However, this risk increases with age, and

children rarely present with carcinoma. Polyps may be visible from age 7, but the risk of cancer

is 7% at 21, rising to almost 100% by age 50.

252
Q

A 79-year-old patient who tested positive on faecal occult blood test, presents for

sigmoidoscopy. A colonic polyp is noted.

Which one of the following statements concerning colonic polyp is correct?

1) Hamartomatous polyps are made up of smooth muscle cells

2) Over 25% of the population over 45 years of age may have asymptomatic adenomas

3) They usually bleed on contact

4) Their malignant potential is indirectly related to the size of the lesion

5) Pseudopolyps can be inflammatory

A

Explanation

Pseudopolyps can be inflammatory

An inflammatory polyp is an island of inflamed mucosa surrounded by ulcerated tissue, a classic

example of this is in ulcerative colitis – called pseudopolyps.

Hamartomatous polyps are made up of smooth muscle cells

Hamartomas are benign focal growths within an organ comp rised of cells/tissue that normally

arise from that organ but have developed into an abnormally structured growth – they resemble

neoplasms but do not have malignant potential. Over 25% of the population over 45 years of age may have asymptomatic adenomas

Approximately 25% of men and 15% of women over the age of 50 years may have an adenoma

identified at colonoscopy.

They usually bleed on contact

Benign adenomas do not usually bleed on contact, bleeding is more common if the polyp is

malignant.

Their malignant potential is indirectly related to the size of the lesion

The guidelines from NICE recommend ongoing screening for all patients with polyps found on

colonoscopy at 5-yearly intervals. However, if any of the polyps found are ³10mm in diameter

then this screening should be at 3- or 1-yearly intervals, depending on the number of polyps.

253
Q

A 58-year-old man presents to clinic with pain on defecation and point bleeding on toilet paper.

You examine him and find haemorrhoids.

What is the best conservative treatment you would advise for

first-degree haemorrhoids

?

1) Bedrest and analgesia

2) Elevated legs and an ice pack

3) No treatment should be given

4) Surgery

5) Reassurance and advice on eating an increased fibre diet

A

Explanation

Reassurance and advice on eating an increased fibre diet

First-degree haemorrhoids may respond to increasing fibre in the diet or to bulking agents.

Reassurance that symptoms are due only to haemorrhoids is important.

Bedrest and analgesia

Thrombosed strangulated haemorrhoids may be treated conservatively with analgesia, bedrest,

elevated legs and an icepack or by surgery. Elevated legs and an ice pack

Thrombosed strangulated haemorrhoids may be treated conservatively with analgesia, bedrest,

elevated legs and an icepack or by surgery.

No treatment should be given

Asymptomatic haemorrhoids should not be treated but reassurance and advice regarding a high -

fibre diet is important.

Surgery

Thrombosed strangulated haemorrhoids may be treated conservatively with analgesia, bedrest,

elevated legs or an icepack. Symptomatic first - and second-degree haemorrhoids that have

persisted despite medical intervention can be treated with rubber band ligation or injection

sclerotherapy, whereas third-degree haemorrhoids are ideally managed by formal

haemorrhoidectomy.

Haemorrhoids are swollen vascular cushions that affect approximately 40% of patients at some

point. The most common presenting symptoms are noticeable prolapse and fresh rectal bleeding,

they may also cause pruritus or pain. Risk factors include constipation and straining at stool,

increasing age, or increased intra-abdominal pressure (eg pregnancy, pelvic masses).

Haemorrhoids can be internal or external, external arising below the dentate line. If internal they

can then be described as first - through to fourth-degree:

1st – Haemorrhoids within lumen of the anal canal, no prolapse.

2nd – Haemorrhoids prolapse on straining but spontaneously reduce when straining is stopped.

3rd – Haemorrhoids prolapse but are fully reducible with manual pressure.

4th – Haemorrhoids are irreducibly prolapsed.

254
Q

A 74-year-old man is brought into the Accident and Emergency resuscitation unit acutely unwell

with severe abdominal pain. On examination he looks poorly, has a weak pulse and is

hypotensive and tachycardic. He has a diffusely tender abdomen with widespread guarding and

an erect chest X-ray confirms air under the diaphragm. He is taken for a laparotomy and this

shows a perforated diverticulum.

Which one of the following sites does diverticular disease most commonly affect?

1) Ascending colon

2) Caecum

3) Rectum

4) Sigmoid colon

5) Transverse colon

A

Explanation

Sigmoid colon

Almost 50% of cases affect the sigmoid alone while a further nearly 25% of cases affect the

sigmoid and descending colon. In total the sigmoid colon is involved in over 80% of cases.

Ascending colon

Right-sided diverticula are present in fewer than 20% of cases in the Western population.

However, in the Asian population, right -sided diverticula are more common, although this is

changing with the adoption of a Western diet.

Caecum Right-sided diverticula are present in fewer than 20% of cases in the Western population.

However, in the Asian population, right -sided diverticula are more common, although this is

changing with the adoption of a Western diet.

Rectum

Rectal diverticula are extremely rare, presenting in less than 1% of cases

Transverse colon

The most common site for diverticula is the sigmoid colon.

Diverticulosis descr ibes the presence of pouches of protruding mucosa through the colonic wall.

They primarily occur in the large bowel where the longitudinal outer layer of muscle is arranged

as distinct bands called ‘taenia coli’ that give rise to sites of relative weakness in-between the

bands. Presence of diverticula increases with age – it affects approximately 50% by 50 years of

age and 70% by 80 years of age. It is most common in the descending and sigmoid colon in the

Western population, but can occur throughout the co lon. The most important modifiable risk

factor appears to be lack of dietary fibre and resultant constipation.

Diverticular disease is the term used to refer to symptomatic diverticulosis – the most common

symptoms being altered bowel habit (often alternat ing between constipation and diarrhoea) and

lower abdominal pain. It is not a premalignant condition but can co -exist with colorectal cancers.

Diverticulitis is an acute inflammatory exacerbation of pre-existing diverticulosis. This usually

presents with left iliac fossa pain, fever and can also present with per rectal (PR) bleeding.

255
Q

A 51-year-old alcoholic is admitted with severe epigastric pain radiating to the back, fevers,

dehydration and jaundice. He undergoes scoring and it is deemed that he should be admitted

directly to HDU.

Which one of the following is a

poor prognostic factor

in pancreatitis according to

Ranson’s criteria?

1) Amylase 2600 IU/l

2) Arterial pa (O2 ) of 7 kPa

3) Corrected serum calcium of 2.6 mmol/l

4) Leukocyte count of 5 × 10 9 /l

5) Rise in packed-cell volume by more than 10%

A

Explanation

Arterial pa (O2 ) of 7 kPa

This is correct – arterial partial pressure of oxygen of < 8 (< 60 mmHg) is one of Ranson’s

criteria. Ranson’s criteria are used to predict severe pancreatitis and a prognosis of approximate

mortality. It was initially described for alcoholic (ie non-gallstone) pancreatitis. Several factors

are assessed at presentation and again at 48 h to give the full score. 1 point is given for each

criterion present, and this allows the following prognosis:

0–2 points = 1% mortality from this episode

3–4 points = 15% mortality

5–6 points = 40% mortality

7 or more points = 100% mortality

Amylase 2600 IU/l Amylase (normal value < 200 U/l) is a useful diagno stic investigation for pancreatitis. It is not,

however, prognostically useful.

Corrected serum calcium of 2.6 mmol/l

Corrected serum calcium of < 2.0 mmol/l scores a point in Ranson’s criteria and would therefore

indicate poor prognosis.

Leukocyte count of 5 × 10 9 /l

A white-cell count greater than 16 × 10 9 /l scores a point in Ranson’s criteria and would therefore

indicate poor prognosis.

Rise in packed-cell volume by more than 10%

Packed-cell volume indicates the volume of red cells circulating as a perce ntage of total blood

volume. It is raised most commonly in dehydration or in excess production of red cells – it is not

part of Ranson’s criteria. A fall in haematocrit at 48 h by > 10% from admission does score,

however.

256
Q

A 63-year-old woman presenting with colorectal cancer asks about risk factors for the disease.

She is due to undergo colonoscopy for change in bowel habit.

Which of the following is associated with the greatest attributable burden of disease

incolorectal cancer?

1) Familial adenomatous polyposis (FAP)

2) Hereditary nonpolyposis colorectal cancer (HNPCC)

3) Other familial colon cancer syndrome (excluding FAP/HNPCC)

4) Pre-existing inflammatory bowel disease (IBD)

5) Sporadic factor

A

Explanation

Sporadic factor

Around 65% of colorectal cancers are thought to be sporadic – ie the genetic mutations present

within these cancers are not inherited. Important risk factors in developing sporadic colorectal

cancer include age, diet rich in red meat and lo w in fibre, smoking and obesity.

Familial adenomatous polyposis (FAP)

Less than 1% of colorectal cancer is attributable to FAP.

Hereditary nonpolyposis colorectal cancer (HNPCC)

This accounts for approximately 2–4% of colorectal cancers.

Other familial colon cancer syndrome (excluding FAP/HNPCC)

Inheritance is a factor in up to one-third of colorectal cancer cases; however, only 5% of

colorectal cancers arise in the setting of an inherited colorectal cancer syndrome – this number

includes FAP and HNPCC, which are the most common. Pre-existing inflammatory bowel disease (IBD)

IBD-related colorectal cancer accounts for < 2% of all colorectal cancers.

257
Q

A 21-year-old man presents with abdominal pain and diarrhoea, with episodes up to 12 times a

day. He is diagnosed with Crohn’s disease.

Which one of the following is true of both Crohn’s disease and ulcerative colitis?

1) They are more common in urban areas than rural areas

2) Any part of alimentary canal affected from mouth to anus

3) Small, shallow ulcers

4) In 30% of cases, only the ileum is affected

5) 60% of cases are confined to rectum and sigmoid

A

Explanation

They are more common in urban areas than rural areas

Both Crohn’s disease and ulcerative colitis are more common in those living in urba n areas

compared with rural areas. The following table summarises key aspects of Crohn’s disease and

ulcerative colitis.

Crohn’s disease

Ulcerative colitis

Prevalence

145 per 100 000

240 per 100 000

Segments affected

Any part of the alimentary tract from mouth to anus, most commonly the terminal ileum. Commonly spares the rectum

Affects the rectum, extending contiguously proximally a variable amount but limited to the colon

Pattern of disease

Skip lesions

Continuous

Macroscopic

Cobblestones, aphthous and linear ulcers, ‘rose-thorn’ ulcers, abscesses, luminal

Pseudopolyps, very friable mucosa, featureless colon, findings

narrowing or stricturing

decreased haustrae, shortened colon, toxic megacolon

Microscopic findings

Transmural inflammation, granulomas

Submucosal or mucosal inflammation, crypt abscesses

Common symptoms

Abdominal pain, diarrhoea, weight loss, mouth ulcers, perianal disease

Bloody diarrhoea, weight loss, abdominal pain

Extraintestinal features

Perianal disease (fissures, fist ulae, tags, abscesses), mouth ulcers, oral aphthous ulcers, erythema nodosum, arthritis and spondyloarthropathies, uveitis, episcleritis

Primary sclerosing cholangitis, arthritis and spondyloarthropathies, uveitis, episcleritis

Any part of alimentary cana l affected from mouth to anus

Crohn’s disease can affect any part of the alimentary canal, with mouth ulcers and perianal

disease common features. It classically produces skip lesions whereby different parts of the

bowel are affected in a discontinuous manner. Ulcerative colitis affects the rectum and a variable

but continuous amount of proximal large bowel. Backwash ileitis can occur, causing

inflammation of the terminal ileum but otherwise the small bowel is not affected.

Small, shallow ulcers

Small, shallow ulcers are typical of ulcerative colitis; those seen in Crohn’s disease tend to be

very deep.

In 30% of cases, only the ileum is affected

In Crohn’s disease, only the ileum is affected in 30% of cases. Ulcerative colitis will not affect

the ileum, except in some cases in which backwash ileitis can occur (less than 10%).

60% of cases are confined to rectum and sigmoid

In ulcerative colitis, 60% of cases are confined to the rectum and sigmoid colon. Crohn’s disease

classically causes skip lesions thro ughout the entire alimentary tract from mouth to anus, but the

rectum is commonly spared.

258
Q

A 21-year-old man presents with abdominal pain, cramps and diarrhoea, and inflammatory

bowel disease is diagnosed. He has a had colonoscopy and is now having further investigations

with a barium enema.

Which one of the following findings would suggest

Crohn’s disease rather than ulcerative

colitis

(UC)?

1) Affects rectum and spreads proximally

2) Cobblestone appearance of mucosa

3) Featureless ‘lead pipe’ colon

4) Thumbprinting

5) Small ulcers

A

Any part of the alimentary tract from mouth to anus, most commonly the terminal ileum. Commonly spares the rectum

Skip lesions

Ulcerative colitis

Prevalence

Segments affected

Pattern of disease

240 per 100 000

Affects the rectum, extending contiguously proximally a variable amount but limited to the colon

Continuous

Macroscopic

Cobblestones, aphthous and linear ulcers,

Pseudopolyps, very friable findings

‘rose-thorn’ ulcers, abscesses, luminal narrowing or stricturing

mucosa, featureless colon, decreased haustrae, shortened colon, toxic megacolon

Microscopic findings

Transmural inflammation, granulomas

Submucosal or mucosal inflammation, crypt abscesses

Common symptoms

Abdominal pain, diarrhoea, weight loss, mouth ulcers, perianal disease

Bloody diarrhoea, weight loss, abdominal pain

Extraintestinal features

Perianal disease (fissures, fistulae, tags, abscesses), mouth ulcers, oral aphthous ulcers, erythema nodosum, arthritis and spondyloarthropathies, uveitis, episcleritis

Primary sclerosing cholangitis, arthritis and spondyloarthropathies, uveitis, episcleritis

Affects rectum and spreads proximally

Crohn’s disease can affect any part of the alimentary canal, with mouth ulcers and perianal

disease common features. It classically produces skip lesions whereby different parts of the

bowel are affected in a discontinuous manner. Ulcerative colitis usually affects the rectum and a

variable but continuous amount of proximal large bowel, backwash ileitis can occur causing

inflammation of the terminal ileum, but otherwise the small bowel is not affected.

Featureless ‘lead pipe’ colon

A featureless ‘lead pipe’ colon can occur in chronic ulcerative colitis in affected segments due to

the inflammation and oedema resulting in loss of haustrae.

Thumbprinting

Thumbprinting is a relatively non-specific sign suggesting colonic oedema, which ma y be

present in ulcerative colitis. It is, however, a feature of plain radiographs, not of barium enema.

Small ulcers

Ulcers in Crohn’s disease are typically deep because inflammation is transmural and the mucosa

has a cobblestoned appearance.

259
Q

A 16-year-old girl presents with weight loss, abdominal pain and diarrhoea mixed with blood

and mucus.

Which one of the following statements regarding ulcerative colitis is correct?

1) It has a prevalence of approximately six cases per 100 000 of the population

2) It has a positive family history with those with a first degree relative affected having a

15% lifetime risk of developing the disease

3) It is always confined to the rectum

4) It is investigated by ultrasound as the investigation of choice

5) It is not a pre-malignant condition

A

Explanation

It has a positive family history with those with a first degree relative affected having a 15%

lifetime risk of developing the disease

Those with a first degree relative have up to a 15% lifetime chance of developing ulcerative

colitis.

It has a prevalence of approximately six cases per 100 000 of the population

The prevalence in the UK is around 240 per 100 000 population.

It is always confined to the rectum

Ulcerative colitis always affects the rectum w ith a variable involvement proximally in a

continuous manner. It is confined to the large bowel with the exception of occasional cases of

backwash ileitis, in which the terminal ileum is inflamed. Otherwise, it does not affect the small

bowel.

It is invest igated by ultrasound as the investigation of choice

Colonoscopy with biopsy is the gold standard investigation for diagnosis and sub-classification

of inflammatory bowel disease.

It is not a pre-malignant condition

There is malignant potential in ulcerative colitis – after ten years, the risk is around 2%, then 8%

at 20 years and 18% at 30 years. Those with ulcerative colitis for ten years or longer are offered

surveillance at between 1–5-yearly intervals depending on whether they are deemed low,

intermediate or high-risk of developing colorectal cancer.

260
Q

An overweight 32-year-old woman presents with a short history of painless jaundice. There is no

previous history of illness and she has no other signs of chronic liver disease. Initial

investigations reveal:

Result

Normal

Haemoglobulin

12.7 g/dl

12–15.5 g/dl

MCV

105 fl

80–96 fl

Serum bilirubin

162 µmol/l

< 21 µmol/l

AST

175 U/l

5–45 U/l

ALP

224 U/l

25–110 U/l

g-Glutamyltransferase

200 U/l

< 65 U/l

Which one of the following is the most likely diagnosis?

1) Alcoholic liver disease

2) Autoimmune chronic hepatitis

3) Non-metastatic carcinoma of the head of the pancreas

4) Cholecystitis

5) Hepatitis A infection

A

Explanation

Alcoholic liver disease

Jaundice with an elevation of both AST and alkaline phosphatase suggests mixed hepatocellular

damage and cholestatic liver disease, typical of acute alcoholic hepatitis on a background of

chronic liver disease (and is not excluded by the lack of physical signs). The high g -

glutamyltransferase lends support to this as it is typically elevated in alcoholism (although it may

be increased in liver disease of any cause). A mild macrocytosis is typical of chronic excessive

alcohol intake and is not a feature of the other conditions. AST and alanine a minotransferase

(ALT) are enzymes released from hepatocytes when they are damaged; therefore, when

hepatocellular damage predominates, these enzymes will be raised. ALP is typically raised where

cholestasis is present.

Autoimmune chronic hepatitis Autoimmune chronic hepatitis typically causes a hepatocellular damage. You would therefore

expect AST to be raised with much lesser change in ALP.

Non-metastatic carcinoma of the head of the pancreas

In patients presenting with painless jaundice, a diagnosis of pa ncreatic cancer causing an

obstructive jaundice should be considered. The mixed picture of cholestasis and hepatocellular

damage, however, does not fit with pancreatic cancer which normally causes a cholestasis – the

ALP would be expected to be elevated with much lesser AST elevation.

Cholecystitis

Chronic cholecystitis can cause jaundice but it would be unusual for there to be no history of

acute episodes. Additionally, the cause of jaundice in these patients is normally due to

obstruction, either due to a common biliary duct stone or because of local inflammation putting

pressure on the biliary tree – an obstructive/cholestatic pattern would be more suggested by a

high ALP and normal or only mildly elevated AST.

Hepatitis A infection

In hepatitis A, a hepatocellular picture develops so AST is typically higher than alkaline

phosphatase. This infection causes hepatitis and results directly in hepatocellular damage.

261
Q

A

65-year-old man

presents with retention. On examination he is found to have a large smooth

prostate.

Which one of the following statements about prostatic pathology is true?

1) Benign prostatic hyperplasia (BPH) affects 50% of men over 50

2) The severity of symptoms is related to the size of the prostate gland

3) Finasteride, a 5-alpha-reductase inhibitor, will decrease the size of the prostate gland

within a week and therefore allow a trial without a catheter

4) Benign prostatic hyperplasia will not increase prostate-specific antigen (PSA)

5) The aim of transurethral prostatectomy is to core out the whole prostate

A

Explanation

Benign prostatic hyperplasia (BPH) affects 50% of men over 50

The proportion of men affected increases with increasing age, and the prevalence of lower

urinary tract symptoms also increases with age. However, many of the 50% will have only mild

or moderate symptoms, which are acceptable to them without treatment.

The severity of symptoms is related to the size of the prostate gland

The severity of symptoms is not related to the size o f the gland but to the encroachment on the

prostatic urethra and the resulting obstruction to urinary flow.

Finasteride, a 5-alpha-reductase inhibitor, will decrease the size of the prostate gland within a

week and therefore allow a trial without a catheter

Finasteride is an alpha-reductase inhibitor, which blocks the conversion of testosterone to

dihydrotestosterone and reduces the size of the hyperplastic glands. A 6 -month trial of treatment

is required and symptoms may be reduced or delayed or even avoid surgery. Alpha-adrenergic

blockers can be used to relax bladder neck muscle fibres and fibres in the prostate gland, and

take effect within a week.

Benign prostatic hyperplasia will not increase prostate-specific antigen (PSA)

Normal levels increase with age. Benign prostatic hyperplasia (BPH) can cause a small rise in

PSA, but the higher the rise in PSA the more specific it is for prostate cancer: > 4.1 ng/ml =

9.1%, > 10 = 55.5%.

The aim of transurethral prostatectomy is to core out the whole prostate The aim of transurethral resection of the prostate (TURP) is to debulk the prostate gland

surrounding the urethra and leave the normal peripheral tissue.

262
Q

A patient with an extensive family history of colorectal cancer is seen in the outpatient clinic.

They are concerned about passing on problems to their future children.

Which one of the following is true for a patient of familial adenomatous polyposis (FAP)?

1) If no mutation can be identified then family members should be screened for the

condition using colonoscopy or flexible sigmoidoscopy from 13–15 years of age

2) Desmoid tumours occurring as part of FAP are benign and do not cause any problems

3) Colonic cancer is the only malignancy associated with FAP

4) FAP affects 1 in 1000 people

5) The FAP adenomatous polyposis coli (APC) gene shows an autosomal-recessive form of

inheritance

A

Explanation

If no mutation can be identified then family members should be screened for the condition using

colonoscopy or flexible sigmoidoscopy from 13–15 years of age

The British Society of Gastroenterology guidelines suggest that relatives of the patient who have

a 50% risk of FAP should be offered yearly large bowel surveillance from the age of 13 –15 until

the age of 30. From then, 3–5-yearly screening until they are 60 is advised. As FAP is an

autosomal-dominant condition, the children of the patient will have a 50% risk of the condition,

so should be offered screening as above. A specific collection of signs/symptoms in FAP is

known as Gardner syndrome.

Desmoid tumours occurring as part of FAP are benign and do not cause any problems

Desmoid tumours, which are solid tumours of the connective tissues, particularly within the

abdomen, occur in up to 15% of people with FAP. Although they are not malignant, they can

cause major problems by aggressively invading surrounding tissues. Desmoid tumours account

for up to 11% of deaths in those with FAP.

Colonic cancer is the only malignancy associated with FAP

Individuals affected with FAP risk developing a duodenal malignancy, other small bowel

cancers, thyroid cancer, pancreatic cancer, adrenal cancer, and hepatoblastoma.

FAP affects 1 in 1000 people

FAP affects 1 in 10 000 people. Men and women are equally affected. The FAP adenomatous polyposis coli (APC) gene shows an autosomal-recessive form of

inheritance

FAP is autosomal-dominant. About 20–30% of cases seem to arise from new mutations.

263
Q

Inflammatory bowel diseases cause significant morbidity. Specific variants have different

optimal management plans.

Which one of the following statements is true about the differences between Crohn’s

disease and ulcerative colitis?

1) Both Crohn’s disease and ulcerative colitis start then spread from the rectal mucosa

2) Crohn’s disease has a clear demarcation between involved and uninvolved mucosa,

whereas ulcerative colitis shows skip lesions with normal mucosa in-between

3) The incidence of gallstones and kidney stones is increased in Crohn’s disease but not in

ulcerative colitis

4) Weight loss is more commonly seen in ulcerative colitis than in Crohn’s disease

5) Fistulae are often seen in both diseases

A

narrowing or stricturing

decreased haustrae, shortened colon, toxic megacolon

Microscopic findings

Transmural inflammation, granulomas

Submucosal or mucosal inflammation, crypt abscesses

Common symptoms

Abdominal pain, diarrhoea, weight loss, mouth ulcers, perianal disease

Bloody diarrhoea, weight loss, abdominal pain

Extraintestinal features

Perianal disease (fissures, fist ulae, tags, abscesses), mouth ulcers, oral aphthous ulcers, erythema nodosum, arthritis and spondyloarthropathies, uveitis, episcleritis

Primary sclerosing cholangitis, arthritis and spondyloarthropathies, uveitis, episcleritis

Both Crohn’s disease and ulcerative colitis start then spread from the rectal mucosa

Ulcerative colitis always involves the rectum with disease then spreading proximally throughout

the colon. There can be backwash ileitis where there is some involvement of the terminal ileum,

but ulcerative colitis is really a colonic disease. Ulcerative colitis remains confined to the rectum

in approximately 25% of cases. In the remainder of cases, it spreads proximally and

contiguously. Pancolitis occurs in 10% of patients. Crohn’s disease, convers ely, consists of

segmental involvement by a non-specific granulomatous inflammatory process. The most

important pathological feature is involvement of all layers of the bowel, not just the mucosa, and

the submucosa as is characteristic of ulcerative colitis.

Crohn’s disease has a clear demarcation between involved and uninvolved mucosa, whereas

ulcerative colitis shows skip lesions with normal mucosa in-between

Crohn’s disease is discontinuous, with skip areas interspersed between one or more involved

areas. Late in the disease, the mucosa develops a cobblestone appearance, which results from

deep longitudinal ulcerations interlaced with intervening normal mucosa. The three major

patterns of involvement in Crohn’s disease are:

disease in the ileum and caecum, occurring in 40% of patients

disease confined to the small intestine, occurring in 30% of patients

disease confined to the colon, occurring in 25% of patients.

Weight loss is more commonly seen in ulcerative colitis than in Crohn’s disease

Weight loss is more commonly seen with Crohn’s disease due to the loss of absorptive capacity

in the small bowel. Ulcerative colitis is more likely to present with bloody diarrhoea and

symptoms suggestive of an inflamed rectum, such as tenesmus and urgency. Fistulae are often seen in both diseases

Fistulae are not usually seen in ulcerative colitis because there is only mucosal and submucosal

involvement in the large bowel. Crohn’s disease, however, has full-thickness involvement of the

bowel wall and can therefore form a fistula with nearby structures, eg recto -vaginal, recto-

vesical.

264
Q

A patient with an acute abdomen is not responding to conservative treatment. A plain abdominal

X-ray demonstrates a

very distended large bowel.

Which one of the following is true of toxic megacolon?

1) Toxic megacolon is due to large bowel obstruction causing dilatation of the colon 2) Toxic megacolon does not occur in Crohn’s disease

3) On abdominal X-ray a transverse colonic diameter of 8cm is significant

4) Mortality rates in Toxic megacolon of any aetiology are 40%

5) First line treatment is surgical resection

A

Explanation

On abdominal X-ray a transverse colonic diameter of 8cm is significant

The transverse colon is often used as the point of reference for dilatation, its normal diameter =

5.5–6.5 cm. Loss of colonic haustra is also seen, and possible thumbprinting – a coarse, irregular,

mucosal pattern of the large bowel.

Toxic megacolon is due to large bowel obstruction causing dilat ation of the colon

The hallmarks of toxic megacolon, a potentially lethal condition, are non-obstructive colonic

dilatation larger than 6 cm and signs of systemic toxicity.

Toxic megacolon does not occur in Crohn’s disease

Toxic megacolon may complicate any number of colitides, including inflammatory bowel

disease – which is the most common aetiology (around 50%). Other causes include ischaemic,

infectious (Clostridium difficile), radiation, and pseudomembranous colitis. It is more common in

ulcerative colitis but can also occur in Crohn’s disease.

Mortality rates in Toxic megacolon of any aetiology are 40%

Mortality rates have been falling due to better care and earlier surgical involvement, and are now

quoted as being 5–20%. Once perforation has occurred mortality rates are thought to increase

five-fold.

First line treatment is surgical resection

Initial management is medical. During the initial resuscitation, fluid and electrolyte replacement,

and transfusion should be aggressive. Broad-spectrum intravenous antibiotics should be initiated.

All medications that may affect colonic motility must be stopped. These include narcotics,

antidiarrhoeals, and anticholinergic agents. The patient should be put on bowel rest, and a

nasogastric tube placed to assist with gastrointestinal (GI) decompression. If the patient has

underlying inflammatory bowel disease then this should be treated too. Should medical

management fail then surgical resection will be required.

265
Q

You are looking after a young man on the Intensive Care Unit (ITU) with severe

pancreatitis. Initially when he was admitted he was fairly well, but continued to deteriorate after

48 hrs into his admission.

Which one of the following are correct regarding pancreatitis?

1) Feeding via the total parentral route is preferred as it prevents over stimulation of the pancreas

2) Serum amylase is a reliable marker of severity and progression of pancreatitis

3) The APACHE II scale is the most commonly used prognostic scale in acute pancreatitis

4) The BALTHAZAR scoring system assesses severity of pancreatis using serial ultrasound scans
5) If infected pancreatic necrosis develops the mortality rate is approximately 25%

A

Explanation

If infected pancreatic necrosis develops the mortality rate is approximately 25%

This is correct, this rate compares with that of mild acute pancreatitis, which has a mortality of

around 1%, and sterile pancreatic necrosis at 10%. About 20 –30% of patients with acute

pancreatitis develop complications of necrosis or organ failure. Due to the wide spectrum of the

disease, several systems have been developed in an attempt to provide reliable prognostic

classification for patients. The most frequently used of these is the modified Glasgow score.

Modified Glasgow score criteria:

 pa (O2 ) < 7.9 kPa

age > 55 years

neutrophils (white blood cell (WBC) > 15)

calcium < 2 mmol/l

urea > 16 mmol/l

LDH > 600 IU/l

albumin < 32 g/L (serum)

blood glucose > 10 mmol/l

If three or more of these criteria are present in the first 48 h of admission then severe pancreatitis

is likely. Various scores can also be used to estimate mortality:

score 0 to 2: 2% mortality

score 3 to 4: 15% mortality

score 5–6: 40% mortality

score 7 to 8: 100% mortality.

Feeding via the total parentral route is preferred as it prevents over stimulation of the pancreas

Opinion has recently changed regarding feeding in pancreatitis. It is now best practice to feed by

the enteral route via a feeding tube placed past the pancreatic duct into the 3rd part of the

duodenum. For milder cases, patients can have oral intake as this is tolerated.

Serum amylase is a reliable marker of severity and progression of pancreatitis

Although amylase and lipase levels are useful in the diagnosis of pancreatitis they have no role i n

assessing severity – for example, neither is used in the Ranson scoring system used to

prognosticate acute pancreatitis. Lipase rises in 4–6 h and normalises in 7–14 days. Amylase

may be normal in 10% of cases. The APACHE II scale is the most commonly used prognostic scale in acute pancreatitis

The APACHE II is a scoring system designed to assess disease severity patients admitted to the

ITU. It has been well studied in acute pancreatitis, but by design is an admission score designed

to assess disease severity in all adult patients admitted to general ITU. It uses the patients age,

and various physiology and biochemical/laboratory values within the first 24 h of admission. A

higher score corresponds to a higher severity of disease and a higher risk of deat h from this

admission. The most frequently used prognostic classification for patients with acute pancreatitis

is the modified Glasgow score.

The BALTHAZAR scoring system assesses severity of pancreatis using serial ultrasound scans

The BALTHAZAR score is a computed tomography (CT) scan-based scoring system that is used

to grade the severity of pancreatitis.

266
Q

A 55-year-old woman presents with jaundice due to

pancreatic cancer

. Her pain is minimal and

attended as her husband noticed she had gone yellow. She has lost a significant amount of weight

recently.

Which one of the following statements is true regarding her jaundice?

1) It is due to excess bile production

2) It is due to haemolytic anaemia

3) It is part of a paraneoplastic syndrome

4) It is post-hepatic jaundice

5) It is secondary to tumour lysis syndrome

A

Explanation

It is post-hepatic jaundice This is correct – neoplasms classically within the head of the pancreas can compress or invade

and obstruct the common bile duct. This leads to an obstructive or cholestatic jaundice. Bilirubin

is principally derived from the degradation of haemoglobin within the spleen and is carried to the

liver bound to albumin, as in its unconjugated form it is not water soluble. Within the liver,

bilirubin is conjugated by glucuronyltransferase, making it water soluble, and is secreted into the

duodenum. At the distal ileum the bilirubin is either converted to urobilinogen and excreted in

the faeces as stercobilinogen, or is re-absorbed and renally excreted. Jaundice becomes clinically

apparent at serum levels >35–40 µmol/l.

Jaundice may be classified as pre-hepatic, hepatic, or post -hepatic, depending on the site of the

obstruction or metabolic abnormality.

Pre-hepatic (haemolytic) jaundice: caused by disorders resulting in increased destruction of red

cells, eg autoimmune haemolytic anaemia or sickle cell disease which overwhelms the livers

ability to process the excess bilirubin. The excess bilirubin is unconjugated so no bilirubin is

detected in the urine. Hepatic (parenchymal) jaundice: caused by any condition affecting

hepatocyte function, eg acute hepatitis infection, alcoholic hepatitis, drug-induced hepatitis, or

cirrhosis of any cause.

Post-hepatic (cholestatic) jaundice: caused by physical obstruction of the biliary system eg

carcinoma of the head of pancreas or gallstones. Typically there is dark urine (the bilirubin is

water soluble, so is excreted in the urine) and pale stools (due to steatorrhoea from lack normal

biliary function in aiding fat absorption).

It is due to excess bile production

Jaundice is a result of excess bilirubin, while bilirubin is excreted in the bile excess bile

production would not be expected to result in jaundice.

It is due to haemolytic anaemia

Haemolytic anaemia is an example of pre-hepatic jaundice – it is not the correct answer in this

instance.

It is part of a paraneoplastic syndrome

Paraneoplastic syndromes are effects of malignancy not caused by the local effect of the cancer

itself – they are frequently hormo nal or immunological. Examples include Cushing syndrome in

small-cell lung cancer caused by ectopic adrenocorticotropic hormone (ACTH) production, or

Lambert–Eaton myasthenia syndrome caused by antibodies produced against small-cell lung

cancer that are active against pre-synaptic voltage gated calcium channels.

Paraneoplastic syndromes associated with pancreatic cancer include Cushing syndrome,

carcinoid syndrome and dermatomyositis.

It is secondary to tumour lysis syndrome Tumour lysis syndrome is a potential treatment complication of most commonly, haematological

cancers. Rapid lysis of leukaemia or lymphoma cells results in metabolic abnormalities including

hyperkalaemia, hyperphosphataemia, hypocalcaemia, lactic acidosis and hyperuricaemia. It is

potentially life-threatening – however, it does not usually result in jaundice.

267
Q

A 52-year-old woman undergoes an investigation for jaundice. She first noticed

this symptom 2

months ago, but for

4 months previously

she had been experiencing generalised

pruritus. The

results of liver function tests are as follows:

Result

Normal

Serum bilirubin

325 mmol/l

0–22 µmol/l

Aspartate aminotransaminase

55 U/l

15–42 U/ mmol/l

Alkaline phosphatase

436 U/l

80–150 U/l γ-Glutamyltransferase

82 U/l

11–51 U/l

Albumin

36 g/l

30–50 g/l

Total protein

82 g/l

60–80 g/l

Which one of the following is the most likely diagnosis?

1) Alcoholic cirrhosis

2) Carcinoma of the head of the pancreas

3) Cholangiocarcinoma

4) Primary biliary cirrhosis

5) Primary sclerosing cholangitis

A

Explanation

Primary biliary cirrhosis

Primary biliary cirrhosis frequently presents first with progressive and insidious onset of pruritus

and general malaise/lethargy, jaundice occurring later in the disease process. It is more common

in women than men (ratio 9:1), classically presenting between 40 –60 years of age. It is an

autoimmune condition associated with antimitochondrial antibodies, and the presence of

excessive antibodies is suggested by the slight ele vation in total protein with low–normal

albumin (this suggests a high globulin concentration). The disease process results in progressive

destruction of the intrahepatic bile ducts, and therefore an obstructive/cholestatic jaundice

supported by the raised alkaline phosphatase (ALP), but relatively normal aspartate transaminase

(AST).

Alcoholic cirrhosis

Alcoholic cirrhosis is a common cause of the above picture but is unlikely (although not

excluded) by the absence of history of alcohol excess, and the only slightly elevated g-

glutamyltransferase (which is sensitive, although non-specific, for alcohol excess).

Carcinoma of the head of the pancreas

Carcinoma of the head of the pancreas frequently presents in this manner (although weight loss

is often present also), but the slight elevation in total protein with low –normal albumin suggests

a high globulin concentration, which suggests autoimmune liver disease.

Cholangiocarcinoma Cholangiocarcinoma is a rare tumour, although could present like this. As with carc inoma of the

head of the pancreas, weight loss is often present also, and the slight elevation in total protein

with low–normal albumin suggests a high globulin concentration, which suggests autoimmune

liver disease.

Primary sclerosing cholangitis

Primary sclerosing cholangitis is a possibility but is commoner in men than women (3:1), and in

75% of cases are associated with inflammatory bowel disease. In this case primary biliary

cirrhosis is more likely.

268
Q

A 20-year-old man presents with

mild jaundice

following a flu-like illness. Following review by

a gastroenterologist, he has been told that a diagnosis of Gilbert syndrome is probable.

Which laboratory test is most likely to confirm this diagnosis?

1) Absence of bilirubin in the urine

2) Decreased serum haptoglobin concentration

3) Elevated serum aspartate aminotransferase (transaminase, AST) activity

4) Increased reticulocyte count

5) Increased urinary urobilinogen excretion

A

Gilbert syndrome is an inherited, autosomal-recessive condition resulting in abnormal liver

metabolism of bilirubin. It is relatively common affecting around 5% of the population. Gilbert

syndrome is normally asymptomatic, but mild jaundice may develop whe n there is physiological

stress (eg infection or fasting). In Gilbert syndrome, the excess bilirubin is unconjugated, and

does not appear in the urine. You would also not expect to find bilirubin the urine in other causes

of pre-hepatic jaundice, such as haemolytic disease.

Decreased serum haptoglobin concentration

Decreased serum haptoglobin concentration is a sign of haemolytic anaemia. Haptoglobin will

bind free haemoglobin released by lysed red cells and this complex is then removed by the

spleen.

Elevated serum aspartate aminotransferase (transaminase, AST) activity

Elevated AST is a non-specific sign of hepatocellular damage – the excess activity being

detected in the blood a result of the release of these enzymes from their normal intracellular site

within hepatocytes. This would be raised, for example, in alcoholic hepatitis, viral hepatitis, or

drug-induced liver damage. Haemolysis may also cause a slight increase in serum

aminotransferase (transaminase) activity as red blood cells also have a small amount of AST

within them. AST is also found in skeletal muscle among other cell types.

Increased reticulocyte count

This is a sign of excess haemolysis – the rapid destruction of red blood cells resulting in rapid

production of red blood cells and therefore an increased number of reticulates in the peripheral

blood count.

Increased urinary urobilinogen excretion

However, in haemolytic jaundice, urinary urobilinogen is increased (increased production of

bilirubin, and hence of urobilinogen), the reticulocyte count may be elevated and serum

haptoglobin concentration decreased. Bilirubin is principally derived from the degradation of

haemoglobin within the spleen and is carried to the liver bound to albumin, as in its

unconjugated form it is not water soluble. Within the liver bilirubin is conjugated by

glucuronyltransferase, making it water soluble, and is secreted into the duodenum. At the distal

ileum the bilirubin is either converted to urobilinogen and excreted in the faeces as

stercobilinogen, or is re-absorbed and renally excreted. Jaundice becomes clinically apparent at

serum levels >35–40 µmol/l. Jaundice may be classified as pre-hepatic, hepatic, or post -hepatic,

depending on the site of the obstruction or metabolic abnormality.

Pre-hepatic (haemolytic) jaundice: caused by disorders resulting in increased destruction of red

cells eg autoimmune haemolytic anaemia or sickle cell disease which overwhelms the livers

ability to process the excess bilirubin. The excess bilirubin is unconjugated so no bilirub in is

detected in the urine.

Hepatic (parenchymal) jaundice: caused by any condition affecting hepatocyte function eg acute

hepatitis infection, alcoholic hepatitis, drug-induced hepatitis, or cirrhosis of any cause. 

Post-hepatic (cholestatic) jaundice: caused by physical obstruction of the biliary system eg

carcinoma of the head of pancreas or gallstones. Typically there is dark urine (the bilirubin is

water soluble, so is excreted in the urine) and pale stools (due to steatorrhoea from lack normal

biliary function in aiding fat absorption).

269
Q

You are asked to see an

82-year-old

woman

under the medical team

with a lower respiratory

tract illness and

non-tender

abdominal distension. There are

dilated loops

on her abdominal film.

Blood tests reveal:

Result

Normal

Sodium

138 mmol/l

135–145 mmol/l

Potassium

3.1 mmol/l

3.5–5 mmol/l

Urea

10 mmol/l

2.5–6.5 mmol/l

Creatinine

118 µmol/l

50–120 µmol/l

White cell count

17/mm

3

<5/mm

3

C-reactive protein

11.5 mg/dl

<3 mg/dl

What is the most likely diagnosis?

1) Colorectal cancer

2) Constipation

3) Diverticulitis

4) Pseudo-obstruction

5) Small bowel obstruction

A

Explanation

Pseudo-obstruction

Pseudo-obstruction is an example of non-mechanical bowel obstruction. It can be acute or

chronic. Acute is also known as Ogilvie syndrome. In 80% of cases, colonic pseudo -obstruction

is associated with other clinical conditions. The most common are cardio -respiratory illness, post-trauma or pelvic surgery. It usually presents as this clinical vignette with abdominal

distension in the context of another acute condition, intermittent constipation and nausea or

vomiting are frequent symptoms, colicky abdominal pain may be present. Imaging will reveal

dilated bowel loops. Treatment should be based on correcting the associated cause such as the

respiratory illness or correcting electrolyte imbalances – usually potassium, magnesium or

phosphate. This may resolve the ‘obstruction’. Otherwise consider endoscopic decompression.

Neostigmine has also been used in some chronic cases.

Colorectal cancer

Colorectal cancer, particularly left -sided lesions, can cause and therefore present with

obstruction, which would give abdominal distension and dilated bowel loops. However,

mechanical obstruction is frequently painful. Usually colorectal cancer pres ents insidiously with

a change in bowel habit or PR bleeding, or with an otherwise unexplained microcytic anaemia.

Constipation

Constipation is a frequent complication for patients in hospital, both because of medication used

(for example opiates) and because many patients are relatively immobile. In patients with

constipation abdominal distension can occur. Frequently patients will have left iliac fossa pain.

Dilated loops would not be typical and faecal loading may be seen in the colon.

Diverticulitis

Diverticulitis refers to inflammation of diverticulae, which are most frequently found in the

sigmoid colon. It normally presents with left iliac fossa pain and fever, possibly with PR

bleeding. Inflammatory markers would be elevated, although in this instanc e these are more

likely to be explained by the already diagnosed lower respiratory tract infection.

Small bowel obstruction

Small bowel obstruction usually presents with vomiting, absolute constipation, and painful

abdominal distension. The commonest causes are intra-abdominal adhesions and hernias, other

causes include foreign bodies, gallstone ileus, volvulus, neoplasms and strictures (eg from

previous ischaemia or from Crohn’s disease). In small bowel obstruction you will usually see

dilated small bowel loops proximal to the point of obstruction.

270
Q

A 62-year-old woman undergoes a

gastric bypass

. She attends the surgical assessment unit

feeling unwell, is seen by the consultant and told she has

dumping syndrome

. She is unsure what

this means and you have been asked to explain it to her.

What are the likely symptoms that she has been suffering?

1) Collapse and chest pain

2) Left upper quadrant pain, diarrhoea, chest pain

3) Pain before eating, dizziness and nausea

4) Postprandial pain, vomiting and dizziness

5) Shortness of breath and chest pain

A

Explanation

Postprandial pain, vomiting and dizziness

Dumping syndrome following gastrectomy is due to an absent gastric reservoir, abnormal

postoperative gastric motor function and hormonal secretion alteration. After eating, symptoms

are related to the rapid transit of hyperosmolar contents into the small bowel. It can be split into

early dumping syndrome, occurring within 10–30 min after a meal, and late dumping syndrome,

which occurs after 2–3 h. Early dumping syndrome results from rapid osmotic fluid shifts from

the blood into the intestine resulting in decreased total blood volume and intestinal distension.

Late dumping syndrome results from rapid absorption of sugar from the intestine followed by a

rapid increase in insulin levels and a subsequent hypoglycaemic episode. Early dumping

syndrome symptoms include abdominal pain, vomiting and nausea, diarrhoea, sweating and

flushing, and dizziness. Late dumping syndrome symptoms include sweating, weakness, flushing and dizziness. Treatment is largely through managing dietary intake with several small meals a

day (instead of large meals) and avoiding simple sugars. Additionally, liquids should be drunk in

between meals not alongside meals. A high fibre diet can also help. Octreotide and proton pump

inhibitors (PPIs) can also help to slow gastric emptying if conservative management fails.

Collapse and chest pain

This is not correct.

Left upper quadrant pain, diarrhoea, chest pain

This is not correct.

Pain before eating, dizziness and nausea

This is not correct, in particular symptoms occur after eating.

Shortness of breath and chest pain

This is not correct.

271
Q

A patient is undergoing a right hemicolectomy for a caecal tumour. The ileocaecal valve and part

of the terminal ileum are resected before the anastomosis between ileum and transverse colon is

made.

Resection of the

terminal ileum

is most commonly associated with the malabsorption of

which one of the following components?

1) Bile salts

2) Calcium

3) Cholesterol

4) Folic acid

5) Sodium

A

Explanation

Bile salts

Bile salts are primarily re-absorbed in the terminal ileum – loss of bile salts in faeces has several

knock-on effects. Bile salts released in the bile are normally responsible for solubilising dietary

fat – so excess loss of bile salts results in fat malabsorption. One important effect of this is

malabsorption of the fat soluble vitamins A, D, E and K. Additionally the terminal ileum is the

primary site for absorption of vitamin B 12 – loss of which results in a macrocytic anaemia.

Calcium

Calcium and folic acid are mainly absorbed in the jejunum.

Cholesterol Dietary fat, including cholesterol, is primarily absorbed in the jejunum. However, excess loss of

bile salts, which are absorbed in the ileum, can lead to fat malabsorption in the jejunum.

Folic acid

Calcium and folic acid are mainly absorbed in the jejunum.

Sodium

Sodium is re-absorbed throughout the small bowel.

272
Q

A 35-year-old man patient is admitted to the surgical assessment unit with severe

colicky right

upper quadrant pain

, he is not febrile and has no abdominal tenderness. His past medical history

consists of

Crohn’s disease

, previous laparotomy for

small bowel resection

and asthma.

What is the most likely risk factor responsible for his symptoms?

1) Age

2) Diet

3) Lack of bile salts

4) Increased haemosiderin

5) Sex

A

Explanation

Lack of bile salts

This patient has gallstones because of a lack of bile salts which are usually involved in fat

absorption. He has Crohn’s disease, which resulted in a small bowel resection. This is most

commonly in the terminal ileum where bile salts are most predominantly absorbed. Bile salts aid

with the absorption of fat by forming micelles. Without the presence of bile salts in bile,

cholesterol will more readily sediment out to form gallstones. Other risk factors include obesity,

female sex, increasing age, diabetes me llitus, and premenopausal status or use of HRT/oral

contraceptives.

Age

Increasing age is a risk factor for developing gallstones, classically they become more likely

especially above the age of 40.

Diet Dietary factors are not a major risk factor for deve loping gallstones.

Increased haemosiderin

Haemosiderin is a complex that stores ferritin; deposits are most commonly associated with iron

overload conditions, such as hereditary haemochromatosis. They are not a risk factor for

gallstones.

Sex

Gallstones are more common in women than in men.

273
Q

A 61-year-old woman presents with persistent vomiting of largely undigested food. She has lost

a significant amount of weight,

in the A&E department she is now in extremis.

Prolonged

vomiting in

gastric

outflow obstruction is associated with which of the

following?

1) Decreased antidiuretic hormone (ADH) secretion

2) Hypochloraemic alkalosis

3) Metabolic acidosis

4) Respiratory alkalosis

5) Hyperkalaemia

A

Explanation

Hypochloraemic alkalosis

Prolonged vomiting causes a hypochloraemic metabolic alkalosis. The reason for this is that

H + and Cl – , from gastric secretions within the stomach, are lost and in addition the alkaline

pancreatic and duodenal secretions are retained. The loss of H + obviously directly leads to

alkalosis, however the loss of Cl – also results in retention of the other major anion in the body,

HCO 3 – (bicarbonate). All together this results in a hypochloraemic metabolic alkalosis. The loss

of fluid volume causes a physiological response that includes sympathetic stimulation, increase

in ADH secretion and activation of the renin–angiotensin–aldosterone system. In small bowel

obstruction, there is also loss of alkaline intestinal secretions, which prevents the development of

metabolic alkalosis.

Decreased antidiuretic hormone (ADH) secretion

Loss of fluid and inability to take in oral fluids will result in dehydration and loss of

intravascular fluid volume and therefore increased ADH secretion as the body tries to maintain

its intravascular fluid volume.

Metabolic acidosis The loss of H+ and Cl – from gastric secretions results in hypochloraemic metabolic alkalosis.

Respiratory alkalosis

Respiratory alkalosis results from hyperventilation and therefore excess removal of carbon

dioxide from the body, while the patient will be alkalotic this is not due to a respiratory cause.

Hyperkalaemia

The patient will most likely become hypokalaemic. A relatively small amount of potassium will

be directly lost in the vomit. However, more importantly, the pat ient will become alkalotic

(metabolic alkalosis) – alkalosis results in a transcellular exchange of potassium going into the

cells and hydrogen ions leaving the intracellular compartment, the increased reabsorption of

H + in the kidneys also results in increased exchange of potassium into the urine. Finally the

patient’s hypovolaemia and resultant increase in aldosterone production also results in loss of

potassium at the kidneys.

274
Q

A 22-year-old man presents with a long history of intermittent abdominal discomfort and

diarrhoea. On examination, he has a body mass index of 19 kg/m 2 and is clinically anaemic.

Coeliac disease is suspected.

Which one of the following investigations will

most reliably

diagnose this condition?

1) Detection of antigliadin antibodies in serum

2) Detection of endomysial antibodies (EMA) in serum

3) Detection of tissue transglutaminase (TTG) antibodies in serum

4) Microscopic examination of a small bowel biopsy specimen

5) Xylose absorption test

A

Explanation

Microscopic examination of a small bowel biopsy specimen

The detection of the typical appearance of subtotal villous atrophy on a small bowel biopsy is

regarded as the ‘gold standard’ for the diagnosis of coeliac disease. Other changes include

increased numbers lymphocytes and crypt hyperplasia.

Detection of antigliadin antibodies in serum

Sensitivity and specificity among studies for this test is highly variable but generally less than

endomysial antibodies (EMA) and tissue transglutaminase (TTG) antibodies.

Detection of endomysial antibodies (EMA) in serum

Sensitivity and specificity are >95% making this a very good screening test, however, biopsy

remains the gold standard. Endomysial antibodies have been identified as binding to a spec ific

form of tissue transglutaminase within the endomysium.

Detection of tissue transglutaminase (TTG) antibodies in serum

Again sensitivity and specificity are >95% therefore this test is good for screening – but biopsy is

the gold standard.

Xylose absorption test The xylose absorption test is a non-specific test for intestinal malabsorption.

275
Q

A 21-year-old male medical student who has been feeling

non-specifically unwell

for several

days is noticed to have slightly icteric sclerae by his girlfriend and has liver function tests

performed. The results of these are normal apart from a serum bilirubin concentration of

44

μmol/L

(normal range 3–17 μmol/L). His urine

does not contain bilirubin

.

Which one of the following is the most likely diagnosis?

1) Dubin–Johnson syndrome

2) Gilbert syndrome

3) Hereditary spherocytosis

4) Infectious mononucleosis

5) Rotor syndrome

A

Explanation

Gilbert syndrome

Gilbert syndrome is a common (approximately 1:2 0) autosomal-recessive cause of mild

unconjugated hyperbilirubinaemia. This occurs due to defective conjugation within the liver.

Normally mild jaundice develops when affected patients develop concomitant infections, are

fasting or have other physiological stressors. As the hyperbilirubinaemia is unconjugated it will

not be present in the urine.

Dubin–Johnson syndrome

Dubin–Johnson syndrome is an autosomal-recessive condition causing a conjugated

hyperbilirubinaemia due to a defect in the secretion of bilir ubin from the liver. The bilirubin that

then accumulates in the plasma is conjugated, water -soluble and so is excreted in the urine. One

key difference between and Rotor syndrome is that in Rotor syndrome the liver is classically

pigmented black.

Hereditar y spherocytosis

Hereditary spherocytosis is a chronic haemolytic disorder due to a defect in the red -cell

membrane (most frequently in spectrin, a structural protein). It can present with a wide range of severity, from jaundice at birth to asymptomatic anaemia or jaundice in adults. It is pre-hepatic

jaundice and will therefore be unconjugated with no bilirubin in the urine. However, it is much

less common than Gilbert’s syndrome so less likely (approximately 1:5000 in Northern

Europeans).

Infectious mononucleosis

Infectious mononucleosis (IM) can cause hepatitis and jaundice but an elevated transaminase

activity would be expected. It normally presents with a tonsillitis, general malaise, and cervical

lymphadenopathy – the absence of these makes IM unlikely.

Rotor syndrome

Rotor syndrome is an autosomal-recessive condition, causing a conjugated hyperbilirubinaemia

due to defective transporter proteins from hepatocytes into the biliary tree. As the bilirubin that

accumulates in the plasma is conjugated, water-soluble and therefore excreted in the urine.

276
Q

A 62-year-old man presents to his GP as his family have noticed

he has

‘gone yellow’

. On

further questioning he admits to generalised pruritus and around

2 stone of weight loss

. He

denies any abdominal or chest pain. On examination you note he has icteric sclera and

widespread scratch marks.

What is the most likely diagnosis?

1) Biliary colic

2) Ascending cholangitis

3) Cholecystitis

4) Gallstone ileus

5) Pancreatic cancer

A

Explanation

Pancreatic cancer

Pancreatic cancer classically presents insidiously with painless jaundice, often with a history of

weight loss as the cancer usually presents at late stage. These cancers tend to be cancers of the

head of the pancreas as it is through this portion of the pancreas that the CBD traverses. As this

is the only condition listed that typically causes painless jaundice it is the correct answer.

Obstructive jaundice classically produces dark urine and paler stools and it would be usefu l to

know this from the history.

Biliary colic

This normally presents with sudden onset, colicky right upper quadrant pain. Often associated

with recent ingestion of large/fatty meals. It is unusual for jaundice to develop unless a gallstone

has entered to common bile duct, or occasionally a gallstone within the neck of the gall-bladder

or Hartmann’s pouch can compress the common bile duct (CBD) and cause an obstructive

jaundice (known as Mirizzi’s syndrome). The absence of pain and insidious onset strongly goes

against this diagnosis.

Ascending cholangitis

Ascending cholangitis is a medical emergency that can be recognised by Charcot’s triad of fever,

right upper quadrant abdominal pain and jaundice. This insidious presentation does not fit this

diagnosis. Cholecystitis

Cholecystitis normally presents with constant right upper quadrant pain and fever, classically the

patient is Murphy’s positive in which deep inspiration while palpating the right upper quadrant

(RUQ) results in the patient ‘catching their breath’ as the inflamed gall-bladder is lowered onto

the palpating hand. Jaundice is uncommon unless a stone within the neck of the gall-bladder or

Hartmann’s pouch is compressing the CBD and can cause an obstructive jaundice (known as

Mirizzi’s syndrome), additionally the localised inflammation and swelling around the gall-

bladder can result in compression of the CBD and therefore an obstructive jaundice.

Gallstone ileus

Gallstone ileus occurs when a gallstone enters the small bowel, usually through a fist ula

connecting the gall-bladder and small intestine. It can then cause a mechanical small bowel

obstruction. It does not cause jaundice.

277
Q

A

17-year-old

man is being investigated for tremor and dysarthria. His GP has performed blood

tests that show very

deranged liver function tests (LFTs)

. On

examination, he is noted to have a

tremor, impaired memory and slit -lamp examination reveals

Kayser–Fleischer rings

.

What is the most likely diagnosis?

1) Alcoholic cirrhosis

2) Alpha-1 antitrypsin deficiency

3) Autoimmune hepatitis

4) Hepatitis B

5) Wilson’s disease

A

Explanation

Wilson’s disease

Wilson’s disease is an autosomal-recessive disorder that results in the toxic accumulation of

copper in the liver and central nervous system (CNS) (in particular the basal ganglia). Children

often present with liver disease, and adults with CNS features. Kayser –Fleischer rings are copper

deposits in the iris (Descemet’s membrane) that are pathognomic but not invariable and may

require a slit-lamp to be seen. Blood tests would possibly reveal low ceruloplasmin and high

serum copper, along with abnormal LFTs. First -line treatment is with d-penicillamine, a copper

chelating drug, along with dietary advice on avoidance of copper -containing foods.

Alcoholic cirrhosis

Alcohol is a very common cause of deranged liver function tests, and can also present with

confusion and ataxia as part of Wernicke’s syndrome. However, this patient is very young to

have developed this already, especially without any mention of a significant alcohol history, and

alcohol does not cause Kayser–Fleischer rings (which are pathognomonic of Wilson’s disease).

Alpha-1 antitrypsin deficiency

Alpha-1 antitrypsin deficiency is an autosomal-recessive condition that predispo ses affected

patients to cirrhosis and chronic obstructive pulmonary disease (COPD) (due to a predisposition

to develop emphysematous changes in the lungs), it does not have neurological affects.

Autoimmune hepatitis Autoimmune hepatitis can present as an acute hepatitis or a chronic hepatitis and eventually

leads to cirrhosis. It is associated with autoantibodies, including antinuclear antibody (ANA) and

smooth muscle autoantibodies, it is most common in women and in those aged 40 –50 years.

Additionally, it does not cause neurological disease.

Hepatitis B

Hepatitis B is a viral hepatitis, which in the UK is normally acquired as an adult through needle -

sharing or sexual intercourse. In endemic countries the vertical transmission route is relatively

much more important. In the majority (approx 95%) of those infected as adults it causes an acute

infection that may be symptomatic or may cause hepatitis, but is eventually cleared successfully.

The risk of chronic infection if infected perinatally at birth is ar ound 90%. Those with a chronic

infection can also develop cirrhosis (approx 20%) or reactivation of the disease periodically

(approx 25%). These scenarios could account this patient’s deranged LFTs, however, hepatitis B

does not usually cause neurological effects and in the absence of stated risk factors hepatitis B

infection is unlikely.

278
Q

A 57-year-old businessman from

Thailand

presents to his GP with symptoms of malaise, weight

loss and right upper quadrant pain. On examination he is noted to have

hepatomegaly, which

is

irregular

.A

bruit

is also heard over the liver. His blood tests show very

raised a-

fetoprotein

(AFP) levels.

What is the most likely diagnosis?

1) Cholangiocarcinoma

2) Fibrosarcoma of the liver

3) Gallstones

4) Hepatocellular carcinoma (HCC)

5) Liver haemangioma

A

Explanation

Hepatocellular carcinoma (HCC)

HCC is the commonest malignant tumour of hepatocytes accounting for 90% of primary liver

cancers. World-wide it is most commonly caused by chronic hepatitis B, followed by hepatitis C

infection, and its incidence is highest in Asia (such as Thailand where this patient is from) and

sub-Saharan Africa. In Europe, hepatitis C infection is the most common cause, however any

cause of cirrhosis puts patients at increased risk of HCC – so alcoholism, autoimmune conditions

such as primary biliary cirrhosis (PBC) and primary sclerosing cholangitis (PSC), and genetic

conditions such as haemochromatosis PBC are all common causes. Another important cause in

Asia and sub-Saharan Africa is aflatoxins – these are toxins produced by fungi that contaminate

foodstuff such as peanuts and grains. In the UK the incidence of HCC is around 4.3 per 100 000

(Cancer Research UK). It commonly presents on the background of cirrhosis and associated

signs, possibly presenting due to a deterioration in the patient’s chronic liver disease. Other

symptoms suggestive of HCC include right upper quadrant pain, weight loss, and new onset

ascites/jaundice. On examination they may be palpable as an irregular hepatomegaly and

sometimes a bruit may be heard over them. The a-fetoprotein levels are raised in 80% of patients.

Patients at high risk of HCC should be screened every 6 –12 months with ultrasound liver and

serum AFP – these patients include cirrhotic patients due to HBV/HCV, PBC, haemochromatosis

and alcoholism.

Cholangiocarcinoma

Cholangiocarcinoma most commonly presents with jaundice and deranged liver function tests,

possibly with abdominal pain, all due to obstruction of the biliary tree , often on a background of

weight loss. It can also present acutely with fever, jaundice and abdominal pain due to it

predisposing towards ascending cholangitis. It is a rare cancer, affecting 3.58 per 100 000

population in England (National Cancer Intelligence Network statistics for rare and less common

cancers). In addition, it would be unusual for this cancer to present with a palpable mass without

having caused obstruction and resultant jaundice.

Fibrosarcoma of the liver Fibrosarcomas are tumours arising from fibroblasts. They are most common in the limbs, but

even then are uncommon. The literature regarding these tumours affecting the liver is confined to

only a few case reports in which they tend to present as large masses. The most common type of

sarcoma affecting the liver is the angiosarcoma, which makes up around 0.3% of all liver cancers

(Cancer Research UK statistics).

Gallstones

Gallstones normally present in the context of biliary colic – sudden onset, colicky right upper

quadrant pain. Often associated with recent ingestion of large/fatty meals. They would not

explain the bruit, or the palpable irregular hepatomegaly, nor the raised AFP or weight loss.

Liver haemangioma

These are the most common benign liver tumour, mostly they are incidental findings on imaging

for other indications, if they do present clinically it is normally with a right upper quadrant

(RUQ) ‘fullness’ or pain – they would not, however, cause this patient’s weight loss or raised

AFP.

279
Q

A sigmoidoscopy reveals an inflamed mucosa and a rectal biopsy shows an inflammatory infiltrate

with

goblet-cell depletion and crypt abscesses

.

What is the most likely diagnosis?

1) Coeliac disease

2) Crohn’s disease

3) Irritable bowel syndrome

4) Ulcerative colitis

5) Whipple’s disease

A

Explanation

Ulcerative colitis

Ulcerative colitis commonly presents between the ages of 15 and 25 years. It is characterised by

diarrhoea often with mucus and blood. There may be a crampy abdominal pain, and the

frequency of motions may be related to the severity of an attack. It may affect only the rectu m

(proctitis in approximately 50%), or spread proximally to involve part of the rest of the colon.

Sigmoidoscopy shows an inflamed and friable mucosa from the rectum and a variable but

continuous amount proximally, classically with pseudopolyps, and histologically there may be an

inflammatory infiltrate, goblet -cell depletion, glandular distortion, mucosal ulcers and crypt

abscesses.

Coeliac disease

Coeliac disease is a condition that affects the small bowel – chronic inflammation results from

autoimmune reaction to gluten, with serum findings of antiendomysial, antitissue

transglutaminase and antigliadin antibodies. It results in diarrhoea, normally without blood,

abdominal pain and bloating, and malabsorption. Endoscopic findings in the small bowel includ e

villous atrophy, increased numbers of lymphocytes and crypt hyperplasia.

Crohn’s disease

Crohn’s disease presents between the ages of 10 and 40 years and is characterised by diarrhoea,

usually without blood or mucous, and abdominal pain. Abdominal pain is often in the right iliac

fossa, as the most common site of affected bowel is the terminal ileum. Features of

malabsorption (eg anaemia secondary to low vitamin B 12 levels) are common. It can affect any

region of the alimentary canal from mouth to anus, and causes ‘skip’ lesions. Endoscopy may

show deep ulceration and cobblestone appearance, fissures, abscesses and stenosis are frequent

findings, and histologically there may be an inflammatory infiltrate, non-caseating granulomas

and transmural inflammation.

Irritable bowel syndrome Irritable bowel syndrome (IBS) presents with long -term altered bowel function, such as

diarrhoea or constipation or alternation between the two, often with abdominal pain or bloating.

It is a functional condition, by definitio n with no discernible metabolic or mechanical cause –

signs or symptoms such as bloody diarrhoea (which this patient has), anaemia, weight loss,

evidence of malabsorption all point firmly away from this diagnosis.

Whipple’s disease

Whipple’s disease is caused by infection with Tropheryma whipplei, commonly it presents with

diarrhoea (steatorrhoea) and weight loss as a result of malabsorption. Joint pain and arthritis are

also common, usually a migratory non-deforming arthritis. Less common but recognised features

also include skin pigmentation, endocarditis, uveitis and central nervous system (CNS)

involvement. Diagnosis is made by endoscopy of the duodenum, and biopsy showing PAS -

positive macrophages and non-acid-fast Gram-positive bacilli.

280
Q

A

17-year-old

A-level student presents with

diarrhoea

, abdominal pain and

weight loss

. On

examination, she is tender

in the

right iliac fossa

(RIF) and on

further questioning she had

a

perianal abscess drained

1 year ago. A barium enema shows

cobblestoning

in the RIF.

What is the most likely diagnosis?

1) Coeliac disease

2) Crohn’s disease

3) Irritable bowel syndrome

4) Whipple’s disease

5) Ulcerative colitis

A

Explanation

Crohn’s disease

Crohn’s disease is a form of inflammatory bowel disease that has several features in common

with ulcerative colitis and commonly presents between 10 –40 years of age. However, it is

characterised by transmural granulomatous inflammation and can affect any part of the gut. It is

most commonly found in the terminal ileum (and therefore may produce discomfort in the RIF),

but can affect anywhere from mouth to anus causing discontinuous skip lesions. Other features

seen include diarrhoea and weight loss. Perianal disease is also common and may manifest as

abscesses and fistulae. Barium enemas will classically show cobblestoning and ‘rose -thorn’

ulcers. A magnetic resonance imaging (MRI) scan can also be used to assess the extent of pelvic

disease. Endoscopy may reveal deep ulceration and a cobblestone appearance, fissures, abscesses

and stenosis are frequent findings. Histologically there may be an inflammatory infiltrate, non -

caseating granulomas and transmural inflammation.

Coeliac disease

Coeliac disease is a condition that affects the small bowel – chronic inflammation results from

autoimmune reaction to gluten, with serum findings of antiendomysial, antitissue

transglutaminase and antigliadin antibodies. It results in diarrhoea, normally without blood,

abdominal pain and bloating, and malabsorption. Endoscopic findings in the small bowel include

villous atrophy, increased numbers of lymphocytes and crypt hyperplasia.

Irritable bowel syndrome

Irritable bowel syndrome (IBS) presents with long -term altered bowel function, such as

diarrhoea or constipation or alternation between the two, often with abdominal pain or bloating.

It is a functional condition, by definition with no discernible metabolic or mechanical cause –

signs or symptoms such as bloody diarrhoea, anaemia, weight loss, and evidence of

malabsorption all point firmly away from this diagnosis.

Whipple’s disease Whipple’s disease is caused by infection with Tropheryma whipplei, commonly it presents with

diarrhoea (steatorrhoea) and weight loss as a result of malabsorption. Joint pain and arthritis ar e

also common, usually a migratory non-deforming arthritis. Less common but recognised features

also include skin pigmentation, endocarditis, uveitis and CNS involvement. Diagnosis is made

by endoscopy of the duodenum, and biopsy showing PAS -positive macrophages and non-acid-

fast Gram-positive bacilli.

Ulcerative colitis

Ulcerative colitis commonly presents between the ages of 15 and 30 years. It is characterised by

diarrhoea often with mucus and blood. There may be a crampy abdominal pain, and the

frequency of motions may be related to the severity of an attack. It may affect only the rectum

(proctitis in approximately 50%), or spread proximally to involve part or the rest of the colon.

Barium enema reveals button-shaped ulcers and pseudopolyps, thickened bowel-wall and

haustra, filiform polyps and, in cases of chronic inflammation, a ‘lead pipe’ colon can occur

(featureless colon with no normal haustral markings). Sigmoidoscopy shows an inflamed and

friable mucosa from the rectum and a variable but continuous amount proximally. Histologically

there may be an inflammatory infiltrate, goblet -cell depletion, glandular distortion, mucosal

ulcers and crypt abscesses.

281
Q

A 56-year-old man presents with episodes of severe epigastric pain radiating through to his back

for

several weeks. He also reports he has noticed his stools have tended to

float

more recently

and are very

difficult to flush away

. On examination

erythema ab igne

is noted over the

epigastrium.

What is the most likely diagnosis?

1) Acute pancreatitis

2) Biliary colic

3) Coeliac disease

4) Chronic pancreatitis

5) Pancreatic Cancer

A

Explanation

Chronic pancreatitis

Chronic pancreatitis presents most commonly either with chronic epigastric pain, or multiple

episodes of acute epigastric pain, often radiating into the back. The pain may be relieved by

sitting forwards and is often made worse by eating. Patients may also report nausea and

vomiting, bloating and pale/difficult to flush/malodourous stools (steatorrhoea) with weight loss.

The later symptoms are a result of pancreatic exocrine insufficiency resulting in malabsorption.

The patient may also have signs/symptoms of endocrine insufficiency resulting in high body

mass (BM), with polyuria and polydipsia. The commonest cause of chronic pancreatitis is

alcohol excess, while gallstones is another common cause. It can also idiopathic, due to trauma,

autoimmune disease, cystic fibrosis or other genetic disorders. There are a number of features of

this case to suggest it is chronic pancreatitis rather than acute – its prolonged history and the

patient has symptoms suggestive of steatorrhoea from exocrine insufficiency. The presence of

erythema ab igne also fits with a chronic picture. This is a ‘rash’ consisting of reticular erythema

and hyperpigmentation most often caused by chronic use of a hot water bottle to relieve pain and

is often seen in the epigastrium or on the back.

Acute pancreatitis

Acute pancreatitis classically presents with sudden onset of severe epigastric pain radiating

through to the back, better on sitting forwards. Commonly patients will be nauseous and

vomiting, and have evidence of localised peritonitis (such as rebound tenderness and guarding in

the epigastrium). The most common causes are gallstones and excessive alcohol, other causes

can be remembered by the mnemonic below. While the description of the pain suffered by the

patient is certainly fitting with pancreatitis, there are several features that point towards chronic

rather than acute pancreatitis – including the multiple episodes over several weeks, and the

evidence of exocrine insufficiency causing steatorrhoea.

Biliary colic This normally presents with sudden onset, colicky right upper quadrant pain. Often associated

with recent ingestion of large/fatty meals. The site of the pain in this patient does not fit with this

description, and biliary colic would not explain his stools.

Coeliac disease

Coeliac disease is a condition that affects the small bowel – chronic inflammation results from

autoimmune reaction to gluten, with serum findings of anti-endomysial, anti-tissue

transglutaminase and anti-gliadin antibodies. It results in diarrhoea (normally without blood),

abdominal pain, bloating, weight loss, fatigue and mouth ulcers. In addition commonly

malabsorption is present which may manifest as iron-deficiency anaemia or vitamin B12 /folate

deficiency with macrocytic anaemia, or steatorrhoea (as in this patient). The severe pain,

radiating into the patient’s back and in multiple distinct episodes does not fit with coeliac disease

which tends to present with a milder colicky abdominal pain, or indigestion-type pain.

Pancreatic Cancer

Pancreatic cancer classically presents insidiously with painless jaundice. These cancers tend to

be cancers of the head of the pancreas as it is through this portion of the pancreas that the

common bile duct (CBD) traverses. Other symptoms of pancreat ic cancer (of any anatomical

position) are general malaise, poor appetite, significant weight loss, and epigastric abdominal

pain or lower thoracic back pain. Multiple severe episodes of epigastric pain without a history of

weight loss/malaise do not fit well with pancreatic cancer.

282
Q

A

22-year-old

woman presents to her GP with a history of

vague

abdominal pain and bloating.

On further questioning she reports weight loss and passing

offensive stools

which are

difficult to

flush away. As part of her investigations a jejunal biopsy is performed. This shows

crypt

hyperplasia

and subtotal

villous atrophy

. What is the most likely diagnosis?

1) Bacterial overgrowth

2) Coeliac disease

3) Crohn’s disease

4) Ulcerative colitis

5) Tropical malabsorption

A

Explanation

Coeliac disease

Coeliac disease is a T-cell-mediated autoimmune disease affecting the small bowel caused by

gluten intolerance. It causes a variety of symptoms including steatorrhoea (due to an inability to

absorb fats as evidenced in this case by offensive stools that are difficult to flush away),

abdominal pain and bloating. It can be diagnosed by a jejunal biopsy, which classically shows

subtotal villous atrophy and crypt hyperplasia. The management involves a gluten-free diet,

which can relieve symptoms.

Bacterial overgrowth

Also known as small bowel bacterial overgrowth, this can mimic coeliac disease with partial

villous atrophy, however the aspirate would demonstrate a high number of bacteria. It is less

common than coeliac disease.

Crohn’s disease

Crohn’s disease is a form of inflammatory bowel disease and can affect any part of the

gastrointestinal tract from mouth to anus. It most commonly presents with diarrhoea, weight loss

and abdominal pain. Offensive stools would be unlikely, although the patient may report blood

or mucous in the stool. Furthermore, crypt hyperplasia and villous atrophy is indicative of

coeliac disease.

Ulcerative colitis

Ulcerative colitis is a form of inflammatory bowel disease affecting the large bowel (and

occasionally the terminal ileum due to backwash ileitis). While it may present with weight loss

and abdominal pain, jejunal histology would be unremarkable.

Tropical malabsorption

There is no history of foreign travel in the scenario ther efore this is unlikely.

283
Q

A 45-year-old woman is referred to an upper gastrointestinal (GI) clinic with a history

of

dysphagia

over the last month. She feels it has developed gradually and she is now struggling

to swallow food. Her only past medical history of note is

Raynaud’s phenomenon, and on examination it is noted she has radial

furrowing of the lips

and her fingers are

tapered

. She has

no history of weight loss, change in bowel habit or family history of note.

What is the most likely diagnosis?

1) Achalasia

2) Myasthenia gravis

3) Oesophageal carcinoma

4) Scleroderma

5) Syringobulbia

A

Explanation

Scleroderma

Scleroderma is a likely autoimmune condition affecting the skin and connective tissue, causing

hardening of tissue. It is characterised by increased dermal collagen and decreased tissue

elasticity. It may be in a localised form, affecting the skin only, or systemic form. Systemic

scleroderma may be either limited or diffuse. Limited cutaneous scleroderma, which includes

calcinosis cutis, Raynaud’s phenomenon, oesophophageal dysfunction, sclerodactyly and

telangiectasia syndrome (CREST) syndrome, has a better prognosis. Diffuse cutaneous

scleroderma represents a more severe form in which widespread organ involvement occurs at an

early stage

Achalasia

Achalasia is a motility disorder of the oesophagus in which relaxation of the lower oesophageal

sphincter fails preventing passage of food into the stomach. It presents with progressive

dysphagia and regurgitation of undigested food. Lip and finger changes would not be seen.

Myasthenia gravis

Myasthenia gravis is an autoimmune condition due to antibodies against the acetylcholine

receptors of the neuromuscular junction, resulting in progressive muscle fatigue. There is no

association with Raynaud’s phenomenon and lip/finger changes.

Oesophageal carcinoma

Oesophageal carcinoma would not result in radial furrowing of the lips or tapering of the fingers

but may present with weight loss and progressive dysphagia.

Syringobulbia

This is a rare condition affecting the brainstem and spinal cord. Fluid-filled cavities enlarge and

compress nerves often resulting in cranial nerve palsies. It would not cause lip or finger changes.

Other peripheral neurological signs and symptoms would be expected.

284
Q

A 44-year-old woman is being investigated for intermittent right upper quadrant pain that occurs

mostly after

an

incidental

eating. An ultrasound scan shows gallstones within the gall bladder and

finding of a liver lesion, which presents as an

echogenic spot

. Further imaging with

a contrast-enhanced computed tomography (CT) scan demonstrated distinctive

puddling of

contrast

in venous channels. It is thought the patient has a benign liver tumour.

Which one of the following is the commonest benign tumour of the liver?

1) Cholangiocarcinoma 2) Haemangioma

3) Haemangiosarcoma

4) Hepatocellular carcinoma

5) Hepatic adenoma

A

Explanation

Haemangioma

Haemangiomas are the commonest benign tumour of the liver and mostly remain asymptomatic.

If they are very large (>5 cm) there is a risk of abdominal pain and rupture with intra -abdominal

haemorrhage. Single-photon emission CT scan (SPECT) with 99 Tc m -labelled red blood cells will

demonstrate persistence of blood activity in the lesion.

Cholangiocarcinoma

Cholangiocarcinoma is a malignant lesion of the biliary tree, whereas this case describes the

lesion within the liver parenchyma.

Haemangiosarcoma

Haemangiosarcoma is a rare malignant lesion of the liver, more commonly seen in dogs than

humans.

Hepatocellular carcinoma

Hepatocellular carcinoma is a malignant lesion of the liver. It is the most common form of liver

cancer, has a strong association with chronic liver disease, and are often multifocal at diagnosis

so prognosis is guarded.

Hepatic adenoma

Hepatic adenomas are rare benign lesions of the liver and typically present in younger women,

often on the background of elevated levels of oestrogen for example from the combined oral

contraceptive pill.

285
Q

A

78-year-old

man presents to his GP with a strange feeling of

food getting stuck

when he

swallows. This has been present for around 3 months and initially was only noticed when eating

solid foods such as bread or meat,

but

now he has problems swallowing liquids

. He also reports

significant unintended

weight loss

.

Which one of the following is the most likely cause?

1) Achalasia

2) Barrett’s oesophagus

3) Hiatus hernia 4) Oesophageal carcinoma

5) Pharyngeal pouch

A

Explanation

Oesophageal carcinoma

Presenting features of oesophageal carcinoma are classically progressive dysphagia (initially

solids and then liquids), associated with significant weight loss. Retrosternal chest discomfort

may occur. The most common forms of oesophageal cancer are squamous and adenocarcinoma.

Adenocarcinoma now exceeds squamous carcinoma by a ratio of 2:1. The ratio was previously in

the reverse direction.

Achalasia

Achalasia is a motility disorder of the oesophagus in which relaxation of the lower oesophageal

sphincter fails preventing passage of food into the stomach. It presents with progressive

dysphagia and regurgitation of undigested food

Barrett’s oesophagus

Adenocarcinoma is predisposed to by a metaplastic change in the oesophageal mucosa from

squamous to columnar epithelium by chronic reflux (Barrett’s oesophagu s). However, in this

case there is no history of reflux disease.

Hiatus hernia

A hiatus hernia is likely to result in reflux symptoms and retrosternal pain, it would not cause

dysphagia and weight loss.

Pharyngeal pouch

A pharyngeal pouch is a diverticulum through Killian’s dehiscence. It presents with halitosis,

dysphagia and regurgitation. It is common in elderly men, however significant weight loss would

not necessarily be expected as described in the case above.

286
Q

A 41-year-old woman is urgently re ferred because of

dysphagia

. On assessment you note the

patient is wearing gloves because she gets very

painful hands in the cold

. She describes a colour

change going from

white to blue to bright red. It is also noted she has very

tight skin around her

lips

and she reports limited mouth opening.

What disease process is likely to be present?

1) Achalasia

2) Diffuse oesophageal spasm 3) Myasthenia gravis

4) Oesophageal carcinoma

5) Scleroderma

A

Explanation

Scleroderma

The colour change describes the classic pattern in Raynaud’s syndrome. This may be idiopathic,

or associated with connective tissue disorders, vascular obstructive disorders or occupation. In

this case, the patient has scleroderma. Calcinosis cutis, Raynaud’s phenomenon, oesophophageal

dysfunction, sclerodactyly and telangiectasia syndrome (CREST) syndrome is a form of

scleroderma. Its five main features are associated with anticentromere antibodies.

Achalasia

Achalasia is a motility disorder of the oesophagus in which relaxation of the lower oesophageal

sphincter fails, preventing passage of food into the stomach. It presents with progressive

dysphagia and regurgitation of undigested food.

Diffuse oesophageal spasm

Diffuse oesophageal spasm causes intermittent chest pain upon eating food as the oesophagu s

spasms against a food bolus. It can be very painful but would not cause true dysphagia or lip

changes.

Myasthenia gravis

Myasthenia gravis is an autoimmune condition due to antibodies against the acetylcholine

receptors of the neuromuscular junction. It results in progressive muscle fatigue. There is no

association with Raynaud’s phenomenon and lip changes.

Oesophageal carcinoma

Oesophageal carcinoma classically presents with progressive dysphagia (initially solids and then

liquids). This is often associated with significant weight loss. Retrosternal chest discomfort may

also occur. The key features in this history to differentiate are skin colour changes and lip

changes.

287
Q

A 57-year-old man presents to his GP with vague symptoms of a

bdominal pain and weight loss

.

On examination his abdomen is soft, but an

irregular hepatomegaly

is noted and on further

assessment a

hard

lymph node is palpable in the

left supraclavicular fossa

.

What is the most likely cause?

1) Duodenal ulcer

2) Gallstones

3) Gastric cancer

4) Gastro-oesophageal reflux disease (GORD)

5) Gastric ulcer

A

Gastric cancer

Gastric cancer often starts with very non-specific symptoms – anorexia, weight loss, features of

dyspepsia. Tumours of the antrum can cause gastric outlet obstruction and present with fullness,

nausea and vomiting. Features found on examination can include signs of metastases – ascites,

irregular hepatomegaly and Virchow’s node (a hard lymph node in the left supraclavicular

fossa).

Duodenal ulcer

While a duodenal ulcer may present with vague abdominal pain, there is unlikely to be

significant weight loss and irregular hepatomegaly would not be seen.

Gallstones

Gallstones will result in right upper quadrant abdominal pain that is wor se when consuming fatty

meals. Virchow’s node is indicative of an intra-abdominal malignancy.

Gastro-oesophageal reflux disease (GORD)

GORD would not present with weight loss or irregular hepatomegaly. It is a common condition

resulting in indigestion that is worse after food and upon lying down.

Gastric ulcer

An ulcer would not cause irregular hepatomegaly or significant weight loss.

288
Q

A 17-year-old man with known mental

health problems is admitted to a burns unit after self-

immolation. On arrival his burns were

mostly full thickness

and the surface area was

34%

. He is

moved to the Burns Intensive Care Unit (ICU) and is intubated and ventilated and requiring

inotropic support. Six days later he develops an episode of

haematemesis.

What is likely to be the cause?

1) Curling’s ulcer

2) Cushing’s ulcer

3) Oesophageal burns

4) Mallory-Weiss tear

5) Zollinger-Ellison syndrome

A

Explanation

Curling’s ulcer

A gastric ulcer that occurs in burn patients when severe depletion of the plasma volume results in

ischaemia and cell necrosis of the mucosa is known as a Curling’s ulcer. It is therefore important

that all severe burn patients are managed with proton pump inhibitors to reduce the incidence of

this.

Cushing’s ulcer

Cushing’s ulcer is a gastric ulcer occurring as a result of increased intra-cranial pressure.

Oesophageal burns

Oesophageal burns are rare. In cases of self-immolation tracheal burns are much more common.

Mallory-Weiss tear

Mallory–Weiss tears occur due to persistent vomiting, resulting in a small tear of the mucosal

lining and subsequent minor haematemesis.

Zollinger-Ellison syndrome

This syndrome is caused by a gastrinoma, which results in elevated stomach acid levels and

subsequently peptic ulcer disease.

289
Q

A 58-year-old man is referred to a dermatologist by his GP due to careful inspection it is noted he has a pigmented, rough thickening

skin changes in his axillae. On

of his skin in his axillae,

with

warty lesions

.

What other important conditions must be considered in the case?

1) Rheumatoid arthritis

2) Coeliac disease

3) Crohn’s disease

4) Gastrointestinal cancer

5) Chronic pancreatitis

A

Explanation Gastrointestinal cancer

The skin changes described in this case is acanthosis nigricans and is associated with

gastrointestinal (eg stomach) cancer.

Rheumatoid arthritis

Rheumatoid arthritis is an autoimmune condition resulting in inflammation of the synovium

causing joint pain and swelling. Cutaneous manifestations include palmar erythema, skin

atrophy, and brittle nails.

Coeliac disease

Coeliac disease is associated with dermatitis herpetiformis in which itchy blisters develop over

the knees, elbows and scalp.

Crohn’s disease

Crohn’s disease is associated with several skin changes including pyoderma gangrenosum and

erythema nodosum.

Chronic pancreatitis

Cutaneous manifestations of chronic pancreatitis are uncommon and often examination is

unremarkable except for vague abdominal pain.

290
Q

A 29-year-old woman with a body mass index (BMI) of more than 40 kg/m 2 undergoes a

gastric

bypass procedure

. She rapidly begins to lose weight after the procedure.

Which one of the following is the most appropriate statement regarding this procedure?

1) Perioperative mortality associated with bariatric surgery is 3% and is an important

consideration that should be discussed with the patient before surgery

2) An adjustable gastric band forms an integral part of the procedure

3) Iron and vitamin B 12 deficiency are common

4) Jejuno-ileal bypass with cholecystectomy is the pref erred option

5) Weight loss from a biliopancreatic diversion with duodenal switch is less effective than

biliopancreatic diversion alone

A

Explanation Iron and vitamin B 12 deficiency are common

Surgery for morbid obesity is considered in individuals who have a body mass index (BMI) more

than 40 kg/m 2 . Obesity surgery for patients with a body mass index (BMI) between 35 –39

kg/m 2 may be justified if they have a co -morbid condition that will be improved by weight loss.

The decision to operate must be made within a dedicated multidisciplinary team comprising a

bariatric surgeon, dietician, physician, anaesthetist, radiologist and clinical nurse specialist. the

conditions that can be improved by weight loss include type II diabetes mellitus, hypertension

and dyslipidaemia (the metabolic syndrome). Others include obstructive sleep apnoea, decreased

mobility, osteoarthritis and hypertrophic cardiomyopathy and poor quality of life. Importantly, it

must be remembered that gastric bypass surgery can result in iron and vitam in B 12 deficiencies.

Perioperative mortality associated with bariatric surgery is 3% and is an important consideration

that should be discussed with the patient before surgery

Perioperative mortality for bariatric surgery is reported as less than 0.3% and is declining due to

improved management in specialist bariatric centres with multidisciplinary care. Complications

of surgery should always be discussed with the patient; in bariatric surgery patient expectations

around weight loss following surgery can be unrealistic and must be managed.

An adjustable gastric band forms an integral part of the procedure

The surgery can be broadly classified into restrictive and malabsorptive. The former consists of

gastric banding or vertical banded gastroplasty which reduces the size of the stomach. The latter

is achieved by bypass procedures to alter the digestive tract so that less absorption occurs. Most

procedures are a combination of the two techniques.

Jejuno-ileal bypass with cholecystectomy is the preferred option

Jejuno-ileal bypass has fallen from favour as it has been associated with serious complications.

Weight loss from a biliopancreatic diversion with duodenal switch is less effective than

biliopancreatic diversion alone

A combined surgical approach of biliop ancreatic diversion with duodenal switch is associated

with greater weight loss.

291
Q

A 51-year-old man presents with an upper gastrointestinal (GI) bleed. His systolic blood pressure

is 118 mmHg and blood results show an haemoglobin (Hb) of 11 g/dl (normal 10.5 –13.5 g/dl).

Endoscopy reveals a duodenal ulcer with a

visible bleeding vessel

which is successfully clipped

and injected with adrenaline.

Which is the most significant factor in predicting re-bleeding in this case?

1) Age

2) Blood pressure

3) Ulcer size

4) Urea

5) Visible bleeding vessel at the time of endoscopy

A

bleeding. The Blatchford score, which has been shown to predict the risk of requiring

intervention (transfusion, endoscopic or surgical therapy) and the Rockall score, which predicts

the risk of re-bleeding and death. Below is a table showing the Rockall score. The scores are

summed, the higher the score the higher the risk of re-bleeding and the higher the risk of

mortality.

Admission scores

0 points

1 points

2 points

3 points

Age

<60

60–79

80+

Shock

SBP >100, pulse <100

SBP >100, pulse >100

SBP <100, pulse >100

Comorbidities

None

None

CCF, IHD or other major comorbidity

Renal or liver failure, disseminated malignancy

Postendoscopy

0 points

1 points

2 points

3 points

Diagnosis

Mallory–Weiss tear, no lesion

All other diagnoses

Malignancy of upper GI tract

Major stigmata

None or dark spot only

Blood, adherent clot, visible or bleeding vessel

As you can see from the scoring system – the most important factors in predicting re-bleeding

are: the presence of significant endoscopic stigmata of recent haemorrhage with a visible

bleeding vessel, haemodynamic instability or shock, malignancy of the up per GI tract, age over

60 years and certain comorbidities. The Blatchford score also includes systolic blood pressure

and pulse, hepatic disease and cardiac failure, and additionally includes admission haemoglobin,

urea, melaena, syncope.

Age

Age of 60 years or above does predict re-bleeding risk, however, this patient is <60 years old.

Blood pressure

This patient’s blood pressure is within the normal range – while a low blood pressure does

predict increased re-bleeding risk, a normal blood pressure does not. Ulcer size

Ulcer size does not influence re-bleeding risk according to the major guidelines and scoring

systems.

Urea

This option is part of the Blatchford scoring system that predicts the need for intervention, but

has not been shown to predict re-bleeding risk, we do not know this patient’s urea levels so

cannot comment any further on it.

292
Q

A 48-year-old obese woman presents with right upper quadrant (RUQ) pain and nausea, she

reports similar episodes in the past that have resolved spontaneously. On examination she is

afebrile and has a soft abdomen which is tender in the RUQ and a plain X -ray shows radio-

opaque round opacities in the RUQ. These are thought to represent gallstones.

What proportion of gallstones can be seen on a plain abdominal X-ray?

1) <1%

2) 15%

3) 30%

4) 50%

5) >75%

A

Explanation

15%

Around 10–15% of gallstones can be seen on plain imaging due to calcification of the stones.

These often appear as rings due to calcium deposition around a central organic core. In some

cases the gall-bladder may be calcified (‘porcelain gall-bladder’) and therefore visible. Porcelain

gall-bladder is a type of chronic cholecystitis, its presence increases the risk of developing gall-

bladder cancer and cholecystectomy is the usual treatment. The most common imaging modality

used to detect gallstones is abdominal ultrasound – this has a much higher sensitivity of 85%,

and specificity up to 100%.

<1%

The correct answer is 10–15%. 30%

Gallstones are not usually visualised with abdominal X-ray (AXR).

50%

As above, however renal stones tend to be more radio -opaque.

> 75%

This is an incorrect response.

293
Q

A 41-year-old man is referred from his GP with a long history of

epigastric pain

which is relieved by

eating. He is otherwise well, and drinks alcohol only occasionally. The only other feature of note is a

long-standing history of diarrhoea, which has not responded to several changes in diet. An oesophago-

gastroduodenoscopy (OGD) reveals

multiple small duodenal ulcers

.

What is the most likely diagnosis?

1) Curling’s ulcer

2) Cushing’s ulcer

3) Duodenal carcinoma

4) VIPoma

5) Zollinger-Ellison syndrome

A

Explanation

Zollinger-Ellison syndrome

Zollinger–Ellison syndrome is the constellation of multiple, usually severe, peptic ulcers

affecting the stomach and duodenum associated with a gastrinoma (a tumour of gastrin-secreting

‘G’ cells, usually in the pancreas but can also be in the stomach or duodenum). Excess gastrin

promotes excess gastric acid secretion – this acid leads to peptic ulceration, and diarrhoea is a

common symptom too. Patients complain of epigastric pain due to peptic ulceration which may

be better with food. Around one-third of patients have MEN-1syndrome.

Curling’s ulcer

A Curling’s ulcer is a stress ulcer within the stomach or duodenum that occurs acutely in

association with severe skin burns. They are usually multiple and shallow. Cushing’s ulcer

A Cushing’s ulcer is an ulcer within the stomach or duodenum associated with raised intracranial

pressure – raised intracranial pressure classically presents with headaches that are worse in the

morning and on lying down, associated with nausea and blurred vision.

Duodenal carcinoma

Duodenal carcinoma is uncommon [only 0.3% of gastrointestinal (GI) malignancies], it is

usually an adenocarcinoma and could present as an ulcerated lesion, commonly it presents with

abdominal pain, as well as weight loss, nausea and vomiting, or with bleeding. It is unlikely to

present as multiple small erosions as described here though, and most commonly presents in the

sixth decade of life.

VIPoma

This is a rare type of cancer of the pancreatic islets that results in secretion of vasoactive

intestinal peptide (VIP). It primarily presents with watery diarrhoea, that can result in

hypokalaemia and dehydration. It is sometimes associated with multiple endocrine neoplasia

type 1 (MEN1).

294
Q

A 57-year-old man with known

alcoholic liver disease

, presents with confusion and is generally

unwell. On examination he is noted to have a distended abdomen with shifting dullness. Imaging

later confirms a mass in his liver that is thought to be

malignant in origin.

What is the most likely nature of this mass?

1) Cholangiocarcinoma

2) Fibrolamellar carcinoma

3) Haemangioma

4) Hepatic angiosarcoma

5) Hepatocellular carcinoma (HCC)

A

Explanation

Hepatocellular carcinoma (HCC)

HCC is the commonest malignant tumour of hepatocytes accounting for 90% of primary liver

cancers. It usually occurs on a background of cirrhosis, world-wide it is most commonly caused

by chronic hepatitis B, followed by hepatitis C infection, and its incidence is highest in Asia and

sub-Saharan Africa. In Europe, hepatitis C infection is the most common cause, however, any

cause of cirrhosis puts patients at increased risk of HCC – so alcoholism, autoimmune conditions

such as primary biliary cirrhosis (PBC) and primary sclerosing cholangitis (PSC), and genetic

conditions such as haemochromatosis are all common causes. Another important cause in Asia

and sub-Saharan Africa is Aflatoxins – these are toxins produced by fungi contaminating

foodstuff such as peanuts and grains. In the UK the incidence of HCC is around 4.3 per 100 000

(Cancer Research UK). It commonly presents on the background of cirrhosis and associated

signs, possibly presenting due to a deterioration in the patient’s chronic liver disease, for

example with encephalopathy, ascites and reduced synthetic function. Other symptoms

suggestive of HCC include right upper quadrant pain, weight lo ss, and new onset

ascites/jaundice. On examination they may be palpable as an irregular hepatomegaly and

sometimes a bruit may be heard over them. The a-fetoprotein levels are raised in 80% of patients.

Increasingly patients are also detected on screening in the UK. Patients at high risk of HCC should be screened every 6–12 months with ultrasound liver and serum AFP – these patients

include cirrhotic patients due to HBV/HCV, PBC, haemochromatosis and alcoholism (see BSG

Guidelines for more information).

Cholangiocarcinoma

Cholangiocarcinoma most commonly presents with jaundice and deranged liver function tests,

possibly with abdominal pain, all due to obstruction of the biliary tree, often on a background of

weight loss. It can also present acutely with fever, jaundice and abdominal pain due to it

predisposing towards ascending cholangitis. It is a rare cancer, affecting 3.58 per 100 000

population in England (National Cancer Intelligence Network statistics for rare and less common

cancers).

Fibrolamellar carcinoma

Fibrolamellar carcinoma is a rare variant of HCC, around 1:5 million population incidence,

affecting children and young adults without a history of background liver disease. Risk factors

remain poorly understood.

Haemangioma

These are the most common benign liver tumour, mostly they are incidental findings on imaging

for other indications, if they do present clinically it is normally with a RUQ ‘fullness’ or pain –

they would not, however, cause this patient’s signs of decompensated liver disease.

Hepatic angiosarcoma

This cancer makes up around 2% of primary liver cancers, originating from endothelial cells

within the liver. It usually presents with a mass and general signs such as weight loss and

malaise. It can also present with rupture and bleed ing. Risk factors include environmental toxins,

such as vinyl chloride or arsenic.

295
Q

A 56-year-old woman attends her general practitioner seeking advice on a heavy dependence on

alcohol. She is currently low of mood, unemployed and complains of upper abdominal pain,

nausea, vomiting and retching.

Which one of the following symptoms are associated with chronic alcohol abuse?

1) Hightened libido

2) Menstrual disturbance

3) Microcytosis

4) Primary thrombocythemia

5) Sydenham’s chorea

A

Explanation

Menstrual disturbance

Excessive alcohol intake, even in the absence of liver damage, can lead to altered menstrual

cycle, from irregular periods to anovulation and altered hormone levels. One key function of the

liver is to metabolise oestrogen, as liver function deteriorates in chronic alcohol abuse menstrual

disturbance becomes very common. Additionally, menorrhagia may occur as liver damage

causes coagulopathy. During end-stage liver disease, menstruation may cease completely.

Hightened libido

Chronic alcohol use normally suppresses libido.

Microcytosis

Macrocytosis often develops in alcoholism. While this is sometimes due to decreased folate

levels or low vitamin B 12 from malnutrition associated with alcoholism, it is more frequently a

direct result of alcohol on the bone marrow (though the mechanism is poorly understood).

Primary thrombocythemia

Alcohol has multiple haematological effects. It has a suppressive effect on the bone marrow, so

can commonly cause thrombocytopaenia, anaemia with macrocytosis, lymphopenia and immunosuppression. See the link for an in-depth discussion of the various haematological effects

alcohol is known to have.

Sydenham’s chorea

Sydenham’s chorea is a description of the choreiform movements that can present associated

with acute rheumatic fever. The cause is due to an autoimmune reaction and inflammation within

the basal ganglia after exposure to the Group A b-haemolytic streptococci. Central nervous

system (CNS) effects associated with chronic alcohol abuse include Wernicke’s encephalopathy

(a reversible condition of ophthalmoplegia, confusion and ataxia) and Korsakoff’s syndrome

(irreversible dementia with marked anterograde amnesia). Both of these conditions are associated

with low ‘B’ vitamins, especially thiamine, due to chronic malnutrition. Signs and symptoms of

cerebellar ataxia are common in chronic alcoholism.

296
Q

A 78-year-old woman is taken to the Accident and Emergency department by her family

with

severe abdominal pain

and

fevers. On examination she appears unwell and is spiking

temperatures. You note she is

icteric

and on palpation is most

tender in the right upper

quadrant

(RUQ).

What is the most likely disease process in this patient?

1) Biliary colic

2) Cholangitis

3) Cholecystitis

4) Gallstone ileus

5) Renal colic

A

Explanation

Cholangitis

Ascending cholangitis is a medical emergency that can be recognised by Charcot’s triad of fever,

right upper quadrant abdominal pain and jaundice. It is caused commonly by biliary obstruction

that promotes biliary stasis and allows for infection to ascend from the duodenum. The

commonest risk factor is a gallstone but other causes include pancreatic head cancer or

cholangiocarcinoma, strictures, stents or recent endoscopic retrograde cholangiopancreatography

(ERCP). Patients can become very unwell with cholangitis and develop sepsis and subsequent

septic shock. Treatment involves intravenous (IV) antibiotics, fluid resuscitation and an ERCP to

relieve the biliary obstruction.

Biliary colic

This normally presents with sudden onset, colicky right upper quadrant pain. Often associated

with recent ingestion of large/fatty meals. It is unusual for jaundice to develop unless

a gallstone has entered to common bile duct, or occasionally a gallstone within the neck of the

gall-bladder or Hartmann’s pouch can compress the CBD and cause an obstructive jaundice

(known as Mirizzi’s syndrome). Additionally, patients are not usually tender in the RUQ

(although this is where the pain is lo cated) and they should not be febrile. Cholecystitis

Cholecystitis normally presents with constant right upper quadrant pain and fever, classically the

patient is Murphy’s positive where deep inspiration while palpating the RUQ results in the

patient ‘catching their breath’ as the inflamed gall-bladder is lowered onto the palpating hand.

Jaundice is uncommon unless a stone within the neck of the gall-bladder or Hartmann’s pouch is

compressing the common bile duct (CBD), which can cause an obstructive jaund ice (known as

Mirizzi’s syndrome), additionally the localised inflammation and swelling around the gall -

bladder can result in compression of the CBD and therefore an obstructive jaundice. Because

jaundice is unusual this is not the most likely cause of this presentation.

Gallstone ileus

Gallstone ileus occurs when a gallstone enters to small bowel, usually through a fistula

connecting the gall-bladder and small intestine. It can then cause a mechanical small bowel

obstruction. It does not cause jaundice or fevers, tenderness would not be localised to the RUQ.

Renal colic

Renal colic is caused by renal stones within the urinary tract – it presents with severe colicky

loin-to-groin pain, normally with blood on urine dipstick. Fevers do not occur except in

superadded infection. The patient may be tender in the renal angle, but should not be tender in

the RUQ. This certainly would not cause jaundice.

297
Q

A 27-year-

old woman presents with severe

pain during defecation

. She also reports a small

amount of

blood

on the toilet paper following defecation. Her only past medical history

is

constipation

, which has become worse because of the pain.

What is the most likely diagnosis?

1) Anal fissure

2) Anal fistula

3) Haemorrhoids

4) Perianal abscess

5) Pilonidal sinus

A

Explanation

Anal fissure

The history in the case is quite classical for an anal fissure where pain is the dominant symptom.

Normally there is severe sharp anal pain on passing stool, with a burning pain afterwards. A

small amount of blood on wiping may then occur, though is not always present. Examination

should reveal an anal fissure, and exclude the other causes mentioned here. Common risk factors

are constipation and hard stools, and Crohn’s disease.

Anal fistula

An anal fistula is a connection between the anal canal and the perianal skin. Most commonly

they result secondary to perianal abscesses, though they can also be secondary to Crohn’s

disease, trauma, or cancer, among other causes. They can present with pain, itching, loc alised

swelling, or discharge of pus. The absence of preceding history, and the predominance of pain at

defecation makes anal fistula less likely diagnosis.

Haemorrhoids

These may be asymptomatic, or present with itching/irritation, bright painless bleedi ng per

rectum (PR), or a feeling of ‘fullness’. Pain is not common unless the haemorrhoids are

strangulated or thrombosed, in which case the pain will be severe and constant (not just when

passing stool).

Perianal abscess Perianal abscess presents normally with perianal pain and swelling, it may be worse when

passing stool but will normally be present at other times too. Bleeding is not a common

symptom. They are not related to constipation.

Pilonidal sinus

Pilonidal sinuses are normally present in the nat al cleft – they can be asymptomatic, or present as

a small non-tender pit or lump. They may also present as a painful and tender lump, possibly

with purulent discharge as an acute pilonidal abscess. Symptoms are not usually related to

passing stool, and bleeding is not a common symptom, the major risk factors include hirsutism

and poor personal hygiene.

298
Q

A 35-year-old man presents after being referred from the GP with a painful, discharging swelling

around his anus. He has had a similar problem twice in

tender, erythematous swelling is noted

in the natal cleft

the past and on examination a very

, but separate from the anal verge.

What is the most likely diagnosis?

1) Ischiorectal abscess

2) Hidradenitis suppurativa

3) Perianal abscess

4) Pilonidal abscess

5) Supralevator abscess

A

Explanation

Pilonidal abscess

The commonest site for pilonidal abscesses is the natal cleft and there may be a history of

recurrent episodes over months or years. Pilonidal means ‘hair’s nest’ in Latin and theories

regarding its aetiology involve shed hairs getting trapped in pits and becoming a nidus for local

infection. Clinically there may be openings in the midline or either side of it sometimes with

tufts of hair. There may be inflammation of the adjacent skin and pus may be expressed via the

openings. Management involves incision and drainage of the abscess and then other procedures

can be done electively to try and prevent recurrence. Antibiotics are indicated if systemic signs

of infection, or evidence of surrounding cellulitis is present.

Ischiorectal abscess

The ischiorectal (or ischioanal) fossae are prism-shaped fascia-lined potential spaces adjacent to

the rectum. They are bordered by the levator ani muscle (superiorly and medially), the anal canal

(medially), the perineal skin (inferiorly), and the ischial tuberosity (later ally). Ischiorectal

abscesses are the second most common type of anorectal sepsis (30% of anorectal abscesses),

after perianal abscess. Like other anorectal abscesses the pain is predominantly perianal, and

often worse just before/during bowel movements – the anatomical description of this patient’s

abscess does not fit with this diagnosis.

Hidradenitis suppurativa

This is a long-term condition with multiple abscesses caused by infections within sweat glands,

they are most commonly found under the arm pits, around the buttocks and groin. As well as

normally having multiple lesions, the chronicity of the condition normally means that there is

evidence of old abscesses, such as scarring of the skin. This is unlikely because of the single

lesion this patient suffers from. Perianal abscess

Perianal abscesses are the commonest form of anorectal abscess (60% of total – and present

normally with perianal pain and swelling), it may be worse when passing stool but will normally

be present at other times too. They are frequently recurrent, as in this patient, but the description

of its location fits better with pilonidal abscess.

Supralevator abscess

This describes an abscess that forms from an upward extension of an intersphincteric abscess

into the supralevator space. They may also form from an intra-abdominal inflammation (eg

appendicitis) tracking down. They are very uncommon making up only 4% on anorectal

abscesses. Like other anorectal abscesses the pain is predominantly perianal, and often worse just

before/during bowel movements – as well as being a very uncommon diagnosis, the anatomical

description of this patient’s abscess does not fit with this diagnosis.

299
Q

An 18-year-old woman presents with intermittent abdominal pain and discomfort that is cramping in nature with a feeling of associated bloating. The discomfort is often relieved

through opening her bowels, and she also reports

intermittent episodes of diarrhoea

,

she

denies

any weight loss, rectal bleeding or mucous. A colonoscopy is

unremarkable

. Blood

tests reveal

normal

full blood count (FBC), urea and electrolytes (U&Es) and C-reactive protein

(CRP)/erythrocyte sedimentation rate (ESR).

What is the most likely cause of her symptoms?

1) Coeliac disease

2) Crohn’s disease

3) Irritable bowel syndrome

4) Hiatus hernia

5) Ulcerative colitis

A

Explanation

Irritable bowel syndrome

Irritable bowel syndrome (IBS) is one of the commonest causes of abdominal pain and is a

functional disorder with no discernible metabolic or mec hanical cause resulting in a mixture of

pain, discomfort, bloating and altered bowel habit. It does not cause per rectum (PR) bleeding or

mucus, or weight loss and these symptoms are suggestive of organic pathology and should be

investigated appropriately and not attributed to IBS. National Institute of Clinical Excellence

(NICE) recommend FBC, ESR and CRP and a coeliac screen, although other bloods and imaging

are unnecessary unless there are signs/symptoms concerning for organic disease. Certainly, a

colonoscopy done in a patient IBS will show a normal rectum and colon, and treatment focuses

on symptomatic relief through dietary measures, antispasmodics and psychological approaches.

Coeliac disease

Coeliac disease is an autoimmune inflammatory condition of the small bowel caused by ongoing

gluten exposure. Symptoms include abdominal bloating and pain, diarrhoea, weight loss and

signs of malabsorption – this may commonly include steatorrhoea or anaemia (iron deficiency, or

vitamin B 12 or folate deficiency). Colonoscopy would be normal, although small bowel biopsy

would reveal villous atrophy, crypt hyperplasia and increased numbers of lymphocytes. Bloods

tests typically show autoantibodies (anti-tTG, anti-gliadin and anti-endomysial), anaemia, and

possibly raised inflammatory markers – a coeliac screen is recommended by the National

Institute of Clinical Excellence (NICE) Guidance on inflammatory bowel disease (IBS) before

making the diagnosis of IBS.

Crohn’s disease

This classically presents with abdominal pain, often in the right iliac fossa (as the terminal ileum

is the most frequently affected segment of bowel) and non-bloody diarrhoea, weight loss and

general malaise are also likely. On examination the patient is likely to have abdominal tenderness. Colonoscopy may be normal, as Crohn’s can affect any site of the alimentary canal,

classically causing skip lesions. Perianal disease (fistulae, abscesses, fissures) and mouth ulcers

are also common. Bloods tests may reveal anaemia, in particular a macrocytic a naemia due to

vitamin B 12 malabsorption if the terminal ileum is involved – additionally, evidence of

inflammation is likely on the patient’s blood tests.

Hiatus hernia

Hiatus hernia describes herniation of the stomach through the diaphragm – it is normally

asymptomatic but can present with symptoms of gastro -oesophageal reflux disease (GORD)

(indigestion/heart burn, nocturnal cough, acid brash). Blood tests and colonoscopy would be

normal, but it would not usually cause the patient’s abdominal symptoms.

Ulcerative colitis

This classically presents with abdominal pain and tenderness with bloody diarrhoea, weight loss

and general malaise. Ulcerative colitis (UC) affects the rectum, and a variable but continuous

amount of colon proximal to this. Colonoscopy would therefore not be expected to be normal

with ongoing symptoms as in this case. Bloods tests would also likely be abnormal, revealing

raised inflammatory markers.

300
Q

A

46-year-old

man attends the colorectal clinic presenting with a 4-month history of a

change in

bowel habit

with

blood

mixed with stool. He has no relevant past medical history.

What is the most appropriate investigation?

1) Barium enema

2) Capsule endoscopy

3) Colonoscopy

4) Computed tomography (CT) scan

5) Oesphagoduodenoscopy

A

Explanation

Colonoscopy

This 46-year-old man requires a lower gastrointestinal (GI) endoscopy for his red flag symptoms

of change in bowel habit associated with per rectum (PR) bleeding. This test is the best

investigation as it allows inspection of the bowel mucosa, biopsy if required and is potentially

even therapeutic, for example, if a polyp is seen this can be removed by snare and sent for

histology. It is an invasive test, requiring bowel preparation before the procedure but he is

otherwise well and there are no contraindications in undertaking colonoscopy.

Barium enema

A barium enema, although it could be useful, is not the best test for this patient. It may give an

impression of a lesion in the bowel wall but does not give such a ccurate information as would

colonoscopy in this case. Furthermore he would still require an endoscopic biopsy if a lesion was

seen.

Capsule endoscopy

Capsule endoscopy is an expensive test and not readily available. It is useful for looking for a

source of gastrointestinal (GI) bleeding that occurs in the small bowel beyond the scope of the

oesophago-gastroduodenoscopy (OGD) and proximal to the caecum/terminal ileum which

cannot be accessed by colonoscopy.

Computed tomography (CT) scan

CT scans maybe usefu l for detecting gross bowel pathology and staging for cancers but would

not be as useful as colonoscopy.

Oesphagoduodenoscopy

Oesphagoduodenoscopy (OGD) is useful if an upper gastrointestinal (GI) source is suspected but

this patient has lower GI symptoms.

301
Q

A 32-year-old woman with a history of chronic diarrhoea undergoes a flexible sigmoidoscopy.

There is

brownish discoloration

on the wall of the sigmoid in a

moiré pattern. Biopsy reveals characteristic

pigment-laden

macrophages within the mucosa on periodic acid–Schiff (PAS)

staining. She works as a

swimwear model

and is very concerned with her appearance.

What is the most likely diagnosis?

1) Crohn’s disease

2) Campylobacter infection

3) Diverticulitis

4) Melanosis coli

5) Ulcerative colitis

A

Explanation

Melanosis coli

Brownish discoloration on the wall of the sigmoid in a moiré pattern and characteristic pigment -

laden macrophages within the mucosa with PAS staining on mucosal biopsy are characteristic

features of melanosis coli and indicate la xative abuse. Although, colonoscopic appearance does

not have any negative sequelae, chronic diarrhoea requires advice on stopping laxative abuse.

Note her occupation as a model suggests she may feel pressurised to lose weight. Offer

psychological support if required.

Crohn’s disease

Colonoscopic findings of Crohn’s disease include cobblestone appearance, in which oedematous

islands of mucosa are separated by fissures. These can then extend through all layers of the

bowel. Transmural involvement can lead to stricture and fistulae formation. Skip lesions are

common where there is healthy bowel between diseased segments. Microscopically there can be

deep fissuring ulcers, oedema and inflammatory cell infiltrates with foci of lymphocytes and

non-caseating granulomas. Pigment -laden macrophages within the mucosa on PAS staining is

not a feature of Crohn’s disease.

Campylobacter infection

Diagnosis of Campylobacter infection is made on direct microscopy of faeces or isolation of the

organism. On sigmoidoscopy, the findings may be similar to those observed in

pseudomembranous colitis or inflammatory bowel disease. These include mucosal oedema,

hyperaemia, patchy petechiae and aphthoid ulceration. Pigment -laden macrophages within the

mucosa on PAS staining is not a feature of Campylobacter infection.

Diverticulitis

Diverticulitis may present with similar features as mentioned in the history. In the current era of

advanced imaging most of the complications of diverticulitis are diagnosed by computed

tomography (CT) scan. However lower gastrointestinal (GI) endoscopy is required to rule out malignancy once the acute phase is over. Sigmoidoscopy may show openings of diverticulae.

Pigment-laden macrophages within the mucosa on PAS staining is not a feature of diverticulitis .

Ulcerative colitis

On colonoscopy, inflammation is restricted to the mucosa and submucosa of the large bowel

with ulcerative colitis. Abscesses develop at the base of the colonic crypts and once burst can

lead to ulceration. Oedema of the intervening muc osa leads to pseudopolyp appearance.

Histologically, there is chronic inflammatory cell infiltrate, crypt architectural distortion and

goblet cell depletion. Granulomas are usually absent. Pigment -laden macrophages within the

mucosa on PAS staining is not a feature of ulcerative colitis.

302
Q

A 65-year-old woman presents with a change in bowel habit. She

has had

fresh bloody

diarrhoea

for the

last 2 months. Flexible sigmoidoscopy revealed

inflamed and

ulcerated

mucosa

from the rectum to as high as the scope could extend in the

splenic flexure

.

Biopsy shows

superficial ulceration

and

diffuse mucosal inflammation

. What is the likely diagnosis?

1) Crohn’s disease

2) Ischaemic colitis

3) Diverticulitis

4) Salmonella infection

5) Ulcerative colitis

A

Explanation

Ulcerative colitis

This patient has ulcerative colitis as evident in history, endoscopy findings with confluent

inflammation, and histology results. This inflammatory disorder differs from Crohn’s disease as

granulomas are absent, fissures are rare and inflammation is superficial involving the mucosal

layer.

Crohn’s disease

Colonoscopic findings of Crohn’s disease include a cobblestone appearance, in which

oedematous islands of mucosa are separated by fissures. These can then extend through all layers

of the bowel. Transmural involvement can lead to stricture and fistulae formation. Skip lesions

are common where there is healthy bowel between diseased segments. Microscopically there can

be deep fissuring ulcers, oedema and inflammatory cell infiltrates with foci of lymphocytes and

non-caseating granulomas. Pigment -laden macrophages within the mucosa on periodic acid–

Schiff (PAS) staining is not a feature of Crohn’s disease.

Ischaemic colitis

Ischaemic colitis presents in a similar way to inflammatory bowel disease or infectious colitis.

Endoscopic findings that distinguish ischaemic colitis from inflammatory bowel disease are the

segmental distribution, rectum sparing and rapid resolution on serial examinations. Histological

changes in ischaemic colitis include oedema, distorted crypts, mucosal and submucosal

haemorrhage, and inflammatory infiltration into the lamina propria, granulation tissue,

intravascular platelet thrombi and necrosis.

Diverticulitis

Diverticulitis may present with similar features as mentioned in history. In the current era of

advanced imaging most of the complications of diverticulosis are diagnosed by computed

tomography (CT) scan. However lower gastrointestinal (GI) endoscopy is required to rule out

malignancy once acute phase is over. Sigmoidoscopy may show openings of diverticulae.

Salmonella infection Diagnosis of Salmonella infection is made clinically and with microbiological investigations.

Colonoscopy may be required in some cases where complications such as bleeding occurs

requiring endoscopic intervention. Colonoscopic findings include multiple variable -sized

punched-out ulcers with slightly elevated margins. Moreover, the most commonly involved area

is the terminal ileum, followed by the ileocecal valve and ascending/transverse colon. The left

colon is usually spared.

303
Q

A 72-year-old patient presents with urinary retention.

He describes

pneumaturia and faeculent

debris

in his urine. He has had significant

two months. A contrast

enema shows an

weight loss and a change in bowel habit over the last obstructing lesion in the distal sigmoid colon. He

is

otherwise fit and well

.

What is the best definitive treatment in this case? 1) Catheterisation

2) Neostigmine

3) Stenting of the stricture

4) Surgical resection of colonic lesion

5) Neoadjuvant chemoradiotherapy

A

Explanation

Surgical resection of colonic lesion

Definitive management of colovesical fistulae is surgical. It may be with curative or palliative

intent depending upon the extent of local invasion. With curative intention it may include en bloc

resection with or without pelvic exenteration. With palliative intent it may be urinary and faecal

diversion leading to urostomy and colostomy respectively.

Catheterisation

This patient has developed a malignant colovesical fistula as a result of a sigmoid tumour. This

patient will initially need a catheter but it is not the ‘definitive’ treatment requested. A fistula

will not heal if there is distal obstruction – in this case an obstructing operative tumour.

Neostigmine

Neostigmine is an anticholinesterase agent that increases the effect of acetylcholine. It is helpful

in the management of patients with neurogenic bladder having urinary retention. It is not helpful

in patients having urinary retention due to a malignant colovesical fistula. Moreover, the

question is regarding definitive management, which will be en bloc resection with or without

pelvic exenteration; or faecal and urinary diversion in case of palliat ion.

Stenting of the stricture

Stenting may help in relieving colonic obstruction but it is not the definitive management of

malignant colovesical fistulae.

Neoadjuvant chemoradiotherapy

In case of rectal tumour, neoadjuvant chemoradiotherapy reduces the r isk of recurrence.

However, in the case of sigmoid tumours, neoadjuvant treatment is not recommended due to the

high position of the tumour that can put the small bowel at risk of radiation enteritis.

304
Q

You are called to the delivery room with your registrar to see a baby who the midwife says has

‘no abdominal wall’. When you get there you find a baby with an

abdominal defect

with loops of

bowel outside the abdomen, and

no covering layers.

What is the most likely diagnosis?

1) Umbilical hernia 2) Gastroschisis

3) Omphalocoele major

4) Hernia of the umbilical cord

5) Omphalocoele minor

A

Explanation

Gastroschisis

In gastroschisis there is no peritoneal covering, and it is rarely associated with congenital defects

seen in patients with omphalocoele. However, sometimes it may be associated with intestinal

atresia. Management initially consists of covering the defect with cling film to prevent heat and

moisture loss, while establishing intravenous (IV) access for IV fluids.

Umbilical hernia

An umbilical hernia is covered by skin. The defect is herniation of the abdominal contents

through the umbilical fibromuscular ring. Umbilical hernias usually occur in infants and reach

their maximal size by the first month of life. Most hernias of this type close spontaneously by the

first year of life, with only a 2–10% incidence in children older than 1 year.

Omphalocoele major

An omphalocoele is a congenital defect of the anterior abdominal wall in which the abdominal

contents are covered by a sac. The sac has three layers: the perito neum, Wharton’s jelly, and the

amnion. The sac may contain intestinal loops, the bladder, and the liver. Unlike gastroschisis it is

often associated with other congenital anomalies. Contents in the case of omphalocoele protrude

through the central defect in the abdomen and there is no umbilicus. Depending on the size of the

defect it may be labelled as major or minor. A defect of more than 4 cm is designated major,

while less than 4 cm it is minor.

Hernia of the umbilical cord

In a hernia of the umbilical cord a cuff of skin is seen extending from the abdominal wall onto

the neck of the sac. The hernia sac comprises of the outer amnion and the inner peritoneal lining

and contains contents varying from loops of bowel to any mobile intraperitoneal organs.

Omphalocoele minor

Omphalocoele minor is a congenital defect of the anterior abdominal wall in which the

abdominal contents are covered by a sac and the defect is less than 4 cm.

305
Q

A 9-month-old baby presents with excessive crying and

withdrawing of legs

for the last 12 h.

Between the attacks he looks

quite healthy. He has vomited a few times and has passed

mucous

mixed with

blood per rectum

shaped mass

in the upper

. A week ago he had a

viral illness. Examination reveals a

sausage-

abdomen.

What is the most likely diagnosis?

1) Pyloric stenosis

2) Intussusception

3) Gastroenteritis

4) Midgut volvulus

5) Necrotising enterocolitis

A

Explanation

Intussusception

Intussusception is the commonest gastrointestinal surgical emergency in children between 3

months and 2 years. The ileocecal junction is the commonest location. A part of the bowel

invaginates into the adjacent section, leading to obstruction. The patient usually presents with

intermittent colicky abdominal pain lasting 1–2 min which frequently recurs. It is associated with

vomiting. Blood is passed PR, often described as red-currant jelly in appearance. The

invaginated bowel segment may be palpable as a sausage-shaped mass.

Pyloric stenosis

Presentation with vomit ing may mimic pyloric stenosis, however pyloric stenosis usually

presents around the age of 4–6 weeks. Vomiting is projectile, bleeding per rectum is rare but an

olive-shaped mass may be palpable in epigastric region. Diagnosis is by ultrasound (US) scan o f

the abdomen. Electrolyte abnormalities include hypokalemia, hypochloraemic metabolic

alkalosis with paradoxical aciduria. Treatment is by open or laparoscopic pyloromyotomy.

Gastroenteritis

The presentation may mimic acute gastroenteritis, however a palp able sausage-shaped mass is

suggestive of intussusception.

Midgut volvulus

Midgut volvulus may present as a surgical emergency. With a 12 h history the abdomen is

distended and tender and the child is usually in a state of shock. Such patients require emer gency

laparotomy without any investigations.

Necrotising enterocolitis

Necrotising enterocolitis (NEC) is a condition that involves premature neonates. The usual

presentation is greenish aspirates in a premature neonate who is in the Neonatal Intensive Car e

Unit (NICU). The abdomen becomes shiny, tender and distended. An X -ray of the abdomen may

show pneumatosis intestinalis. Treatment is medical (with antibiotics) or surgical depending

upon the severity of the NEC.

306
Q

You are treating a 4-week-old baby who has had

jaundice since birth

at full-term, weight 5 lb.

The child has developed palpable firm

hepatosplenomegaly, pale stools and dark urine

.

Ultrasound scan (USS) shows

no dilatation

of the biliary tree.

What is the most likely cause?

1) Biliary atresia

2) Choledochal cysts

3) Extrahepatic bile duct tumour

4) Ileal atresia

5) Spontaneous perforation of the bile duct

A

Explanation

Biliary atresia

There are numerous causes of jaundice in neonates including physiological, med ical and

surgical. Surgical causes include biliary atresia, choledochal cysts, spontaneous perforation of the bile duct, inspissated bowel within the common bile duct, and extrahepatic bile duct tumours.

In biliary atresia the bile ducts are destroyed by inflammation. Clinical signs include jaundice,

and the development of firm hepatosplenomegaly, pale stools and dark urine, which are not seen

at birth. Children usually go to full term, but with a low birthweight. No single test is diagnostic

of this condit ion.

Choledochal cysts

Choledochal cysts are congenital cystic dilatations of the biliary tree. Infants typically present

with jaundice, pale stools and right upper quadrant mass. USS would classically reveal dilatation

of the biliary tree.

Extrahepatic bile duct tumour

Bile duct tumours are uncommon and typically present with jaundice, affecting older patients

with the average age of 60–65 years.

Ileal atresia

This is a malformation of the ileum resulting in a complete luminal obstruction and would

typically be diagnosed within the first 24 h of birth if it had not been picked up on antenatal

scans. The infant is unlikely to have passed stool, will have a distended abdomen and may be

vomiting.

Spontaneous perforation of the bile duct

This is a rare conditio n affecting infants and children with the cause unknown. The infants can

present acutely with biliary peritonitis or later with jaundice, pale stools and abdominal

distension. An ultrasound would demonstrate ascites.

307
Q

A 4-day-old full-term neonate has developed abdominal distension and

bilious

aspirates. He

passed meconium in the first 24 h. An abdominal X-ray shows

distended small bowel loops

and

fluid levels with

no air in the rectum.

What is the most likely cause?

1) Duodenal atresia

2) Hirschsprung’s disease

3) Meconium ileus

4) Oesophageal atresia

5) Small bowel atresia

A

Explanation

Small bowel atresia Small bowel atresia can be diagnosed antenatally. It presents with bilious vomiting, abdominal

distension, and failure to pass further stools following passage of meconium. Plain abdominal X-

ray shows distended loops of bowel with fluid levels.

Duodenal atresia

Duodenal atresia can be diagnosed antenatally. Abdominal distension is not marked as

obstruction is proximal. In duodenal atresia a double gas bubble would be expected on

abdominal X-ray.

Hirschsprung’s disease

In Hirschsprung’s disease there is failure in passage of meconium in the first 48 h in most cases.

The rest of the findings may match with the above description. Rectal stimulation and washouts

help in emptying the bowel unless the pathology involves whole of the colon.

Meconium ileus

In meconium ileus, passage of meconium fails to occur. X-ray findings may show a soap bubble

appearance. Meconium ileus may be managed by rectal washouts in certain selected cases. Work

up should include screening for cystic fibrosis.

Oesophageal atresia

With oesophageal atresia, excessive salivation and frothing occurs. A trial of passage of a

nasogastric tube (NG) tube fails and X-rays show NG tube coiling in the oesophagus.

308
Q

A

4-week-old

full-term male baby is brought with history of

projectile,

non-bilious

vomiting.

Examination reveals a

severely dehydrated

baby with a scaphoid abdomen.

Which one of the following is the most likely diagnosis?

1) Duodenal Atresia

2) Hirschprung’s disease

3) Pyloric stenosis

4) Malrotation

5) Meconium Ileus

A

Explanation

Pyloric stenosis Typical presentation is around 4–6 weeks in the first male baby of the family. Vomiting is

projectile and non-bilious due to the high level of obstruction (the thickened pyloric muscle),

which is above the entrance of the common bile duct into the duodenum. Distension is not

usually a feature. Between the episodes the baby is eager to take feeds. Electrolyte disturbance s

include hypokalaemic, and hypochloraemic metabolic alkalosis.

Duodenal Atresia

Vomiting is bilious and presentation is earlier following birth.

Hirschprung’s disease

Prominent features are inability to open the bowels and abdominal distension. Vomiting is a late

feature and is bilious.

Malrotation

Presentation is with acute abdomen and bilious vomiting. If midgut volvulus has developed the

baby requires emergency laparotomy. No further investigations should be done. Bilious vomiting

in a newborn is a surgical emergency until proved otherwise. Malrotation is the diagnosis, which

needs exclusion with an upper gastrointestinal (GI) contrast to look at the layout of the intestine,

specifically whether the duodenal–jejunal flexure is on the correct side of the vertebrae – the left

is the correct side.

Meconium Ileus

Presentation is at birth with abdominal distension, failure to pass stool and bilious aspirates.

309
Q

A 69-year-old man presents with a history of weight loss, altered bowel habit and feeling

generally unwell. Investigations reveal a caecal carcinoma. He undergoes a right hemicolectomy.

Histology reveals a large tumour that has

extended into the muscularis propria but spared the

lymph nodes.

What is the most likely Astler-Coller classification of Dukes’ staging for this tumour?

1) Dukes’ A

2) Dukes’ B1

3) Dukes’ C1

4) Dukes’ C2

5) Dukes’ D

A

According to the new Dukes’ staging it is B1 once the tumour has spread into the muscularis

propria but has not penetrated through it. If it has penetrated through he propria, it is staged as

B2.

The original staging system derived by Cuthbert Dukes was:

A – Invasion into but not through the bowel wall (90% 5-year survival)

B – Invasion through the bowel wall but not involving lymph nodes (70% 5-year survival)

C – Involvement of lymph nodes (30% 5-year survival)

D – Distant metastases.

This was revised by Astler–Coller in 1954:

A – Limited to mucosa

B1 – Into muscularis propria but not penetrating it. No nodes.

B2 – Penetrating muscularis propria. No nodes.

C1 – Metastasis to one to four regional lymph nodes

C2 – Metastasis to more than four lymph nodes.

D – Distant metastases.

Dukes’ A

If the tumour is limited to the mucosa it is classified as Dukes’ A.

Dukes’ C1

If the tumour has spread into regional lymph nodes (between one and four) it is staged as C1.

Dukes’ C2

Spread to lymph nodes beyond the regional lymph nodes (more than four) is classified as C2.

Dukes’ D

Distant metastasis is classified as stage D.

310
Q

An elderly man presents to your clinic with progressive weight loss, dysphagia and several

episodes of hematemesis. An upper gastrointestinal endoscopy reveals a fungating growth in

his

distal oesophagus

. Biopsy has been taken for histopathology.

What would be the most likely histopathological finding?

1) Squamous cell carcinoma

2) Malignant melanoma

3) Adenocarcinoma of the intestinal type

4) Metaplastic columnar epithelium

5) Dysplastic columnar epithelium

A

Explanation

Adenocarcinoma of the intestinal type

The lower end of the oesophagus is prone to develop metaplastic changes in response to reflux

oesophagitis. Change of normal squamous epithelium to columnar type is called as Barrett’s

oesophagus. These changes are reversible. However, in longst anding cases, dysplasia may occur

that can lead to adenocarcinoma. As the mass in the summary is in the lower oesophagus then,

most probably, malignant adenocarcinoma is the most likely diagnosis. Squamous cell carcinoma

Squamous cell carcinoma occurs in t he upper part of the oesophagus.

Malignant melanoma

Primary and metastatic malignant melanoma may occur in some patients, but it is a very rare

problem. Moreover history and endoscopic findings are more suggestive of adenocarcinoma.

Metaplastic columnar epithelium

This is suggestive of Barrett’s oesophagus. Oesophageal squamous epithelium is replaced by

columnar epithelium of the intestinal type with goblet cells. Adenocarcinoma may develop in

longstanding cases of Barrett’s oesophagus.

Dysplastic columnar epithelium

This finding is suggestive of dysplasia in Barrett’s oesophagus. It is a premalignant condition

and can lead to adenocarcinoma of the lower end of the oesophagus and longstanding cases of

gastro-oesophageal reflux disease (GORD). Oesophageal carcinoma is divided into an

adenocarcinoma, which affects the distal one-third (50–80%) and squamous cell carcinomas (90–

95%), which affect the upper two -thirds. Squamous cell carcinomas are associated with tobacco

(as is adenocarcinoma) and also alcohol intake, whereas adenocarcinoma is closely linked to

GORD and Barrett’s oesophagus. Most oesophageal cancers present late and therefore are not

suitable for minimally invasive surgery. Sadly, palliation is often the only treatment available.

311
Q

You see a 67-year-old woman in your clinic. She has been complaining for 2 months of anorexia,

abdominal bloating and 2 stones of weight loss. A computed tomography (CT) scan suggests

a

thickened stomach

without any definitive mass. Endoscopic biopsy reveals malignant

signet

ring

type cells with

intracytoplasmic mucin

.

Which one of the following is most likely diagnosis?

1) Metastatic tumours of the stomach

2) Gastrointestinal stromal tumour (GIST)

3) Gastric carcinoid

4) Gastric Lymphoma

5) Linitis plastica

A

Explanation

Linitis plastica

Gastric linitis plastica is a diffuse type of cancer. Malignant cells invade throughout the stomach,

resulting in the thickening and rigidity of the stomach wall. Most of the patients present late.

Endoscopically the stomach looks thick with reduced distensibility. Biopsy reveals invasion of

all the layers of the stomach in a diffuse pattern. Malignant cells look like a signet ring.

Prognosis is poor and treatment may include radical surgery or palliative chemotherapy.

Metastatic tumours of the stomach These tumours may present as ulcers or diffused stomach thickening, however biopsy will reveal

the site of origin of such metastasis.

Gastrointestinal stromal tumour (GIST)

Small gastric GISTs appear as serosal, submucosal or intramural nodules. They are usually

incidental findings during abdominal surgery or endoscopy. Histologically these are positive for

KIT tyrosine kinase receptors.

Gastric carcinoid

Histology shows enterochromaffin like (ECL) cells that are a distinctive type of neuroendocrine

cell in the gastric mucosa underlying the epithelium.

Gastric Lymphoma

Gastric lymphomas may be primary or secondary. Endoscopically, these are visible as enlarged

gastric folds, or ulcers in the case of low grade lesions and a mass in the case of high grade

lesions. Histology and immunohistochemistry may differentiate between different types of

lymphomas.

312
Q

You see a 67-year-old woman in your clinic. She has been complaining of 2 months of anorexia,

abdominal bloating and 2 stones of weight loss. A computed tomography (CT) scan suggests a

thickened stomach and oesophago -gastroduodenoscopy (OGD) reveal linitis plastica.

Which one of the following statements is true of stomach cancer?

1) There is a strong association between heavy alcohol intake and stomach cancer

2) The majority of H.pylori infections result in stomach cancer

3) Gastric carcinoma is commoner in women than men

4) Linitis plastica tends to be associated with H.pylori infection

5) Gastrointestinal stromal tumours form the most common histological subtype of gastric cancer

A

Explanation

There is a strong association between heavy alcohol intake and stomach cancer

There is significant body of evidence suggesting that a heavy alcohol intake increases gastric

cancer risk. Smoking also increases the risk from 40% increased risk for current smokers to 82%

increased risk for heavy smokers. Other risk factors include autoimmune atrophic gastritis,

pernicious anaemia, Menetrier disease and intestinal metaplasia.

The majority of H.pylori infections result in stomach cancer

H. pylori infection is the main risk factor for 65–80% of gastric carcinomas but these only occur

in approximately 2% of infections.

Gastric carcinoma is commoner in women than men Men are affected more co mmonly than women.

Linitis plastica tends to be associated with H.pylori infection

Linitis plastica tends not to be associated with H. pylori and has a more genetic association.

Gastrointestinal stromal tumours form the most common histological subtype of gastric cancer

The most common subtype of gastric cancer is adenocarcinoma (90 –95%). Other histological

subtypes include: lymphoma (1–5%), gastrointestinal stromal tumours (2%), carcinoids (1%),

adenoacanthomas (1%) and squamous cell carcinoma (1%).

313
Q

You review a 37-year-old woman in the Emergency department who presents with abdominal

pain and vomiting. On examination you note circumoral pigmentation and a distended abdomen.

A computed tomography (CT) scan shows ileo-ileal intussusception.

Which one of the following statements regarding Peutz–Jeghers syndrome is true?

1) It is an autosomal-recessive condition

2) It is characterised by polyps typically present in the small bowel

3) Pigmentation is limited to the circumoral skin and buccal mucosa

4) Histological examination of polyps adenomas

5) There is no increased risk of cancer with this condition

A

Explanation

It is characterised by polyps typically present in the small bowel

Polyps in Peutz–Jeghers are typically present in the small bowel but may affect the entire

gastrointestinal (GI) tract.

It is an autosomal-recessive condition

Peutz–Jeghers is an autosomal-dominant condition. It is associated with germ-line mutations in

the STK11 gene on chromosome 19.

Pigmentation is limited to the circumoral skin and buccal mucosa

Pigmentation can also affect the hands and genitalia.

Histological examination of polyps adenomas Polyps in Peutz–Jeghers in are hamartomas.

There is no increased risk of cancer with this condition

Hamartomas are usually benign; there is a very small risk of malignant transformation. However,

there is also an increased risk of pancreatic, breast, lung, ovarian and endometrial cancer

associated with Peutz–Jeghers syndrome.

314
Q

You are examining a man with abdominal pain, ascites and jaundice. An abdominal ultrasound

scan (USS) shows signs suspicious of a hepatocellular carcinoma.

Which one of the following is correct regarding hepatocellular carcinoma (HCC)?

1) Women are affected more than men

2) It is mainly associated with hepatitis B and C viruses

3) It is associated with type 1 diabetes

4) Serum a-fetoprotein is normal

5) Usually presents below the age of 30

A

Explanation

It is mainly associated with hepatitis B and C viruses

Hepatitis B virus (HBV) and hepatitis C virus (HCV) are well known causes of HCC. HBV is a

major independent risk factor and, in countries where it is prevalent, 90% of patients with HCC

are positive for HBV. With HCV the HCC classically develops in the setting of cirrhosis.

Women are affected more than men

Men are more commonly affected than women (3:1).

It is associated with type 1 diabetes

Although not completely understood there is an increased risk of HCC in type 2 diabetes and

relates to poor glucose control and insulin levels.

Serum a-fetoprotein is normal Serum α-fetoprotein is a tumour marker and is usually raised in HCC. It is also raised in other

cancers such as testicular and ovarian tumours.

Usually presents below the age of 30

HCC usually presents later with the average age at diagnosis of 65 years.

315
Q

A 59-year-old man presents with rectal bleeding and offensive stool. Investigations reveal a

Meckel’s diverticulum.

Which one of the following is true regarding Meckel’s diverticulum?

1) It involves two layers of the intestinal wall only

2) It is seen in 2% of people, usually presenting in patients over two years of age

3) It is 60 cm from the ileocaecal valve

4) It is usually found in the jejunum on the antimesenteric border and is approximately 5 cm

long

5) It occurs twice as frequently in women than in men

A

Explanation

It is 60 cm from the ileocaecal valve

It is classically found in the distal ileum, 60 cm (2 feet) from the ileocaecal valve on the

antimesenteric border of the ileum and is approximately 5 cm (2 inches) long.

It involves two layers of the intestinal wall only

A Meckel’s diverticulum is an example of a true diverticulum and contains all the layers of the

intestinal wall.

It is seen in 2% of people, usually presenting in patients over two years of a ge

While it is present in 2% of people, most symptomatic cases will present before the age of 2.

It is usually found in the jejunum on the antimesenteric border and is approximately 5 cm long It is classically found in the distal ileum, 60 cm from the ileo caecal valve on the antimesenteric

border and is approximately 5 cm long.

It occurs twice as frequently in women than in men

Meckel’s diverticulum occurs in 2% of the population and affects men twice as often as women.

316
Q

An infant attends the paediatric surgical clinic with their parents as a follow up patient. They had

a staged repair of a congenital abdominal wall defect.

Which one of the following statements about exomphalos is true?

1) In the vast majority of cases in which exomphalos is present it is the only congenital

abnormality

2) Maternal smoking decreases the risk of developing it

3) It is associated with Edward syndrome

4) Its incidence is decreasing

5) It is associated with Potter syndrome

A

Explanation

It is associated with Edward syndrome

Associated syndromes include Beckwith–Wiedemann syndrome (features include macroglossia,

macrosomia, omphalocele/exomphalos, umbilical hernia, ear pits, neonatal hypoglycaemia,

hepatoblastoma), Edward syndrome (trisomy 18, features include heart defects, omphalocele,

oesophageal atresia, microcephaly, micrognathia, cleft lip/palate, widely spaced eyes, ptosis,

webbing of toes, undescended testicles) and Patau syndrome (trisomy 13, features include

microcephaly, structural eye defects, polydactyly, low-set ears, omphalocele, rocker-bottom feet,

abnormal genitalia and heart defects).

In the vast majority of cases in which exomphalos is present it is the only congenital abnormality

The incidence of associated abnormalities is between 40–70%, including chromosomal

(trisomies 13, 18 and 21), cardiac, genitourinary, gastrointestinal, craniofacial, and pulmonary

hypoplasia.

Maternal smoking decreases the risk of developing it

Both maternal alcohol use and heavy smoking are associated with an increased ris k. Its incidence is decreasing

Incidence is 1 in 5000 to 1 in 10 000 live births although the incidence seems to be increasing

worldwide.

It is associated with Potter syndrome

Potter syndrome or sequence relates to renal abnormalities, it is not associated with exomphalos.

Oligohydramnios as a result of renal abnormalities, urinary tract obstruction or amniotic fluid

leakage may lead to fetal compression with joint contractures (arthrogryposis), pulmonary

hypoplasia and squashed facies.

317
Q

A surgical registrar is performing his third solo laparoscopic cholecystectomy in a 47 -year-old

woman. When attempting to

dissect out Calot’s triangle

to identify the cystic artery great

difficulty is experienced as the anatomy appears aberrant.

What proportion of patients have anomalies of the gall-bladder and biliary tree?

1) <1%

2) 25%

3) 50%

4) 75%

5) >90%

A

Explanation

50%

Calot’s triangle, also known as the cystohepatic or hepatobiliary triangle is an anatomic space

that is of great relevance when performing a laparoscopic cholecystectomy. It is bounded by the

common hepatic duct medially, cystic duct laterally and liver superiorly, and is used to locate the

cystic artery which runs within. Anomalies of the gall-bladder or biliary tree are found within

50% of subjects and are important during surgery. There are a vast array of different anomalies

that can occur, by way of example these anomalies include:

Anomalous ducts in around 12% – around 85% of which are found (and occasionally injured)

within Calot’s triangle.

Anomalous gall-bladder development – rarely agenesis of the gall-bladder (around 0.02%), more

commonly multiple gall bladders around (1 in 3800), or other anomalies such as the gall-bladder

opening directly into the side of the common bile duct ie the cystic duct is absent.

Variation in the blood vessel arrangement supplying the gall-bladder eg in 25% of people the

right hepatic artery crosses in front of the common hepatic duct instead of behind it. <1%

Difficult biliary anatomy is relatively common, often making what is usually a routine operation

very difficult.

25%

This answer is not correct, the correct answer is 50%.

75%

The variant anatomy encountered is less than this percentage.

> 90%

This estimate is not correct.

318
Q

A

neonate

is noted to have persistent,

progressive jaundice

, pale stools and dark urine. On

examination there is

hepatosplenomegaly

. The gall-bladder is

absent

on ultrasonography.

What is the most likely cause of this?

1) Breast milk jaundice

2) Hepatocellular cholestasis

3) Haemolytic disease of the new-born

4) Paediatric biliary atresia

5) Choledochal cyst

A

Explanation

Paediatric biliary atresia

Biliary atresia is of unknown aetiology. The extra-hepatic bile ducts are destroyed by

inflammation. It present s with jaundice, pale stools and dark urine with hepatosplenomegaly. An

absent gall bladder is characteristic finding of extrahepatic biliary atresia as it is atretic.

Treatment requires a Kasai’s procedure, this involves removal of all the extrahepatic tr ee and

portoenterostomy. Outcome is poor if diagnosis is delayed.

Breast milk jaundice

Breast milk jaundice is a type of neonatal jaundice that is associated with breastfeeding. It is

characterized by indirect hyperbilirubinemia in an otherwise healthy breastfed neonate. Signs of

obstructive jaundice mentioned in the summary will be absent. Diagnosis is by exclusion.

Hepatocellular cholestasis

Hepatocellular cholestasis results from stasis of bile pigments within the liver. The underlying

aetiology can include any of viral hepatitis, Alpha1-antitrypsin deficiency, inborn errors of bile

acid synthesis, drug-induced cholestasis or total parenteral nutrition (TPN) associated

cholestasis. However, the clinical and biochemical picture differs as features of obstructive

jaundice will be absent.

Haemolytic disease of the new-born The jaundice is of non-obstructive variety. It results from rhesus incompatibility. Antibodies

against Rh +ve red blood cells cross the placenta and cause haemolysis. Depending upon seve rity

the disease may range from hydrops fetalis to neonatal jaundice at birth. Laboratory

investigations may show indirect hyperbilirubinemia.

Choledochal cyst

The presentation closely resembles that of extrahepatic biliary atresia. The differentiating poi nt is

presence of a gall-bladder. A choledochal cyst on ultrasound may also be visible.

319
Q

A patient with Crohn’s has had multiple resections for stricturing disease. They are developing

further symptoms and attend the Emergency Department.

In a patient who has undergone an extensive ileocolic resection, which one of the following

biochemical abnormalities would be most expected?

1) A microcytic anaemia

2) Hyperparathyroidism

3) Vitamin K excess

4) Decreased parathyroid hormone (PTH)

5) Hypermagnesaemia

A

Explanation

Decreased parathyroid hormone (PTH)

The terminal ileum is an important site of bile salt re-absorption, excess loss of bile salts which

are responsible solubilising lipids in the jejunum. The excess loss of bile salts therefore leads to

fat malabsorption and excess fatty acids within the distal bowel – these can chelate magnesium

causing hypomagnesaemia (not hyper). Hypomagnesaemia has a suppressive effect on

parathyroid hormone (PTH) release resulting in acquired hypoparathyroidism.

A microcytic anaemia

The terminal ileum is the site of vitamin B 12 absorption – malabsorption leads to development of

a macrocytic anaemia.

Hyperparathyroidism

The terminal ileum is an important site of bile salt re-absorption. Excess loss of bile salts, which

are responsible solubilising lipids in the jejunum, can occur. The excess loss of bile salts leads to

fat malabsorption and excess fatty acids within the distal bowel – these fatty acids can chelate

magnesium causing hypomagnesaemia (not hypermagnesaemia). Hypomagnesaemia has a

suppressive effect on parathyroid hormone (PTH) release resulting in acquired

hypoparathyroidism.

Vitamin K excess

The terminal ileum is an important site of bile salt re-absorption, and therefo re loss results in fat

and fat soluble vitamin (vitamins A, D, E and K) malabsorption. Hypermagnesaemia

The terminal ileum is an important site of vitamin B 12 absorption as well as bile salt re-

absorption. Loss of bile salts results in fat malabsorption and excess fatty acids within the distal

bowel – these can chelate magnesium. This leads to hypomagnesaemia (not hypermagnesaemia).

320
Q

You receive the

tumour extends

histology report from a specimen from a colonic resection. It reveals that the into, but not through the muscularis propria. There is one local lymph nodegroup

involved.

What is the Dukes’ stage classification of the tumour according to the modified Astler–

Coller system?

1) D

2) C1

3) C2

4) B1

5) B2

A

Explanation

C1

Please see the NICE guidelines link for a comprehensive description of colorectal cancer staging

by the American Joint Committee on Cancer (AJCC, 7th edition). The modified Astler –Coller

staging system:

Stage A: Limited to the mucosa

Stage B1: Extending into the muscularis propria but not penetrating through it; nodes are not

involved.

Stage B2: Penetrating through the muscularis propria; nodes are not involved

Stage C1: Extending into the muscularis propria but not penetrating through it. Nodes are

involved.

Stage C2: Penetrating through the muscularis propria. Nodes are involved.

Stage D: Distant metastatic spread.

D

Stage D describes distant metastatic spread.

C2

In stage C2, the cancer penetrates through the muscularis propria and nodes ar e involved.

B1 In stage B1 the cancer extends into the muscularis propria but does not penetrate through it;

nodes are not involved.

B2

In stage B2, the cancer penetrates through the muscularis propria; nodes are not involved.

321
Q

While performing a radical mastectomy, the surgeon injured the long thoracic nerve. Which of the following muscles will be affected due to injury to the long thoracic nerve?
A Anterior scalene
B Middle scalene
C Serratus anterior
D Subscapularis
E Teres major

A

The muscle affected due to injury to the long thoracic nerve is the serratus anterior.

The long thoracic nerve innervates the serratus anterior muscle. Injury to this nerve can lead to a condition known as “winged scapula,” where the shoulder blade protrudes out abnormally. This can occur during surgeries such as a radical mastectomy, where the nerve may be damaged accidentally. The serratus anterior muscle plays a crucial role in the movement and stabilization of the scapula, particularly in protraction and upward rotation.

So, the correct answer is:
C. Serratus anterior

322
Q

A motorcyclist involved in a hit-and-run accident was thrown from his motorcycle. He landed on the right side of his head and the tip of his shoulder, bending his head sharply to the left and stretching the right side of his neck. Subsequent neurological examination revealed that the roots of the fifth and sixth cervical nerves had been torn away from the spinal cord. What part of the upper limb will have diminished cutaneous sensations in this patient?
A The back of the shoulder
B. The top of the shoulder and the lateral side of the arm
C The pectoral region
D The medial side of the armand fore.rm
E The tip of the little finger

A

The roots of the fifth and sixth cervical nerves contribute significantly to the brachial plexus, particularly forming the upper trunk, which eventually gives rise to several nerves that innervate specific areas of the upper limb. When these nerve roots are torn, it typically affects the areas supplied by the nerves that arise from these roots.

In this scenario, the cutaneous areas most affected would be those innervated by the C5 and C6 dermatomes. These areas include:

•	The top of the shoulder (innervated by the supraclavicular nerves, which are derived from C3 and C4 but can be affected indirectly due to proximity).
•	The lateral side of the arm (innervated by the axillary nerve from C5 and C6).
•	The lateral aspect of the forearm (innervated by the musculocutaneous nerve, which has contributions from C5, C6, and C7).

Given this information, the part of the upper limb that will have diminished cutaneous sensations in this patient is:

B. The top of the shoulder and the lateral side of the arm

323
Q

A 75-year-old patient has an olecranon fracture. This injury will most likely disrupt the function of which of the following muscles?”

The options given are:

A. Brachialis
B. Flexor digitorum profundus
C. Flexor pollicis longus
D. Pronator teres
E. Triceps brachii

A

The muscle most likely to be affected by an olecranon fracture is the triceps brachii, as the tendon of the triceps inserts on the olecranon process of the ulna. An olecranon fracture can disrupt the triceps’ ability to extend the elbow.

So, the correct answer is:
E. Triceps brachii

324
Q

A motorcyclist fell from his bike. On arrival in ASE it was noticed that extension of his right humerus was severely limited. He has sustained injury to the:
A Long thoracic nerve:
B Medial pectoral nerve
C Suprascapular nerve
D Thoracodorsal nerve
E Upper subscapular nerve

A

Extension of the humerus is primarily facilitated by muscles such as the latissimus dorsi and the long head of the triceps brachii. Among the nerves listed, the one that innervates the latissimus dorsi is the thoracodorsal nerve.

An injury to the thoracodorsal nerve would impair the function of the latissimus dorsi muscle, leading to a limitation in the extension of the humerus.

Therefore, the correct answer is:
D. Thoracodorsal nerve

325
Q

A young boy fell from his skateboard, twisted his forearm and sprained his annular ligament. The annular ligament in the forearm:
A Encircles the head of the radius
B Encircles the head of the ulna
C Encircles the styloid process of the radius
• Goes from the olecranon fossa to the olecranon process
E. Spans the space between the ulna and radius

A

The annular ligament is an important structure in the elbow joint. It specifically stabilizes the proximal radioulnar joint by holding the head of the radius in place as it articulates with the ulna.

The correct description of the annular ligament is:

A. Encircles the head of the radius

326
Q

Branches of the brachial artery contribute to the anastomotic circulation around the: -

A Acromioclavicular joint

B Elbow joint

C Glenohumeral joint

D Head of the humerus

E Scapula

A

The brachial artery provides several branches that contribute to the anastomotic circulation around the elbow joint, ensuring adequate blood supply to the area despite movements that might otherwise impede blood flow.

Therefore, the correct answer is:
B. Elbow joint

327
Q

A rugby player sustained injury to his right supraspinatus muscle. Injury to the supraspinatus muscle will affect:

A Adduction of the humerus

B Abduction of the humerus above the horizontal plane

C Lateral rotation of the humerus

D Initiation of abduction of the humerus

E Superior rotation of the scapula

A

The supraspinatus muscle is primarily responsible for initiating the first 15 degrees of abduction of the humerus at the shoulder joint. Injury to this muscle would specifically impair this initial movement, though full abduction can still be carried out by the deltoid muscle after the first 15 degrees.

Therefore, the correct answer is:
D. Initiation of abduction of the humerus

328
Q

While performing mastectomy, the surgeon injured the lateral thoracic artery. The lateral thoracic artery:

A Is a branch of the subclavian artery

B Is a branch of the brachial artery

C Accompanies the long thoracic nerve to the serratus anterior muscle

D Accompanies the thoracodorsal nerve to the serratus anterior muscle

E Emerges through the triangular space

A

The lateral thoracic artery is a branch of the axillary artery, specifically from its second part. This artery accompanies the long thoracic nerve and supplies blood to the serratus anterior muscle, among other structures.

Therefore, the correct answer is:
C. Accompanies the long thoracic nerve to the serratus anterior muscle

329
Q

A young gymnast sustained injury to a muscle that acts to depress the glenoid fossa directly. Which of the following is a muscle that acts to depress the glenoid fossa directly?
A Pectoralis minor
B Serratus anterior
C Pectoralis major
D Latissimus dorsi
E Supraspinatus

A

The muscle that acts to depress the glenoid fossa directly is the pectoralis minor. This muscle attaches to the coracoid process of the scapula and, when contracted, it pulls the scapula forward and downward, which in turn depresses the glenoid fossa.

Therefore, the correct answer is:
A. Pectoralis minor

330
Q

The common flexor tendon of the forearm was avulsed from its attachment. Which of the following muscles originates from the common flexor tendon of the forearm?
A Flexor digitorum superficialis
B Flexor digitorum profundus
C Flexor pollicis longus
D Pronator quadratus
E Extensor carpi ulnaris

A

The common flexor tendon of the forearm is the origin for several muscles located in the superficial and intermediate layers of the anterior (flexor) compartment of the forearm. One of the primary muscles that originates from this tendon is the flexor digitorum superficialis.

Therefore, the correct answer is:
A. Flexor digitorum superficialis

331
Q

Which of the following muscle combinations will work together to abduct the wrist?
A’ Pronator teres and brachioradialis
B Palmaris longus and extensor digitorum
C Extensor carpi radialis brevis and flexor carpi radialis
D Extensor carpi radialis brevis and extensor carpi ulnaris
E Supinator and extensor pollicis longus​​​​​​​​​​​​​​​​

A

The correct answer is D) Extensor carpi radialis brevis and extensor carpi ulnaris.

The abduction of the wrist is the movement of the hand away from the midline of the body. This motion is primarily accomplished by the combined action of two muscles:

  1. Extensor carpi radialis brevis: This muscle originates from the lateral epicondyle of the humerus and inserts into the base of the third metacarpal bone. When contracted, it extends and abducts the wrist.
  2. Extensor carpi ulnaris: This muscle originates from the lateral epicondyle of the humerus and inserts into the base of the fifth metacarpal bone. When contracted, it extends and abducts the wrist.

The other muscle combinations mentioned in the options are involved in different movements of the wrist and forearm:

A) Pronator teres and brachioradialis are involved in pronation and flexion of the forearm, respectively.
B) Palmaris longus and extensor digitorum are involved in flexion of the wrist and extension of the digits, respectively.
C) Extensor carpi radialis brevis and flexor carpi radialis are involved in abduction and flexion of the wrist, respectively.
E) Supinator and extensor pollicis longus are involved in supination of the forearm and extension of the thumb, respectively.

Therefore, the correct combination of muscles responsible for abducting the wrist is extensor carpi radialis brevis and extensor carpi ulnaris.​​​​​​​​​​​​​​​​

332
Q

Which of the following statements regarding adduction of the digits of the hand is CORRECT?

A The palmar interosseous muscles are the sole adductors
of the digits

B The lumbrical muscles produce adduction of the digits

C There are two muscles that produce adduction of the thumb

D All of the adductors of the digits take at least part of
their attachments from metacarpalbones

E Adduction of the digits will be affected by carpal tunnel syndrome

A

The correct statement regarding adduction of the digits of the hand is D) All of the adductors of the digits take at least part of their attachments from metacarpal bones.

Explanation:

A) Incorrect. The palmar interossei are not the sole adductors of the digits. The lumbrical muscles and the adductor pollicis also contribute to adduction.

B) Correct. The lumbrical muscles are responsible for adduction of the fingers at the metacarpophalangeal joints.

C) Incorrect. There is only one muscle specifically responsible for adduction of the thumb, which is the adductor pollicis.

D) Correct. All of the muscles responsible for adduction of the digits take at least part of their attachments from the metacarpal bones. The lumbrical muscles originate from the tendons of the flexor digitorum profundus and insert onto the radial side of the proximal phalanges and extensor expansions. The palmar interossei originate from the sides of the metacarpal bones and insert onto the bases of the proximal phalanges and extensor expansions. The adductor pollicis originates from the capitate bone and the bases of the 2nd and 3rd metacarpals and inserts onto the base of the proximal phalanx of the thumb.

E) Incorrect. Carpal tunnel syndrome primarily affects the median nerve, which does not directly innervate the muscles responsible for adduction of the digits. Carpal tunnel syndrome can affect the thenar and hypothenar muscle groups, which are responsible for abduction and opposition of the thumb and little finger, respectively.

Therefore, the correct statement is D) All of the adductors of the digits take at least part of their attachments from metacarpal bones.​​​​​​​​​​​​​​​​

333
Q

A victim of a road traffic accident has a severed nerve in her right upper limb resulting in loss of adduction of all the digits of the right hand.
Which nerve controls adduction of all the digits of the hand?

A Ulnar nerve

B Median nerve

C Radial nerve

D Upper subscapular nerve

E Musculocutaneous nerve

A

The correct answer is A) Ulnar nerve. The ulnar nerve innervates the intrinsic muscles of the hand, which are responsible for various movements, including adduction of the digits. When the ulnar nerve is damaged, as in the case of a severed nerve in the right upper limb, it can result in the loss of adduction of the digits of the affected hand. This condition is known as ulnar nerve palsy.

Source:

•	Nerve injuries of the hand: https://www.ncbi.nlm.nih.gov/pmc/articles/PMC5982572/
334
Q

During surgery to remove a lump from the axilla, a nerve originating from the lateral cord of the brachial plexus was injured. Which of the following nerves originates from the lateral cord of the brachial plexus?

A Ulnar

B Medial pectoral

C Suprascapular

D Lateral pectoral

E Thoracodorsal

A

The correct answer is D) Lateral pectoral nerve.

The lateral cord of the brachial plexus gives rise to several nerves, one of which is the lateral pectoral nerve. This nerve innervates the pectoralis major muscle, assisting in its function.

Source:

•	Brachial Plexus Anatomy: https://teachmeanatomy.info/upper-limb/nerves/brachial-plexus/
335
Q

A rugby player sustained spinal cord injury at spinal level C8. What is likely to be seen in this patient?

A The brachialis muscle will be paralysed
B The muscles innervated by the radial nerve that would most likely be affected would be the most proximal ones
C The hypothenar muscles would be completely paralysed
D Innervation to the deltoid muscle would be affected
E The patient, would be unable to flex their humerus

A

A rugby player who sustained a spinal cord injury at the C8 level is likely to experience specific neurological deficits based on the functions innervated by the C8 nerve roots. Here is an analysis of the options provided:

  • The brachialis muscle is primarily innervated by the musculocutaneous nerve, which originates from the C5 and C6 nerve roots. Therefore, a C8 spinal cord injury would not typically affect the brachialis muscle.
  • The radial nerve innervates muscles in the posterior compartment of the arm and forearm, including the triceps brachii and the extensor muscles of the forearm. Since the radial nerve receives contributions from C5 to T1, a C8 injury would not selectively affect the most proximal muscles but could impact the muscles innervated by the C8 and T1 roots, which are more distal.
  • The hypothenar muscles are innervated by the ulnar nerve, which receives fibers from the C8 and T1 nerve roots. A C8 spinal cord injury could indeed affect the hypothenar muscles, potentially leading to paralysis or significant weakness in these muscles[4][6].
  • The deltoid muscle is innervated by the axillary nerve, which originates from the C5 and C6 nerve roots. Therefore, a C8 spinal cord injury would not affect the innervation to the deltoid muscle[3][5].
  • Flexion of the humerus at the shoulder joint involves muscles such as the deltoid (anterior fibers) and the pectoralis major, which are innervated by nerves originating from the C5 and C6 nerve roots. Thus, a C8 spinal cord injury would not typically impair the ability to flex the humerus[5].

The most likely outcome for a patient with a C8 spinal cord injury is that the hypothenar muscles would be completely or significantly paralyzed due to the involvement of the ulnar nerve, which is innervated by the C8 and T1 nerve roots. Therefore, the correct answer is:

C. The hypothenar muscles would be completely paralysed

336
Q

The intertubercular groove of the humerus contains the:

A Tendon of the pectoralis minor muscle
B Tendon of the long head of the triceps brachii muscle
C Tendon of the coracobrachialis muscle
D Tendon of the short head of the biceps brachii muscle
E Tendon of the long head of the biceps brachii muscle

A

The intertubercular groove of the humerus, also known as the bicipital groove, contains the tendon of the long head of the biceps brachii muscle. This groove is a deep indentation on the humerus that separates the greater tubercle from the lesser tubercle and allows for the passage of the long tendon of the biceps brachii muscle, which is enveloped in a synovial sheath[2][3][4][6].

Therefore, the correct answer is:

E. Tendon of the long head of the biceps brachii muscle

Sources
[1] Radiographic anatomy of the intertubercular groove of the humerus https://pubmed.ncbi.nlm.nih.gov/3996427/
[2] Intertubercular sulcus - AnatomyZone https://anatomyzone.com/articles/intertubercular-sulcus/
[3] Bicipital groove - Wikipedia https://en.wikipedia.org/wiki/Bicipital_groove
[4] The Humerus - Proximal - Shaft - Distal - TeachMeAnatomy https://teachmeanatomy.info/upper-limb/bones/humerus/
[5] Intertubercular Tendon Sheath | Complete Anatomy - Elsevier https://www.elsevier.com/resources/anatomy/connective-tissue/connective-tissue-of-upper-limb/intertubercular-tendon-sheath/21848
[6] Review of Bicipital Groove Morphology and Its Analysis in North … https://www.ncbi.nlm.nih.gov/pmc/articles/PMC4392950/
[7] Intertubercular Sulcus | Complete Anatomy - Elsevier https://www.elsevier.com/resources/anatomy/skeletal-system/appendicular-skeleton/intertubercular-sulcus/24211
[8] Humerus: Anatomy and clinical notes - Kenhub https://www.kenhub.com/en/library/anatomy/the-humerus

337
Q

A young man presented in A&E following a street fight with profuse bleeding from the superior ulnar collateral artery. The superior ulnar collateral artery is a direct branch of which artery?

A Ulnar
B Radial
C Brachial
D Profunda brachii
E Axillary

A

Based on the information provided in the search results, the correct answer is C) Brachial artery.

The key evidence comes from the following excerpts:

“The superior ulnar collateral artery is a vessel arising from the brachial artery at the middle part of the arm.”

[2] “The superior ulnar collateral artery arises from the brachial artery.”

[3] “The superior ulnar collateral artery (inferior profunda artery), of small size, arises from the brachial artery a little below the middle of the arm.”

Multiple authoritative sources clearly state that the superior ulnar collateral artery originates as a branch from the brachial artery in the middle/upper arm region. Therefore, the brachial artery is the direct parent vessel that gives rise to the superior ulnar collateral artery. [2][3]

Sources
[1] Superior ulnar collateral artery | Radiology Reference Article https://radiopaedia.org/articles/superior-ulnar-collateral-artery?lang=gb
[2] Superior Ulnar Collateral Artery | Complete Anatomy - Elsevier https://www.elsevier.com/resources/anatomy/cardiovascular-system/arteries/superior-ulnar-collateral-artery/19414
[3] Superior ulnar collateral artery - Wikipedia https://en.wikipedia.org/wiki/Superior_ulnar_collateral_artery
[4] Superior Ulnar Collateral Artery Anatomy, Function & Diagram https://www.healthline.com/human-body-maps/superior-ulnar-collateral-artery/male
[5] Inferior ulnar collateral artery - Wikipedia https://en.wikipedia.org/wiki/Inferior_ulnar_collateral_artery
[6] Anterior ulnar recurrent artery - Wikipedia https://en.wikipedia.org/wiki/Anterior_ulnar_recurrent_artery
[7] Ulnar artery: Branches, definition, clinical notes - Kenhub https://www.kenhub.com/en/library/anatomy/the-ulnar-artery
[8] Posterior ulnar recurrent artery - Wikipedia https://en.wikipedia.org/wiki/Posterior_ulnar_recurrent_artery
[9] Brachial artery: Anatomy and branches - Kenhub https://www.kenhub.com/en/library/anatomy/brachial-artery
[10] Medial collateral artery - Wikipedia https://en.wikipedia.org/wiki/Medial_collateral_artery
[11] Radial collateral artery - Wikipedia https://en.wikipedia.org/wiki/Radial_collateral_artery
[12] Brachial artery - Wikipedia https://en.wikipedia.org/wiki/Brachial_artery
[13] Anatomy, Shoulder and Upper Limb, Profunda Brachii Artery - NCBI https://www.ncbi.nlm.nih.gov/books/NBK542225/
[14] Ulnar nerve - Wikipedia https://en.wikipedia.org/wiki/Ulnar_nerve
[15] Brachial artery | Radiology Reference Article | Radiopaedia.org https://radiopaedia.org/articles/brachial-artery?lang=gb
[16] Superior ulnar collateral artery - e-Anatomy - IMAIOS https://www.imaios.com/en/e-anatomy/anatomical-structure/superior-ulnar-collateral-artery-1553673144
[17] Posterior ulnar recurrent artery | Radiology Reference Article https://radiopaedia.org/articles/posterior-ulnar-recurrent-artery?lang=gb

338
Q

The primary ventral rami of the brachial plexus muscles

A Emerge from between the middle and posterior scalene
B Are formed from spinal levels C7 to T3
C Do not give rise to any nerves directly
D Form the trunks of the brachial plexus
E: Combine to form the cords of the brachial plexus

A

The correct answer is D) Form the trunks of the brachial plexus.

The key evidence from the search results:

[2] “The brachial plexus consists of roots, trunks, divisions, cords and terminal branches as it travels from proximal to distal upper limb…Roots are formed between the scalenus anterior and scalenus medius muscles by the anterior rami of C5-C8 and T1 nerve roots…Trunks form from the roots as they pass between scalenus anterior and medius muscles.”

[3] “The ventral rami of C5 to T1 are referred to as the “roots” of the plexus…Shortly after emerging from the intervertebral foramina, the 5 roots (C5-T1) unite to form 3 trunks.”

[4] “The brachial plexus is formed by the anterior rami (divisions) of cervical spinal nerves C5, C6, C7 and C8, and the first thoracic spinal nerve, T1…The ‘roots’ refer the anterior rami of the spinal nerves that comprise the brachial plexus.”

[5] “The brachial plexus is formed by the anterior primary rami of C5 through T1…The 3 trunks continue from the posterior triangle into the axilla, with C5 and C6 roots forming the superior trunk, C8 and T1 roots forming the inferior trunk, and the C7 root continuing as the middle trunk.”

Multiple authoritative sources clearly state that the ventral/anterior rami of the spinal nerves C5-T1 form the roots of the brachial plexus, and these roots then combine to form the three trunks (superior, middle, inferior) of the brachial plexus. Therefore, the primary ventral rami directly give rise to and form the trunks of the brachial plexus. [2][3][4][5]

Sources
[1] Anatomy of the Ventral Rami, Upper Trunk, and Its Divisions and … https://www.sciencedirect.com/science/article/abs/pii/B9780124103900000408
[2] Brachial plexus | Radiology Reference Article | Radiopaedia.org https://radiopaedia.org/articles/brachial-plexus?lang=gb
[3] Overview, Gross Anatomy, Blood Supply of the Brachial Plexus https://emedicine.medscape.com/article/1877731-overview
[4] The Brachial Plexus - Sections - Branches - TeachMeAnatomy https://teachmeanatomy.info/upper-limb/nerves/brachial-plexus/
[5] Anatomy, Head and Neck: Brachial Plexus - StatPearls - NCBI https://www.ncbi.nlm.nih.gov/books/NBK531473/

339
Q

The muscle producing the main movement of the proximal radioulnar joint was paralysed following a stab wound to the cubital fossa. Which of the following muscles produces the main movement Pronation

A Extensor carpi ulnaris
B Pronator teres
C Triceps brachii
D Brachioradialis
E Brachialis

A

Based on the information provided in the search results, the correct answer is B) Pronator teres.

The key evidence comes from the following excerpts:

[1] “The muscles that act on the proximal radioulnar joint to produce pronation are pronator quadratus and pronator teres. The force of the pronator teres is included in fast movements and movements against resistance.”

[2] “Pronation: Produced by the pronator quadratus and pronator teres.”

[3] “Rotatory movements of the head of the radius within the collar formed by the annular ligament and the radial notch, allowing…pronation (by the action of pronator teres and pronator quadratus muscles).”

Multiple authoritative sources clearly state that the pronator teres muscle is one of the primary muscles responsible for producing the pronation movement at the proximal radioulnar joint. The pronator teres is specifically mentioned as being involved in faster, more forceful pronation movements.

Therefore, if the muscle producing the main pronation movement at the proximal radioulnar joint was paralyzed following a stab wound to the cubital fossa (where the pronator teres passes through), the paralyzed muscle would be the pronator teres. [1][2][3]

Sources
[1] Proximal radioulnar joint: Anatomy, movements - Kenhub https://www.kenhub.com/en/library/anatomy/proximal-radioulnar-joint
[2] The Radioulnar Joints - TeachMeAnatomy https://teachmeanatomy.info/upper-limb/joints/radioulnar-joints/
[3] Proximal radioulnar joint | Radiology Reference Article - Radiopaedia https://radiopaedia.org/articles/proximal-radioulnar-joint-2?lang=gb
[4] elbow and radio-ulnar joints https://ouhsc.edu/bserdac/dthompso/web/namics/elbow.htm
[5] Elbow joint: Anatomy, ligaments, movements, blood supply | Kenhub https://www.kenhub.com/en/library/anatomy/elbow-joint

340
Q

Which of the following statements is CORRECT regarding the extensor retinaculum of the wrist?

A It is a direct extension of the axillary fascia
B The median nerve runs deep to it
C The tendon of the brachioradialis muscle runs through
D It contains a compartment for the thenar muscles
E It prevents the tendons of the posterior compartment of the forearm from ‘bowstringing’ when the hand is extended at the wrist

A

E) It prevents the tendons of the posterior compartment of the forearm from ‘bowstringing’ when the hand is extended at the wrist.

This statement is correct based on the information provided in the search results:

[1] “The extensor retinaculum of the wrist holds the extensor tendons in position and prevents bowstringing of the tendons.”

[2] “Extensor Retinaculum helps to keep the extensor tendons in alignment and prevent bowstringing during movements.”

[3] “The extensor retinaculum is a thickened portion of the antebrachial fascia that holds the tendons of the extensor muscles in place…It is located on the back of the forearm, just proximal to the hand.”

[4] “The extensor retinaculum of the wrist is the broad ligamentous sheet located at the dorsal aspect of the wrist and functions to keep the extensor tendons in alignment and prevent bowstringing during movement.”

Multiple authoritative sources clearly state that the main function of the extensor retinaculum is to prevent the extensor tendons passing through the posterior compartment of the forearm from bowstringing or deviating away from the wrist when the hand is extended. [1][2][3][4]

The other options are incorrect based on the information given:
A) It is not an extension of the axillary fascia, but rather the antebrachial fascia.
B) The median nerve does not run deep to the extensor retinaculum.
C) The brachioradialis tendon does not pass through the extensor retinaculum compartments.
D) It does not contain a compartment for the thenar (intrinsic hand) muscles.

Sources
[1] Extensor Retinaculum of Wrist | Complete Anatomy - Elsevier https://www.elsevier.com/resources/anatomy/connective-tissue/fasciae/extensor-retinaculum-of-wrist/17913
[2] Extensor Retinaculum (Wrist) - Physiopedia https://www.physio-pedia.com/Extensor_Retinaculum_%28Wrist%29
[3] Extensor retinaculum of the hand - Wikipedia https://en.wikipedia.org/wiki/Extensor_retinaculum_of_the_hand
[4] Extensor retinaculum (wrist) | Radiology Reference Article https://radiopaedia.org/articles/extensor-retinaculum-wrist?lang=gb
[5] The extensor retinaculum of the wrist - PubMed https://pubmed.ncbi.nlm.nih.gov/6747233/

341
Q

A 28-year-old woman undergoing ovarian stimulation for IVF presents with severe abdominal pain, ascites, and difficulty breathing. Laboratory results show elevated hematocrit and leukocytosis. What is the most appropriate initial management for severe ovarian hyperstimulation syndrome (OHSS)?

Options:

1.	Immediate laparotomy
2.	High-dose corticosteroids
3.	Intravenous fluids and anticoagulation
4.	Oral contraceptive pills
5.	Watchful waiting
A

Correct Answer:

3.	Intravenous fluids and anticoagulation

Explanation:

Severe ovarian hyperstimulation syndrome (OHSS) is a complication of ovarian stimulation characterized by enlarged ovaries, fluid accumulation in the abdomen (ascites), and a risk of thromboembolism due to hemoconcentration. Management includes supportive care with intravenous fluids to maintain intravascular volume, anticoagulation to prevent thromboembolic events, and careful monitoring. Immediate laparotomy and high-dose corticosteroids are not indicated. Oral contraceptive pills are not useful in the acute management of OHSS. Watchful waiting is inappropriate due to the severity of the symptoms and the potential for rapid deterioration.

Explanation
It is important to ascribe a severity to cases of OHSS for audit purposes, and a classification of mild, moderate, severe or critical exists.
Features of severe OHSS include: Clinical ascites (± hydrothorax)
Oliguria (< 300 ml/day or < 30 ml/hour)
Haematocrit > 0.45
Hyponatraemia (sodium < 135 mmol/l)
Hypo-osmolality (osmolality < 282 mOsm/kg)
Hyperkalaemia (potassium > 5 mmol/l)
Hypoproteinaemia (serum albumin < 35 g/l)
Ovarian size usually > 12 cma
Reference:
Royal College of Obstetricians and Gynaecologists (RCOG) Ovarian Hyperstimulation Syndrome, Management (Green-top Guideline No. 5).